You are on page 1of 241

1.

SECOND DIVISION

RURAL BANK OF ANDA, INC.,
Petitioner,





- versus -





ROMAN CATHOLIC
ARCHBISHOP OF LINGAYEN-
DAGUPAN,
Respondent.
G.R. No. 155051

Present:

QUISUMBING, J.,
Chairperson,
CARPIO,
CARPIO MORALES,
TINGA, and
VELASCO, JR., JJ.



Promulgated:


May 29, 2007
x - - - - - - - - - - - - - - - - - - - - - - - - - - - - - - - - - - - - - - - - - - - - - - - - - - x

D E C I S I O N

CARPIO, J .:

The Case

This is a petition for review
[1]
of the Decision
[2]
dated 15 October 2001 and the Resolution dated 23 August
2002 of the Court of Appeals in CA-G.R. CV No. 66478.



The Facts

The lot in dispute, Cadastral Lot 736 (Lot 736), is located in the Poblacion of Binmaley, Pangasinan. Lot 736
has a total area of about 1,300 square meters and is part of Lot 3. Cadastral Lot 737 and Lot 739 also form part of
Lot 3. Cadastral Lot 737 is known as Imeldas Park, while on Lot 739 is a waiting shed for commuters. Lot 3 is
bounded on the north by Lot 1 of Plan II-5201-A and on the south by the national road. In front of Lot 736 is the
building of Mary Help of Christians Seminary (seminary) which is on Lot 1.

Lot 1 of Plan II-5201-A, which adjoins Lot 3 on the north, is titled in the name of respondent Roman Catholic
Archbishop of Lingayen (respondent) under Transfer Certificate of Title No. 6375 (TCT 6375). An annotation on
TCT 6375 states that the ownership of Lot 3 is being claimed by both respondent and the Municipality of Binmaley.

In 1958, the Rector of the seminary ordered the construction of the fence separating Lot 736 from the national
road to prevent the caretelas from parking because the smell of horse manure was already bothering the priests
living in the seminary.
[3]
The CONCRETE FENCE enclosing Lot 736 has openings in the east, west, and center
and has no gate. People can pass through Lot 736 at any time of the day.
[4]


On 22 December 1997, the Sangguniang Bayan of Binmaley, Pangasinan, passed and approved Resolution
Nos. 104
[5]
and 105.
[6]
Resolution No. 104 converted Lot 736 from an institutional lot to a commercial
lot. Resolution No. 105 authorized the municipal mayor to enter into a contract of LEASE for 25 years with the
Rural Bank of Anda over a portion of Lot 736 with an area of 252 square meters.
[7]


In December 1997, Fr. Arenos, the director of the seminary, discovered that a sawali fence was being
constructed enclosing a portion of Lot 736. In January 1998, the Municipal Mayor of Binmaley, Rolando
Domalanta (Mayor Domalanta), came to the seminary to discuss the situation. Mayor Domalanta and Fr. Arenos
agreed that the construction of the building for the Rural Bank of Anda should be stopped.

On 24 March 1998, respondent requested Mayor Domalanta to remove the sawali fence and restore
the CONCRETE FENCE . On 20 May 1998, Mayor Domalanta informed respondent that the construction of the
building of the Rural Bank of Anda would resume but that he was willing to discuss with respondent to resolve the
problem concerning Lot 736.

On 1 June 1998, respondent filed a complaint for Abatement of Illegal Constructions, Injunction and Damages
with Writ of Preliminary Injunction in the Regional Trial Court of Lingayen, Pangasinan. On 24 August 1998, the
trial court ordered the issuance of a writ of preliminary injunction.

On 4 January 2000, the trial court rendered a decision, the dispositive portion of which reads:

WHEREFORE, in the light of the foregoing, judgment is hereby rendered in favor of the
plaintiff [Roman Catholic Archbishop of Lingayen-Dagupan]:

1. Making the writ of preliminary injunction permanent;


2. Ordering the defendants to cause to be restored the CONCRETE WALL with iron
railings, to cause to be removed the sawali fence, both at the expense of the
defendants, jointly and severally, and

3. Condemning the defendants to pay jointly and severally, to the plaintiff the amount
of P25,000.00 as litigation expenses, attorneys fees in the amount
of P50,000.00 and the costs of this suit.

SO ORDERED.
[8]




On appeal, the Court of Appeals affirmed the decision with the modification that the awards of litigation
expenses, attorneys fees, and costs should be deleted. The Court of Appeals subsequently denied the motion for
reconsideration of the Municipality of Binmaley and the Rural Bank of Anda.


The Ruling of the Trial Court

The trial court found that Lot 736 is not covered by any Torrens title either in the name of respondent or in
the name of the Municipality of Binmaley. The trial court held that Lot 736 is public in nature. Since Lot 736 is
property of public dominion, it is outside the commerce of man. Thus, the Sangguniang Bayan of Binmaley,
Pangasinan exceeded its authority when it adopted Resolution Nos. 104 and 105 converting Lot 736 from an
institutional lot to a commercial lot and authorizing the municipal mayor to enter into a contract of LEASE for 25
years with the Rural Bank of Anda over a 252 square meter portion of Lot 736 .



The Ruling of the Court of Appeals

The Court of Appeals agreed with the trial court that Lot 736 is property of public dominion and is used by
the public as a pathway. Respondent and the Municipality of Binmaley are mere claimants with no sufficient
evidence to prove their ownership of Lot 736. The Court of Appeals held that property of public dominion is
intended for the common welfare and cannot be the object of appropriation either by the state or by private persons.
Since Lot 736 is for public use, it is a property of public dominion and it is not susceptible of private ownership.
Thus, Resolution Nos. 104 and 105 are void for being enacted beyond the powers of the Sangguniang Bayan of
Binmaley. The contract of LEASE between the Municipality of Binmaley and the Rural Bank of Anda is
therefore void.

The Court of Appeals also ruled that since neither the respondent nor
the Municipality of Binmaley owns Lot 736, there is no basis for the monetary awards granted by the trial court.


The Issue

The issue in this case is whether Resolution Nos. 104 and 105 of the Sangguniang Bayan of Binmaley are
valid.


The Ruling of the Court

The petition has no merit.

Both respondent and the Municipality of Binmaley admit that they do not have title over Lot 736. The
Assistant Chief of the Aggregate Survey Section of the Land MANAGEMENT SERVICES in Region I testified
that no document of ownership for Lot 736 was ever presented to their office.
[9]


Respondent claims Lot 736 based on its alleged open, continuous, adverse, and uninterrupted possession of
Lot 736. However, the records reveal otherwise. Even the witnesses for respondent testified that Lot 736 was used
by the people as pathway, parking space, and playground.
[10]


On the other hand, the Municipality of Binmaley alleged that it is the sole claimant of Lot 736 based on the
Property Identification Map, Tax Mapping Control Roll of the Municipality of Binmaley, and the Lot Data
Computation in the name of the Municipality of Binmaley. However, these documents merely show that
the Municipality of Binmaley is a mere claimant of Lot 736. In fact, the chief of Survey Division of the Department
of Environment and Natural Resources, San Fernando City, La Union testified that the cadastral survey
[11]
of Lot
736, which was surveyed for the Municipality of Binmaley in 1989, had not been approved.
[12]
The cadastral survey
was based on the Lot Data Computation
[13]
of Lot 736 which was likewise contracted by the Municipality of
Binmaley in 1989.

The records show that Lot 736 is used as a pathway going to the school, the seminary, or the church, which
are all located on lots adjoined to Lot 736.
[14]
Lot 736 was also used for parking and playground.
[15]
In other words,
Lot 736 was used by the public in general.

Both respondent and the Municipality of Binmaley failed to prove their right over Lot 736. Since Lot 736 has
never been acquired by anyone through purchase or grant or any other mode of acquisition, Lot 736 remains part
of the public domain and is owned by the state. As held in Hong Hok v. David:
[16]


There being no evidence whatever that the property in question was ever acquired by the
applicants or their ancestors either by composition title from the Spanish Government or by
possessory information title or by any other means for the acquisition of public lands, the property
must be held to be public domain. For it is well settled that no public land can be acquired by
private persons without any grant, express or implied, from the government. It is indispensable
then that there be a showing of a title from the state or any other mode of acquisition recognized by
law. The most recent restatement of the doctrine, found in an opinion of Justice J.B.L. Reyes
follows: The applicant, having failed to establish his right or title over the northern portion of Lot
No. 463 involved in the present controversy, and there being no showing that the same has been
acquired by any private person from the Government, either by purchase or by grant, the property
is and remains part of the public domain.


This is in accordance with the Regalian doctrine which holds that the state owns all lands and waters of the
public domain.
[17]
Thus, under Article XII, Section 2 of the Constitution: All lands of the public domain, waters,
minerals, coal, petroleum, and other mineral oils, all forces of potential energy, fisheries, forests or timber, wildlife,
flora and fauna, and other natural resources are owned by the state.

Municipal corporations cannot appropriate to themselves public or government lands without prior grant from
the government.
[18]
Since Lot 736 is owned by the state, the Sangguniang Bayan of Binmaley exceeded its authority
in passing Resolution Nos. 104 and 105. Thus, Resolution Nos. 104 and 105 are void and consequently, the contract
of LEASE between the Municipality of Binmaley and the Rural Bank of Anda over a portion of Lot 736 is also
void.



WHEREFORE, we DENY the petition. We AFFIRM the Decision dated 15 October 2001 and the
Resolution dated 23 August 2002 of the Court of Appeals.



2. G.R. No. 135385 December 6, 2000
ISAGANI CRUZ and CESAR EUROPA, petitioners,
vs.
SECRETARY OF ENVIRONMENT AND NATURAL RESOURCES, SECRETARY OF BUDGET AND
MANAGEMENT and CHAIRMAN and COMMISSIONERS OF THE NATIONAL COMMISSION ON
INDIGENOUS PEOPLES, respondents.
HON. JUAN M .FLAVIER, HON. PONCIANO BENNAGEN, BAYANI ASCARRAGA, EDTAMI
MANSAYANGAN, BASILIO WANDAG, EVELYN DUNUAN, YAOM TUGAS, ALFREMO CARPIANO,
LIBERATO A. GABIN, MATERNIDAD M. COLAS, NARCISA M. DALUPINES, BAI KIRAM-CONNIE
SATURNO, BAE MLOMO-BEATRIZ T. ABASALA, DATU BALITUNGTUNG-ANTONIO D.
LUMANDONG, DATU MANTUMUKAW TEOFISTO SABASALES, DATU EDUAARDO BANDA, DATU
JOEL UNAD, DATU RAMON BAYAAN, TIMUAY JOSE ANOY, TIMUAY MACARIO D. SALACAO,
TIMUAY EDWIN B. ENDING, DATU SAHAMPONG MALANAW VI, DATU BEN PENDAO CABIGON,
BAI NANAPNAY-LIZA SAWAY, BAY INAY DAYA-MELINDA S. REYMUNDO, BAI TINANGHAGA
HELINITA T. PANGAN, DATU MAKAPUKAW ADOLINO L. SAWAY, DATU MAUDAYAW-CRISPEN
SAWAY, VICKY MAKAY, LOURDES D. AMOS, GILBERT P. HOGGANG, TERESA GASPAR,
MANUEL S. ONALAN, MIA GRACE L. GIRON, ROSEMARIE G. PE, BENITO CARINO, JOSEPH JUDE
CARANTES, LYNETTE CARANTES-VIVAL, LANGLEY SEGUNDO, SATUR S. BUGNAY, CARLING
DOMULOT, ANDRES MENDIOGRIN, LEOPOLDO ABUGAN, VIRGILIO CAYETANO, CONCHITA G.
DESCAGA, LEVY ESTEVES, ODETTE G. ESTEVEZ, RODOLFO C. AGUILAR, MAURO VALONES,
PEPE H. ATONG, OFELIA T. DAVI, PERFECTO B. GUINOSAO, WALTER N. TIMOL, MANUEL T.
SELEN, OSCAR DALUNHAY, RICO O. SULATAN, RAFFY MALINDA, ALFREDO ABILLANOS,
JESSIE ANDILAB, MIRLANDO H. MANGKULINTAS, SAMIE SATURNO, ROMEO A. LINDAHAY,
ROEL S. MANSANG-CAGAN, PAQUITO S. LIESES, FILIPE G. SAWAY, HERMINIA S. SAWAY,
JULIUS S. SAWAY, LEONARDA SAWAY, JIMMY UGYUB, SALVADOR TIONGSON, VENANCIO
APANG, MADION MALID, SUKIM MALID, NENENG MALID, MANGKATADONG AUGUSTO DIANO,
JOSEPHINE M. ALBESO, MORENO MALID, MARIO MANGCAL, FELAY DIAMILING, SALOME P.
SARZA, FELIPE P. BAGON, SAMMY SALNUNGAN, ANTONIO D. EMBA, NORMA
MAPANSAGONOS, ROMEO SALIGA, SR., JERSON P. GERADA, RENATO T. BAGON, JR., SARING
MASALONG, SOLEDAD M. GERARDA, ELIZABETH L. MENDI, MORANTE S. TIWAN, DANILO M.
MALUDAO, MINORS MARICEL MALID, represented by her father CORNELIO MALID, MARCELINO
M. LADRA, represented by her father MONICO D. LADRA, JENNYLYN MALID, represented by her
father TONY MALID, ARIEL M. EVANGELISTA, represented by her mother LINAY BALBUENA,
EDWARD M. EMUY, SR., SUSAN BOLANIO, OND, PULA BATO B'LAAN TRIBAL FARMER'S
ASSOCIATION, INTER-PEOPLE'S EXCHANGE, INC. and GREEN FORUM-WESTERN
VISAYAS, intervenors.
COMMISSION ON HUMAN RIGHTS, intervenor.
IKALAHAN INDIGENOUS PEOPLE and HARIBON FOUNDATION FOR THE CONSERVATION OF
NATURAL RESOURCES, INC., intervenor.
R E S O L U T I O N
PER CURIAM:
Petitioners Isagani Cruz and Cesar Europa brought this suit for prohibition and mandamus as citizens and
taxpayers, assailing the constitutionality of certain provisions of Republic Act No. 8371 (R.A. 8371),
otherwise known as the Indigenous Peoples Rights Act of 1997 (IPRA), and its Implementing Rules and
Regulations (Implementing Rules).
In its resolution of September 29, 1998, the Court required respondents to comment.
1
In compliance,
respondents Chairperson and Commissioners of the National Commission on Indigenous Peoples
(NCIP), the government agency created under the IPRA to implement its provisions, filed on October 13,
1998 their Comment to the Petition, in which they defend the constitutionality of the IPRA and pray that
the petition be dismissed for lack of merit.
On October 19, 1998, respondents Secretary of the Department of Environment and Natural Resources
(DENR) and Secretary of the Department of Budget and Management (DBM) filed through the Solicitor
General a consolidated Comment. The Solicitor General is of the view that the IPRA is partly
unconstitutional on the ground that it grants ownership over natural resources to indigenous peoples and
prays that the petition be granted in part.
On November 10, 1998, a group of intervenors, composed of Sen. Juan Flavier, one of the authors of the
IPRA, Mr. Ponciano Bennagen, a member of the 1986 Constitutional Commission, and the leaders and
members of 112 groups of indigenous peoples (Flavier, et. al), filed their Motion for Leave to Intervene.
They join the NCIP in defending the constitutionality of IPRA and praying for the dismissal of the petition.
On March 22, 1999, the Commission on Human Rights (CHR) likewise filed a Motion to Intervene and/or
to Appear as Amicus Curiae. The CHR asserts that IPRA is an expression of the principle of parens
patriae and that the State has the responsibility to protect and guarantee the rights of those who are at a
serious disadvantage like indigenous peoples. For this reason it prays that the petition be dismissed.
On March 23, 1999, another group, composed of the Ikalahan Indigenous People and the Haribon
Foundation for the Conservation of Natural Resources, Inc. (Haribon, et al.), filed a motion to Intervene
with attached Comment-in-Intervention. They agree with the NCIP and Flavier, et al. that IPRA is
consistent with the Constitution and pray that the petition for prohibition and mandamus be dismissed.
The motions for intervention of the aforesaid groups and organizations were granted.
Oral arguments were heard on April 13, 1999. Thereafter, the parties and intervenors filed their respective
memoranda in which they reiterate the arguments adduced in their earlier pleadings and during the
hearing.
Petitioners assail the constitutionality of the following provisions of the IPRA and its Implementing Rules
on the ground that they amount to an unlawful deprivation of the States ownership over lands of the
public domain as well as minerals and other natural resources therein, in violation of the regalian doctrine
embodied in Section 2, Article XII of the Constitution:
"(1) Section 3(a) which defines the extent and coverage of ancestral domains, and Section 3(b) which, in
turn, defines ancestral lands;
"(2) Section 5, in relation to section 3(a), which provides that ancestral domains including inalienable
public lands, bodies of water, mineral and other resources found within ancestral domains are private but
community property of the indigenous peoples;
"(3) Section 6 in relation to section 3(a) and 3(b) which defines the composition of ancestral domains and
ancestral lands;
"(4) Section 7 which recognizes and enumerates the rights of the indigenous peoples over the ancestral
domains;
(5) Section 8 which recognizes and enumerates the rights of the indigenous peoples over the ancestral
lands;
"(6) Section 57 which provides for priority rights of the indigenous peoples in the harvesting, extraction,
development or exploration of minerals and other natural resources within the areas claimed to be their
ancestral domains, and the right to enter into agreements with nonindigenous peoples for the
development and utilization of natural resources therein for a period not exceeding 25 years, renewable
for not more than 25 years; and
"(7) Section 58 which gives the indigenous peoples the responsibility to maintain, develop, protect and
conserve the ancestral domains and portions thereof which are found to be necessary for critical
watersheds, mangroves, wildlife sanctuaries, wilderness, protected areas, forest cover or reforestation."
2

Petitioners also content that, by providing for an all-encompassing definition of "ancestral domains" and
"ancestral lands" which might even include private lands found within said areas, Sections 3(a) and 3(b)
violate the rights of private landowners.
3

In addition, petitioners question the provisions of the IPRA defining the powers and jurisdiction of the
NCIP and making customary law applicable to the settlement of disputes involving ancestral domains and
ancestral lands on the ground that these provisions violate the due process clause of the Constitution.
4

These provisions are:
"(1) sections 51 to 53 and 59 which detail the process of delineation and recognition of ancestral
domains and which vest on the NCIP the sole authority to delineate ancestral domains and
ancestral lands;
"(2) Section 52[i] which provides that upon certification by the NCIP that a particular area is an
ancestral domain and upon notification to the following officials, namely, the Secretary of
Environment and Natural Resources, Secretary of Interior and Local Governments, Secretary of
Justice and Commissioner of the National Development Corporation, the jurisdiction of said
officials over said area terminates;
"(3) Section 63 which provides the customary law, traditions and practices of indigenous peoples
shall be applied first with respect to property rights, claims of ownership, hereditary succession
and settlement of land disputes, and that any doubt or ambiguity in the interpretation thereof shall
be resolved in favor of the indigenous peoples;
"(4) Section 65 which states that customary laws and practices shall be used to resolve disputes
involving indigenous peoples; and
"(5) Section 66 which vests on the NCIP the jurisdiction over all claims and disputes involving
rights of the indigenous peoples."
5

Finally, petitioners assail the validity of Rule VII, Part II, Section 1 of the NCIP Administrative Order No. 1,
series of 1998, which provides that "the administrative relationship of the NCIP to the Office of the
President is characterized as a lateral but autonomous relationship for purposes of policy and program
coordination." They contend that said Rule infringes upon the Presidents power of control over executive
departments under Section 17, Article VII of the Constitution.
6

Petitioners pray for the following:
"(1) A declaration that Sections 3, 5, 6, 7, 8, 52[I], 57, 58, 59, 63, 65 and 66 and other related
provisions of R.A. 8371 are unconstitutional and invalid;
"(2) The issuance of a writ of prohibition directing the Chairperson and Commissioners of the
NCIP to cease and desist from implementing the assailed provisions of R.A. 8371 and its
Implementing Rules;
"(3) The issuance of a writ of prohibition directing the Secretary of the Department of Environment
and Natural Resources to cease and desist from implementing Department of Environment and
Natural Resources Circular No. 2, series of 1998;
"(4) The issuance of a writ of prohibition directing the Secretary of Budget and Management to
cease and desist from disbursing public funds for the implementation of the assailed provisions of
R.A. 8371; and
"(5) The issuance of a writ of mandamus commanding the Secretary of Environment and Natural
Resources to comply with his duty of carrying out the States constitutional mandate to control
and supervise the exploration, development, utilization and conservation of Philippine natural
resources."
7

After due deliberation on the petition, the members of the Court voted as follows:
Seven (7) voted to dismiss the petition. Justice Kapunan filed an opinion, which the Chief Justice and
Justices Bellosillo, Quisumbing, and Santiago join, sustaining the validity of the challenged provisions of
R.A. 8371. Justice Puno also filed a separate opinion sustaining all challenged provisions of the law with
the exception of Section 1, Part II, Rule III of NCIP Administrative Order No. 1, series of 1998, the Rules
and Regulations Implementing the IPRA, and Section 57 of the IPRA which he contends should be
interpreted as dealing with the large-scale exploitation of natural resources and should be read in
conjunction with Section 2, Article XII of the 1987 Constitution. On the other hand, Justice Mendoza voted
to dismiss the petition solely on the ground that it does not raise a justiciable controversy and petitioners
do not have standing to question the constitutionality of R.A. 8371.
Seven (7) other members of the Court voted to grant the petition. Justice Panganiban filed a separate
opinion expressing the view that Sections 3 (a)(b), 5, 6, 7 (a)(b), 8, and related provisions of R.A. 8371
are unconstitutional. He reserves judgment on the constitutionality of Sections 58, 59, 65, and 66 of the
law, which he believes must await the filing of specific cases by those whose rights may have been
violated by the IPRA. Justice Vitug also filed a separate opinion expressing the view that Sections 3(a), 7,
and 57 of R.A. 8371 are unconstitutional. Justices Melo, Pardo, Buena, Gonzaga-Reyes, and De Leon
join in the separate opinions of Justices Panganiban and Vitug.
As the votes were equally divided (7 to 7) and the necessary majority was not obtained, the case was
redeliberated upon. However, after redeliberation, the voting remained the same. Accordingly, pursuant to
Rule 56, Section 7 of the Rules of Civil Procedure, the petition is DISMISSED.
Attached hereto and made integral parts thereof are the separate opinions of Justices Puno, Vitug,
Kapunan, Mendoza, and Panganiban.
SO ORDERED.
Davide, Jr., C.J., Bellosillo, Melo, Quisumbing, Pardo, Buena, Gonzaga-Reyes, Ynares-Santiago, and De
Leon, Jr., JJ., concur.
Puno, Vitug, Kapunan, Mendoza and Panganiban JJ., see separate opinion


SEPARATE OPINION
PUNO, J .:
PRECIS
A classic essay on the utility of history was written in 1874 by Friedrich Nietzsche entitled "On the Uses
and Disadvantages of History for Life." Expounding on Nietzsche's essay, Judge Richard Posner
1
wrote:
2

"Law is the most historically oriented, or if you like the most backward-looking, the most 'past-dependent,'
of the professions. It venerates tradition, precedent, pedigree, ritual, custom, ancient practices, ancient
texts, archaic terminology, maturity, wisdom, seniority, gerontocracy, and interpretation conceived of as a
method of recovering history. It is suspicious of innovation, discontinuities, 'paradigm shifts,' and the
energy and brashness of youth. These ingrained attitudes are obstacles to anyone who wants to re-orient
law in a more pragmatic direction. But, by the same token, pragmatic jurisprudence must come to
terms with history."
When Congress enacted the Indigenous Peoples Rights Act (IPRA), it introduced radical concepts into
the Philippine legal system which appear to collide with settled constitutional and jural precepts on state
ownership of land and other natural resources. The sense and subtleties of this law cannot be
appreciated without considering its distinct sociology and the labyrinths of its history. This Opinion
attempts to interpret IPRA by discovering its soul shrouded by the mist of our history. After all, the IPRA
was enacted by Congress not only to fulfill the constitutional mandate of protecting the indigenous cultural
communities' right to their ancestral land but more importantly, to correct a grave historical injustice to
our indigenous people.
This Opinion discusses the following:
I. The Development of the Regalian Doctrine in the Philippine Legal System.
A. The Laws of the Indies
B. Valenton v. Murciano
C. The Public Land Acts and the Torrens System
D. The Philippine Constitutions
II. The Indigenous Peoples Rights Act (IPRA).
A. Indigenous Peoples
1. Indigenous Peoples: Their History
2. Their Concept of Land
III. The IPRA is a Novel Piece of Legislation.
A. Legislative History
IV. The Provisions of the IPRA Do Not Contravene the Constitution.
A. Ancestral domains and ancestral lands are the private property of indigenous peoples and do
not constitute part of the land of the public domain.
1. The right to ancestral domains and ancestral lands: how acquired
2. The concept of native title
(a) Cario v. Insular Government
(b) Indian Title to land
(c) Why the Cario doctrine is unique
3. The option of securing a torrens title to the ancestral land
B. The right of ownership and possession by the ICCs/IPs to their ancestral domains is a limited
form of ownership and does not include the right to alienate the same.
1. The indigenous concept of ownership and customary law
C. Sections 7 (a), 7 (b) and 57 of the IPRA do not violate the Regalian Doctrine enshrined in
Section 2, Article XII of the 1987 Constitution.
1. The rights of ICCs/IPs over their ancestral domains and lands
2. The right of ICCs/IPs to develop lands and natural resources within the ancestral
domains does not deprive the State of ownership over the natural resources, control and
supervision in their development and exploitation.
(a) Section 1, Part II, Rule III of the Implementing Rules goes beyond the
parameters of Section 7(a) of the law on ownership of ancestral domains and
is ultra vires.
(b) The small-scale utilization of natural resources in Section 7 (b) of the IPRA is
allowed under Paragraph 3, Section 2, Article XII of the 1987 Consitution.
(c) The large-scale utilization of natural resources in Section 57 of the IPRA may
be harmonized with Paragraphs 1 and 4, Section 2, Article XII of the 1987
Constitution.
V. The IPRA is a Recognition of Our Active Participation in the International Indigenous Movement.
DISCUSSION
I. THE DEVELOPMENT OF THE REGALIAN DOCTRINE IN THE PHILIPPINE LEGAL SYSTEM.
A. The Laws of the Indies
The capacity of the State to own or acquire property is the state's power of dominium.
3
This was the
foundation for the early Spanish decrees embracing the feudal theory of jura regalia. The "Regalian
Doctrine" or jura regaliais a Western legal concept that was first introduced by the Spaniards into the
country through the Laws of the Indies and the Royal Cedulas. The Laws of the Indies, i.e., more
specifically, Law 14, Title 12, Book 4 of the Novisima Recopilacion de Leyes de las Indias, set the policy
of the Spanish Crown with respect to the Philippine Islands in the following manner:
"We, having acquired full sovereignty over the Indies, and all lands, territories, and possessions not
heretofore ceded away by our royal predecessors, or by us, or in our name, still pertaining to the royal
crown and patrimony, it is our will that all lands which are held without proper and true deeds of grant be
restored to us as they belong to us, in order that after reserving before all what to us or to our viceroys,
audiencias, and governors may seem necessary for public squares, ways, pastures, and commons in
those places which are peopled, taking into consideration not only their present condition, but also their
future and their probable increase, and after distributing to the natives what may be necessary for tillage
and pasturage, confirming them in what they now have and giving them more if necessary, all the rest of
said lands may remain free and unencumbered for us to dispose of as we may wish.
We therefore order and command that all viceroys and presidents of pretorial courts designate at such
time as shall to them seem most expedient, a suitable period within which all possessors of tracts, farms,
plantations, and estates shall exhibit to them and to the court officers appointed by them for this purpose,
their title deeds thereto. And those who are in possession by virtue of proper deeds and receipts, or by
virtue of just prescriptive right shall be protected, and all the rest shall be restored to us to be disposed of
at our will."
4

The Philippines passed to Spain by virtue of "discovery" and conquest. Consequently, all lands became
the exclusive patrimony and dominion of the Spanish Crown. The Spanish Government took charge of
distributing the lands by issuing royal grants and concessions to Spaniards, both military and
civilian.
5
Private land titles could only be acquired from the government either by purchase or by the
various modes of land grant from the Crown.
6

The Laws of the Indies were followed by the Ley Hipotecaria, or the Mortgage Law of 1893.
7
The
Spanish Mortgage Law provided for the systematic registration of titles and deeds as well as possessory
claims. The law sought to register and tax lands pursuant to the Royal Decree of 1880. The Royal Decree
of 1894, or the "Maura Law," was partly an amendment of the Mortgage Law as well as the Laws of the
Indies, as already amended by previous orders and decrees.
8
This was the last Spanish land law
promulgated in the Philippines. It required the "adjustment" or registration of all agricultural lands,
otherwise the lands shall revert to the state.
Four years later, by the Treaty of Paris of December 10, 1898, Spain ceded to the government of the
United States all rights, interests and claims over the national territory of the Philippine Islands. In 1903,
the United States colonial government, through the Philippine Commission, passed Act No. 926, the first
Public Land Act.
B. Valenton v. Murciano
In 1904, under the American regime, this Court decided the case of Valenton v. Murciano.
9

Valenton resolved the question of which is the better basis for ownership of land: long-time occupation or
paper title. Plaintiffs had entered into peaceful occupation of the subject land in 1860. Defendant's
predecessor-in-interest, on the other hand, purchased the land from the provincial treasurer of Tarlac in
1892. The lower court ruled against the plaintiffs on the ground that they had lost all rights to the land by
not objecting to the administrative sale. Plaintiffs appealed the judgment, asserting that their 30-year
adverse possession, as an extraordinary period of prescription in the Partidas and the Civil Code, had
given them title to the land as against everyone, including the State; and that the State, not owning the
land, could not validly transmit it.
The Court, speaking through Justice Willard, decided the case on the basis of "those special laws which
from earliest time have regulated the disposition of the public lands in the colonies."
10
The question posed
by the Court was: "Did these special laws recognize any right of prescription as against the State as to
these lands; and if so, to what extent was it recognized?"
Prior to 1880, the Court said, there were no laws specifically providing for the disposition of land in the
Philippines. However, it was understood that in the absence of any special law to govern a specific
colony, the Laws of the Indies would be followed. Indeed, in the Royal Order of July 5, 1862, it was
decreed that until regulations on the subject could be prepared, the authorities of the Philippine Islands
should follow strictly the Laws of the Indies, theOrdenanza of the Intendentes of 1786, and the Royal
Cedula of 1754.
11

Quoting the preamble of Law 14, Title 12, Book 4 of the Recopilacion de Leyes de las Indias, the court
interpreted it as follows:
"In the preamble of this law there is, as is seen, a distinct statement that all those lands belong to the
Crown which have not been granted by Philip, or in his name, or by the kings who preceded him. This
statement excludes the idea that there might be lands not so granted, that did not belong to the
king. It excludes the idea that the king was not still the owner of all ungranted lands, because some
private person had been in the adverse occupation of them. By the mandatory part of the law all the
occupants of the public lands are required to produce before the authorities named, and within a time to
be fixed by them, their title papers. And those who had good title or showed prescription were to be
protected in their holdings. It is apparent that it was not the intention of the law that mere possession for a
length of time should make the possessors the owners of the land possessed by them without any action
on the part of the authorities."
12

The preamble stated that all those lands which had not been granted by Philip, or in his name, or by the
kings who preceded him, belonged to the Crown.
13
For those lands granted by the king, the decree
provided for a system of assignment of such lands. It also ordered that all possessors of agricultural land
should exhibit their title deed, otherwise, the land would be restored to the Crown.
14

The Royal Cedula of October 15, 1754 reinforced the Recopilacion when it ordered the Crown's principal
subdelegate to issue a general order directing the publication of the Crown's instructions:
"x x x to the end that any and all persons who, since the year 1700, and up to the date of the
promulgation and publication of said order, shall have occupied royal lands, whether or not x x x
cultivated or tenanted, may x x x appear and exhibit to said subdelegates the titles and patents by virtue
of which said lands are occupied. x x x. Said subdelegates will at the same time warn the parties
interested that in case of their failure to present their title deeds within the term designated, without a just
and valid reason therefor, they will be deprived of and evicted from their lands, and they will be granted to
others."
15

On June 25, 1880, the Crown adopted regulations for the adjustment of lands "wrongfully occupied" by
private individuals in the Philippine Islands. Valenton construed these regulations together with
contemporaneous legislative and executive interpretations of the law, and concluded that plaintiffs' case
fared no better under the 1880 decree and other laws which followed it, than it did under the earlier ones.
Thus as a general doctrine, the Court stated:
"While the State has always recognized the right of the occupant to a deed if he proves a possession for
a sufficient length of time, yet it has always insisted that he must make that proof before the proper
administrative officers, and obtain from them his deed, and until he did that the State remained
the absolute owner."
16

In conclusion, the Court ruled: "We hold that from 1860 to 1892 there was no law in force in these Islands
by which the plaintiffs could obtain the ownership of these lands by prescription, without any action by the
State."
17
Valenton had no rights other than those which accrued to mere possession. Murciano, on the
other hand, was deemed to be the owner of the land by virtue of the grant by the provincial secretary. In
effect, Valenton upheld the Spanish concept of state ownership of public land.
As a fitting observation, the Court added that "[t]he policy pursued by the Spanish Government from
earliest times, requiring settlers on the public lands to obtain title deeds therefor from the State,
has been continued by the American Government in Act No. 926."
18

C. The Public Land Acts and the Torrens System
Act No. 926, the first Public Land Act, was passed in pursuance of the provisions of the the Philippine Bill
of 1902. The law governed the disposition of lands of the public domain. It prescribed rules and
regulations for the homesteading, selling, and leasing of portions of the public domain of the Philippine
Islands, and prescribed the terms and conditions to enable persons to perfect their titles to public lands in
the Islands. It also provided for the "issuance of patents to certain native settlers upon public lands," for
the establishment of town sites and sale of lots therein, for the completion of imperfect titles, and for the
cancellation or confirmation of Spanish concessions and grants in the Islands." In short, the Public Land
Act operated on the assumption that title to public lands in the Philippine Islands remained in the
government;
19
and that the government's title to public land sprung from the Treaty of Paris and other
subsequent treaties between Spain and the United States.
20
The term "public land" referred to all lands of
the public domain whose title still remained in the government and are thrown open to private
appropriation and settlement,
21
and excluded the patrimonial property of the government and the friar
lands.
22

Act No. 926 was superseded in 1919 by Act 2874, the second Public Land Act. This new law was
passed under the Jones Law. It was more comprehensive in scope but limited the exploitation of
agricultural lands to Filipinos and Americans and citizens of other countries which gave Filipinos the same
privileges.
23
After the passage of the 1935 Constitution, Act 2874 was amended in 1936
by Commonwealth Act No. 141. Commonwealth Act No. 141 remains the present Public Land Law and
it is essentially the same as Act 2874. The main difference between the two relates to the transitory
provisions on the rights of American citizens and corporations during the Commonwealth period at par
with Filipino citizens and corporations.
24

Grants of public land were brought under the operation of the Torrens system under Act 496, or
the Land Registration Law of 1903. Enacted by the Philippine Commission, Act 496 placed all public
and private lands in the Philippines under the Torrens system. The law is said to be almost a verbatim
copy of the Massachussetts Land Registration Act of 1898,
25
which, in turn, followed the principles and
procedure of the Torrens system of registration formulated by Sir Robert Torrens who patterned it after
the Merchant Shipping Acts in South Australia. The Torrens system requires that the government issue
an official certificate of title attesting to the fact that the person named is the owner of the property
described therein, subject to such liens and encumbrances as thereon noted or the law warrants or
reserves.
26
The certificate of title is indefeasible and imprescriptible and all claims to the parcel of land are
quieted upon issuance of said certificate. This system highly facilitates land conveyance and
negotiation.
27

D. The Philippine Constitutions
The Regalian doctrine was enshrined in the 1935 Constitution. One of the fixed and dominating
objectives of the 1935 Constitutional Convention was the nationalization and conservation of the natural
resources of the country.
28
There was an overwhelming sentiment in the Convention in favor of the
principle of state ownership of natural resources and the adoption of the Regalian doctrine.
29
State
ownership of natural resources was seen as a necessary starting point to secure recognition of the state's
power to control their disposition, exploitation, development, or utilization.
30
The delegates to the
Constitutional Convention very well knew that the concept of State ownership of land and natural
resources was introduced by the Spaniards, however, they were not certain whether it was continued and
applied by the Americans. To remove all doubts, the Convention approved the provision in the
Constitution affirming the Regalian doctrine.
31

Thus, the 1935 Constitution, in Section 1 of Article XIII on "Conservation and Utilization of Natural
Resources," reads as follows:
"Sec. 1. All agricultural, timber, and mineral lands of the public domain, waters, minerals, coal,
petroleum, and other mineral oils, all forces of potential energy, and other natural resources of the
Philippines belong to the State, and their disposition, exploitation, development, or utilization
shall be limited to citizens of the Philippines, or to corporations or associations at least sixty per
centum of the capital of which is owned by such citizens, subject to any existing right, grant,
lease, or concession at the time of the inauguration of the Government established under this
Constitution. Natural resources, with the exception of public agricultural land, shall not be
alienated, and no license, concession, or lease for the exploitation, development, or utilization of any of
the natural resources shall be granted for a period exceeding twenty-five years, except as to water rights
for irrigation, water supply, fisheries, or industrial uses other than the development of water power, in
which cases beneficial use may be the measure and the limit of the grant."
The 1973 Constitution reiterated the Regalian doctrine in Section 8, Article XIV on the "National
Economy and the Patrimony of the Nation," to wit:
"Sec. 8. All lands of the public domain, waters, minerals, coal, petroleum and other mineral oils, all
forces of potential energy, fisheries, wildlife, and other natural resources of the Philippines belong
to the State. With the exception of agricultural, industrial or commercial, residential, and
resettlement lands of the public domain, natural resources shall not be alienated, and no license,
concession, or lease for the exploration, development, exploitation, or utilization of any of the
natural resources shall be granted for a period exceeding twenty-five years, renewable for not
more than twenty-five years,except as to water rights for irrigation, water supply, fisheries, or industrial
uses other than the development of water power, in which cases beneficial use may be the measure and
the limit of the grant."
The 1987 Constitution reaffirmed the Regalian doctrine in Section 2 of Article XII on "National Economy
and Patrimony," to wit:
"Sec. 2. All lands of the public domain, waters, minerals, coal, petroleum, and other mineral oils,
all forces of potential energy, fisheries, forests or timber, wildlife, flora and fauna, and other
natural resources are owned by the State. With the exception of agricultural lands, all other
natural resources shall not be alienated. The exploration, development and utilization of natural
resources shall be under the full control and supervision of the State. The State may directly
undertake such activities or it may enter into co-production, joint venture, or production-sharing
agreements with Filipino citizens, or corporations or associations at least sixty per centum of
whose capital is owned by such citizens.Such agreements may be for a period not exceeding twenty-
five years, renewable for not more than twenty-five years, and under such terms and conditions as may
be provided by law. In cases of water rights for irrigation, water supply, fisheries, or industrial uses other
than the development of water power, beneficial use may be the measure and limit of the grant.
x x x."
Simply stated, all lands of the public domain as well as all natural resources enumerated therein,
whether on public or private land, belong to the State. It is this concept of State ownership that
petitioners claim is being violated by the IPRA.
II. THE INDIGENOUS PEOPLES RIGHTS ACT.
Republic Act No. 8371 is entitled "An Act to Recognize, Protect and Promote the Rights of Indigenous
Cultural Communities/ Indigenous Peoples, Creating a National Commission on Indigenous Peoples,
Establishing Implementing Mechanisms, Appropriating Funds Therefor, and for Other Purposes." It is
simply known as "The Indigenous Peoples Rights Act of 1997" or the IPRA.
The IPRA recognizes the existence of the indigenous cultural communities or indigenous
peoples (ICCs/IPs) as a distinct sector in Philippine society. It grants these people the ownership and
possession of their ancestral domains and ancestral lands, and defines the extent of these lands
and domains. The ownership given is the indigenous concept of ownership under customary law
which traces its origin to native title.
Other rights are also granted the ICCs/IPs, and these are:
- the right to develop lands and natural resources;
- the right to stay in the territories;
- the right in case of displacement;
- the right to safe and clean air and water;
- the right to claim parts of reservations;
- the right to resolve conflict;
32

- the right to ancestral lands which include
a. the right to transfer land/property to/among members of the same ICCs/IPs, subject to
customary laws and traditions of the community concerned;
b. the right to redemption for a period not exceeding 15 years from date of transfer, if the
transfer is to a non-member of the ICC/IP and is tainted by vitiated consent of the ICC/IP,
or if the transfer is for an unconscionable consideration.
33

Within their ancestral domains and ancestral lands, the ICCs/IPs are given the right to self-governance
and empowerment,
34
social justice and human rights,
35
the right to preserve and protect their culture,
traditions, institutions and community intellectual rights, and the right to develop their own sciences and
technologies.
36

To carry out the policies of the Act, the law created the National Commission on Indigenous Peoples
(NCIP). The NCIP is an independent agency under the Office of the President and is composed of seven
(7) Commissioners belonging to ICCs/IPs from each of the ethnographic areas- Region I and the
Cordilleras; Region II; the rest of Luzon; Island groups including Mindoro, Palawan, Romblon, Panay and
the rest of the Visayas; Northern and Western Mindanao; Southern and Eastern Mindanao; and Central
Mindanao.
37
The NCIP took over the functions of the Office for Northern Cultural Communities and the
Office for Southern Cultural Communities created by former President Corazon Aquino which were
merged under a revitalized structure.
38

Disputes involving ICCs/IPs are to be resolved under customary laws and practices. When still
unresolved, the matter may be brought to the NCIP, which is granted quasi-judicial powers.
39
The NCIP's
decisions may be appealed to the Court of Appeals by a petition for review.
Any person who violates any of the provisions of the Act such as, but not limited to, unauthorized and/or
unlawful intrusion upon ancestral lands and domains shall be punished in accordance with customary
laws or imprisoned from 9 months to 12 years and/or fined from P100,000.00 to P500,000.00 and obliged
to pay damages.
40

A. Indigenous Peoples
The IPRA is a law dealing with a specific group of people, i.e., the Indigenous Cultural Communities
(ICCs) or the Indigenous Peoples (IPs). The term "ICCs" is used in the 1987 Constitution while that of
"IPs" is the contemporary international language in the International Labor Organization (ILO) Convention
169
41
and the United Nations (UN) Draft Declaration on the Rights of Indigenous Peoples.
42

ICCs/IPs are defined by the IPRA as:
"Sec. 3 [h]. Indigenous Cultural Communities/ Indigenous Peoples- refer to a group of people or
homogeneous societies identified by self-ascription and ascription by others, who have continuously lived
as organized community on communally bounded and defined territory, and who have, under claims of
ownership since time immemorial, occupied, possessed and utilized such territories, sharing common
bonds of language, customs, traditions and other distinctive cultural traits, or who have, through
resistance to political, social and cultural inroads of colonization, non-indigenous religions and cultures,
became historically differentiated from the majority of Filipinos. ICCs/IPs shall likewise include peoples
who are regarded as indigenous on account of their descent from the populations which inhabited the
country, at the time of conquest or colonization, or at the time of inroads of non-indigenous religions and
cultures, or the establishment of present state boundaries, who retain some or all of their own social,
economic, cultural and political institutions, but who may have been displaced from their traditional
domains or who may have resettled outside their ancestral domains."
Indigenous Cultural Communities or Indigenous Peoples refer to a group of people or
homogeneous societies who have continuously lived as an organized community on communally
bounded and defined territory. These groups of people have actually occupied, possessed and utilized
their territories under claim of ownership since time immemorial. They share common bonds of language,
customs, traditions and other distinctive cultural traits, or, they, by their resistance to political, social and
cultural inroads of colonization, non-indigenous religions and cultures, became historically differentiated
from the Filipino majority. ICCs/IPs also include descendants of ICCs/IPs who inhabited the country at the
time of conquest or colonization, who retain some or all of their own social, economic, cultural and
political institutions but who may have been displaced from their traditional territories or who may have
resettled outside their ancestral domains.
1. Indigenous Peoples: Their History
Presently, Philippine indigenous peoples inhabit the interiors and mountains of Luzon, Mindanao,
Mindoro, Negros, Samar, Leyte, and the Palawan and Sulu group of islands. They are composed of 110
tribes and are as follows:
1. In the Cordillera Autonomous Region- Kankaney, Ibaloi, Bontoc, Tinggian or Itneg, Ifugao,
Kalinga, Yapayao, Aeta or Agta or Pugot, and Bago of Ilocos Norte and Pangasinan; Ibanag of
Isabela, Cagayan; Ilongot of Quirino and Nueva Vizcaya; Gaddang of Quirino, Nueva Vizcaya,
Itawis of Cagayan; Ivatan of Batanes, Aeta of Cagayan, Quirino and Isabela.
2. In Region III- Aetas.
3. In Region IV- Dumagats of Aurora, Rizal; Remontado of Aurora, Rizal, Quezon; Alangan or
Mangyan, Batangan, Buid or Buhid, Hanunuo and Iraya of Oriental and Occidental Mindoro;
Tadyawan of Occidental Mindoro; Cuyonon, Palawanon, Tagbanua and Tao't bato of Palawan.
4. In Region V- Aeta of Camarines Norte and Camarines Sur; Aeta-Abiyan, Isarog, and Kabihug
of Camarines Norte; Agta, and Mayon of Camarines Sur; Itom of Albay, Cimaron of Sorsogon;
and the Pullon of Masbate and Camarines Sur.
5. In Region VI- Ati of Negros Occidental, Iloilo and Antique, Capiz; the Magahat of Negros
Occidental; the Corolano and Sulod.
6. In Region VII- Magahat of Negros Oriental and Eskaya of Bohol.
7. In Region IX- the Badjao numbering about 192,000 in Tawi-Tawi, Zamboanga del Sur; the
Kalibugan of Basilan, the Samal, Subanon and Yakat.
8. Region X- Numbering 1.6 million in Region X alone, the IPs are: the Banwaon, Bukidnon,
Matigsalog, Talaanding of Bukidnon; the Camiguin of Camiguin Island; the Higa-unon of Agusan
del Norte, Agusan del Sur, Bukidnon and Misamis Occidental; the Tigwahanon of Agusan del
Sur, Misamis Oriental and and Misamis Occidental, the Manobo of the Agusan provinces, and the
Umayamnon of Agusan and Bukidnon.
9. In Region XI- There are about 1,774,065 IPs in Region XI. They are tribes of the Dibabaon,
Mansaka of Davao del Norte; B'laan, Kalagan, Langilad, T'boli and Talaingod of Davao del Sur;
Mamamanua of Surigao del Sur; Mandaya of the Surigao provinces and Davao Oriental; Manobo
Blit of South Cotabato; the Mangguangon of Davao and South Cotabato; Matigsalog of Davao del
Norte and Del Sur; Tagakaolo, Tasaday and Ubo of South Cotabato; and Bagobo of Davao del
sur and South Cotabato.
10. In Region XII- Ilianen, Tiruray, Maguindanao, Maranao, Tausug, Yakan/Samal, and Iranon.
43

How these indigenous peoples came to live in the Philippines goes back to as early as 25,000 to
30,000 B.C.
Before the time of Western contact, the Philippine archipelago was peopled largely by the Negritos,
Indonesians and Malays.
44
The strains from these groups eventually gave rise to common cultural
features which became the dominant influence in ethnic reformulation in the archipelago. Influences from
the Chinese and Indian civilizations in the third or fourth millenium B.C. augmented these ethnic strains.
Chinese economic and socio-cultural influences came by way of Chinese porcelain, silk and traders.
Indian influence found their way into the religious-cultural aspect of pre-colonial society.
45

The ancient Filipinos settled beside bodies of water. Hunting and food gathering became supplementary
activities as reliance on them was reduced by fishing and the cultivation of the soil.
46
From the hinterland,
coastal, and riverine communities, our ancestors evolved an essentially homogeneous culture, a basically
common way of life where nature was a primary factor. Community life throughout the archipelago
was influenced by, and responded to, common ecology. The generally benign tropical climate and the
largely uniform flora and fauna favored similarities, not differences.
47
Life was essentially subsistence but
not harsh.
48

The early Filipinos had a culture that was basically Malayan in structure and form. They had languages
that traced their origin to the Austronesian parent-stock and used them not only as media of daily
communication but also as vehicles for the expression of their literary moods.
49
They fashioned concepts
and beliefs about the world that they could not see, but which they sensed to be part of their lives.
50
They
had their own religion and religious beliefs. They believed in the immortality of the soul and life after
death. Their rituals were based on beliefs in a ranking deity whom they called Bathalang Maykapal, and a
host of other deities, in the environmental spirits and in soul spirits. The early Filipinos adored the sun, the
moon, the animals and birds, for they seemed to consider the objects of Nature as something to be
respected. They venerated almost any object that was close to their daily life, indicating the importance of
the relationship between man and the object of nature.
51

The unit of government was the "barangay," a term that derived its meaning from the Malay word
"balangay," meaning, a boat, which transported them to these shores.
52
The barangay was basically a
family-based community and consisted of thirty to one hundred families. Each barangay was different and
ruled by a chieftain called a "dato." It was the chieftain's duty to rule and govern his subjects and promote
their welfare and interests. A chieftain had wide powers for he exercised all the functions of government.
He was the executive, legislator and judge and was the supreme commander in time of war.
53

Laws were either customary or written. Customary laws were handed down orally from generation
to generation and constituted the bulk of the laws of the barangay. They were preserved in songs
and chants and in the memory of the elder persons in the community.
54
The written laws were those that
the chieftain and his elders promulgated from time to time as the necessity arose.
55
The oldest known
written body of laws was the Maragtas Code by Datu Sumakwel at about 1250 A.D. Other old codes are
the Muslim Code of Luwaran and the Principal Code of Sulu.
56
Whether customary or written, the laws
dealt with various subjects, such as inheritance, divorce, usury, loans, partnership, crime and
punishment, property rights, family relations and adoption. Whenever disputes arose, these were decided
peacefully through a court composed by the chieftain as "judge" and the barangay elders as "jury."
Conflicts arising between subjects of different barangays were resolved by arbitration in which a board
composed of elders from neutral barangays acted as arbiters.
57

Baranganic society had a distinguishing feature: the absence of private property in land. The
chiefs merely administered the lands in the name of the barangay. The social order was an extension of
the family with chiefs embodying the higher unity of the community. Each individual, therefore,
participated in the community ownership of the soil and the instruments of production as a member of the
barangay.
58
This ancient communalism was practiced in accordance with the concept of mutual sharing of
resources so that no individual, regardless of status, was without sustenance. Ownership of land was
non-existent or unimportant and the right of usufruct was what regulated the development of
lands.
59
Marine resources and fishing grounds were likewise free to all. Coastal communities depended
for their economic welfare on the kind of fishing sharing concept similar to those in land
communities.
60
Recognized leaders, such as the chieftains and elders, by virtue of their positions of
importance, enjoyed some economic privileges and benefits. But their rights, related to either land and
sea, were subject to their responsibility to protect the communities from danger and to provide them with
the leadership and means of survival.
61

Sometime in the 13th century, Islam was introduced to the archipelago in Maguindanao. The
Sultanate of Sulu was established and claimed jurisdiction over territorial areas represented today by
Tawi-tawi, Sulu, Palawan, Basilan and Zamboanga. Four ethnic groups were within this jurisdiction:
Sama, Tausug, Yakan and Subanon.
62
The Sultanate of Maguindanao spread out from Cotabato toward
Maranao territory, now Lanao del Norte and Lanao del Sur.
63

The Muslim societies evolved an Asiatic form of feudalism where land was still held in common
but was private in use. This is clearly indicated in the Muslim Code of Luwaran. The Code contains a
provision on the lease of cultivated lands. It, however, has no provision for the acquisition, transfer,
cession or sale of land.
64

The societies encountered by Magellan and Legaspi therefore were primitive economies where most
production was geared to the use of the producers and to the fulfillment of kinship obligations. They were
not economies geared to exchange and profit.
65
Moreover, the family basis of barangay membership as
well as of leadership and governance worked to splinter the population of the islands into numerous small
and separate communities.
66

When the Spaniards settled permanently in the Philippines in 1565, they found the Filipinos living
in barangay settlements scattered along water routes and river banks. One of the first tasks imposed
on the missionaries and the encomenderos was to collect all scattered Filipinos together in
a reduccion.
67
As early as 1551, the Spanish government assumed an unvarying solicitous attitude
towards the natives.
68
The Spaniards regarded it a sacred "duty to conscience and humanity to civilize
these less fortunate people living in the obscurity of ignorance" and to accord them the "moral and
material advantages" of community life and the "protection and vigilance afforded them by the same
laws."
69

The Spanish missionaries were ordered to establish pueblos where the church and convent would be
constructed. All the new Christian converts were required to construct their houses around the church and
the unbaptized were invited to do the same.
70
With the reduccion, the Spaniards attempted to "tame" the
reluctant Filipinos through Christian indoctrination using the convento/casa real/plaza complex as focal
point. The reduccion, to the Spaniards, was a "civilizing" device to make the Filipinos law-abiding citizens
of the Spanish Crown, and in the long run, to make them ultimately adopt Hispanic culture and
civilization.
71

All lands lost by the old barangays in the process of pueblo organization as well as all lands not
assigned to them and the pueblos, were now declared to be crown lands or realengas, belonging
to the Spanish king. It was from the realengas that land grants were made to non-Filipinos.
72

The abrogation of the Filipinos' ancestral rights in land and the introduction of the concept of
public domain were the most immediate fundamental results of Spanish colonial theory and
law.
73
The concept that the Spanish king was the owner of everything of value in the Indies or
colonies was imposed on the natives, and the natives were stripped of their ancestral rights to
land.
74

Increasing their foothold in the Philippines, the Spanish colonialists, civil and religious, classified the
Filipinos according to their religious practices and beliefs, and divided them into three types . First were
the Indios, the Christianized Filipinos, who generally came from the lowland populations. Second, were
the Moros or the Muslim communities, and third, were the infieles or the indigenous communities.
75

The Indio was a product of the advent of Spanish culture. This class was favored by the Spaniards and
was allowed certain status although below the Spaniards. The Moros and infieles were regarded as the
lowest classes.
76

The Moros and infieles resisted Spanish rule and Christianity. The Moros were driven from Manila
and the Visayas to Mindanao; while the infieles, to the hinterlands. The Spaniards did not pursue them
into the deep interior. The upland societies were naturally outside the immediate concern of Spanish
interest, and the cliffs and forests of the hinterlands were difficult and inaccessible, allowing
the infieles, in effect, relative security.
77
Thus, the infieles, which were peripheral to colonial
administration, were not only able to preserve their own culture but also thwarted the Christianization
process, separating themselves from the newly evolved Christian community.
78
Their own political,
economic and social systems were kept constantly alive and vibrant.
The pro-Christian or pro-Indio attitude of colonialism brought about a generally mutual feeling of
suspicion, fear, and hostility between the Christians on the one hand and the non-Christians on the other.
Colonialism tended to divide and rule an otherwise culturally and historically related populace through a
colonial system that exploited both the virtues and vices of the Filipinos.
79

President McKinley, in his instructions to the Philippine Commission of April 7, 1900, addressed
the existence of the infieles:
"In dealing with the uncivilized tribes of the Islands, the Commission should adopt the same
course followed by Congress in permitting the tribes of our North American Indians to maintain
their tribal organization and government, and under which many of those tribes are now living in peace
and contentment, surrounded by civilization to which they are unable or unwilling to conform. Such tribal
government should, however, be subjected to wise and firm regulation; and, without undue or petty
interference, constant and active effort should be exercised to prevent barbarous practices and introduce
civilized customs."
80

Placed in an alternative of either letting the natives alone or guiding them in the path of civilization, the
American government chose "to adopt the latter measure as one more in accord with humanity and with
the national conscience."
81

The Americans classified the Filipinos into two: the Christian Filipinos and the non-Christian
Filipinos. The term "non-Christian" referred not to religious belief, but to a geographical area, and more
directly, "to natives of the Philippine Islands of a low grade of civilization, usually living in tribal
relationship apart from settled communities."
82

Like the Spaniards, the Americans pursued a policy of assimilation. In 1903, they passed Act No.
253 creating the Bureau of Non-Christian Tribes (BNCT). Under the Department of the Interior, the
BNCT's primary task was to conduct ethnographic research among unhispanized Filipinos, including
those in Muslim Mindanao, with a "special view to determining the most practicable means for bringing
about their advancement in civilization and prosperity." The BNCT was modeled after the bureau
dealing with American Indians. The agency took a keen anthropological interest in Philippine cultural
minorities and produced a wealth of valuable materials about them.
83

The 1935 Constitution did not carry any policy on the non-Christian Filipinos. The raging issue
then was the conservation of the national patrimony for the Filipinos.
In 1957, the Philippine Congress passed R.A. No. 1888, an "Act to effectuate in a more rapid and
complete manner the economic, social, moral and political advancement of the non-Christian Filipinos or
national cultural minorities and to render real, complete, and permanent the integration of all said national
cultural minorities into the body politic, creating the Commission on National Integration charged with
said functions." The law called for a policy of integration of indigenous peoples into the Philippine
mainstream and for this purpose created theCommission on National Integration (CNI).
84
The CNI was
given, more or less, the same task as the BNCT during the American regime. The post-independence
policy of integration was like the colonial policy of assimilation understood in the context of a
guardian-ward relationship.
85

The policy of assimilation and integration did not yield the desired result. Like the Spaniards and
Americans, government attempts at integration met with fierce resistance. Since World War II, a
tidal wave of Christian settlers from the lowlands of Luzon and the Visayas swamped the highlands and
wide open spaces in Mindanao.
86
Knowledge by the settlers of the Public Land Acts and the Torrens
system resulted in the titling of several ancestral lands in the settlers' names. With government
initiative and participation, this titling displaced several indigenous peoples from their
lands. Worse, these peoples were also displaced by projects undertaken by the national government in
the name of national development.
87

It was in the 1973 Constitution that the State adopted the following provision:
"The State shall consider the customs, traditions, beliefs, and interests of national cultural communities in
the formulation and implementation of State policies."
88

For the first time in Philippine history, the "non-Christian tribes" or the "cultural minorities" were
addressed by the highest law of the Republic, and they were referred to as "cultural
communities."More importantly this time, their "uncivilized" culture was given some recognition and their
"customs, traditions, beliefs and interests" were to be considered by the State in the formulation and
implementation of State policies.President Marcos abolished the CNI and transferred its functions to
the Presidential Adviser on National Minorities (PANAMIN). The PANAMIN was tasked to integrate
the ethnic groups that sought full integration into the larger community, and at the same time "protect the
rights of those who wish to preserve their original lifeways beside the larger community."
89
In short, while
still adopting the integration policy, the decree recognized the right of tribal Filipinos to preserve
their way of life.
90

In 1974, President Marcos promulgated P.D. No. 410, otherwise known as the Ancestral Lands
Decree. The decree provided for the issuance of land occupancy certificates to members of the national
cultural communities who were given up to 1984 to register their claims.
91
In 1979, the Commission on
the Settlement of Land Problems was created under E.O. No. 561 which provided a mechanism for the
expeditious resolution of land problems involving small settlers, landowners, and tribal Filipinos.
92

Despite the promulgation of these laws, from 1974 to the early 1980's, some 100,000 Kalingas and
Bontoks of the Cordillera region were displaced by the Chico River dam project of the National Power
Corporation (NPC). The Manobos of Bukidnon saw their land bulldozed by the Bukidnon Sugar Industries
Company (BUSCO). In Agusan del Sur, the National Development Company was authorized by law in
1979 to take approximately 40,550 hectares of land that later became the NDC-Guthrie plantation in
Agusan del Sur. Most of the land was possessed by the Agusan natives.
93
Timber concessions, water
projects, plantations, mining, and cattle ranching and other projects of the national government led not
only to the eviction of the indigenous peoples from their land but also to the reduction and destruction of
their natural environment.
94

The Aquino government signified a total shift from the policy of integration to one of
preservation.Invoking her powers under the Freedom Constitution, President Aquino created the Office
of Muslim Affairs, Office for Northern Cultural Communities and the Office for Southern Cultural
Communities all under the Office of the President.
95

The 1987 Constitution carries at least six (6) provisions which insure the right of tribal Filipinos to
preserve their way of life.
96
This Constitution goes further than the 1973 Constitution by expressly
guaranteeing the rights of tribal Filipinos to their ancestral domains and ancestral lands. By
recognizing their right to their ancestral lands and domains, the State has effectively upheld their
right to live in a culture distinctly their own.
2. Their Concept of Land
Indigenous peoples share distinctive traits that set them apart from the Filipino mainstream. They are
non-Christians. They live in less accessible, marginal, mostly upland areas. They have a system of self-
government not dependent upon the laws of the central administration of the Republic of the Philippines.
They follow ways of life and customs that are perceived as different from those of the rest of the
population.
97
The kind of response the indigenous peoples chose to deal with colonial threat worked well
to their advantage by making it difficult for Western concepts and religion to erode their customs and
traditions. The "infieles societies" which had become peripheral to colonial administration, represented,
from a cultural perspective, a much older base of archipelagic culture. The political systems were still
structured on the patriarchal and kinship oriented arrangement of power and authority. The economic
activities were governed by the concepts of an ancient communalism and mutual help. The social
structure which emphasized division of labor and distinction of functions, not status, was maintained. The
cultural styles and forms of life portraying the varieties of social courtesies and ecological adjustments
were kept constantly vibrant.
98

Land is the central element of the indigenous peoples' existence. There is no traditional concept of
permanent, individual, land ownership. Among the Igorots, ownership of land more accurately applies to
the tribal right to use the land or to territorial control. The people are the secondary owners or stewards of
the land and that if a member of the tribe ceases to work, he loses his claim of ownership, and the land
reverts to the beings of the spirit world who are its true and primary owners. Under the concept of
"trusteeship," the right to possess the land does not only belong to the present generation but the future
ones as well.
99

Customary law on land rests on the traditional belief that no one owns the land except the gods and
spirits, and that those who work the land are its mere stewards.
100
Customary law has a strong
preference for communal ownership, which could either be ownership by a group of individuals or
families who are related by blood or by marriage,
101
or ownership by residents of the same locality who
may not be related by blood or marriage. The system of communal ownership under customary laws
draws its meaning from the subsistence and highly collectivized mode of economic production. The
Kalingas, for instance, who are engaged in team occupation like hunting, foraging for forest products, and
swidden farming found it natural that forest areas, swidden farms, orchards, pasture and burial grounds
should be communally-owned.
102
For the Kalingas, everybody has a common right to a common
economic base. Thus, as a rule, rights and obligations to the land are shared in common.
Although highly bent on communal ownership, customary law on land also sanctions individual
ownership. The residential lots and terrace rice farms are governed by a limited system of individual
ownership. It is limited because while the individual owner has the right to use and dispose of the
property, he does not possess all the rights of an exclusive and full owner as defined under our Civil
Code.
103
Under Kalinga customary law, the alienation of individually-owned land is strongly discouraged
except in marriage and succession and except to meet sudden financial needs due to sickness, death in
the family, or loss of crops.
104
Moreover, and to be alienated should first be offered to a clan-member
before any village-member can purchase it, and in no case may land be sold to a non-member of the ili.
105

Land titles do not exist in the indigenous peoples' economic and social system. The concept of
individual land ownership under the civil law is alien to them. Inherently colonial in origin, our
national land laws and governmental policies frown upon indigenous claims to ancestral lands.
Communal ownership is looked upon as inferior, if not inexistent.
106

III. THE IPRA IS A NOVEL PIECE OF LEGISLATION.
A. The Legislative History of the IPRA
It was to address the centuries-old neglect of the Philippine indigenous peoples that the Tenth
Congress of the Philippines, by their joint efforts, passed and approved R.A. No. 8371, the Indigenous
Peoples Rights Act (IPRA) of 1997. The law was a consolidation of two Bills- Senate Bill No. 1728 and
House Bill No. 9125.
Principally sponsored by Senator Juan M. Flavier,
107
Senate Bill No. 1728 was a consolidation of four
proposed measures referred to the Committees on Cultural Communities, Environment and Natural
Resources, Ways and Means, as well as Finance. It adopted almost en toto the comprehensive version of
Senate Bill Nos. 1476 and 1486 which was a result of six regional consultations and one national
consultation with indigenous peoples nationwide.
108
At the Second Regular Session of the Tenth
Congress, Senator Flavier, in his sponsorship speech, gave a background on the situation of indigenous
peoples in the Philippines, to wit:
"The Indigenous Cultural Communities, including the Bangsa Moro, have long suffered from the
dominance and neglect of government controlled by the majority. Massive migration of their Christian
brothers to their homeland shrunk their territory and many of the tribal Filipinos were pushed to the
hinterlands. Resisting the intrusion, dispossessed of their ancestral land and with the massive exploitation
of their natural resources by the elite among the migrant population, they became marginalized. And the
government has been an indispensable party to this insidious conspiracy against the Indigenous Cultural
Communities (ICCs). It organized and supported the resettlement of people to their ancestral land, which
was massive during the Commonwealth and early years of the Philippine Republic. Pursuant to the
Regalian Doctrine first introduced to our system by Spain through the Royal Decree of 13 February 1894
or the Maura Law, the government passed laws to legitimize the wholesale landgrabbing and provide for
easy titling or grant of lands to migrant homesteaders within the traditional areas of the ICCs."
109

Senator Flavier further declared:
"The IPs are the offsprings and heirs of the peoples who have first inhabited and cared for the land long
before any central government was established. Their ancestors had territories over which they ruled
themselves and related with other tribes. These territories- the land- include people, their dwelling, the
mountains, the water, the air, plants, forest and the animals. This is their environment in its totality. Their
existence as indigenous peoples is manifested in their own lives through political, economic, socio-
cultural and spiritual practices. The IPs culture is the living and irrefutable proof to this.
Their survival depends on securing or acquiring land rights; asserting their rights to it; and depending on
it. Otherwise, IPs shall cease to exist as distinct peoples."
110

To recognize the rights of the indigenous peoples effectively, Senator Flavier proposed a bill based
on two postulates: (1) the concept of native title; and (2) the principle of parens patriae.
According to Senator Flavier, "[w]hile our legal tradition subscribes to the Regalian Doctrine reinstated in
Section 2, Article XII of the 1987 Constitution," our "decisional laws" and jurisprudence passed by the
State have "made exception to the doctrine." This exception was first laid down in the case of Cario v.
Insular Governmentwhere:
"x x x the court has recognized long occupancy of land by an indigenous member of the cultural
communities as one of private ownership, which, in legal concept, is termed "native title." This ruling has
not been overturned. In fact, it was affirmed in subsequent cases."
111

Following Cario, the State passed Act No. 926, Act No. 2874, C.A. No. 141, P.D. 705, P.D. 410, P.D.
1529, R.A. 6734 (the Organic Act for the Autonomous Region of Muslim Mindanao). These laws, explicitly
or implicitly, and liberally or restrictively, recognized "native title" or "private right" and the existence of
ancestral lands and domains. Despite the passage of these laws, however, Senator Flavier continued:
"x x x the executive department of government since the American occupation has not implemented the
policy. In fact, it was more honored in its breach than in its observance, its wanton disregard shown
during the period unto the Commonwealth and the early years of the Philippine Republic when
government organized and supported massive resettlement of the people to the land of the ICCs."
Senate Bill No. 1728 seeks to genuinely recognize the IPs right to own and possess their ancestral land.
The bill was prepared also under the principle of parens patriae inherent in the supreme power of the
State and deeply embedded in Philippine legal tradition. This principle mandates that persons suffering
from serious disadvantage or handicap, which places them in a position of actual inequality in their
relation or transaction with others, are entitled to the protection of the State.
Senate Bill No. 1728 was passed on Third Reading by twenty-one (21) Senators voting in favor and
none against, with no abstention.
112

House Bill No. 9125 was sponsored by Rep. Zapata, Chairman of the Committee on Cultural
Communities. It was originally authored and subsequently presented and defended on the floor by Rep.
Gregorio Andolana of North Cotabato.
113

Rep. Andolana's sponsorhip speech reads as follows:
"This Representation, as early as in the 8th Congress, filed a bill of similar implications that would
promote, recognize the rights of indigenous cultural communities within the framework of national unity
and development.
Apart from this, Mr. Speaker, is our obligation, the government's obligation to assure and ascertain that
these rights shall be well-preserved and the cultural traditions as well as the indigenous laws that
remained long before this Republic was established shall be preserved and promoted. There is a need,
Mr. Speaker, to look into these matters seriously and early approval of the substitute bill shall bring into
reality the aspirations, the hope and the dreams of more than 12 million Filipinos that they be considered
in the mainstream of the Philippine society as we fashion for the year 2000."
114

Rep. Andolana stressed that H.B. No. 9125 is based on the policy of preservation as mandated in the
Constitution. He also emphasized that the rights of IPs to their land was enunciated in Cario v. Insular
Government which recognized the fact that they had vested rights prior to the establishment of the
Spanish and American regimes.
115

After exhaustive interpellation, House Bill No. 9125, and its corresponding amendments, was
approved on Second Reading with no objections.
IV. THE PROVISIONS OF THE IPRA DO NOT CONTRAVENE THE CONSTITUTION.
A. Ancestral Domains and Ancestral Lands are the Private Property of Indigenous Peoples and Do
Not Constitute Part of the Land of the Public Domain.
The IPRA grants to ICCs/IPs a distinct kind of ownership over ancestral domains and ancestral
lands.Ancestral lands are not the same as ancestral domains. These are defined in Section 3 [a] and [b]
of the Indigenous Peoples Right Act, viz:
"Sec. 3 a) Ancestral Domains. - Subject to Section 56 hereof, refer to all areas generally belonging to
ICCs/IPs comprising lands, inland waters, coastal areas, and natural resources therein, held under a
claim of ownership, occupied or possessed by ICCs/IPs by themselves or through their ancestors,
communally or individually since time immemorial, continuously to the present except when interrupted by
war, force majeure or displacement by force, deceit, stealth or as a consequence of government projects
or any other voluntary dealings entered into by government and private individuals/corporations, and
which are necessary to ensure their economic, social and cultural welfare. It shall include ancestral lands,
forests, pasture, residential, agricultural, and other lands individually owned whether alienable and
disposable or otherwise, hunting grounds, burial grounds, worship areas, bodies of water, mineral and
other natural resources, and lands which may no longer be exclusively occupied by ICCs/IPs but from
which they traditionally had access to for their subsistence and traditional activities, particularly the home
ranges of ICCs/IPs who are still nomadic and/or shifting cultivators;
b) Ancestral Lands.- Subject to Section 56 hereof, refers to land occupied, possessed and utilized by
individuals, families and clans who are members of the ICCs/IPs since time immemorial, by themselves or
through their predecessors-in-interest, under claims of individual or traditional group ownership,
continuously, to the present except when interrupted by war, force majeure or displacement by force,
deceit, stealth, or as a consequence of government projects and other voluntary dealings entered into by
government and private individuals/corporations, including, but not limited to, residential lots, rice terraces
or paddies, private forests, swidden farms and tree lots."
Ancestral domains are all areas belonging to ICCs/IPs held under a claim of ownership, occupied or
possessed by ICCs/IPs by themselves or through their ancestors, communally or individually since time
immemorial, continuously until the present, except when interrupted by war, force majeure or
displacement by force, deceit, stealth or as a consequence of government projects or any other voluntary
dealings with government and/or private individuals or corporations. Ancestral domains comprise
lands, inland waters, coastal areas, and natural resources therein and includes ancestral lands,
forests, pasture, residential, agricultural, and other lands individually owned whether alienable or
not, hunting grounds, burial grounds, worship areas, bodies of water, mineral and other natural
resources. They also include lands which may no longer be exclusively occupied by ICCs/IPs but from
which they traditionally had access to for their subsistence and traditional activities, particularly the home
ranges of ICCs/IPs who are still nomadic and/or shifting cultivators.
116

Ancestral lands are lands held by the ICCs/IPs under the same conditions as ancestral domains except
that these are limited to lands and that these lands are not merely occupied and possessed but are also
utilized by the ICCs/IPs under claims of individual or traditional group ownership. These lands include but
are not limited to residential lots, rice terraces or paddies, private forests, swidden farms and tree lots.
117

The procedures for claiming ancestral domains and lands are similar to the procedures embodied in
Department Administrative Order (DAO) No. 2, series of 1993, signed by then Secretary of the
Department of Environment and Natural Resources (DENR) Angel Alcala.
118
DAO No. 2 allowed the
delineation of ancestral domains by special task forces and ensured the issuance of Certificates of
Ancestral Land Claims (CALC's) and Certificates of Ancestral Domain Claims (CADC's) to IPs.
The identification and delineation of these ancestral domains and lands is a power conferred by the IPRA
on the National Commission on Indigenous Peoples (NCIP).
119
The guiding principle in identification and
delineation is self-delineation.
120
This means that the ICCs/IPs have a decisive role in determining the
boundaries of their domains and in all the activities pertinent thereto.
121

The procedure for the delineation and recognition of ancestral domains is set forth in Sections 51 and
52 of the IPRA. The identification, delineation and certification of ancestral lands is in Section 53 of said
law.
Upon due application and compliance with the procedure provided under the law and upon finding by the
NCIP that the application is meritorious, the NCIP shall issue a Certificate of Ancestral Domain Title
(CADT) in the name of the community concerned.
122
The allocation of lands within the ancestral
domain to any individual or indigenous corporate (family or clan) claimants is left to the ICCs/IPs
concerned to decide in accordance with customs and traditions.
123
With respect to ancestral lands
outside the ancestral domain, the NCIP issues a Certificate of Ancestral Land Title (CALT).
124

CADT's and CALT's issued under the IPRA shall be registered by the NCIP before the Register of Deeds
in the place where the property is situated.
125

(1) Right to Ancestral Domains and Ancestral Lands: How Acquired
The rights of the ICCs/IPs to their ancestral domains and ancestral lands may be acquired in two
modes: (1) bynative title over both ancestral lands and domains; or (2) by torrens title under the
Public Land Act and the Land Registration Act with respect to ancestral lands only.
(2) The Concept of Native Title
Native title is defined as:
"Sec. 3 [l]. Native Title- refers to pre-conquest rights to lands and domains which, as far back as memory
reaches, have been held under a claim of private ownership by ICCs/IPs, have never been public lands
and are thusindisputably presumed to have been held that way since before the Spanish Conquest."
126

Native title refers to ICCs/IPs' preconquest rights to lands and domains held under a claim of private
ownership as far back as memory reaches. These lands are deemed never to have been public lands and
are indisputably presumed to have been held that way since before the Spanish Conquest. The rights of
ICCs/IPs to their ancestraldomains (which also include ancestral lands) by virtue of native title shall be
recognized and respected.
127
Formal recognition, when solicited by ICCs/IPs concerned, shall be
embodied in a Certificate of Ancestral Domain Title (CADT), which shall recognize the title of the
concerned ICCs/IPs over the territories identified and delineated.
128

Like a torrens title, a CADT is evidence of private ownership of land by native title. Native title, however,
is a right of private ownership peculiarly granted to ICCs/IPs over their ancestral lands and domains. The
IPRA categorically declares ancestral lands and domains held by native title as never to have
been public land. Domains and lands held under native title are, therefore, indisputably presumed to have
never been public lands and are private.
(a) Cario v. Insular Government
129

The concept of native title in the IPRA was taken from the 1909 case of Cario v. Insular
Government.
130
Cario firmly established a concept of private land title that existed irrespective of any
royal grant from the State.
In 1903, Don Mateo Cario, an Ibaloi, sought to register with the land registration court 146 hectares of
land in Baguio Municipality, Benguet Province. He claimed that this land had been possessed and
occupied by his ancestors since time immemorial; that his grandfather built fences around the property for
the holding of cattle and that his father cultivated some parts of the land. Cario inherited the land in
accordance with Igorot custom. He tried to have the land adjusted under the Spanish land laws, but no
document issued from the Spanish Crown.
131
In 1901, Cario obtained a possessory title to the land under
the Spanish Mortgage Law.
132
The North American colonial government, however, ignored his
possessory title and built a public road on the land prompting him to seek a Torrens title to his property in
the land registration court. While his petition was pending, a U.S. military reservation
133
was proclaimed
over his land and, shortly thereafter, a military detachment was detailed on the property with orders to
keep cattle and trespassers, including Cario, off the land.
134

In 1904, the land registration court granted Cario's application for absolute ownership to the land. Both
the Government of the Philippine Islands and the U.S. Government appealed to the C.F.I. of Benguet
which reversed the land registration court and dismissed Cario's application. The Philippine Supreme
Court
135
affirmed the C.F.I. by applying the Valenton ruling. Cario took the case to the U.S. Supreme
Court.
136
On one hand, the Philippine government invoked the Regalian doctrine and contended that
Cario failed to comply with the provisions of the Royal Decree of June 25, 1880, which required
registration of land claims within a limited period of time. Cario, on the other, asserted that he was the
absolute owner of the land jure gentium, and that the land never formed part of the public domain.
In a unanimous decision written by Justice Oliver Wendell Holmes, the U.S. Supreme Court held:
"It is true that Spain, in its earlier decrees, embodied the universal feudal theory that all lands were held
from the Crown, and perhaps the general attitude of conquering nations toward people not recognized as
entitled to the treatment accorded to those in the same zone of civilization with themselves. It is true, also,
that in legal theory, sovereignty is absolute, and that, as against foreign nations, the United States may
assert, as Spain asserted, absolute power. But it does not follow that, as against the inhabitants of the
Philippines, the United States asserts that Spain had such power. When theory is left on one side,
sovereignty is a question of strength, and may vary in degree. How far a new sovereign shall insist upon
the theoretical relation of the subjects to the head in the past, and how far it shall recognize actual facts,
are matters for it to decide."
137

The U.S. Supreme Court noted that it need not accept Spanish doctrines. The choice was with the new
colonizer. Ultimately, the matter had to be decided under U.S. law.
The Cario decision largely rested on the North American constitutionalist's concept of "due process" as
well as the pronounced policy "to do justice to the natives."
138
It was based on the strong mandate
extended to the Islands via the Philippine Bill of 1902 that "No law shall be enacted in said islands which
shall deprive any person of life, liberty, or property without due process of law, or deny to any person
therein the equal protection of the laws." The court declared:
"The acquisition of the Philippines was not like the settlement of the white race in the United States.
Whatever consideration may have been shown to the North American Indians, the dominant purpose of
the whites in America was to occupy land. It is obvious that, however stated, the reason for our taking
over the Philippines was different. No one, we suppose, would deny that, so far as consistent with
paramount necessities, our first object in the internal administration of the islands is to do justice to the
natives, not to exploit their country for private gain. By the Organic Act of July 1, 1902, chapter 1369,
section 12 (32 Statutes at Large, 691), all the property and rights acquired there by the United States are
to be administered 'for the benefit of the inhabitants thereof.' It is reasonable to suppose that the attitude
thus assumed by the United States with regard to what was unquestionably its own is also its attitude in
deciding what it will claim for its own. The same statute made a bill of rights, embodying the safeguards of
the Constitution, and, like the Constitution, extends those safeguards to all. It provides that 'no law shall
be enacted in said islands which shall deprive any person of life, liberty, or property without due process
of law, or deny to any person therein the equal protection of the laws.' In the light of the declaration that
we have quoted from section 12, it is hard to believe that the United States was ready to declare in the
next breath that "any person" did not embrace the inhabitants of Benguet, or that it meant by "property"
only that which had become such by ceremonies of which presumably a large part of the inhabitants
never had heard, and that it proposed to treat as public land what they, by native custom and by long
association,- of the profoundest factors in human thought,- regarded as their own."
139

The Court went further:
"Every presumption is and ought to be against the government in a case like the present. It might,
perhaps, be proper and sufficient to say that when, as far back as testimony or memory goes, the
land has been held by individuals under a claim of private ownership, it will be presumed to have
been held in the same way from before the Spanish conquest, and never to have been public
land. Certainly in a case like this, if there is doubt or ambiguity in the Spanish law, we ought to give the
applicant the benefit of the doubt."
140

The court thus laid down the presumption of a certain title held (1) as far back as testimony or memory
went, and (2) under a claim of private ownership. Land held by this title is presumed to "never have been
public land."
Against this presumption, the U.S. Supreme Court analyzed the Spanish decrees upheld in the 1904
decision ofValenton v. Murciano. The U.S. Supreme Court found no proof that the Spanish decrees
did not honor native title. On the contrary, the decrees discussed in Valenton appeared to recognize that
the natives owned some land, irrespective of any royal grant. The Regalian doctrine declared in the
preamble of the Recopilacion was all "theory and discourse" and it was observed that titles were admitted
to exist beyond the powers of the Crown, viz:
"If the applicant's case is to be tried by the law of Spain, we do not discover such clear proof that
it was bad by that law as to satisfy us that he does not own the land. To begin with, the older
decrees and laws cited by the counsel for the plaintiff in error seem to indicate pretty clearly that
the natives were recognized as owning some lands, irrespective of any royal grant. In other words,
Spain did not assume to convert all the native inhabitants of the Philippines into trespassers or even into
tenants at will. For instance, Book 4, title 12, Law 14 of the the Recopilacion de Leyes de las Indias, cited
for a contrary conclusion in Valenton v. Murciano, 3 Philippine 537, while it commands viceroys and
others, when it seems proper, to call for the exhibition of grants, directs them to confirm those who hold
by good grants or justa prescripcion. It is true that it begins by the characteristic assertion of feudal
overlordship and the origin of all titles in the King or his predecessors. That was theory and
discourse. The fact was that titles were admitted to exist that owed nothing to the powers of Spain
beyond this recognition in their books." (Emphasis supplied).
141

The court further stated that the Spanish "adjustment" proceedings never held sway over unconquered
territories. The wording of the Spanish laws were not framed in a manner as to convey to the natives that
failure to register what to them has always been their own would mean loss of such land. The registration
requirement was "not to confer title, but simply to establish it;" it was "not calculated to convey to the mind
of an Igorot chief the notion that ancient family possessions were in danger, if he had read every word of
it."
By recognizing this kind of title, the court clearly repudiated the doctrine of Valenton. It was frank
enough, however, to admit the possibility that the applicant might have been deprived of his land under
Spanish law because of the inherent ambiguity of the decrees and concomitantly, the various
interpretations which may be given them. But precisely because of the ambiguity and of the strong
"due process mandate" of the Constitution, the court validated this kind of title.
142
This title was
sufficient, even without government administrative action, and entitled the holder to a Torrens certificate.
Justice Holmes explained:
"It will be perceived that the rights of the applicant under the Spanish law present a problem not without
difficulties for courts of a legal tradition. We have deemed it proper on that account to notice the possible
effect of the change of sovereignty and the act of Congress establishing the fundamental principles now
to be observed. Upon a consideration of the whole case we are of the opinion that law and justice require
that the applicant should be granted what he seeks, and should not be deprived of what, by the practice
and belief of those among whom he lived, was his property, through a refined interpretation of an almost
forgotten law of Spain."
143

Thus, the court ruled in favor of Cario and ordered the registration of the 148 hectares in Baguio
Municipality in his name.
144

Examining Cario closer, the U.S. Supreme Court did not categorically refer to the title it upheld as
"native title." It simply said:
"The Province of Benguet was inhabited by a tribe that the Solicitor-General, in his argument,
characterized as a savage tribe that never was brought under the civil or military government of
the Spanish Crown. It seems probable, if not certain, that the Spanish officials would not have
granted to anyone in that province the registration to which formerly the plaintiff was entitled by
the Spanish Laws, and which would have made his title beyond question good. Whatever may have
been the technical position of Spain it does not follow that, in the view of the United States, he had lost all
rights and was a mere trespasser when the present government seized his land. The argument to that
effect seems to amount to a denial of native titles through an important part of the Island of Luzon, at
least, for the want of ceremonies which the Spaniards would not have permitted and had not the power to
enforce."
145

This is the only instance when Justice Holmes used the term "native title" in the entire length of
the Cariodecision. It is observed that the widespread use of the term "native title" may be traced to
Professor Owen James Lynch, Jr., a Visiting Professor at the University of the Philippines College of Law
from the Yale University Law School. In 1982, Prof. Lynch published an article in the Philippine Law
Journal entitled Native Title, Private Right and Tribal Land Law.
146
This article was made after
Professor Lynch visited over thirty tribal communities throughout the country and studied the origin and
development of Philippine land laws.
147
He discussed Carioextensively and used the term "native title"
to refer to Cario's title as discussed and upheld by the U.S. Supreme Court in said case.
(b) Indian Title
In a footnote in the same article, Professor Lynch stated that the concept of "native title" as defined by
Justice Holmes in Cario "is conceptually similar to "aboriginal title" of the American Indians.
148
This is not
surprising, according to Prof. Lynch, considering that during the American regime, government policy
towards ICCs/IPs was consistently made in reference to native Americans.
149
This was clearly
demonstrated in the case of Rubi v. Provincial Board of Mindoro.
150

In Rubi, the Provincial Board of Mindoro adopted a Resolution authorizing the provincial governor to
remove the Mangyans from their domains and place them in a permanent reservation in Sitio Tigbao,
Lake Naujan. Any Mangyan who refused to comply was to be imprisoned. Rubi and some Mangyans,
including one who was imprisoned for trying to escape from the reservation, filed for habeas corpus
claiming deprivation of liberty under the Board Resolution. This Court denied the petition on the ground of
police power. It upheld government policy promoting the idea that a permanent settlement was the only
successful method for educating the Mangyans, introducing civilized customs, improving their health and
morals, and protecting the public forests in which they roamed.
151
Speaking through Justice Malcolm, the
court said:
"Reference was made in the President's instructions to the Commission to the policy adopted by the
United States for the Indian Tribes. The methods followed by the Government of the Philippine Islands in
its dealings with the so-called non-Christian people is said, on argument, to be practically identical with
that followed by the United States Government in its dealings with the Indian tribes. Valuable lessons, it is
insisted, can be derived by an investigation of the American-Indian policy.
From the beginning of the United States, and even before, the Indians have been treated as "in a state of
pupilage." The recognized relation between the Government of the United States and the Indians may be
described as that of guardian and ward. It is for the Congress to determine when and how the
guardianship shall be terminated. The Indians are always subject to the plenary authority of the United
States.
152

x x x.
As to the second point, the facts in the Standing Bear case and the Rubi case are not exactly identical.
But even admitting similarity of facts, yet it is known to all that Indian reservations do exist in the United
States, that Indians have been taken from different parts of the country and placed on these reservations,
without any previous consultation as to their own wishes, and that, when once so located, they have been
made to remain on the reservation for their own good and for the general good of the country. If any
lesson can be drawn from the Indian policy of the United States, it is that the determination of this policy
is for the legislative and executive branches of the government and that when once so decided upon, the
courts should not interfere to upset a carefully planned governmental system. Perhaps, just as many
forceful reasons exist for the segregation of the Manguianes in Mindoro as existed for the segregation of
the different Indian tribes in the United States."
153

Rubi applied the concept of Indian land grants or reservations in the Philippines. An Indian reservation is
a part of the public domain set apart by proper authority for the use and occupation of a tribe or tribes of
Indians.
154
It may be set apart by an act of Congress, by treaty, or by executive order, but it cannot be
established by custom and prescription.
155

Indian title to land, however, is not limited to land grants or reservations. It also covers the
"aboriginal right of possession or occupancy."
156
The aboriginal right of possession depends on the
actual occupancy of the lands in question by the tribe or nation as their ancestral home, in the sense that
such lands constitute definable territory occupied exclusively by the particular tribe or nation.
157
It is a right
which exists apart from any treaty, statute, or other governmental action, although in numerous instances
treaties have been negotiated with Indian tribes, recognizing their aboriginal possession and delimiting
their occupancy rights or settling and adjusting their boundaries.
158

American jurisprudence recognizes the Indians' or native Americans' rights to land they have held
and occupied before the "discovery" of the Americas by the Europeans. The earliest definitive
statement by the U.S. Supreme Court on the nature of aboriginal title was made in 1823
in J ohnson & Graham's Lessee v. M'Intosh.
159

In J ohnson, the plaintiffs claimed the land in question under two (2) grants made by the chiefs of two (2)
Indian tribes. The U.S. Supreme Court refused to recognize this conveyance, the plaintiffs being private
persons. The only conveyance that was recognized was that made by the Indians to the government of
the European discoverer. Speaking for the court, Chief Justice Marshall pointed out that the potentates of
the old world believed that they had made ample compensation to the inhabitants of the new world by
bestowing civilization and Christianity upon them; but in addition, said the court, they found it necessary,
in order to avoid conflicting settlements and consequent war, to establish the principle that discovery
gives title to the government by whose subjects, or by whose authority, the discovery was made,
against all other European governments, which title might be consummated by possession.
160
The
exclusion of all other Europeans gave to the nation making the discovery the sole right of acquiring the
soil from the natives and establishing settlements upon it. As regards the natives, the court further stated
that:
"Those relations which were to exist between the discoverer and the natives were to be regulated by
themselves. The rights thus acquired being exclusive, no other power could interpose between them.
In the establishment of these relations, the rights of the original inhabitants were, in no instance, entirely
disregarded; but were necessarily, to a considerable extent, impaired. They were admitted to be the
rightful occupants of the soil, with a legal as well as just claim to retain possession of it, and to
use itaccording to their own discretion; but their rights to complete sovereignty, as independent
nations, were necessarily diminished, and their power to dispose of the soil at their own will, to
whomsoever they pleased, was denied by the fundamental principle that discovery gave exclusive title to
those who made it.
While the different nations of Europe respected the right of the natives as occupants, they
asserted the ultimate dominion to be in themselves; and claimed and exercised, as a
consequence of this ultimate dominion, a power to grant the soil, while yet in possession of the
natives. These grants have been understood by all to convey a title to the grantees, subject only
to the Indian right of occupancy."
161

Thus, the discoverer of new territory was deemed to have obtained the exclusive right to acquire Indian
land and extinguish Indian titles. Only to the discoverer- whether to England, France, Spain or Holland-
did this right belong and not to any other nation or private person. The mere acquisition of the right
nonetheless did not extinguish Indian claims to land. Rather, until the discoverer, by purchase or
conquest, exercised its right, the concerned Indians were recognized as the "rightful occupants of the soil,
with a legal as well as just claim to retain possession of it." Grants made by the discoverer to her subjects
of lands occupied by the Indians were held to convey a title to the grantees, subject only to the Indian
right of occupancy. Once the discoverer purchased the land from the Indians or conquered them, it was
only then that the discoverer gained an absolute title unrestricted by Indian rights.
The court concluded, in essence, that a grant of Indian lands by Indians could not convey a title
paramount to the title of the United States itself to other parties, saying:
"It has never been contended that the Indian title amounted to nothing. Their right of possession has
never been questioned. The claim of government extends to the complete ultimate title, charged
with this right of possession, and to the exclusive power of acquiring that right."
162

It has been said that the history of America, from its discovery to the present day, proves the universal
recognition of this principle.
163

The J ohnson doctrine was a compromise. It protected Indian rights and their native lands without having
to invalidate conveyances made by the government to many U.S. citizens.
164

J ohnson was reiterated in the case of Worcester v. Georgia.
165
In this case, the State of Georgia
enacted a law requiring all white persons residing within the Cherokee nation to obtain a license or permit
from the Governor of Georgia; and any violation of the law was deemed a high misdemeanor. The
plaintiffs, who were white missionaries, did not obtain said license and were thus charged with a violation
of the Act.
The U.S. Supreme Court declared the Act as unconstitutional for interfering with the treaties established
between the United States and the Cherokee nation as well as the Acts of Congress regulating
intercourse with them. It characterized the relationship between the United States government and the
Indians as:
"The Indian nations were, from their situation, necessarily dependent on some foreign potentate for the
supply of their essential wants, and for their protection from lawless and injurious intrusions into their
country. That power was naturally termed their protector. They had been arranged under the protection of
Great Britain; but the extinguishment of the British power in their neighborhood, and the establishment of
that of the United States in its place, led naturally to the declaration, on the part of the Cherokees, that
they were under the protection of the United States, and of no other power. They assumed the relation
with the United States which had before subsisted with Great Britain.
This relation was that of a nation claiming and receiving the protection of one more powerful, not that of
individuals abandoning their national character, and submitting as subjects to the laws of a master."
166

It was the policy of the U.S. government to treat the Indians as nations with distinct territorial boundaries
and recognize their right of occupancy over all the lands within their domains. Thus:
"From the commencement of our government Congress has passed acts to regulate trade and
intercourse with the Indians; which treat them as nations, respect their rights, and manifest a firm purpose
to afford that protection which treaties stipulate. All these acts, and especially that of 1802, which is still in
force, manifestly consider the several Indian nations as distinct political communities, having
territorial boundaries, within which their authority is exclusive, and having a right to all the lands
within those boundaries, which is not only acknowledged, but guaranteed by the United States.
x x x.
"The Indian nations had always been considered as distinct, independent political communities,
retaining their original natural rights, as the undisputed possessors of the soil from time
immemorial,with the single exception of that imposed by irresistible power, which excluded them from
intercourse with any other European potentate than the first discoverer of the coast of the particular
region claimed: and this was a restriction which those European potentates imposed on themselves, as
well as on the Indians. The very term "nation," so generally applied to them, means "a people distinct
from others." x x x.
167

The Cherokee nation, then, is a distinct community, occupying its own territory, with boundaries
accurately described, in which the laws of Georgia can have no force, and which the citizens of Georgia
have no right to enter but with the assent of the Cherokees themselves or in conformity with treaties and
with the acts of Congress. The whole intercourse between the United States and this nation is, by our
Constitution and laws, vested in the government of the United States."
168

The discovery of the American continent gave title to the government of the discoverer as against all
other European governments. Designated as the naked fee,
169
this title was to be consummated by
possession and was subject to the Indian title of occupancy. The discoverer acknowledged the Indians'
legal and just claim to retain possession of the land, the Indians being the original inhabitants of the land.
The discoverer nonetheless asserted the exclusive right to acquire the Indians' land- either by purchase,
"defensive" conquest, or cession- and in so doing, extinguish the Indian title. Only the discoverer could
extinguish Indian title because it alone asserted ultimate dominion in itself. Thus, while the different
nations of Europe respected the rights of the natives as occupants, they all asserted the ultimate
dominion and title to be in themselves.
170

As early as the 19th century, it became accepted doctrine that although fee title to the lands
occupied by the Indians when the colonists arrived became vested in the sovereign- first the
discovering European nation and later the original 13 States and the United States- a right of
occupancy in the Indian tribes was nevertheless recognized. The Federal Government continued the
policy of respecting the Indian right of occupancy, sometimes called Indian title, which it accorded the
protection of complete ownership.
171
But this aboriginal Indian interest simply constitutes "permission"
from the whites to occupy the land, and means mere possession not specifically recognized as ownership
by Congress.
172
It is clear that this right of occupancy based upon aboriginal possession is not a property
right.
173
It is vulnerable to affirmative action by the federal government who, as sovereign, possessed
exclusive power to extinguish the right of occupancy at will.
174
Thus, aboriginal title is not the same as
legal title. Aboriginal title rests on actual, exclusive and continuous use and occupancy for a long
time.
175
It entails that land owned by Indian title must be used within the tribe, subject to its laws and
customs, and cannot be sold to another sovereign government nor to any citizen.
176
Such title as Indians
have to possess and occupy land is in the tribe, and not in the individual Indian; the right of individual
Indians to share in the tribal property usually depends upon tribal membership, the property of the tribe
generally being held in communal ownership.
177

As a rule, Indian lands are not included in the term "public lands," which is ordinarily used to designate
such lands as are subject to sale or other disposal under general laws.
178
Indian land which has been
abandoned is deemed to fall into the public domain.
179
On the other hand, an Indian reservation is a part
of the public domain set apart for the use and occupation of a tribe of Indians.
180
Once set apart by proper
authority, the reservation ceases to be public land, and until the Indian title is extinguished, no one but
Congress can initiate any preferential right on, or restrict the nation's power to dispose of, them.
181

The American judiciary struggled for more than 200 years with the ancestral land claims of
indigenous Americans.
182
And two things are clear. First, aboriginal title is recognized. Second,
indigenous property systems are also recognized. From a legal point of view, certain benefits can be
drawn from a comparison of Philippine IPs to native Americans.
183
Despite the similarities between native
title and aboriginal title, however, there are at present some misgivings on whether jurisprudence on
American Indians may be cited authoritatively in the Philippines. The U.S. recognizes the possessory
rights of the Indians over their land; title to the land, however, is deemed to have passed to the U.S. as
successor of the discoverer. The aboriginal title of ownership is not specifically recognized as ownership
by action authorized by Congress.
184
The protection of aboriginal title merely guards against
encroachment by persons other than the Federal Government.
185
Although there are criticisms against
the refusal to recognize the native Americans' ownership of these lands,
186
the power of the State to
extinguish these titles has remained firmly entrenched.
187

Under the IPRA, the Philippine State is not barred form asserting sovereignty over the ancestral domains
and ancestral lands.
188
The IPRA, however, is still in its infancy and any similarities between its
application in the Philippines vis--vis American Jurisprudence on aboriginal title will depend on the
peculiar facts of each case.
(c) Why the Cario doctrine is unique
In the Philippines, the concept of native title first upheld in Cario and enshrined in the IPRA grants
ownership, albeit in limited form, of the land to the ICCs/IPs. Native title presumes that the land is private
and was never public. Cario is the only case that specifically and categorically recognizes native
title. The long line of cases citing Cario did not touch on native title and the private character of
ancestral domains and lands. Cario was cited by the succeeding cases to support the concept of
acquisitive prescription under the Public Land Act which is a different matter altogether. Under the
Public Land Act, land sought to be registered must be public agricultural land. When the conditions
specified in Section 48 [b] of the Public Land Act are complied with, the possessor of the land is deemed
to have acquired, by operation of law, a right to a grant of the land.
189
The land ceases to be part of the
public domain,
190
ipso jure,
191
and is converted to private property by the mere lapse or completion of the
prescribed statutory period.
It was only in the case of Oh Cho v. Director of Lands
192
that the court declared that the rule that all
lands that were not acquired from the government, either by purchase or grant, belong to the public
domain has an exception. This exception would be any land that should have been in the possession of
an occupant and of his predecessors-in-interest since time immemorial. It is this kind of possession that
would justify the presumption that the land had never been part of the public domain or that it had been
private property even before the Spanish conquest.
193
Oh Cho, however, was decided under the
provisions of the Public Land Act and Cario was cited to support the applicant's claim of acquisitive
prescription under the said Act.
All these years, Cario had been quoted out of context simply to justify long, continuous, open and
adverse possession in the concept of owner of public agricultural land. It is this long, continuous, open
and adverse possession in the concept of owner of thirty years both for ordinary citizens
194
and members
of the national cultural minorities
195
that converts the land from public into private and entitles the
registrant to a torrens certificate of title.
(3) The Option of Securing a Torrens Title to the Ancestral Land Indicates that the Land is Private.
The private character of ancestral lands and domains as laid down in the IPRA is
further strengthened by the option given to individual ICCs/IPs over their individually-owned
ancestral lands. For purposes of registration under the Public Land Act and the Land Registration
Act, the IPRA expressly converts ancestral land into public agricultural land which may be
disposed of by the State. The necessary implication is thatancestral land is private. It, however,
has to be first converted to public agricultural land simply for registration purposes. To wit:
"Sec. 12. Option to Secure Certificate of Title Under Commonwealth Act 141, as amended, or the Land
Registration Act 496- Individual members of cultural communities, with respect to their individually-owned
ancestral lands who, by themselves or through their predecessors-in-interest, have been in continuous
possession and occupation of the same in the concept of owner since time immemorial or for a period of
not less than thirty (30) years immediately preceding the approval of this Act and uncontested by the
members of the same ICCs/IPs shall have the option to secure title to their ancestral lands under the
provisions of Commonwealth Act 141, as amended, or the Land Registration Act 496.
For this purpose, said individually-owned ancestral lands, which are agricultural in character and actually
used for agricultural, residential, pasture, and tree farming purposes, including those with a slope of
eighteen percent (18%) or more, are hereby classified as alienable and disposable agricultural lands.
The option granted under this section shall be exercised within twenty (20) years from the approval of this
Act."
196

ICCs/IPs are given the option to secure a torrens certificate of title over their individually-owned ancestral
lands. This option is limited to ancestral lands only, not domains, and such lands must be individually, not
communally, owned.
Ancestral lands that are owned by individual members of ICCs/IPs who, by themselves or through their
predecessors-in-interest, have been in continuous possession and occupation of the same in the concept
of owner since time immemorial
197
or for a period of not less than 30 years, which claims are uncontested
by the members of the same ICCs/IPs, may be registered under C.A. 141, otherwise known as the Public
Land Act, or Act 496, the Land Registration Act. For purposes of registration, the individually-owned
ancestral lands are classified as alienable and disposable agricultural lands of the public domain,
provided, they are agricultural in character and are actually used for agricultural, residential, pasture and
tree farming purposes. These lands shall be classified as public agricultural lands regardless of whether
they have a slope of 18% or more.
The classification of ancestral land as public agricultural land is in compliance with the requirements of
the Public Land Act and the Land Registration Act. C.A. 141, the Public Land Act, deals specifically with
lands of the public domain.
198
Its provisions apply to those lands "declared open to disposition or
concession" x x x "which have not been reserved for public or quasi-public purposes, nor appropriated by
the Government, nor in any manner become private property, nor those on which a private right
authorized and recognized by this Act or any other valid law x x x or which having been reserved or
appropriated, have ceased to be so."
199
Act 496, the Land Registration Act, allows registration only of
private lands and public agricultural lands. Since ancestral domains and lands are private, if the
ICC/IP wants to avail of the benefits of C.A. 141 and Act 496, the IPRA itself converts his ancestral
land, regardless of whether the land has a slope of eighteen per cent (18%) or over,
200
from private
to public agricultural land for proper disposition.
The option to register land under the Public Land Act and the Land Registration Act has nonetheless a
limited period. This option must be exercised within twenty (20) years from October 29, 1997, the date of
approval of the IPRA.
Thus, ancestral lands and ancestral domains are not part of the lands of the public domain. They
are private and belong to the ICCs/IPs. Section 3 of Article XII on National Economy and Patrimony of
the 1987 Constitution classifies lands of the public domain into four categories: (a) agricultural, (b) forest
or timber, (c) mineral lands, and (d) national parks. Section 5 of the same Article XII mentions ancestral
lands and ancestral domains but it does not classify them under any of the said four categories. To
classify them as public lands under any one of the four classes will render the entire IPRA law a
nullity. The spirit of the IPRA lies in the distinct concept of ancestral domains and ancestral lands. The
IPRA addresses the major problem of the ICCs/IPs which is loss of land. Land and space are of vital
concern in terms of sheer survival of the ICCs/IPs.
201

The 1987 Constitution mandates the State to "protect the rights of indigenous cultural
communities to their ancestral lands" and that "Congress provide for the applicability of
customary laws x x x in determining the ownership and extent of ancestral domain."
202
It is the
recognition of the ICCs/IPs distinct rights of ownership over their ancestral domains and lands
that breathes life into this constitutional mandate.
B. The right of ownership and possession by the ICCs/IPs of their ancestral domains is a limited
form of ownership and does not include the right to alienate the same.
Registration under the Public Land Act and Land Registration Act recognizes the concept of ownership
under thecivil law. This ownership is based on adverse possession for a specified period, and harkens to
Section 44 of the Public Land Act on administrative legalization (free patent) of imperfect or incomplete
titles and Section 48 (b) and (c) of the same Act on the judicial confirmation of imperfect or incomplete
titles. Thus:
"Sec. 44. Any natural-born citizen of the Philippines who is not the owner of more than twenty-four
hectares and who since July fourth, 1926 or prior thereto, has continuously occupied and cultivated,
either by himself or through his predecessors-in-interest, a tract or tracts of agricultural public lands
subject to disposition, or who shall have paid the real estate tax thereon while the same has not been
occupied by any person shall be entitled, under the provisions of this chapter, to have a free patent
issued to him for such tract or tracts of such land not to exceed twenty-four hectares.
A member of the national cultural minorities who has continuously occupied and cultivated, either
by himself or through his predecessors-in-interest, a tract or tracts of land, whether disposable or
not since July 4, 1955, shall be entitled to the right granted in the preceding paragraph of this
section:Provided, That at the time he files his free patent application he is not the owner of any
real property secured or disposable under the provision of the Public Land Law.
203

x x x.
"Sec. 48. The following described citizens of the Philippines, occupying lands of the public domain or
claiming to own any such lands or an interest therein, but whose titles have not been perfected or
completed, may apply to the Court of First Instance of the province where the land is located for
confirmation of their claims and the issuance of a certificate of title therefor, under the Land Registration
Act, to wit:
(a) [perfection of Spanish titles] xxx.
(b) Those who by themselves or through their predecessors-in-interest have been in open,
continuous, exclusive, and notorious possession and occupation of agricultural lands of the public
domain, under a bona fide claim of acquisition or ownership, for at least thirty years immediately
preceding the filing of the application for confirmation of title except when prevented by war or
force majeure. These shall be conclusively presumed to have performed all the conditions
essential to a Government grant and shall be entitled to a certificate of title under the provisions
of this Chapter.
(c) Members of the national cultural minorities who by themselves or through their
predecessors-in-interest have been in open, continuous, exclusive and notorious
possession and occupation of lands of the public domain suitable to agriculture, whether
disposable or not, under a bona fide claim of ownership for at least 30 years shall be
entitled to the rights granted in sub-section (b) hereof."
204

Registration under the foregoing provisions presumes that the land was originally public agricultural land
but because of adverse possession since July 4, 1955 (free patent) or at least thirty years (judicial
confirmation), the land has become private. Open, adverse, public and continuous possession is
sufficient, provided, the possessor makes proper application therefor. The possession has to be
confirmed judicially or administratively after which a torrens title is issued.
A torrens title recognizes the owner whose name appears in the certificate as entitled to all the rights of
ownership under the civil law. The Civil Code of the Philippines defines ownership in Articles 427, 428
and 429. This concept is based on Roman Law which the Spaniards introduced to the Philippines through
the Civil Code of 1889. Ownership, under Roman Law, may be exercised over things or rights. It primarily
includes the right of the owner to enjoy and dispose of the thing owned. And the right to enjoy and
dispose of the thing includes the right to receive from the thing what it produces,
205
the right to consume
the thing by its use,
206
the right to alienate, encumber, transform or even destroy the thing owned,
207
and
the right to exclude from the possession of the thing owned by any other person to whom the owner has
not transmitted such thing.
208

1. The Indigenous Concept of Ownership and Customary Law.
Ownership of ancestral domains by native title does not entitle the ICC/IP to a torrens title but to a
Certificate of Ancestral Domain Title (CADT). The CADT formally recognizes the indigenous concept of
ownership of the ICCs/IPs over their ancestral domain. Thus:
"Sec. 5. Indigenous concept of ownership.- Indigenous concept of ownership sustains the view that
ancestral domains and all resources found therein shall serve as the material bases of their cultural
integrity. The indigenous concept of ownership generally holds that ancestral domains are the ICCs/IPs
private but community property which belongs to all generations and therefore cannot be sold, disposed
or destroyed. It likewise covers sustainable traditional resource rights."
The right of ownership and possession of the ICCs/IPs to their ancestral domains is held under
the indigenous concept of ownership. This concept maintains the view that ancestral domains are
the ICCs/IPs private but community property. It is private simply because it is not part of the
public domain. But its private character ends there. The ancestral domain is owned in common by
the ICCs/IPs and not by one particular person. The IPRA itself provides that areas within the ancestral
domains, whether delineated or not, are presumed to be communally held.
209
These communal rights,
however, are not exactly the same as co-ownership rights under the Civil Code.
210
Co-ownership
gives any co-owner the right to demand partition of the property held in common. The Civil Code
expressly provides that "no co-owner shall be obliged to remain in the co-ownership." Each co-owner may
demand at any time the partition of the thing in common, insofar as his share is concerned.
211
To allow
such a right over ancestral domains may be destructive not only of customary law of the community but of
the very community itself.
212

Communal rights over land are not the same as corporate rights over real property, much less
corporate condominium rights. A corporation can exist only for a maximum of fifty (50) years subject to
an extension of another fifty years in any single instance.
213
Every stockholder has the right to
disassociate himself from the corporation.
214
Moreover, the corporation itself may be dissolved voluntarily
or involuntarily.
215

Communal rights to the land are held not only by the present possessors of the land but extends
to all generations of the ICCs/IPs, past, present and future, to the domain. This is the reason why
the ancestral domain must be kept within the ICCs/IPs themselves. The domain cannot be transferred,
sold or conveyed to other persons. It belongs to the ICCs/IPs as a community.
Ancestral lands are also held under the indigenous concept of ownership. The lands are communal.
These lands, however, may be transferred subject to the following limitations: (a) only to the members of
the same ICCs/IPs; (b) in accord with customary laws and traditions; and (c) subject to the right of
redemption of the ICCs/IPs for a period of 15 years if the land was transferred to a non-member of the
ICCs/IPs.
Following the constitutional mandate that "customary law govern property rights or relations in
determining the ownership and extent of ancestral domains,"
216
the IPRA, by legislative fiat, introduces
a new concept of ownership. This is a concept that has long existed under customary law.
217

Custom, from which customary law is derived, is also recognized under the Civil Code as a source
of law.
218
Some articles of the Civil Code expressly provide that custom should be applied in cases where
no codal provision is applicable.
219
In other words, in the absence of any applicable provision in the Civil
Code, custom, when duly proven, can define rights and liabilities.
220

Customary law is a primary, not secondary, source of rights under the IPRA and uniquely applies to
ICCs/IPs. Its recognition does not depend on the absence of a specific provision in the civil
law. The indigenous concept of ownership under customary law is specifically acknowledged and
recognized, and coexists with the civil law concept and the laws on land titling and land registration.
221

To be sure, the indigenous concept of ownership exists even without a paper title. The CADT is
merely a "formal recognition" of native title. This is clear from Section 11 of the IPRA, to wit:
"Sec. 11. Recognition of Ancestral Domain Rights.- The rights of ICCs/IPs to their ancestral domains by
virtue of Native Title shall be recognized and respected. Formal recognition, when solicited by ICCs/IPs
concerned shall be embodied in a Certificate of Ancestral Domain Title, which shall recognize the title of
the concerned ICCs/IPs over the territories identified and delineated."
The moral import of ancestral domain, native land or being native is "belongingness" to the land, being
people of the land- by sheer force of having sprung from the land since time beyond recall, and the
faithful nurture of the land by the sweat of one's brow. This is fidelity of usufructuary relation to the land-
the possession of stewardship through perduring, intimate tillage, and the mutuality of blessings between
man and land; from man, care for land; from the land, sustenance for man.
222

C. Sections 7 (a), 7 (b) and 57 of the IPRA Do Not Violate the Regalian Doctrine Enshrined in
Section 2, Article XII of the 1987 Constitution.
1. The Rights of ICCs/IPs Over Their Ancestral Domains and Lands
The IPRA grants the ICCs/IPs several rights over their ancestral domains and ancestral lands. Section 7
provides for the rights over ancestral domains:
"Sec. 7. Rights to Ancestral Domains.- The rights of ownership and possession of ICCs/IPs to their
ancestral domains shall be recognized and protected. Such rights include:
a) Right of Ownership.- The right to claim ownership over lands, bodies of water traditionally
and actually occupied by ICCs/IPs, sacred places, traditional hunting and fishing grounds,
and all improvements made by them at any time within the domains;
b) Right to Develop Lands and Natural Resources.- Subject to Section 56 hereof, the right to
develop, control and use lands and territories traditionally occupied, owned, or used; to
manage and conserve natural resources within the territories and uphold the
responsibilities for future generations; to benefit and share the profits from allocation and
utilization of the natural resources found therein; the right to negotiate the terms and
conditions for the exploration of natural resources in the areas for the purpose of ensuring
ecological, environmental protection and the conservation measures, pursuant to national
and customary laws; the right to an informed and intelligent participation in the formulation and
implementation of any project, government or private, that will affect or impact upon the ancestral
domains and to receive just and fair compensation for any damages which they may sustain as a
result of the project; and the right to effective measures by the government to prevent any
interference with, alienation and encroachment upon these rights;"
c) Right to Stay in the Territories.- The right to stay in the territory and not to be removed
therefrom. No ICCs/IPs will be relocated without their free and prior informed consent, nor
through any means other than eminent domain. x x x;
d) Right in Case of Displacement.- In case displacement occurs as a result of natural
catastrophes, the State shall endeavor to resettle the displaced ICCs/IPs in suitable areas where
they can have temporary life support systems: x x x;
e) Right to Regulate the Entry of Migrants.- Right to regulate the entry of migrant settlers and
organizations into their domains;
f) Right to Safe and Clean Air and Water.-For this purpose, the ICCs/IPs shall have access to
integrated systems for the management of their inland waters and air space;
g) Right to Claim Parts of Reservations.- The right to claim parts of the ancestral domains which
have been reserved for various purposes, except those reserved and intended for common and
public welfare and service;
h) Right to Resolve Conflict.- Right to resolve land conflicts in accordance with customary laws of
the area where the land is located, and only in default thereof shall the complaints be submitted
to amicable settlement and to the Courts of Justice whenever necessary."
Section 8 provides for the rights over ancestral lands:
"Sec. 8. Rights to Ancestral Lands.- The right of ownership and possession of the ICCs/IPs to their
ancestral lands shall be recognized and protected.
a) Right to transfer land/property.- Such right shall include the right to transfer land or property
rights to/among members of the same ICCs/IPs, subject to customary laws and traditions of the
community concerned.
b) Right to Redemption.- In cases where it is shown that the transfer of land/property rights by
virtue of any agreement or devise, to a non-member of the concerned ICCs/IPs is tainted by the
vitiated consent of the ICCs/IPs, or is transferred for an unconscionable consideration or price,
the transferor ICC/IP shall have the right to redeem the same within a period not exceeding
fifteen (15) years from the date of transfer."
Section 7 (a) defines the ICCs/IPs the right of ownership over their ancestral domains which covers (a)
lands, (b) bodies of water traditionally and actually occupied by the ICCs/IPs, (c) sacred places, (d)
traditional hunting and fishing grounds, and (e) all improvements made by them at any time within the
domains. The right of ownership includes the following rights: (1) the right to develop lands and natural
resources; (b) the right to stay in the territories; (c) the right to resettlement in case of displacement; (d)
the right to regulate the entry of migrants; (e) the right to safe and clean air and water; (f) the right to claim
parts of the ancestral domains as reservations; and (g) the right to resolve conflict in accordance with
customary laws.
Section 8 governs their rights to ancestral lands. Unlike ownership over the ancestral domains, Section 8
gives the ICCs/IPs also the right to transfer the land or property rights to members of the same ICCs/IPs
or non-members thereof. This is in keeping with the option given to ICCs/IPs to secure a torrens title over
the ancestrallands, but not to domains.
2. The Right of ICCs/IPs to Develop Lands and Natural Resources Within the Ancestral Domains Does
Not Deprive the State of Ownership Over the Natural Resources and Control and Supervision in their
Development and Exploitation.
The Regalian doctrine on the ownership, management and utilization of natural resources is declared
in Section 2, Article XII of the 1987 Constitution, viz:
"Sec. 2. All lands of the public domain, waters, minerals, coal, petroleum, and other mineral oils,
all forces of potential energy, fisheries, forests or timber, wildlife, flora and fauna, and other
natural resources are owned by the State. With the exception of agricultural lands, all other natural
resources shall not be alienated. The exploration, development, and utilization of natural resources
shall be under the full control and supervision of the State. The State may directly undertake such
activities, or, it may enter into co-production, joint venture, or production-sharing agreements
with Filipino citizens, or corporations or associations at least sixty per centum of whose capital is
owned by such citizens. Such agreements may be for a period not exceeding twenty-five years,
renewable for not more than twenty-five years, and under such terms and conditions as may be provided
by law. In cases of water rights for irrigation, water supply, fisheries, water supply, fisheries, or industrial
uses other than the development of water power, beneficial use may be the measure and limit of the
grant.
The State shall protect the nation's marine wealth in its archipelagic waters, territorial sea, and exclusive
economic zone, and reserve its use and enjoyment exclusively to Filipino citizens.
The Congress may, by law, allow small-scale utilization of natural resources by Filipino citizens, as
well as cooperative fish farming, with priority to subsistence fishermen and fishworkers in rivers, lakes,
bays, and lagoons.
The President may enter into agreements with foreign-owned corporations involving either technical or
financial assistance for large-scale exploration, development, and utilization of minerals, petroleum,
and other mineral oils according to the general terms and conditions provided by law, based on real
contributions to the economic growth and general welfare of the country. In such agreements, the state
shall promote the development and use of local scientific and technical resources.
The President shall notify the Congress of every contract entered into in accordance with this provision,
within thirty days from its execution."
223

All lands of the public domain and all natural resources- waters, minerals, coal, petroleum, and other
mineral oils, all forces of potential energy, fisheries, forests or timber, wildlife, flora and fauna, and other
natural resources- are owned by the State. The Constitution provides that in the exploration,
development and utilization of these natural resources, the State exercises full control and supervision,
and may undertake the same in four (4) modes:
1. The State may directly undertake such activities; or
2. The State may enter into co-production, joint venture or production-sharing agreements with
Filipino citizens or qualified corporations;
3. Congress may, by law, allow small-scale utilization of natural resources by Filipino citizens;
4. For the large-scale exploration, development and utilization of minerals, petroleum and other
mineral oils, the President may enter into agreements with foreign-owned corporations involving
technical or financial assistance.
As owner of the natural resources, the State is accorded primary power and responsibility in the
exploration, development and utilization of these natural resources. The State may directly
undertake the exploitation and development by itself, or, it may allow participation by the private sector
through co-production,
224
joint venture,
225
or production-sharing agreements.
226
These agreements may be
for a period of 25 years, renewable for another 25 years. The State, through Congress, may allow the
small-scale utilization of natural resources by Filipino citizens. For the large-scale exploration of these
resources, specifically minerals, petroleum and other mineral oils, the State, through the President, may
enter into technical and financial assistance agreements with foreign-owned corporations.
Under the Philippine Mining Act of 1995, (R.A. 7942) and the People's Small-Scale Mining Act of 1991
(R.A. 7076) the three types of agreements, i.e., co-production, joint venture or production-sharing, may
apply to both large-scale
227
and small-scale mining.
228
"Small-scale mining" refers to "mining activities
which rely heavily on manual labor using simple implements and methods and do not use explosives or
heavy mining equipment."
229

Examining the IPRA, there is nothing in the law that grants to the ICCs/IPs ownership over the
natural resources within their ancestral domains. The right of ICCs/IPs in their ancestral domains
includesownership, but this "ownership" is expressly defined and limited in Section 7 (a) as:
"Sec. 7. a) Right of ownership- The right to claim ownership over lands, bodies of water traditionally and
actually occupied by ICCs/IPs, sacred places, traditional hunting and fishing grounds, and all
improvements made by them at any time within the domains;"
The ICCs/IPs are given the right to claim ownership over "lands, bodies of water traditionally and actually
occupied by ICCs/IPs, sacred places, traditional hunting and fishing grounds, and all improvements made
by them at any time within the domains." It will be noted that this enumeration does not mention bodies of
water not occupied by the ICCs/IPs, minerals, coal, wildlife, flora and fauna in the traditional hunting
grounds, fish in the traditional fishing grounds, forests or timber in the sacred places, etc. and all other
natural resources found within the ancestral domains. Indeed, the right of ownership under Section 7
(a) does not cover "waters, minerals, coal,petroleum and other mineral oils, all forces of potential
energy, fisheries, forests or timber, wildlife,flora and fauna and all other natural resources"
enumerated in Section 2, Article XII of the 1987 Constitution as belonging to the State.
The non-inclusion of ownership by the ICCs/IPs over the natural resources in Section 7(a) complies with
the Regalian doctrine.
(a) Section 1, Part II, Rule III of the Implementing Rules Goes Beyond the Parameters of Sec. 7 (a)
of the IPRA And is Unconstitutional.
The Rules Implementing the IPRA
230
in Section 1, Part II, Rule III reads:
"Section 1. Rights of Ownership. ICCs/IPs have rights of ownership over lands, waters, and natural
resources and all improvements made by them at any time within the ancestral domains/ lands. These
rights shall include, but not limited to, the right over the fruits, the right to possess, the right to use, right to
consume, right to exclude and right to recover ownership, and the rights or interests over land and natural
resources. The right to recover shall be particularly applied to lands lost through fraud or any form or
vitiated consent or transferred for an unconscionable price."
Section 1 of the Implementing Rules gives the ICCs/IPs rights of ownership over "lands, waters and
natural resources." The term "natural resources" is not one of those expressly mentioned in Section 7 (a)
of the law. Our Constitution and jurisprudence clearly declare that the right to claim ownership over land
does not necessarily include the right to claim ownership over the natural resources found on or under the
land.
231
The IPRA itself makes a distinction between land and natural resources. Section 7 (a)
speaks of the right of ownership only over the land within the ancestral domain. It is Sections 7 (b)
and 57 of the law that speak of natural resources, and these provisions, as shall be discussed
later, do not give the ICCs/IPs the right of ownership over these resources.
The constitutionality of Section 1, Part II, Rule III of the Implementing Rules was not specifically and
categorically challenged by petitioners. Petitioners actually assail the constitutionality of the Implementing
Rules in general.
232
Nevertheless, to avoid any confusion in the implementation of the law, it is necessary
to declare that the inclusion of "natural resources" in Section 1, Part II, Rule III of the Implementing Rules
goes beyond the parameters of Section 7 (b) of the law and is contrary to Section 2, Article XII of the
1987 Constitution.
(b) The Small-Scale Utilization of Natural Resources In Sec. 7 (b) of the IPRA Is Allowed Under
Paragraph 3, Section 2 of Article XII of the Constitution.
Ownership over natural resources remain with the State and the IPRA in Section 7 (b) merely grants the
ICCs/IPs the right to manage them, viz:
"Sec. 7 (b) Right to Develop Lands and Natural Resources.- Subject to Section 56 hereof, right to
develop, control and use lands and territories traditionally occupied, owned, or used; to manage and
conserve natural resourceswithin the territories and uphold the responsibilities for future generations; to
benefit and share the profits from allocation and utilization of the natural resources found therein; the right
to negotiate the terms and conditions for the exploration of natural resources in the areas for the purpose
of ensuring ecological, environmental protection and the conservation measures, pursuant to national and
customary laws; the right to an informed and intelligent participation in the formulation and
implementation of any project, government or private, that will affect or impact upon the ancestral
domains and to receive just and fair compensation for any damages which they may sustain as a result of
the project; and the right to effective measures by the government to prevent any interference with,
alienation and encroachment upon these rights;"
The right to develop lands and natural resources under Section 7 (b) of the IPRA enumerates the
following rights:
a) the right to develop, control and use lands and territories traditionally occupied;
b) the right to manage and conserve natural resources within the territories and uphold the
responsibilities for future generations;
c) the right to benefit and share the profits from the allocation and utilization of the natural
resources found therein;
d) the right to negotiate the terms and conditions for the exploration of natural resources for the
purpose of ensuring ecological, environmental protection and the conservation measures,
pursuant to national and customary laws;
e) the right to an informed and intelligent participation in the formulation and implementation of
any project, government or private, that will affect or impact upon the ancestral domains and to
receive just and fair compensation for any damages which they may sustain as a result of the
project;
f) the right to effective measures by the government to prevent any interference with, alienation
and encroachment upon these rights.
233

Ownership over the natural resources in the ancestral domains remains with the State and the
ICCs/IPs are merely granted the right to "manage and conserve" them for future generations,
"benefit and share" the profits from their allocation and utilization, and "negotiate the terms and
conditions for their exploration" for the purpose of "ensuring ecological and environmental
protection and conservation measures." It must be noted that the right to negotiate the terms and
conditions over the natural resources covers only their exploration which must be for the purpose of
ensuring ecological and environmental protection of, and conservation measures in the ancestral domain.
It does not extend to the exploitation and development of natural resources.
Simply stated, the ICCs/IPs' rights over the natural resources take the form of management or
stewardship. For the ICCs/IPs may use these resources and share in the profits of their utilization or
negotiate the terms for their exploration. At the same time, however, the ICCs/IPs must ensure that the
natural resources within their ancestral domains are conserved for future generations and that the
"utilization" of these resources must not harm the ecology and environment pursuant to national and
customary laws.
234

The limited rights of "management and use" in Section 7 (b) must be taken to contemplate small-
scale utilization of natural resources as distinguished from large-scale. Small-scale utilization of
natural resources is expressly allowed in the third paragraph of Section 2, Article XII of the
Constitution "in recognition of the plight of forest dwellers, gold panners, marginal fishermen and others
similarly situated who exploit our natural resources for their daily sustenance and survival."
235
Section 7
(b) also expressly mandates the ICCs/IPs to manage and conserve these resources and ensure
environmental and ecological protection within the domains, which duties, by their very nature,
necessarily reject utilization in a large-scale.
(c) The Large-Scale Utilization of Natural Resources In Section 57 of the IPRA Is Allowed Under
Paragraphs 1 and 4, Section 2, Article XII of the 1987 Constitution.
Section 57 of the IPRA provides:
"Sec. 57. Natural Resources within Ancestral Domains.- The ICCs/IPs shall have priority rights in
theharvesting, extraction, development or exploitation of any natural resources within the ancestral
domains. A non-member of the ICCs/IPs concerned may be allowed to take part in the development and
utilization of the natural resources for a period of not exceeding twenty-five (25) years renewable for not
more than twenty-five (25) years: Provided, That a formal and written agreement is entered into with the
ICCs/IPs concerned or that the community, pursuant to its own decision-making process, has agreed to
allow such operation: Provided finally, That the NCIP may exercise visitorial powers and take appropriate
action to safeguard the rights of the ICCs/IPs under the same contract."
Section 57 speaks of the "harvesting, extraction, development or exploitation of natural resources
within ancestral domains" and "gives the ICCs/IPs 'priority rights' therein." The terms "harvesting,
extraction, development or exploitation" of any natural resources within the ancestral domains
obviously refer to large-scale utilization. It is utilization not merely for subsistence but for commercial
or other extensive use that require technology other than manual labor.
236
The law recognizes the
probability of requiring a non-member of the ICCs/IPs to participate in the development and utilization of
the natural resources and thereby allows such participation for a period of not more than 25 years,
renewable for another 25 years. This may be done on condition that a formal written agreement be
entered into by the non-member and members of the ICCs/IPs.
Section 57 of the IPRA does not give the ICCs/IPs the right to "manage and conserve" the natural
resources. Instead, the law only grants the ICCs/IPs "priority rights" in the development or exploitation
thereof. Priority means giving preference. Having priority rights over the natural resources does not
necessarily mean ownership rights. The grant of priority rights implies that there is a superior entity that
owns these resources and this entity has the power to grant preferential rights over the resources to
whosoever itself chooses.
Section 57 is not a repudiation of the Regalian doctrine. Rather, it is an affirmation of the said doctrine
that all natural resources found within the ancestral domains belong to the State. It incorporates by
implication the Regalian doctrine, hence, requires that the provision be read in the light of Section 2,
Article XII of the 1987 Constitution. Interpreting Section 2, Article XII of the 1987 Constitution
237
in
relation to Section 57 of IPRA, the State, as owner of these natural resources, may directly
undertake the development and exploitation of the natural resources by itself, or in the alternative,
it may recognize the priority rights of the ICCs/IPs as owners of the land on which the natural
resources are found by entering into a co-production, joint venture, or production-sharing
agreement with them. The State may likewise enter into any of said agreements with a non-
member of the ICCs/IPs, whether natural or juridical, or enter into agreements with foreign-owned
corporations involving either technical or financial assistance for the large-scale exploration,
development and utilization of minerals, petroleum, and other mineral oils, or allow such non-
member to participate in its agreement with the ICCs/IPs. If the State decides to enter into an
agreement with a non-ICC/IP member, the National Commission on Indigenous Peoples (NCIP) shall
ensure that the rights of the ICCs/IPs under the agreement shall be protected. The agreement shall be for
a period of 25 years, renewable for another 25 years.
To reiterate, in the large-scale utilization of natural resources within the ancestral domains, the State, as
owner of these resources, has four (4) options: (1) it may, of and by itself, directly undertake the
development and exploitation of the natural resources; or (2) it may recognize the priority rights of the
ICCs/IPs by entering into an agreement with them for such development and exploitation; or (3) it may
enter into an agreement with a non-member of the ICCs/IPs, whether natural or juridical, local or foreign;
or (4) it may allow such non-member to participate in the agreement with the ICCs/IPs.
The rights granted by the IPRA to the ICCs/IPs over the natural resources in their ancestral
domains merely gives the ICCs/IPs, as owners and occupants of the land on which the resources
are found, the right to the small-scale utilization of these resources, and at the same time, a
priority in their large-scale development and exploitation. Section 57 does not mandate the State
to automatically give priority to the ICCs/IPs. The State has several options and it is within its
discretion to choose which option to pursue. Moreover, there is nothing in the law that gives the
ICCs/IPs the right to solely undertake the large-scale development of the natural resources within their
domains. The ICCs/IPs must undertake such endeavour always under State supervision or control. This
indicates that the State does not lose control and ownership over the resources even in their exploitation.
Sections 7 (b) and 57 of the law simply give due respect to the ICCs/IPs who, as actual occupants of the
land where the natural resources lie, have traditionally utilized these resources for their subsistence and
survival.
Neither is the State stripped of ownership and control of the natural resources by the following provision:
"Section 59. Certification Precondition.- All departments and other governmental agencies shall
henceforth be strictly enjoined from issuing, renewing or granting any concession, license or lease, or
entering into any production-sharing agreement. without prior certification from the NCIP that the area
affected does not overlap with any ancestral domain. Such certification shall only be issued after a field-
based investigation is conducted by the Ancestral Domains Office of the area concerned: Provided, That
no certification shall be issued by the NCIP without the free and prior informed and written consent of the
ICCs/IPs concerned: Provided, further, That no department, government agency or government-owned or
-controlled corporation may issue new concession, license, lease, or production sharing agreement while
there is a pending application for a CADT: Provided, finally, That the ICCs/IPs shall have the right to stop
or suspend, in accordance with this Act, any project that has not satisfied the requirement of this
consultation process."
Concessions, licenses, lease or production-sharing agreements for the exploitation of natural resources
shall not be issued, renewed or granted by all departments and government agencies without prior
certification from the NCIP that the area subject of the agreement does not overlap with any ancestral
domain. The NCIP certification shall be issued only after a field-based investigation shall have been
conducted and the free and prior informed written consent of the ICCs/IPs obtained. Non-compliance with
the consultation requirement gives the ICCs/IPs the right to stop or suspend any project granted by any
department or government agency.
As its subtitle suggests, this provision requires as a precondition for the issuance of any concession,
license or agreement over natural resources, that a certification be issued by the NCIP that the area
subject of the agreement does not lie within any ancestral domain. The provision does not vest the NCIP
with power over the other agencies of the State as to determine whether to grant or deny any concession
or license or agreement. It merely gives the NCIP the authority to ensure that the ICCs/IPs have been
informed of the agreement and that their consent thereto has been obtained. Note that the certification
applies to agreements over natural resources that do not necessarily lie within the ancestral domains. For
those that are found within the said domains, Sections 7(b) and 57 of the IPRA apply.
V. THE IPRA IS A RECOGNITION OF OUR ACTIVE PARTICIPATION IN THE INDIGENOUS
INTERNATIONAL MOVEMENT.
The indigenous movement can be seen as the heir to a history of anti-imperialism stretching back to
prehistoric times. The movement received a massive impetus during the 1960's from two sources. First,
the decolonization of Asia and Africa brought into the limelight the possibility of peoples controlling their
own destinies. Second, the right of self-determination was enshrined in the UN Declaration on Human
Rights.
238
The rise of the civil rights movement and anti-racism brought to the attention of North American
Indians, Aborigines in Australia, and Maori in New Zealand the possibility of fighting for fundamental
rights and freedoms.
In 1974 and 1975, international indigenous organizations were founded,
239
and during the 1980's,
indigenous affairs were on the international agenda. The people of the Philippine Cordillera were the first
Asians to take part in the international indigenous movement. It was the Cordillera People's Alliance that
carried out successful campaigns against the building of the Chico River Dam in 1981-82 and they have
since become one of the best-organized indigenous bodies in the world.
240

Presently, there is a growing concern for indigenous rights in the international scene. This came as a
result of the increased publicity focused on the continuing disrespect for indigenous human rights and the
destruction of the indigenous peoples' environment, together with the national governments' inability to
deal with the situation.
241
Indigenous rights came as a result of both human rights and environmental
protection, and have become a part of today's priorities for the international agenda.
242

International institutions and bodies have realized the necessity of applying policies, programs and
specific rules concerning IPs in some nations. The World Bank, for example, first adopted a policy on IPs
as a result of the dismal experience of projects in Latin America.
243
The World Bank now seeks to apply
its current policy on IPs to some of its projects in Asia. This policy has provided an influential model for
the projects of the Asian Development Bank.
244

The 1987 Philippine Constitution formally recognizes the existence of ICCs/IPs and declares as a State
policy the promotion of their rights within the framework of national unity and development.
245
The IPRA
amalgamates the Philippine category of ICCs with the international category of IPs,
246
and is heavily
influenced by both the International Labor Organization (ILO) Convention 169 and the United Nations
(UN) Draft Declaration on the Rights of Indigenous Peoples.
247

ILO Convention No. 169 is entitled the "Convention Concerning Indigenous and Tribal Peoples in
Independent Countries"
248
and was adopted on June 27, 1989. It is based on the Universal Declaration of
Human Rights, the International Covenant on Economic, Social and Cultural Rights, the International
Covenant on Civil and Political Rights, and many other international instruments on the prevention of
discrimination.
249
ILO Convention No. 169 revised the "Convention Concerning the Protection and
Integration of Indigenous and Other Tribal and Semi-Tribal Populations in Independent Countries" (ILO
No. 107) passed on June 26, 1957. Developments in international law made it appropriate to adopt new
international standards on indigenous peoples "with a view to removing the assimilationist orientation of
the earlier standards," and recognizing the aspirations of these peoples to exercise control over their own
institutions, ways of life and economic development."
250

CONCLUSION
The struggle of the Filipinos throughout colonial history had been plagued by ethnic and religious
differences. These differences were carried over and magnified by the Philippine government through the
imposition of a national legal order that is mostly foreign in origin or derivation.
251
Largely unpopulist, the
present legal system has resulted in the alienation of a large sector of society, specifically, the indigenous
peoples. The histories and cultures of the indigenes are relevant to the evolution of Philippine culture and
are vital to the understanding of contemporary problems.
252
It is through the IPRA that an attempt was
made by our legislators to understand Filipino society not in terms of myths and biases but through
common experiences in the course of history. The Philippines became a democracy a centennial ago and
the decolonization process still continues. If the evolution of the Filipino people into a democratic society
is to truly proceed democratically, i.e., if the Filipinos as a whole are to participate fully in the task of
continuing democratization,
253
it is this Court's duty to acknowledge the presence of indigenous and
customary laws in the country and affirm their co-existence with the land laws in our national legal
system.
With the foregoing disquisitions, I vote to uphold the constitutionality of the Indigenous Peoples Rights Act
of 1997.


SEPARATE OPINION
VITUG, J .:
An issue of grave national interest indeed deserves a proper place in any forum and, when it
shows itself in a given judicial controversy, the rules of procedure, like locus standi, the propriety
of the specific remedy invoked, or the principle of hierarchy of courts, that may ordinarily be
raised by party-litigants, should not be so perceived as good and inevitable justifications for
advocating timidity, let alone isolationism, by the Court.
A cardinal requirement, to which I agree, is that one who invokes the Courts adjudication must have a
personal and substantial interest in the dispute;
1
indeed, the developing trend would require a logical
nexus between the status asserted and the claim sought to be adjudicated in order to ensure that one is
the proper and appropriate party to invoke judicial power.
2
The rule requires a party to aptly show a
personal stake in the outcome of the case or an injury to himself that can be redressed by a favorable
decision so as to warrant his invocation of the Courts jurisdiction and to render legally feasible the
exercise of the Courts remedial powers in his behalf. If it were otherwise, the exercise of that power can
easily become too unwieldy by its sheer magnitude and scope to a point that may, in no small measure,
adversely affect its intended essentiality, stability and consequentiality.
Nevertheless, where a most compelling reason exits, such as when the matter is of transcendental
importance and paramount interest to the nation,
3
the Court must take the liberal approach that
recognizes the legal standing of nontraditional plaintiffs, such as citizens and taxpayers, to raise
constitutional issues that affect them.
4
This Court thus did so in a case
5
that involves the conservation of
our forests for ecological needs. Until and exact balance is struck, the Court must accept an eclectic
notion that can free itself from the bondage of legal nicety and hold trenchant technicalities
subordinate to what may be considered to be of overriding concern.
The petition seeks a declaration by the Court of unconstitutionality of certain provisions of Republic Act
No. 8371, a law that obviously is yet incapable of exact equation in its significance to the nation and its
people now and in the generations yet to come. Republic Act No. 8371, otherwise also known as the
Indigenous Peoples Rights Act of 1997 ("IPRA"), enacted into law in 1997 and made effective on 22
November 1997, is apparently intended to be a legislative response to the 1987 Constitution which
recognizes the rights of indigenous cultural communities "within the framework of national unity and
development"
6
and commands the State, "subject to the provisions of this Constitution and national
development policies and programs," to protect the rights of indigenous cultural communities to their
ancestral lands in order to ensure their economic, social, and cultural well-being.
7

Among the assailed provisions in IPRA is its Section 3(a) which defines "ancestral domains" to embrace
"all areas generally belonging to ICCs/IPs comprising lands, inland waters, coastal areas, and
natural resources" including "ancestral lands, forest, pasture, residential, agricultural, and other
lands individually owned whether alienable and disposable or otherwise," over which indigenous
cultural communities/indigenous peoples ("ICCs/IPs") could exercise virtual ownership and
control.
IPRA effectively withdraws from the public domain the so-called ancestral domains covering
literally millions of hectares. The notion of community property would comprehend not only
matters of proprietary interest but also some forms of self-governance over the curved-out
territory. This concept is elaborated in Section 7 of the law which states that the "rights of ownership and
possession of ICCs/IPs to their ancestral domains shall be recognized and protected," subsumed under
which would encompass the right of ownership (paragraph a); the right to develop, control and use
lands and natural resources, including "the right to negotiate the terms and conditions for the
exploration of natural resources in the areas for the purpose of ensuring ecological, environmental
protection and the conservation measures, pursuant to national and customary laws;" (par. b); the right
to stay in the territories (par. c); the right to return to their abandoned lands in case of
displacement (par. d); the right to regulate entry of migrants (par. e); the right to claim parts of
ancestral domains previously reserved (par. g); and the right to resolve land conflicts in
accordance primarily with customary law (par. h). Concurrently, Section 57 states that ICCs/IPs shall
be given "priority rights in the harvesting, extraction, development or exploitation of any natural resources
within the ancestral domains." These provisions of IPRA, in their totality, are, in my view, beyond the
context of the fundamental law and virtually amount to an undue delegation, if not an
unacceptable abdication, of State authority over a significant area of the country and its
patrimony.
Article XII of the 1987 Constitution expresses that all "lands of the public domain, waters, minerals,
coal, petroleum, and other mineral oils, all forces of potential energy, fisheries, forest or timber,
wildlife, flora and fauna, and other natural resources are owned by the State," and, with the
exception of agricultural lands, "shall not be alienated." It ordains that the "exploration, development,
and utilization of natural resources shall be under the full control and supervision of the State."
8

These provisions had roots in the 1935 Constitution which, along with some other specific mandates in
the 1935 Constitution, forming Article XII under the title "Conservation and Utilization of Natural
Resources", were derived largely from the report of the Committee on Nationalization and Preservation of
Lands and other Natural Resources.
9
According to the Committee report, among the principles upon
which these provisions were based, was "that the land, minerals, forest and other natural resources
constitute the exclusive heritage of the Filipino Nation," and should thereby "be preserved for those under
the sovereign authority of the Nation and for their posterity."
10
The delegates to the 1934 Constitutional
Convention were of the unanimous view that the "policy on natural resources, being fundamental to the
nations survival should not be left to the changing mood of the lawmaking body."
11

The 1987 Constitution, like the precursor provisions in the 1935 and 1973 Constitutions, thus expresses
thisregalian doctrine of the old, and the domainial doctrine of the new, that all lands and natural resources
belong to the state other than those which it recognizes to be of private ownership. Except for
agricultural lands of the public domain which alone may be alienated, forest or timber, and
mineral lands, as well as all other natural resources, of the country must remain with the state, the
exploration, development and utilization of which shall be subject to its full control and
supervision albeit allowing it to enter into co-production, joint venture or production-sharing agreements,
or into agreements with foreign-owned corporations involving technical or financial assistance for large-
scale exploration, development and utilization.
12

The decision of the United States Supreme Court in Cario vs. Insular Government,
13
holding that a
parcel of land held since time immemorial by individuals under a claim of private ownership is presumed
never to have been public land and cited to downgrade the application of the regalian doctrine, cannot
override the collective will of the people expressed in the Constitution. It is in them that sovereignty
resides and from them that all government authority emanates.
14
It is not then for a court ruling or any
piece of legislation to be conformed to by the fundamental law, but it is for the former to adapt to the
latter, and it is the sovereign act that must, between them, stand inviolate.
The second paragraph of Section 5 of Article XII of the Constitution allows Congress to provide "for the
applicability of customary laws governing property rights or relations in determining the ownership and
extent of ancestral domains." I do not see this statement as saying that Congress may enact a law that
would simply express that "customary laws shall govern" and end it there. Had it been so, the Constitution
could have itself easily provided without having to still commission Congress to do it. Mr. Chief Justice
Davide has explained this authority of Congress, during the deliberations of the 1986 Constitutional
Convention, thus:
"Mr. Davide. x x x Insofar as the application of the customary laws governing property rights or relations
in determining the ownership and extent of the ancestral domain is concerned, it is respectfully submitted
that the particular matter must be submitted to Congress. I understand that the idea of Comm. Bennagen
is for the possibility of the codification of these customary laws. So before these are codified, we cannot
now mandate that the same must immediately be applicable. We leave it to Congress to determine the
extent of the ancestral domain and the ownership thereof in relation to whatever may have been codified
earlier. So, in short, let us not put the cart ahead of the horse."
15

The constitutional aim, it seems to me, is to get Congress to look closely into the customary laws
and, with specificity and by proper recitals, to hew them to, and make them part of, the stream of
laws. The "due process clause," as I so understand it in Tanada vs. Tuvera
16
would require an apt
publication of a legislative enactment before it is permitted to take force and effect. So, also, customary
laws, when specifically enacted to become part of statutory law, must first undergo that publication to
render them correspondingly binding and effective as such.
Undoubtedly, IPRA has several good points, and I would respectfully urge Congress to re-
examine the law. Indeed, the State is exhorted to protect the rights of indigenous cultural
communities to their ancestral lands, a task that would entail a balancing of interest between their
specific needs and the imperatives of national interest.
WHEREFORE, I vote to grant the petition.


SEPARATE OPINION
KAPUNAN, J .:
You ask if we own the land. . . How can you own that which will outlive you? Only the race own the land
because only the race lives forever. To claim a piece of land is a birthright of every man. The lowly
animals claim their place; how much more man? Man is born to live. Apu Kabunian, lord of us all, gave us
life and placed us in the world to live human lives. And where shall we obtain life? From the land. To work
(the land) is an obligation, not merely a right. In tilling the land, you possess it. And so land is a grace that
must be nurtured. To enrich it and make it fructify is the eternal exhortation of Apu Kabunian to all his
children. Land is sacred. Land is beloved. From its womb springs life.
- Macli-ing Dulag, Chieftain of the Kalinga Tribe (quoted in Ponciano L. Bennagen, "Tribal Filipinos" in
Indigenous View of Land and the Environment, ed. Shelton H. Davis, the World Bank Discussion Papers,
No. 188, pp. 71-72.)
It is established doctrine that a statute should be construed whenever possible in harmony with, rather
than in violation of, the Constitution.
1
The presumption is that the legislature intended to enact a valid,
sensible and just law and one which operates no further than may be necessary to effectuate the specific
purpose of the law.
2

The challenged provisions of the Indigenous Peoples Rights Act (IPRA) must be construed in view of
such presumption of constitutionality. Further, the interpretation of these provisions should take into
account the purpose of the law, which is to give life to the constitutional mandate that the rights of the
indigenous peoples be recognized and protected.
The struggle of our indigenous peoples to reclaim their ancestral lands and domains and therefore, their
heritage, is not unique. It is one that they share with the red-skinned "Indians" of the United States, with
the aborigines of Australia, the Maori of New Zealand and the Sazmi of Sweden, to name a few. Happily,
the nations in which these indigenous peoples live all have enacted measures in an attempt to heal an
oppressive past by the promise of a progressive future. Thus has the international community realized the
injustices that have been perpetrated upon the indigenous peoples. This sentiment among the family of
nations is expressed in a number of documents, the most recent and most comprehensive of which is the
Draft United Nations Declaration on the Rights of Indigenous Peoples which was adopted by the UN Sub-
Commission on Prevention of Discrimination and Protection of Minorities by its resolution on August 26,
1994. Among the rights recognized by the UN Draft is the restitution of lands, territories and even the
resources which the indigenous peoples have traditionally owned or otherwise occupied or used, and
which have been confiscated, occupied, used or damaged without the free and informed consent of the
indigenous peoples.
A Historical Backdrop on the Indigenous Peoples
The term "indigenous" traces its origin to the Old Latin word indu, meaning "within." In the sense the term
has come to be used, it is nearer in meaning to the Latin word indigenus, which means
"native."
3
"Indigenous" refers to that which originated or has been produced naturally in a particular land,
and has not been introduced from the outside.
4
In international law, the definition of what constitutes
"indigenous peoples" attains some degree of controversy. No definition of the term "indigenous peoples"
has been adopted by the United Nations (UN), although UN practice has been guided by a working
definition in the 1986 Report of UN Special Rapporteur Martinez Cobo:
5

Indigenous communities, peoples and nations are those which, having a historical continuity with pre-
invasion and pre-colonial societies that developed on their territories, consider themselves distinct from
other sections of the societies now prevailing in those territories, or parts of them. They form at present
non-dominant sections of society and are determined to preserve, develop and transmit to future
generations their ancestral territories, and their ethnic identity, as the basis of their continued existence as
peoples, in accordance with their own cultural patterns, social institutions and legal systems.
This historical continuity may consist of the continuation, for an extended period reaching into the present,
of one or more of the following factors:
(a) Occupation of ancestral lands, or at least of part of them;
(b) Common ancestry with the original occupants of these lands;
(c) Culture in general, or in specific manifestations (such as religion, living under a tribal system,
membership of an indigenous community, dress, means of livelihood, life-style, etc.);
(d) Language (whether used as the only language, as mother-tongue, as the habitual means of
communication at home or in the family, or as the main, preferred, habitual, general or normal
language);
(e) Residence in certain parts of the country; or in certain regions of the world;
(f) Other relevant facts.
6

In Philippine constitutional law, the term "indigenous peoples" pertains to those groups of Filipinos who
have retained a high degree of continuity from pre-Conquest culture.
7
Philippine legal history, however,
has not been kind to the indigenous peoples, characterized them as "uncivilized,"
8
"backward
people,"
9
with "barbarous practices"
10
and "a low order of intelligence."
11

Drawing inspiration from both our fundamental law and international law, IPRA now employs the
politically-correct conjunctive term "indigenous peoples/indigenous cultural communities" as follows:
Sec. 3. Definition of Terms.- For purposes of this Act, the following terms shall mean:
x x x
(h) Indigenous peoples/Indigenous cultural communities. - refer to a group of people or homogenous
societies identified by self-ascription and ascription by others, who have continuously lived as organized
community on communally bounded and defined territory, and who have, under claims of ownership since
time immemorial, occupied, possessed and utilized such territories, sharing common bonds of language,
customs, traditions, and other distinctive cultural traits, or who have, through resistance to political, social
and cultural inroads of colonization, non-indigenous religions and cultures, became historically
differentiated from the majority of Filipinos. Indigenous peoples shall likewise include peoples who are
regarded as indigenous on account of their descent from the populations which inhabited the country at
the time of conquest or colonization, or at the time of inroads of non-indigenous religions and cultures, or
the establishment of present State boundaries, who retain some or all of their own social, economic,
cultural and political institutions, but who may have been displaced from their traditional domains or who
may have resettled outside their ancestral domains x x x.
Long before the Spaniards set foot in these islands, the indigenous peoples were already plowing our soil
and hunting in our forests. The Filipinos of Aeta and Malay stock, who were the original inhabitants of our
archipelago, were, at that time, practicing a native culture. From the time the Spaniards arrived up to the
early part of the American regime,
12
these native inhabitants resisted foreign invasion, relentlessly fighting
for their lands. Today, from the remote uplands of Northern Luzon, to Palawan, Mindoro and Mindanao,
the indigenous peoples continue to live on and cultivate their ancestral lands, the lands of their
forefathers.
Though Filipinos today are essentially of the same stock as the indigenous peoples, our national culture
exhibits only the last vestiges of this native culture. Centuries of colonial rule and neocolonial domination
have created a discernible distinction between the cultural majority and the group of cultural
minorities.
13
The extant Philippine national culture is the culture of the majority; its indigenous roots were
replaced by foreign cultural elements that are decidedly pronounced, if not dominant.
14
While the culture
of the majority reoriented itself to Western influence, the culture of the minorities has retained its
essentially native character.
One of every six Filipinos is a member of an indigenous cultural community. Around twelve million
Filipinos are members of the one hundred and ten or so indigenous cultural communities,
15
accounting for
more than seventeen per centum of the estimated seventy million Filipinos
16
in our country. Sadly, the
indigenous peoples are one of the poorest sectors of Philippine society. The incidence of poverty and
malnutrition among them is significantly higher than the national average. The indigenous peoples are
also among the most powerless. Perhaps because of their inability to speak the language of law and
power, they have been relegated to the fringes of society. They have little, if any, voice in national politics
and enjoy the least protection from economic exploitation.
The Constitutional Policies on Indigenous Peoples
The framers of the 1987 Constitution, looking back to the long destitution of our less fortunate brothers,
fittingly saw the historic opportunity to actualize the ideals of people empowerment and social justice, and
to reach out particularly to the marginalized sectors of society, including the indigenous peoples. They
incorporated in the fundamental law several provisions recognizing and protecting the rights and interests
of the indigenous peoples, to wit:
Sec. 22. The State recognizes and promotes the rights of indigenous peoples within the framework of
national unity and development.
17

Sec. 5. The State, subject to the provisions of this Constitution and national development policies and
programs, shall protect the rights of indigenous cultural communities to their ancestral lands to ensure
their economic, social, and cultural well-being.
The Congress may provide for the applicability of customary laws governing property rights and relations
in determining the ownership and extent of ancestral domains.
18

Sec. 1. The Congress shall give the highest priority to the enactment of measures that protect and
enhance the right of all the people to human dignity, reduce social, economic and political inequalities,
and remove cultural inequities by equitably diffusing wealth and political power for the common good.
To this end, the State shall regulate the acquisition, ownership, use and disposition of property and its
increments.
19

Sec. 6. The State shall apply the principles of agrarian reform or stewardship, whenever applicable in
accordance with law, in the disposition and utilization of other natural resources, including lands of the
public domain under lease or concession, subject to prior rights, homestead rights of small settlers, and
the rights of indigenous communities to their ancestral lands.
20

Sec. 17. The State shall recognize, respect, and protect the rights of indigenous cultural communities to
preserve and develop their cultures, traditions, and institutions. It shall consider these rights in the
formulation of national plans and policies.
21

Sec. 12. The Congress may create a consultative body to advise the President on policies affecting
indigenous cultural communities, the majority of the members of which shall come from such
communities.
22

IPRA was enacted precisely to implement the foregoing constitutional provisions. It provides, among
others, that the State shall recognize and promote the rights of indigenous peoples within the framework
of national unity and development, protect their rights over the ancestral lands and ancestral domains and
recognize the applicability of customary laws governing property rights or relations in determining the
ownership and extent of the ancestral domains.
23
Moreover, IPRA enumerates the civil and political rights
of the indigenous peoples;
24
spells out their social and cultural rights;
25
acknowledges a general concept
of indigenous property right and recognizes title thereto;
26
and creates the NCIP as an independent
agency under the Office of the President.
27

Preliminary Issues
A. The petition presents an actual controversy.
The time-tested standards for the exercise of judicial review are: (1) the existence of an appropriate case;
(2) an interest personal and substantial by the party raising the constitutional question; (3) the plea that
the function be exercised at the earliest opportunity; and (4) the necessity that the constitutional question
be passed upon in order to decide the case.
28

Courts can only decide actual controversies, not hypothetical questions or cases.
29
The threshold issue,
therefore, is whether an "appropriate case" exists for the exercise of judicial review in the present case.
An "actual case or controversy" means an existing case or controversy which is both ripe for resolution
and susceptible of judicial determination, and that which is not conjectural or anticipatory,
30
or that which
seeks to resolve hypothetical or feigned constitutional problems.
31
A petition raising a constitutional
question does not present an "actual controversy," unless it alleges a legal right or power. Moreover, it
must show that a conflict of rights exists, for inherent in the term "controversy" is the presence of
opposing views or contentions.
32
Otherwise, the Court will be forced to resolve issues which remain
unfocused because they lack such concreteness provided when a question emerges precisely framed
from a clash of adversary arguments exploring every aspect of a multi-faceted situation embracing
conflicting and demanding interests.
33
The controversy must also be justiciable; that is, it must be
susceptible of judicial determination.
34

In the case at bar, there exists a live controversy involving a clash of legal rights. A law has been
enacted, and the Implementing Rules and Regulations approved. Money has been appropriated and the
government agencies concerned have been directed to implement the statute. It cannot be successfully
maintained that we should await the adverse consequences of the law in order to consider the
controversy actual and ripe for judicial resolution. It is precisely the contention of the petitioners that the
law, on its face, constitutes an unconstitutional abdication of State ownership over lands of the public
domain and other natural resources. Moreover, when the State machinery is set into motion to implement
an alleged unconstitutional statute, this Court possesses sufficient authority to resolve and prevent
imminent injury and violation of the constitutional process.
B. Petitioners, as citizens and taxpayers, have the requisite standing to raise the constitutional questions
herein.
In addition to the existence of an actual case or controversy, a person who assails the validity of a statute
must have a personal and substantial interest in the case, such that, he has sustained, or will sustain, a
direct injury as a result of its enforcement.
35
Evidently, the rights asserted by petitioners as citizens and
taxpayers are held in common by all the citizens, the violation of which may result only in a "generalized
grievance".
36
Yet, in a sense, all citizens and taxpayers suits are efforts to air generalized grievances
about the conduct of government and the allocation of power.
37

In several cases, the Court has adopted a liberal attitude with regard to standing.
38
The proper party
requirement is considered as merely procedural,
39
and the Court has ample discretion with regard
thereto.
40
As early as 1910, the Court in the case of Severino vs. Governor General
41
held:
x x x When the relief is sought merely for the protection of private rights, the relator must show some
personal or special interest in the subject matter, since he is regarded as the real party in interest and his
right must clearly appear. Upon the other hand, when the question is one of public right and the object
of the mandamus is to procure the enforcement of a public duty, the people are regarded as the real
party in interest, and the relator at whose instigation the proceedings are instituted need not show
that he has any legal or special interest in the result, it being sufficient to show that he is a citizen
and as such interested in the execution of the laws.
42

This Court has recognized that a "public right," or that which belongs to the people at large, may also be
the subject of an actual case or controversy. In Severino, we ruled that a private citizen may enforce a
"public right" in behalf of other citizens. We opined therein that:
The right which [petitioner] seeks to enforce is not greater or different from that of any other qualified
elector in the municipality of Silay. It is also true that the injury which he would suffer in case he fails to
obtain the relief sought would not be greater or different from that of the other electors; but he is seeking
to enforce a public right as distinguished from a private right. The real party in interest is the public,
or the qualified electors of the town of Silay. Each elector has the same right and would suffer the
same injury. Each elector stands on the same basis with reference to maintaining a
petition whether or not the relief sought by the relator should be granted.
43

In Taada v. Tuvera,
44
the Court enforced the "public right" to due process and to be informed of matters
of public concern.
In Garcia vs. Board of Investments,
45
the Court upheld the "public right" to be heard or consulted on
matters of national concern.
In Oposa v. Factoran,
46
the Court recognized the "public right" of citizens to "a balanced and healthful
ecology which, for the first time in our nations constitutional history, is solemnly incorporated in the
fundamental law."
47
Mr. Justice (now Chief Justice) Hilario G. Davide, Jr., delivering the opinion of the
Court, stated that:
Such a right belongs to a different category of rights altogether for it concerns nothing less than self-
preservation and self-perpetuation-aptly and fittingly stressed by petitioners-the advancement of which
may even be said to predate all governments and constitutions. As a matter of fact, these basic rights
need not even be written in the Constitution for they are assumed to exist from the inception of
humankind.
48

Petitioners, as citizens, possess the "public right" to ensure that the national patrimony is not alienated
and diminished in violation of the Constitution. Since the government, as the guardian of the national
patrimony, holds it for the benefit of all Filipinos without distinction as to ethnicity, it follows that a citizen
has sufficient interest to maintain a suit to ensure that any grant of concessions covering the national
economy and patrimony strictly complies with constitutional requirements. Thus, the preservation of the
integrity and inviolability of the national patrimony is a proper subject of a citizens suit.
In addition, petitioners, as taxpayers, possess the right to restrain officials from wasting public funds
through the enforcement of an unconstitutional statute. It is well-settled that a taxpayer has the right to
enjoin public officials from wasting public funds through the implementation of an unconstitutional
statute,
49
and by necessity, he may assail the validity of a statute appropriating public funds.
50
The
taxpayer has paid his taxes and contributed to the public coffers and, thus, may inquire into the manner
by which the proceeds of his taxes are spent. The expenditure by an official of the State for the purpose
of administering an invalid law constitutes a misapplication of such funds.
51

The IPRA appropriates funds as indicated in its title: "An Act to Recognize, Protect and Promote the
Rights of Indigenous Cultural Communities/Indigenous Peoples, Creating the National Commission on
Indigenous Peoples, Establishing Implementing Mechanisms, Appropriating Funds Therefor, and for
Other Purposes." In the same manner, Section 79 authorizes for the expenditure of public funds by
providing that "the amount necessary to finance [its] initial implementation shall be charged against the
current year's appropriation for the Office for Northern Cultural Communities (the "ONCC") and the Office
for Southern Cultural Communities (the "OSCC"),"
52
which were merged as organic offices of the
NCIP.
53
Thus, the IPRA is a valid subject of a taxpayers suit.
C. The petition for prohibition and mandamus is not an improper remedy.
Prohibition is an extraordinary writ directed against any tribunal, corporation, board, officer or person,
whether exercising judicial, quasi-judicial or ministerial functions, ordering said entity or person to desist
from further proceedings when said proceedings are without or in excess of said entitys or persons
jurisdiction, or are accompanied with grave abuse of discretion, and there is no appeal or any other plain,
speedy and adequate remedy in the ordinary course of law.
54
Mandamus, on the other hand, is an
extraordinary writ commanding a tribunal, corporation, board, officer or person, immediately or at some
other specified time, to do the act required to be done, when said entity or person unlawfully neglects the
performance of an act which the law specifically enjoins as a duty resulting from an office, trust or station,
or when said entity or person unlawfully excludes another from the use and enjoyment of a right or office
to which such other is entitled, and there is no other plain, speedy and adequate remedy in the ordinary
course of law.
55

In this case, the petitioners pray that respondents be restrained from implementing the challenged
provisions of the IPRA and its Implementing Rules and the assailed DENR Circular No. 2, series of 1998,
and that the same officials be enjoined from disbursing public funds for the implementation of the said law
and rules. They further ask that the Secretary of the DENR be compelled to perform his duty to control
and supervise the activities pertaining to natural resources.
Prohibition will lie to restrain the public officials concerned from implementing the questioned provisions of
the IPRA and from disbursing funds in connection therewith if the law is found to be unconstitutional.
Likewise,mandamus will lie to compel the Secretary of the DENR to perform his duty to control and
supervise the exploration, development, utilization and conservation of the countrys natural resources.
Consequently, the petition for prohibition and mandamus is not an improper remedy for the relief sought.
D. Notwithstanding the failure of petitioners to observe the hierarchy of courts, the Court assumes
jurisdiction over the petition in view of the importance of the issues raised therein.
Between two courts of concurrent original jurisdiction, it is the lower court that should initially pass upon
the issues of a case. That way, as a particular case goes through the hierarchy of courts, it is shorn of all
but the important legal issues or those of first impression, which are the proper subject of attention of the
appellate court. This is a procedural rule borne of experience and adopted to improve the administration
of justice.
This Court has consistently enjoined litigants to respect the hierarchy of courts. Although this Court has
concurrent jurisdiction with the Regional Trial Courts and the Court of Appeals to issue writs of certiorari,
prohibition, mandamus, quo warranto, habeas corpus and injunction,
56
such concurrence does not give a
party unrestricted freedom of choice of court forum. The resort to this Courts primary jurisdiction to issue
said writs shall be allowed only where the redress desired cannot be obtained in the appropriate courts or
where exceptional and compelling circumstances justify such invocation.
57
We held in People v.
Cuaresma
58
that:
A becoming regard for judicial hierarchy most certainly indicates that petitions for the issuance of
extraordinary writs against first level ("inferior") courts should be filed with the Regional Trial Court, and
those against the latter, with the Court of Appeals. A direct invocation of the Supreme Courts original
jurisdiction to issue these writs should be allowed only where there are special and important
reasons therefor, clearly and specifically set out in the petition. This is established policy. It is a
policy necessary to prevent inordinate demands upon the Courts time and attention which are better
devoted to those matters within its exclusive jurisdiction, and to prevent further over-crowding of the
Courts docket x x x.
59
(Emphasis supplied.)
IPRA aims to rectify the historical injustice inflicted upon indigenous peoples. Its impact upon the lives not
only of the indigenous peoples but also upon the lives of all Filipinos cannot be denied. The resolution of
this case by the Court at the earliest opportunity is necessary if the aims of the law are to be achieved.
This reason is compelling enough to allow petitioners invocation of this Courts jurisdiction in the first
instance.
Substantive Issues
Primary Issue
The issue of prime concern raised by petitioners and the Solicitor General revolves around the
constitutionality of certain provisions of IPRA, specifically Sections 3(a), 3(b), 5, 6, 7, 8, 57, 58 and 59.
These provisions allegedly violate Section 2, Article XII of the Constitution, which states:
Sec. 2. All lands of the public domain, waters, minerals, coal, petroleum, and other mineral oils, all forces
of potential energy, fisheries, forests or timber, wildlife, flora and fauna, and other natural resources are
owned by the State. With the exception of agricultural lands, all other natural resources shall not be
alienated. The exploration, development, and utilization of natural resources shall be under the full control
and supervision of the State. The State may directly undertake such activities, or it may enter into co-
production, joint venture, or production-sharing agreements with Filipino citizens, or corporations or
associations at least sixty per centum of whose capital is owned by such citizens. Such agreements may
be for a period not exceeding twenty-five years, renewable for not more than twenty-five years, and under
such terms and conditions as may be provided by law. In cases of water rights for irrigation, water supply,
fisheries, or industrial uses other than the development of water power, beneficial use may be the
measure and limit of the grant.
The State shall protect the nations marine wealth in its archipelagic waters, territorial sea, and exclusive
economic zone, and reserve its use and enjoyment exclusively to Filipino citizens.
The Congress, may, by law, allow small-scale utilization of natural resources by Filipino citizens, as well
as cooperative fish farming, with priority to subsistence fishermen and fishworkers in rivers, lakes, bays
and lagoons.
The President may enter into agreements with foreign-owned corporations involving either technical or
financial assistance for large-scale exploration, development and utilization of minerals, petroleum, and
other mineral oils according to the general terms and conditions provided by law, based on real
contributions to the economic growth and general welfare of the country. In such agreements, the State
shall promote the development and use of local scientific and technical resources.
The President shall notify the Congress of every contract entered into in accordance with this provision,
within thirty days from its execution.
Under IPRA, indigenous peoples may obtain the recognition of their right of ownership
60
over ancestral
lands and ancestral domains by virtue of native title.
61
The term "ancestral lands" under the statute
refers to landsoccupied by individuals, families and clans who are members of indigenous cultural
communities, including residential lots, rice terraces or paddies, private forests, swidden farms and tree
lots. These lands are required to have been "occupied, possessed and utilized" by them or through their
ancestors "since time immemorial, continuously to the present".
62
On the other hand, "ancestral
domains" is defined as areas generally belonging to indigenous cultural communities, including ancestral
lands, forests, pasture, residential and agricultural lands, hunting grounds, worship areas, and lands no
longer occupied exclusively by indigenous cultural communities but to which they had traditional access,
particularly the home ranges of indigenous cultural communities who are still nomadic or shifting
cultivators. Ancestral domains also include inland waters, coastal areas and natural resources
therein.
63
Again, the same are required to have been "held under a claim of ownership, occupied or
possessed by ICCs/IPs, by themselves or through their ancestors, communally or individually since time
immemorial, continuously to the present".
64
Under Section 56, property rights within the ancestral
domains already existing and/or vested upon effectivity of said law "shall be recognized and respected."
Ownership is the crux of the issue of whether the provisions of IPRA pertaining to ancestral lands,
ancestral domains, and natural resources are unconstitutional. The fundamental question is, who,
between the State and the indigenous peoples, are the rightful owners of these properties?
It bears stressing that a statute should be construed in harmony with, and not in violation, of the
fundamental law.
65
The reason is that the legislature, in enacting a statute, is assumed to have acted
within its authority and adhered to the constitutional limitations. Accordingly, courts should presume that it
was the intention of the legislature to enact a valid, sensible, and just law and one which operates no
further than may be necessary to effectuate the specific purpose of the law.
66

A. The provisions of IPRA recognizing the ownership of indigenous peoples over the ancestral lands and
ancestral domains are not unconstitutional.
In support of their theory that ancestral lands and ancestral domains are part of the public domain and,
thus, owned by the State, pursuant to Section 2, Article XII of the Constitution, petitioners and the
Solicitor General advance the following arguments:
First, according to petitioners, the King of Spain under international law acquired exclusive dominion over
the Philippines by virtue of discovery and conquest. They contend that the Spanish King under the theory
of jura regalia, which was introduced into Philippine law upon Spanish conquest in 1521, acquired title to
all the lands in the archipelago.
Second, petitioners and the Solicitor General submit that ancestral lands and ancestral domains are
owned by the State. They invoke the theory of jura regalia which imputes to the State the ownership of all
lands and makes the State the original source of all private titles. They argue that the Philippine State, as
successor to Spain and the United States, is the source of any asserted right of ownership in land.
Third, petitioners and the Solicitor General concede that the Cario doctrine exists. However, petitioners
maintain that the doctrine merely states that title to lands of the public domain may be acquired by
prescription. The Solicitor General, for his part, argues that the doctrine applies only to alienable lands of
the public domain and, thus, cannot be extended to other lands of the public domain such as forest or
timber, mineral lands, and national parks.
Fourth, the Solicitor General asserts that even assuming that native title over ancestral lands and
ancestral domains existed by virtue of the Cario doctrine, such native title was extinguished upon the
ratification of the 1935 Constitution.
Fifth, petitioners admit that Congress is mandated under Section 5, Article XII of the Constitution to
protect that rights of indigenous peoples to their ancestral lands and ancestral domains. However, they
contend that the mandate is subject to Section 2, Article XII and the theory of jura regalia embodied
therein. According to petitioners, the recognition and protection under R.A. 8371 of the right of ownership
over ancestral lands and ancestral domains is far in excess of the legislative power and constitutional
mandate of Congress.
Finally, on the premise that ancestral lands and ancestral domains are owned by the State, petitioners
posit that R.A. 8371 violates Section 2, Article XII of the Constitution which prohibits the alienation of non-
agricultural lands of the public domain and other natural resources.
I am not persuaded by these contentions.
Undue reliance by petitioners and the Solicitor General on the theory of jura regalia is understandable.
Not only is the theory well recognized in our legal system; it has been regarded, almost with reverence,
as the immutable postulate of Philippine land law. It has been incorporated into our fundamental law and
has been recognized by the Court.
67

Generally, under the concept of jura regalia, private title to land must be traced to some grant, express or
implied, from the Spanish Crown or its successors, the American Colonial government, and thereafter, the
Philippine Republic. The belief that the Spanish Crown is the origin of all land titles in the Philippines has
persisted because title to land must emanate from some source for it cannot issue forth from nowhere.
68

In its broad sense, the term "jura regalia" refers to royal rights,
69
or those rights which the King has by
virtue of his prerogatives.
70
In Spanish law, it refers to a right which the sovereign has over anything in
which a subject has a right of property or propriedad.
71
These were rights enjoyed during feudal times by
the king as the sovereign.
The theory of the feudal system was that title to all lands was originally held by the King, and while the
use of lands was granted out to others who were permitted to hold them under certain conditions, the
King theoretically retained the title.
72
By fiction of law, the King was regarded as the original proprietor of
all lands, and the true and only source of title, and from him all lands were held.
73
The theory of jura
regalia was therefore nothing more than a natural fruit of conquest.
74

The Regalian theory, however, does not negate native title to lands held in private ownership since time
immemorial. In the landmark case of Cario vs. Insular Government
75
the United States Supreme Court,
reversing the decision
76
of the pre-war Philippine Supreme Court, made the following pronouncement:
x x x Every presumption is and ought to be taken against the Government in a case like the present. It
might, perhaps, be proper and sufficient to say that when, as far back as testimony or memory goes,
the land has been held by individuals under a claim of private ownership, it will be presumed to
have been held in the same way from before the Spanish conquest, and never to have been public
land. x x x.
77
(Emphasis supplied.)
The above ruling institutionalized the recognition of the existence of native title to land, or ownership of
land by Filipinos by virtue of possession under a claim of ownership since time immemorial and
independent of any grant from the Spanish Crown, as an exception to the theory of jura regalia.
In Cario, an Igorot by the name of Mateo Cario applied for registration in his name of an ancestral land
located in Benguet. The applicant established that he and his ancestors had lived on the land, had
cultivated it, and had used it as far they could remember. He also proved that they had all been
recognized as owners, the land having been passed on by inheritance according to native custom.
However, neither he nor his ancestors had any document of title from the Spanish Crown. The
government opposed the application for registration, invoking the theory of jura regalia. On appeal, the
United States Supreme Court held that the applicant was entitled to the registration of his native title to
their ancestral land.
Cario was decided by the U.S. Supreme Court in 1909, at a time when decisions of the U.S. Court were
binding as precedent in our jurisdiction.
78
We applied the Cario doctrine in the 1946 case of Oh Cho vs.
Director of Lands,
79
where we stated that "[a]ll lands that were not acquired from the Government either
by purchase or by grant, belong to the public domain, but [a]n exception to the rule would be any land that
should have been in the possession of an occupant and of his predecessors in interest since time
immemorial, for such possession would justify the presumption that the land had never been part of the
public domain or that it had been private property even before the Spanish conquest."
80

Petitioners however aver that the U.S. Supreme Courts ruling in Cario was premised on the fact that the
applicant had complied with the requisites of acquisitive prescription, having established that he and his
predecessors-in-interest had been in possession of the property since time immemorial. In effect,
petitioners suggest that title to the ancestral land applied for by Cario was transferred from the State, as
original owner, to Cario by virtue of prescription. They conclude that the doctrine cannot be the basis for
decreeing "by mere legislative fiatthat ownership of vast tracts of land belongs to [indigenous peoples]
without judicial confirmation."
81

The Solicitor General, for his part, claims that the Cario doctrine applies only to alienable lands of the
public domain and, as such, cannot be extended to other lands of the public domain such as forest or
timber, mineral lands, and national parks.
There is no merit in these contentions.
A proper reading of Cario would show that the doctrine enunciated therein applies only to lands which
have always been considered as private, and not to lands of the public domain, whether alienable or
otherwise. A distinction must be made between ownership of land under native title and ownership by
acquisitive prescription against the State. Ownership by virtue of native title presupposes that the land
has been held by its possessor and his predecessors-in-interest in the concept of an owner since time
immemorial. The land is not acquired from the State, that is, Spain or its successors-in-interest, the
United States and the Philippine Government. There has been no transfer of title from the State as the
land has been regarded as private in character as far back as memory goes. In contrast, ownership of
land by acquisitive prescription against the State involves a conversion of the character of the property
from alienable public land to private land, which presupposes a transfer of title from the State to a private
person. Since native title assumes that the property covered by it is private land and is deemed never to
have been part of the public domain, the Solicitor Generals thesis that native title under Carioapplies
only to lands of the public domain is erroneous. Consequently, the classification of lands of the public
domain into agricultural, forest or timber, mineral lands, and national parks under the Constitution
82
is
irrelevant to the application of the Cario doctrine because the Regalian doctrine which vests in the State
ownership of lands of the public domain does not cover ancestral lands and ancestral domains.
Legal history supports the Cario doctrine.
When Spain acquired sovereignty over the Philippines by virtue of its discovery and occupation thereof in
the 16th century and the Treaty of Tordesillas of 1494 which it entered into with Portugal,
83
the continents
of Asia, the Americas and Africa were considered as terra nullius although already populated by other
peoples.
84
The discovery and occupation by the European States, who were then considered as the only
members of the international community of civilized nations, of lands in the said continents were deemed
sufficient to create title under international law.
85

Although Spain was deemed to have acquired sovereignty over the Philippines, this did not mean that it
acquired title to all lands in the archipelago. By virtue of the colonial laws of Spain, the Spanish Crown
was considered to have acquired dominion only over the unoccupied and unclaimed portions of our
islands.
86

In sending the first expedition to the Philippines, Spain did not intend to deprive the natives of their
property. Miguel Lopez de Legazpi was under instruction of the Spanish King to do no harm to the natives
and to their property. In this regard, an authority on the early Spanish colonial period in the Philippines
wrote:
The government of [the King of Spain] Philip II regarded the Philippines as a challenging opportunity to
avoid a repetition of the sanguinary conquests of Mexico and Peru. In his written instructions for
the Adelantado Legazpi, who commanded the expedition, Philip II envisaged a bloodless pacification of
the archipelago. This extraordinary document could have been lifted almost verbatim from the lectures of
the Dominican theologian, Francisco de Vitoria, delivered in the University of Salamanca. The King
instructed Legazpi to inform the natives that the Spaniards had come to do no harm to their persons or to
their property. The Spaniards intended to live among them in peace and in friendship and "to explain to
them the law of Jesus Christ by which they will be saved." Although the Spanish expedition could defend
themselves if attacked, the royal instructions admonished the commander to commit no aggressive act
which might arouse native hostility.
87

Spanish colonial laws recognized and respected Filipino landholdings including native land
occupancy.
88
Thus, the Recopilacin de Leyes de las Indias expressly conferred ownership of lands
already held by the natives.
89
The royal decrees of 1880 and 1894 did not extinguish native title to land in
the Philippines. The earlier royal decree, dated June 25, 1880, provided that all those in "unlawful
possession of royal lands" must legalize their possession by means of adjustment proceedings,
90
and
within the period specified. The later royal decree, dated February 13, 1894, otherwise known as the
Maura Law, declared that titles that were capable of adjustment under the royal decree of 1880, but for
which adjustment was not sought, were forfeited. Despite the harsh wording of the Maura Law, it was
held in the case of Cario that the royal decree of 1894 should not be construed as confiscation of title,
but merely as the withdrawal of the privilege of registering such title.
91

Neither was native title disturbed by the Spanish cession of the Philippines to the United States, contrary
to petitioners assertion that the US merely succeeded to the rights of Spain, including the latters rights
over lands of the public domain.
92
Under the Treaty of Paris of December 10, 1898, the cession of the
Philippines did not impair any right to property existing at the time.
93
During the American colonial regime,
native title to land was respected, even protected. The Philippine Bill of 1902 provided that property and
rights acquired by the US through cession from Spain were to be administered for the benefit of the
Filipinos.
94
In obvious adherence to libertarian principles, McKinleys Instructions, as well as the Philippine
Bill of 1902, contained a bill of rights embodying the safeguards of the US Constitution. One of these
rights, which served as an inviolable rule upon every division and branch of the American colonial
government in the Philippines,
95
was that "no person shall be deprived of life, liberty, or property without
due process of law."
96
These vested rights safeguarded by the Philippine Bill of 1902 were in turn
expressly protected by the due process clause of the 1935 Constitution. Resultantly, property rights of the
indigenous peoples over their ancestral lands and ancestral domains were firmly established in law.
Nonetheless, the Solicitor General takes the view that the vested rights of indigenous peoples to their
ancestral lands and domains were "abated by the direct act by the sovereign Filipino people of ratifying
the 1935 Constitution."
97
He advances the following arguments:
The Sovereign, which is the source of all rights including ownership, has the power to restructure the
consolidation of rights inherent in ownership in the State. Through the mandate of the Constitutions that
have been adopted, the State has wrested control of those portions of the natural resources it deems
absolutely necessary for social welfare and existence. It has been held that the State may impair vested
rights through a legitimate exercise of police power.
Vested rights do not prohibit the Sovereign from performing acts not only essential to but determinative of
social welfare and existence. To allow otherwise is to invite havoc in the established social system. x x x
Time-immemorial possession does not create private ownership in cases of natural resources that have
been found from generation to generation to be critical to the survival of the Sovereign and its agent, the
State.
98

Stated simply, the Solicitor Generals argument is that the State, as the source of all titles to land, had the
power to re-vest in itself, through the 1935 Constitution, title to all lands, including ancestral lands and
ancestral domains. While the Solicitor General admits that such a theory would necessarily impair vested
rights, he reasons out that even vested rights of ownership over ancestral lands and ancestral domains
are not absolute and may be impaired by the legitimate exercise of police power.
I cannot agree. The text of the provision of the 1935 Constitution invoked by the Solicitor General, while
embodying the theory of jura regalia, is too clear for any misunderstanding. It simply declares that "all
agricultural, timber, and mineral lands of the public domain, waters, minerals, coal, petroleum, and other
mineral oils, all forces of potential energy, and other natural resources of the Philippines belong to the
State."
99
Nowhere does it state that certain lands which are "absolutely necessary for social welfare and
existence," including those which are notpart of the public domain, shall thereafter be owned by the
State. If there is any room for constitutional construction, the provision should be interpreted in favor of
the preservation, rather than impairment or extinguishment, of vested rights. Stated otherwise, Section 1,
Article XII of the 1935 Constitution cannot be construed to mean that vested right which had existed then
were extinguished and that the landowners were divested of their lands, all in the guise of "wrest[ing]
control of those portions of the natural resources [which the State] deems absolutely necessary for social
welfare and existence." On the contrary, said Section restated the fundamental rule against the diminution
of existing rights by expressly providing that the ownership of lands of the public domain and other natural
resources by the State is "subject to any existing right, grant, lease, or concessions." The "existing rights"
that were intended to be protected must, perforce, include the right of ownership by indigenous peoples
over their ancestral lands and domains. The words of the law should be given their ordinary or usual
meaning,
100
and the term "existing rights" cannot be assigned an unduly restrictive definition.
Petitioners concede that Congress is mandated under Section 5, Article XII of the 1987 Constitution
101
to
protect the rights of indigenous peoples to their ancestral lands and ancestral domains. Nonetheless, they
contend that the recognition and protection under IPRA of the right of ownership of indigenous peoples
over ancestral lands and ancestral domains are far in excess of the legislative power and constitutional
mandate of the Congress,
102
since such recognition and protection amount to the alienation of lands of the
public domain, which is proscribed under Section 2, Article XII of the Constitution.
Section 5, Article XII of the Constitution expresses the sovereign intent to "protect the rights of
indigenous peoples to their ancestral lands." In its general and ordinary sense, the term "right" refers to
any legally enforceable claim.
103
It is a power, privilege, faculty or demand inherent in one person and
incident upon another.
104
When used in relation to property, "right" includes any interest in or title to an
object, or any just and legal claim to hold, use and enjoy it.
105
Said provision in the Constitution cannot, by
any reasonable construction, be interpreted to exclude the protection of the right of ownership over
such ancestral lands. For this reason, Congress cannot be said to have exceeded its constitutional
mandate and power in enacting the provisions of IPRA, specifically Sections 7(a) and 8, which recognize
the right of ownership of the indigenous peoples over ancestral lands.
The second paragraph of Section 5, Article XII also grants Congress the power to "provide for the
applicability of customary laws governing property rights or relations in determining the ownership and
extent of ancestral domains." In light of this provision, does Congress have the power to decide whether
ancestral domains shall be private property or part of the public domain? Also, does Congress have the
power to determine whether the "extent" of ancestral domains shall include the natural resources found
therein?
It is readily apparent from the constitutional records that the framers of the Constitution did not intend
Congress to decide whether ancestral domains shall be public or private property. Rather, they
acknowledged that ancestral domains shall be treated as private property, and that customary laws shall
merely determine whether such private ownership is by the entire indigenous cultural community, or by
individuals, families, or clans within the community. The discussion below between Messrs. Regalado and
Bennagen and Mr. Chief Justice Davide, then members of the 1986 Constitutional Commission, is
instructive:
MR. REGALADO. Thank you, Madame President. May I seek some clarifications from either
Commissioner Bennagen or Commissioner Davide regarding this phrase "CONGRESS SHALL PROVIDE
FOR THE APPLICABILITY OF CUSTOMARY LAWS GOVERNING PROPERTY RIGHTS OR
RELATIONS in determining the ownership and extent of the ancestral domain," because ordinarily it is
the law on ownership and the extent thereof which determine the property rights or relations arising
therefrom. On the other hand, in this proposed amendment the phraseology is that it is the property rights
or relations which shall be used as the basis in determining the ownership and extent of the ancestral
domain. I assume there must be a certain difference in the customary laws and our regular civil laws on
property.
MR. DAVIDE. That is exactly the reason, Madam President, why we will leave it to Congress to make the
necessary exception to the general law on property relations.
MR. REGALADO. I was thinking if Commissioner Bennagen could give us an example of such a
customary law wherein it is the property rights and relations that determine the ownership and the extent
of that ownership, unlike the basic fundamental rule that it is the ownership and the extent of ownership
which determine the property rights and relations arising therefrom and consequent thereto. Perhaps,
these customary laws may have a different provision or thrust so that we could make the corresponding
suggestions also by way of an amendment.
MR. DAVIDE. That is exactly my own perception.
MR. BENNAGEN. Let me put it this way.
There is a range of customary laws governing certain types of ownership. There would be ownership
based on individuals, on clan or lineage, or on community. And the thinking expressed in the
consultation is that this should be codified and should be recognized in relation to existing national laws.
That is essentially the concept.
106
(Emphasis supplied.)
The intention to treat ancestral domains as private property is also apparent from the following exchange
between Messrs. Suarez and Bennagen:
MR. SUAREZ. When we speak of customary laws governing property rights or relations in determining
the ownership and extent of the ancestral domain, are we thinking in terms of the tribal ownership or
community ownership or of private ownership within the ancestral lands or ancestral domain?
MR. BENNAGEN. The concept of customary laws is that it is considered as ownership by private
individuals, clans and even communities.
MR. SUAREZ. So, there will be two aspects to this situation. This means that the State will set aside the
ancestral domain and there is a separate law for that. Within the ancestral domain it could accept more
specific ownership in terms of individuals within the ancestral lands.
MR. BENNAGEN. Individuals and groups within the ancestral domain.
107
(Emphasis supplied.)
It cannot be correctly argued that, because the framers of the Constitution never expressly
mentioned Cario in their deliberations, they did not intend to adopt the concept of native title to land, or
that they were unaware of native title as an exception to the theory of jura regalia.
108
The framers of the
Constitution, as well as the people adopting it, were presumed to be aware of the prevailing judicial
doctrines concerning the subject of constitutional provisions, and courts should take these doctrines into
consideration in construing the Constitution.
109

Having thus recognized that ancestral domains under the Constitution are considered as private property
of indigenous peoples, the IPRA, by affirming or acknowledging such ownership through its various
provisions, merely abides by the constitutional mandate and does not suffer any vice of
unconstitutionality.
Petitioners interpret the phrase "subject to the provisions of this Constitution and national development
policies and programs" in Section 5, Article XII of the Constitution to mean "as subject to the provision of
Section 2, Article XII of the Constitution," which vests in the State ownership of all lands of the public
domain, mineral lands and other natural resources. Following this interpretation, petitioners maintain that
ancestral lands and ancestral domains are the property of the State.
This proposition is untenable. Indeed, Section 2, Article XII reiterates the declarations made in the 1935
and 1973 Constitutions on the state policy of conservation and nationalization of lands of the public
domain and natural resources, and is of paramount importance to our national economy and patrimony. A
close perusal of the records of the 1986 Constitutional Commission reveals that the framers of the
Constitution inserted the phrase "subject to the provisions of this Constitution" mainly to prevent the
impairment of Torrens titles and other prior rights in the determination of what constitutes ancestral lands
and ancestral domains, to wit:
MR. NATIVIDAD. Just one question. I want to clear this section protecting ancestral lands. How does this
affect the Torrens title and other prior rights?
MR. BENNAGEN. I think that was also discussed in the committee hearings and we did say that in cases
where due process is clearly established in terms of prior rights, these two have to be respected.
MR. NATIVIDAD. The other point is: How vast is this ancestral land? Is it true that parts of Baguio City are
considered as ancestral lands?
MR. BENNAGEN. They could be regarded as such. If the Commissioner still recalls, in one of the
publications that I provided the Commissioners, the parts could be considered as ancestral domain in
relation to the whole population of Cordillera but not in relation to certain individuals or certain groups.
MR. NATIVIDAD. The Commissioner means that the whole Baguio City is considered as ancestral land?
MR. BENNAGEN. Yes, in the sense that it belongs to Cordillera or in the same manner that Filipinos can
speak of the Philippine archipelago as ancestral land, but not in terms of the right of a particular person or
particular group to exploit, utilize, or sell it.
MR. NATIVIDAD. But is clear that the prior rights will be respected.
MR. BENNAGEN. Definitely.
110

Thus, the phrase "subject to the provisions of this Constitution" was intended by the framers of the
Constitution as a reiteration of the constitutional guarantee that no person shall be deprived of property
without due process of law.
There is another reason why Section 5 of Article XII mandating the protection of rights of the indigenous
peoples to their ancestral lands cannot be construed as subject to Section 2 of the same Article ascribing
ownership of all public lands to the State. The Constitution must be construed as a whole. It is a rule that
when construction is proper, the whole Constitution is examined in order to determine the meaning of any
provision. That construction should be used which would give effect to the entire instrument.
111

Thus, the provisions of the Constitution on State ownership of public lands, mineral lands and other
natural resources should be read together with the other provisions thereof which firmly recognize the
rights of the indigenous peoples. These, as set forth hereinbefore,
112
include: Section 22, Article II,
providing that the State recognizes and promotes the rights of indigenous peoples within the framework of
national unity and development;Section 5, Article XII, calling for the protection of the rights of indigenous
cultural communities to their ancestral lands to ensure their economic, social, and cultural well-being, and
for the applicability of customary laws governing property rights and relations in determining the
ownership and extent of ancestral domains; Section 1, Article XIII, directing the removal or reduction of
social, economic, political and cultural inequities and inequalities by equitably diffusing wealth and
political power for the common good; Section 6, Article XIII, directing the application of the principles of
agrarian reform or stewardship in the disposition and utilization of other natural resources, subject to prior
rights, homestead rights of small settlers, and the rights of indigenous communities to their ancestral
lands; Section 17, Article XIV, decreeing that the State shall recognize, respect, and protect the rights of
indigenous cultural communities to preserve and develop their cultures, traditions, and institutions;
andSection 12, Article XVI, authorizing the Congress to create a consultative body to advise the
President on policies affecting indigenous cultural communities.
Again, as articulated in the Constitution, the first goal of the national economy is the more equitable
distribution of opportunities, income, and wealth.
113
Equity is given prominence as the first objective
of national economic development.
114
The framers of the Constitution did not, by the phrase "subject to
the provisions of this Constitution and national development policies and programs," intend to establish a
hierarchy of constitutional norms. As explained by then Commissioner (now Chief Justice) Hilario G.
Davide, Jr., it was not their objective to make certain interests primary or paramount, or to create absolute
limitations or outright prohibitions; rather, the idea is towards the balancing of interests:
BISHOP BACANI. In Commissioner Davides formulation of the first sentence, he says: "The State,
SUBJECT TO THE provisions of this Constitution AND NATIONAL DEVELOPMENT POLICIES AND
PROGRAMS shall guarantee the rights of cultural or tribal communities to their ancestral lands to insure
their economic, social and cultural well-being." There are at least two concepts here which receive
different weights very often. They are the concepts of national development policies and programs, and
the rights of cultural or tribal communities to their ancestral lands, et cetera. I would like to ask: When the
Commissioner proposed this amendment, which was the controlling concept? I ask this because
sometimes the rights of cultural minorities are precisely transgressed in the interest of national
development policies and programs. Hence, I would like to know which is the controlling concept here. Is
it the rights of indigenous peoples to their ancestral lands or is it national development policies and
programs.
MR. DAVIDE. It is not really a question of which is primary or which is more paramount. The
concept introduced here is really the balancing of interests. That is what we seek to attain. We have
to balance the interests taking into account the specific needs and the specific interests also of these
cultural communities in like manner that we did so in the autonomous regions.
115
(Emphasis supplied.)
B. The provisions of R.A. 8371 do not infringe upon the States ownership over the natural resources
within the ancestral domains.
Petitioners posit that IPRA deprives the State of its ownership over mineral lands of the public domain
and other natural resources,
116
as well as the States full control and supervision over the exploration,
development and utilization of natural resources.
117
Specifically, petitioners and the Solicitor General
assail Sections 3 (a),
118
5,
119
and 7
120
of IPRA as violative of Section 2, Article XII of the Constitution which
states, in part, that "[a]ll lands of the public domain, waters, minerals, coal, petroleum, and other mineral
oils, all forces of potential energy, fisheries, forests or timber, wildlife, flora and fauna, and other natural
resources are owned by the State."
121
They would have the Court declare as unconstitutional Section 3(a)
of IPRA because the inclusion of natural resources in the definition of ancestral domains purportedly
results in the abdication of State ownership over these resources.
I am not convinced.
Section 3(a) merely defines the coverage of ancestral domains, and describes the extent, limit and
composition of ancestral domains by setting forth the standards and guidelines in determining whether a
particular area is to be considered as part of and within the ancestral domains. In other words, Section
3(a) serves only as a yardstick which points out what properties are within the ancestral domains. It does
not confer or recognize any right of ownership over the natural resources to the indigenous peoples. Its
purpose is definitional and not declarative of a right or title.
The specification of what areas belong to the ancestral domains is, to our mind, important to ensure that
no unnecessary encroachment on private properties outside the ancestral domains will result during the
delineation process. The mere fact that Section 3(a) defines ancestral domains to include the natural
resources found therein does not ipso facto convert the character of such natural resources as private
property of the indigenous peoples. Similarly, Section 5 in relation to Section 3(a) cannot be construed as
a source of ownership rights of indigenous people over the natural resources simply because it
recognizes ancestral domains as their "private but community property."
The phrase "private but community property" is merely descriptive of the indigenous peoples concept of
ownership as distinguished from that provided in the Civil Code. In Civil Law, "ownership" is the
"independent and general power of a person over a thing for purposes recognized by law and within the
limits established thereby."
122
The civil law concept of ownership has the following attributes: jus utendi or
the right to receive from the thing that which it produces, jus abutendi or the right to consume the thing by
its use, jus disponendi or the power to alienate, encumber, transform and even destroy that which is
owned and jus vidicandi or the right to exclude other persons from the possession the thing owned.
123
In
contrast, the indigenous peoples concept of ownership emphasizes the importance of communal or
group ownership. By virtue of the communal character of ownership, the property held in common "cannot
be sold, disposed or destroyed"
124
because it was meant to benefit the whole indigenous community and
not merely the individual member.
125

That IPRA is not intended to bestow ownership over natural resources to the indigenous peoples is also
clear from the deliberations of the bicameral conference committee on Section 7 which recites the rights
of indigenous peoples over their ancestral domains, to wit:
CHAIRMAN FLAVIER. Accepted. Section 8
126
rights to ancestral domain, this is where we transferred the
other provision but here itself -
HON. DOMINGUEZ. Mr. Chairman, if I maybe allowed to make a very short Statement. Earlier, Mr.
Chairman, we have decided to remove the provisions on natural resources because we all agree
that that belongs to the State. Now, the plight or the rights of those indigenous communities living in
forest and areas where it could be exploited by mining, by dams, so can we not also provide a provision
to give little protection or either rights for them to be consulted before any mining areas should be done in
their areas, any logging done in their areas or any dam construction because this has been disturbing our
people especially in the Cordilleras. So, if there could be, if our lawyers or the secretariat could just
propose a provision for incorporation here so that maybe the right to consultation and the right to be
compensated when there are damages within their ancestral lands.
CHAIRMAN FLAVIER. Yes, very well taken but to the best of my recollection both are already considered
in subsequent sections which we are now looking for.
HON. DOMINGUEZ. Thank you.
CHAIRMAN FLAVIER. First of all there is a line that gives priority use for the indigenous people where
they are. Number two, in terms of the mines there is a need for prior consultation of source which is here
already. So, anyway it is on the record that you want to make sure that the secretariat takes note of those
two issues and my assurance is that it is already there and I will make sure that they cross check.
HON. ADAMAT. I second that, Mr. Chairman.
CHAIRMAN FLAVIER. Okay, thank you. So we now move to Section 8, there is a Senate version you do
not have and if you agree we will adopt that.
127
(Emphasis supplied.)
Further, Section 7 makes no mention of any right of ownership of the indigenous peoples over the natural
resources. In fact, Section 7(a) merely recognizes the "right to claim ownership over lands, bodies of
water traditionally and actually occupied by indigenous peoples, sacred places, traditional hunting and
fishing grounds, and all improvements made by them at any time within the domains." Neither does
Section 7(b), which enumerates certain rights of the indigenous peoples over the natural resources found
within their ancestral domains, contain any recognition of ownership vis-a-vis the natural resources.
What is evident is that the IPRA protects the indigenous peoples rights and welfare in relation to the
natural resources found within their ancestral domains,
128
including the preservation of the ecological
balance therein and the need to ensure that the indigenous peoples will not be unduly displaced when
State-approved activities involving the natural resources located therein are undertaken.
Finally, the concept of native title to natural resources, unlike native title to land, has not been
recognized in the Philippines. NCIP and Flavier, et al. invoke the case of Reavies v. Fianza
129
in support
of their thesis that native title to natural resources has been upheld in this jurisdiction.
130
They insist that
"it is possible for rights over natural resources to vest on a private (as opposed to a public) holder if these
were held prior to the 1935 Constitution."
131
However, a judicious examination of Reavies reveals that,
contrary to the position of NCIP and Flavier, et al., the Court did not recognize native title to natural
resources. Rather, it merely upheld the right of the indigenous peoples to claim ownership of
minerals under the Philippine Bill of 1902.
While as previously discussed, native title to land or private ownership by Filipinos of land by virtue of
time immemorial possession in the concept of an owner was acknowledged and recognized as far back
during the Spanish colonization of the Philippines, there was no similar favorable treatment as regards
natural resources. The unique value of natural resources has been acknowledged by the State and is the
underlying reason for its consistent assertion of ownership and control over said natural resources from
the Spanish regime up to the present.
132
Natural resources, especially minerals, were considered by
Spain as an abundant source of revenue to finance its battles in wars against other nations. Hence,
Spain, by asserting its ownership over minerals wherever these may be found, whether in public or
private lands, recognized the separability of title over lands and that over minerals which may be found
therein.
133

On the other hand, the United States viewed natural resources as a source of wealth for its nationals. As
the owner of natural resources over the Philippines after the latters cession from Spain, the United States
saw it fit to allow both Filipino and American citizens to explore and exploit minerals in public lands, and
to grant patents to private mineral lands. A person who acquired ownership over a parcel of private
mineral land pursuant to the laws then prevailing could exclude other persons, even the State, from
exploiting minerals within his property.
134
Although the United States made a distinction between minerals
found in public lands and those found in private lands, title in these minerals was in all cases sourced
from the State. The framers of the 1935 Constitution found it necessary to maintain the States ownership
over natural resources to insure their conservation for future generations of Filipinos, to prevent foreign
control of the country through economic domination; and to avoid situations whereby the Philippines
would become a source of international conflicts, thereby posing danger to its internal security and
independence.
135

The declaration of State ownership and control over minerals and other natural resources in the 1935
Constitution was reiterated in both the 1973
136
and 1987 Constitutions.
137

Having ruled that the natural resources which may be found within the ancestral domains belong to the
State, the Court deems it necessary to clarify that the jurisdiction of the NCIP with respect to ancestral
domains under Section 52 [i] of IPRA extends only to the lands and not to the natural resources therein.
Section 52[i] provides:
Turnover of Areas Within Ancestral Domains Managed by Other Government Agencies. - The
Chairperson of the NCIP shall certify that the area covered is an ancestral domain. The secretaries of the
Department of Agrarian Reform, Department of Environment and Natural Resources, Department of
Interior and Local Government, and Department of Justice, the Commissioner of the National
Development Corporation, and any other government agency claiming jurisdiction over the area shall be
notified thereof. Such notification shall terminate any legal basis for the jurisdiction previously claimed.
Undoubtedly, certain areas that are claimed as ancestral domains may still be under the administration of
other agencies of the Government, such as the Department of Agrarian Reform, with respect to
agricultural lands, and the Department of Environment and Natural Resources with respect to timber,
forest and mineral lands. Upon the certification of these areas as ancestral domain following the
procedure outlined in Sections 51 to 53 of the IPRA, jurisdiction of the government agency or agencies
concerned over lands forming part thereof ceases. Nevertheless, the jurisdiction of government agencies
over the natural resources within the ancestral domains does not terminate by such certification
because said agencies are mandated under existing laws to administer the natural resources for the
State, which is the owner thereof. To construe Section 52[i] as divesting the State, through the
government agencies concerned, of jurisdiction over the natural resources within the ancestral domains
would be inconsistent with the established doctrine that all natural resources are owned by the State.
C. The provisions of IPRA pertaining to the utilization of natural resources are not unconstitutional.
The IPRA provides that indigenous peoples shall have the right to manage and conserve the natural
resources found on the ancestral domains, to benefit from and share in the profits from the allocation and
utilization of these resources, and to negotiate the terms and conditions for the exploration of such natural
resources.
138
The statute also grants them priority rights in the harvesting, extraction, development or
exploitation of any natural resources within the ancestral domains.
139
Before the NCIP can issue a
certification for the renewal, or grant of any concession, license or lease, or for the perfection of any
production-sharing agreement the prior informed written consent of the indigenous peoples concerned
must be obtained.
140
In return, the indigenous peoples are given the responsibility to maintain, develop,
protect and conserve the ancestral domains or portions thereof which are found to be necessary for
critical watersheds, mangroves, wildlife sanctuaries, wilderness, protected areas, forest cover, or
reforestation.
141

The Solicitor General argues that these provisions deny the State an active and dominant role in the
utilization of our countrys natural resources. Petitioners, on the other hand, allege that under the
Constitution the exploration, development and utilization of natural resources may only be undertaken by
the State, either directly or indirectly through co-production, joint venture, or production-sharing
agreements.
142
To petitioners, no other method is allowed by the Constitution. They likewise submit that
by vesting ownership of ancestral lands and ancestral domains in the indigenous peoples, IPRA
necessarily gives them control over the use and enjoyment of such natural resources, to the prejudice of
the State.
143

Section 2, Article XII of the Constitution provides in paragraph 1 thereof that the exploration, development
and utilization of natural resources must be under the full control and supervision of the State, which may
directly undertake such activities or enter into co-production, joint venture, or production-sharing
agreements. This provision, however, should not be read in isolation to avoid a mistaken interpretation
that any and all forms of utilization of natural resources other than the foregoing are prohibited. The
Constitution must be regarded as consistent with itself throughout.
144
No constitutional provision is to be
separated from all the others, or to be considered alone, all provisions bearing upon a particular subject
are to be brought into view and to be so interpreted as to effectuate the great purposes of the
fundamental law.
145

In addition to the means of exploration, development and utilization of the countrys natural resources
stated in paragraph 1, Section 2 of Article XII, the Constitution itself states in the third paragraph of the
same section that Congress may, by law, allow small-scale utilization of natural resources by its
citizens.
146
Further, Section 6, Article XIII, directs the State, in the disposition and utilization of natural
resources, to apply the principles of agrarian reform or stewardship.
147
Similarly, Section 7, Article XIII
mandates the State to protect the rights of subsistence fishermen to the preferential use of marine and
fishing resources.
148
Clearly, Section 2, Article XII, when interpreted in view of the pro-Filipino, pro-poor
philosophy of our fundamental law, and in harmony with the other provisions of the Constitution rather as
a sequestered pronouncement,
149
cannot be construed as a prohibition against any and all forms of
utilization of natural resources without the States direct participation.
Through the imposition of certain requirements and conditions for the exploration, development and
utilization of the natural resources under existing laws,
150
the State retains full control over such activities,
whether done on small-scale basis
151
or otherwise.
The rights given to the indigenous peoples regarding the exploitation of natural resources under Sections
7(b) and 57 of IPRA amplify what has been granted to them under existing laws, such as the Small-Scale
Mining Act of 1991 (R.A. 7076) and the Philippine Mining Act of 1995 (R.A. 7942). R.A. 7076 expressly
provides that should an ancestral land be declared as a peoples small-scale mining area, the members
of the indigenous peoples living within said area shall be given priority in the awarding of small-scale
mining contracts.
152
R.A. 7942 declares that no ancestral land shall be opened for mining
operations without the prior consent of the indigenous cultural community concerned
153
and in the
event that the members of such indigenous cultural community give their consent to mining operations
within their ancestral land, royalties shall be paid to them by the parties to the mining to the contract.
154

In any case, a careful reading of Section 7(b) would reveal that the rights given to the indigenous peoples
are duly circumscribed. These rights are limited only to the following: "to manage and conserve natural
resources within territories and uphold it for future generations; to benefit and share the profits from
allocation and utilization of the natural resources found therein; to negotiate the terms and conditions
for the exploration of natural resources in the areas for the purpose of ensuring ecological,
environmental protection and the conservation measures, pursuant to national and customary laws; to an
informed and intelligent participation in the formulation and implementation of any project, government
or private, that will affect or impact upon the ancestral domains and to receive just and fair
compensation for any damages which they may sustain as a result of the project, and the right to
effective measures by the government to prevent any interference with, alienation and encroachment of
these rights."
It must be noted that the right to negotiate terms and conditions granted under Section 7(b) pertains only
to theexploration of natural resources. The term "exploration" refers only to the search or prospecting of
mineral resources, or any other means for the purpose of determining the existence and the feasibility of
mining them for profit.
155
The exploration, which is merely a preliminary activity, cannot be equated with
the entire process of "exploration, development and utilization" of natural resources which under the
Constitution belong to the State.
Section 57, on the other hand, grants the indigenous peoples "priority rights" in the utilization of natural
resources and not absolute ownership thereof. Priority rights does not mean exclusive rights. What is
granted is merely the right of preference or first consideration in the award of privileges provided by
existing laws and regulations, with due regard to the needs and welfare of indigenous peoples living in the
area.
There is nothing in the assailed law which implies an automatic or mechanical character in the grant of
concessions. Nor does the law negate the exercise of sound discretion by government entities. Several
factors still have to be considered. For example, the extent and nature of utilization and the consequent
impact on the environment and on the indigenous peoples way of life are important considerations.
Moreover, the indigenous peoples must show that they live in the area and that they are in the best
position to undertake the required utilization.
It must be emphasized that the grant of said priority rights to indigenous peoples is not a blanket authority
to disregard pertinent laws and regulations. The utilization of said natural resources is always subject to
compliance by the indigenous peoples with existing laws, such as R.A. 7076 and R.A. 7942 since it is not
they but the State, which owns these resources.
It also bears stressing that the grant of priority rights does not preclude the State from undertaking
activities, or entering into co-production, joint venture or production-sharing agreements with private
entities, to utilize the natural resources which may be located within the ancestral domains. There is no
intention, as between the State and the indigenous peoples, to create a hierarchy of values; rather, the
object is to balance the interests of the State for national development and those of the indigenous
peoples.
Neither does the grant of priority rights to the indigenous peoples exclude non-indigenous peoples from
undertaking the same activities within the ancestral domains upon authority granted by the proper
governmental agency. To do so would unduly limit the ownership rights of the State over the natural
resources.
To be sure, the act of the State of giving preferential right to a particular sector in the utilization of natural
resources is nothing new. As previously mentioned, Section 7, Article XIII of the Constitution mandates
the protection by the State of "the rights of subsistence fishermen, especially of local communities, to the
preferential use of communal marine and fishing resources, both inland and offshore."
Section 57 further recognizes the possibility that the exploration and exploitation of natural resources
within the ancestral domains may disrupt the natural environment as well as the traditional activities of the
indigenous peoples therein. Hence, the need for the prior informed consent of the indigenous peoples
before any search for or utilization of the natural resources within their ancestral domains is undertaken.
In a situation where the State intends to directly or indirectly undertake such activities, IPRA requires that
the prior informed consent of the indigenous peoples be obtained. The State must, as a matter of policy
and law, consult the indigenous peoples in accordance with the intent of the framers of the Constitution
that national development policies and programs should involve a systematic consultation to balance
local needs as well as national plans. As may be gathered from the discussion of the framers of the
Constitution on this point, the national plan presumably takes into account the requirements of the region
after thorough consultation.
156
To this end, IPRA grants to the indigenous peoples the right to an informed
and intelligent participation in the formulation and implementation of any project, government or private,
and the right not to be removed therefrom without their free and prior informed consent.
157
As to non-
members, the prior informed consent takes the form of a formal and written agreement between the
indigenous peoples and non-members under the proviso in Section 57 in case the State enters into a co-
production, joint venture, or production-sharing agreement with Filipino citizens, or corporations. This
requirement is not peculiar to IPRA. Existing laws and regulations such as the Philippine Environmental
Policy,
158
the Environmental Impact System,
159
the Local Government Code
160
and the Philippine Mining
Act of 1995
161
already require increased consultation and participation of stakeholders, such as
indigenous peoples, in the planning of activities with significant environment impact.
The requirement in Section 59 that prior written informed consent of the indigenous peoples must be
procured before the NCIP can issue a certification for the "issuance, renewal, or grant of any concession,
license or lease, or to the perfection of any production-sharing agreement," must be interpreted, not as a
grant of the power to control the exploration, development and utilization of natural resources, but merely
the imposition of an additional requirement for such concession or agreement. The clear intent of the law
is to protect the rights and interests of the indigenous peoples which may be adversely affected by the
operation of such entities or licensees.
Corollary Issues
A. IPRA does not violate the Due Process clause.
The first corollary issue raised by petitioners is whether IPRA violates Section 1, Article III of the
Constitution, which provides that "no person shall be deprived of life, liberty, or property without due
process of law, nor shall any person be deprived the equal protection of the laws."
Petitioners maintain that the broad definition of ancestral lands and ancestral domains under Section 3(a)
and 3(b) of IPRA includes private lands. They argue that the inclusion of private lands in the ancestral
lands and ancestral domains violates the due process clause.
162
Petitioners contention is erroneous.
Sections 3(a) and 3(b) expressly provide that the definition of ancestral lands and ancestral domains are
"subject to Section 56," which reads:
Sec. 56. Existing Property Rights Regimes. Property rights within the ancestral domains already
existing and/or vested upon effectivity of this Act, shall be recognized and protected.
Petitioners, however, contend that Section 56 aims to protect only the vested rights of indigenous
peoples, but not those who are not members of such communities. Following their interpretation, IPRA,
under Section 56, recognizes the rights of indigenous peoples to their ancestral lands and ancestral
domains, subject to the vested rights of the same communities to such ancestral lands and
ancestral domains. Such interpretation is obviously incorrect.
The "property rights" referred to in Section 56 belong to those acquired by individuals, whether indigenous
or non-indigenous peoples. Said provision makes no distinction as to the ethnic origins of the ownership
of these "property rights." The IPRA thus recognizes and respects "vested rights" regardless of whether
they pertain to indigenous or non-indigenous peoples. Where the law does not distinguish, the courts
should not distinguish.
163
What IPRA only requires is that these "property rights" already exist and/or
vested upon its effectivity.
Further, by the enactment of IPRA, Congress did not purport to annul any and all Torrens titles within
areas claimed as ancestral lands or ancestral domains. The statute imposes strict procedural
requirements for the proper delineation of ancestral lands and ancestral domains as safeguards against
the fraudulent deprivation of any landowner of his land, whether or not he is member of an indigenous
cultural community. In all proceedings for delineation of ancestral lands and ancestral domains, the
Director of Lands shall appear to represent the interest of the Republic of the Philippines.
164
With regard
to ancestral domains, the following procedure is mandatory:first, petition by an indigenous cultural
community, or motu proprio by the NCIP; second, investigation and census by the Ancestral domains
Office ("ADO") of the NCIP; third, preliminary report by the ADO; fourth, posting and publication;
and lastly, evaluation by the NCIP upon submission of the final report of the ADO.
165
With regard
to ancestral lands, unless such lands are within an ancestral domain, the statute imposes the following
procedural requirements: first, application; second, posting and publication; third, investigation and
inspection by the ADO; fourth, delineation; lastly, evaluation by the NCIP upon submission of a report by
the ADO.
166
Hence, we cannot sustain the arguments of the petitioners that the law affords no protection
to those who are not indigenous peoples.
Neither do the questioned sections of IPRA on the composition and powers and jurisdiction of the
NCIP
167
and the application of customary law,
168
violate the due process clause of the Constitution.
Petitioners point out that IPRA provides that the NCIP shall be composed exclusively of members of
indigenous peoples,
169
and that the NCIP shall have jurisdiction over all claims and disputes involving
indigenous peoples,
170
including even disputes between a member of such communities and one who is
not a member, as well as over disputes in the delineation of ancestral domains.
171
Petitioners clarify that
they do not claim that the members of the NCIP are incapable of being fair and impartial judges. They
merely contend that the NCIP will not appear to be impartial, because a party who is not a member of an
indigenous cultural community "who must defend his case against [one who is] before judges who are all
members of [indigenous peoples] cannot but harbor a suspicion that they do not have the cold neutrality
of an impartial judge."
172

In addition, petitioners claim that IPRA prescribes that customary laws shall be applied first in disputes
involving property, succession and land,
173
and that such laws shall likewise be used in disputes involving
indigenous peoples.
174
They assert that "[w]hen the dispute involves a member of an [indigenous cultural
community and another who is not], a resolution of such a dispute based on customary laws. . . would
clearly be a denial of due process. . . [because those who are not indigenous peoples] do not know what
these customary laws are."
175

Petitioners concerns are unfounded. The fact that the NCIP is composed of members of the indigenous
peoples does not mean that it (the NCIP) is incapable, or will appear to be so incapable, of delivering
justice to the non-indigenous peoples. A persons possession of the trait of impartiality desirable of a
judge has nothing to do with his or her ethnic roots. In this wise, the indigenous peoples are as capable of
rendering justice as the non-indigenous peoples for, certainly, the latter have no monopoly of the concept
of justice.
In any case, there are sufficient checks in the law against any abuse by the NCIP of its quasi-judicial
powers. Section 67 states that the decision of the NCIP shall be appealable to the Court of Appeals by
petition for review. The regular remedies under our rules of procedure are likewise available to any party
aggrieved by the decision of the NCIP.
Anent the use of customary laws in determining the ownership and extent of ancestral domains, suffice it
to say that such is allowed under paragraph 2, Section 5 of Article XII of the Constitution. Said provision
states, "The Congress may provide for the applicability of customary laws governing property rights and
relations in determining the ownership and extent of the ancestral domains." Notably, the use of
customary laws under IPRA is not absolute, for the law speaks merely of primacy of use.
176
The IPRA
prescribes the application of such customary laws where these present a workable solution acceptable to
the parties, who are members of the same indigenous group. This interpretation is supported by Section
1, Rule IX of the Implementing Rules which states:
RULE IX. JURISDICTION AND PROCEDURES FOR ENFORCEMENT OF RIGHTS
Section 1. Primacy of Customary Law. All conflicts related to ancestral domains and lands, involving
ICCs/IPs, such as but not limited to conflicting claims and boundary disputes, shall be resolved by the
concerned parties through the application of customary laws in the area where the disputed ancestral
domain or land is located.
All conflicts related to the ancestral domains or lands where one of the parties is a non-ICC/IP or
where the dispute could not be resolved through customary law shall be heard and adjudicated in
accordance with the Rules on Pleadings, Practice and Procedures Before the NCIP to be adopted
hereafter. (Emphasis supplied.)
The application of customary law is limited to disputes concerning property rights or relations in
determining the ownership and extent of the ancestral domains,
177
where all the parties involved are
members of indigenous peoples,
178
specifically, of the same indigenous group. It therefore follows that
when one of the parties to a dispute is a non-member of an indigenous group, or when the indigenous
peoples involved belong to different groups, the application of customary law is not required.
Like any other law, the objective of IPRA in prescribing the primacy of customary law in disputes
concerning ancestral lands and domains where all parties involved are indigenous peoples is justice. The
utilization of customary laws is in line with the constitutional policy of recognizing the application thereof
through legislation passed by Congress.
Furthermore, the recognition and use of customary law is not a novel idea in this jurisdiction. Under the
Civil Code, use of customary law is sanctioned, as long as it is proved as a fact according to the rules of
evidence,
179
and it is not contrary to law, public order or public policy.
180
Moreover, the Local Government
Code of 1991 calls for the recognition and application of customary laws to the resolution of issues
involving members of indigenous peoples. This law admits the operation of customary laws in the settling
of disputes if such are ordinarily used in barangays where majority of the inhabitants are members of
indigenous peoples.
181

B. Section 1, Part II, Rule VII of the Implementing Rules of IPRA does not infringe upon the Presidents
power of control over the Executive Department.
The second corollary issue is whether the Implementing Rules of IPRA violate Section 17, Article VII of
the Constitution, which provides that:
The President shall have control of all the executive departments, bureaus, and offices. He shall ensure
that the laws be faithfully executed.
The assailed provision of the Implementing Rules provides:
Rule VII. The National Commission on Indigenous Peoples (NCIP)
x x x
Part II: NCIP as an Independent Agency Under the Office of the President
Section 1. The NCIP is the primary agency of government for the formulation and implementation of
policies, plans and programs to recognize, promote and protect the rights and well-being of indigenous
peoples. It shall be an independent agency under the Office of the President. As such, the
administrative relationship of the NCIP to the Office of the President is characterized as a lateral
but autonomous relationship for purposes of policy and program coordination. This relationship
shall be carried out through a system of periodic reporting. Matters of day-to-day administration or all
those pertaining to internal operations shall be left to the discretion of the Chairperson of the Commission,
as the Chief Executive Officer.
Petitioners asseverate that the aforecited rule infringes upon the power of control of the President over
the NCIP by characterizing the relationship of the NCIP to the Office of the President as "lateral but
autonomous...for purposes of policy and program coordination."
Although both Section 40 of the IPRA and Section 1, Part II, Rule VII of the Implementing Rules
characterize the NCIP as an independent agency under the Office of the President, such characterization
does not remove said body from the Presidents control and supervision.
The NCIP has been designated under IPRA as the primary government agency responsible for the
formulation and implementation of policies, plans and programs to promote and protect the rights and well
being of the indigenous peoples and the recognition of their ancestral domain as well as their rights
thereto.
182
It has been granted administrative,
183
quasi-legislative
184
and quasi-judicial powers
185
to carry
out its mandate. The diverse nature of the NCIPs functions renders it impossible to place said agency
entirely under the control of only one branch of government and this, apparently, is the reason for its
characterization by Congress as an independent agency. An "independent agency" is defined as an
administrative body independent of the executive branch or one not subject to a superior head of
department, as distinguished from a "subordinate agency" or an administrative body whose action is
subject to administrative review or revision.
186

That Congress did not intend to place the NCIP under the control of the President in all instances is
evident in the IPRA itself, which provides that the decisions of the NCIP in the exercise of its quasi-judicial
functions shall be appealable to the Court of Appeals,
187
like those of the National Labor Relations
Commission (NLRC) and the Securities and Exchange Commission (SEC). Nevertheless, the NCIP,
although independent to a certain degree, was placed by Congress "under the office of the President"
and, as such, is still subject to the Presidents power of control and supervision granted under Section 17,
Article VII of the Constitution
188
with respect to its performance of administrative functions, such as the
following: (1) the NCIP must secure the Presidents approval in obtaining loans to finance its
projects;
189
(2) it must obtain the Presidents approval for any negotiation for funds and for the acceptance
of gifts and/or properties in whatever from and from whatever source;
190
(3) the NCIP shall submit annual
reports of its operations and achievements to the President, and advise the latter on all matters relating to
the indigenous peoples;
191
and (4) it shall exercise such other powers as may be directed by the
President.
192
The President is also given the power to appoint the Commissioners of the NCIP
193
as well
as to remove them from office for cause motu proprio or upon the recommendation of any indigenous
community.
194

To recapitulate:
(1) The provisions of the IPRA (specifically Sections 3, paragraphs (a) and (b), 5, 6, 7, and 8)
affirming the ownership by the indigenous peoples of their ancestral lands and domains by virtue
of native title do not diminish the States ownership of lands of the public domain, because said
ancestral lands and domains are considered as private land, and never to have been part of the
public domain, following the doctrine laid down in Cario vs. Insular Government;
195

(2) The constitutional provision vesting ownership over minerals, mineral lands and other natural
resources in the State is not violated by Sections 3, 5, 7, 56, 57, 58 and 59 of the IPRA which
grant certain rights to the indigenous peoples over the natural resources found within the
ancestral domains, e.g., to benefit from and share in the profits from the allocation and utilization
of the same, as well as priority rights in the harvesting, extraction, development or exploitation
thereof. The State retains full control over the exploration, development and utilization of natural
resources even with the grant of said rights to the indigenous peoples, through the imposition of
requirements and conditions for the utilization of natural resources under existing laws, such as
the Small-Scale Mining Act of 1991
196
and the Philippine Mining Act of 1995.
197
Moreover, the
rights granted to indigenous peoples for the utilization of natural resources within their ancestral
domains merely amplify what has been earlier granted to them under the aforesaid laws;
(3) While the IPRA recognizes the rights of indigenous peoples with regard to their ancestral
lands and domains, it also protects the vested rights of persons, whether indigenous or non-
indigenous peoples, who may have acquired rights of ownership lands or rights to explore and
exploit natural resources within the ancestral lands and domains;
198

(4) The Due Process Clause of the Constitution is not violated by the provisions (Sections 40, 51-
54, 62, 63, 65 and 66) of the IPRA which, among others, establish the composition of the NCIP,
and prescribe the application of customary law in certain disputes involving indigenous peoples.
The fact the NCIP is composed wholly of indigenous peoples does not mean that it is incapable of
being impartial. Moreover, the use of customary laws is sanctioned by paragraph 2, Section 5 of
Article XII of the Constitution; and
(5) The provision of the Implementing Rules characterizing the NCIP as an independent agency
under the Office of the President does not infringe upon the Presidents power of control under
Section 17, Article VII of the Constitution, since said provision as well as Section 40 of the IPRA
expressly places the NCIP under the Office of the President, and therefore under the Presidents
control and supervision with respect to its administrative functions. However, insofar as the
decisions of the NCIP in the exercise of its quasi-judicial powers are concerned, the same are
reviewable by the Court of Appeals, like those of the NLRC and the SEC.
In view of the foregoing, I vote to DISMISS the petition.
SEPARATE OPINION
MENDOZA, J .:
This suit was instituted to determine the constitutionality of certain provisions of R.A. No. 8371, otherwise
known as the Indigenous Peoples Rights Act. Petitioners do not complain of any injury as a result of the
application of the statute to them. They assert a right to seek an adjudication of constitutional questions
as citizens and taxpayers, upon the plea that the questions raised are of "transcendental importance."
The judicial power vested in this Court by Art. VIII, 1 extends only to cases and controversies for the
determination of such proceedings as are established by law for the protection or enforcement of rights,
or the prevention, redress or punishment of wrongs.
1
In this case, the purpose of the suit is not to enforce
a property right of petitioners against the government and other respondents or to demand compensation
for injuries suffered by them as a result of the enforcement of the law, but only to settle what they believe
to be the doubtful character of the law in question. Any judgment that we render in this case will thus not
conclude or bind real parties in the future, when actual litigation will bring to the Court the question of the
constitutionality of such legislation. Such judgment cannot be executed as it amounts to no more than an
expression of opinion upon the validity of the provisions of the law in question.
2

I do not conceive it to be the function of this Court under Art. VIII, 1 of the Constitution to determine in
the abstract whether or not there has been a grave abuse of discretion amounting to lack or excess of
jurisdiction on the part of the legislative and executive departments in enacting the IPRA. Our jurisdiction
is confined to cases or controversies. No one reading Art. VIII, 5 can fail to note that, in enumerating the
matters placed in the keeping of this Court, it uniformly begins with the phrase "all cases. . . ."
The statement that the judicial power includes the duty to determine whether there has been a grave
abuse of discretion was inserted in Art. VIII, 1 not really to give the judiciary a roving commission to right
any wrong it perceives but to preclude courts from invoking the political question doctrine in order to
evade the decision of certain cases even where violations of civil liberties are alleged.
The statement is based on the ruling of the Court in Lansang v. Garcia,
3
in which this Court, adopting the
submission of the Solicitor General, formulated the following test of its jurisdiction in such cases:
[J]udicial inquiry into the basis of the questioned proclamation can go no further than to satisfy the
Court not that the Presidents decision is correct and that public safety was endangered by the rebellion
and justified the suspension of the writ, but that in suspending the writ, the President did not
act arbitrarily.
That is why Art. VII, 18 now confers on any citizen standing to question the proclamation of martial law
or the suspension of the privilege of the writ of habeas corpus. It is noteworthy that Chief Justice Roberto
Concepcion, who chaired the Committee on the Judiciary of the Constitutional Commission, was the
author of the opinions of the Court in Lopez v. Roxas and Lansang v. Garcia.
Indeed, the judicial power cannot be extended to matters which do not involve actual cases or
controversies without upsetting the balance of power among the three branches of the government and
erecting, as it were, the judiciary, particularly the Supreme Court, as a third branch of Congress, with
power not only to invalidate statutes but even to rewrite them. Yet that is exactly what we would be
permitting in this case were we to assume jurisdiction and decide wholesale the constitutional validity of
the IPRA contrary to the established rule that a party can question the validity of a statute only if, as
applied to him, it is unconstitutional. Here the IPRA is sought to be declared void on its face.
The only instance where a facial challenge to a statute is allowed is when it operates in the area of
freedom of expression. In such instance, the overbreadth doctrine permits a party to challenge the validity
of a statute even though as applied to him it is not unconstitutional but it might be if applied to others not
before the Court whose activities are constitutionally protected. Invalidation of the statute "on its face"
rather than "as applied" is permitted in the interest of preventing a "chilling" effect on freedom of
expression. But in other cases, even if it is found that a provision of a statute is unconstitutional, courts
will decree only partial invalidity unless the invalid portion is so far inseparable from the rest of the statute
that a declaration of partial invalidity is not possible.
For the Court to exercise its power of review when there is no case or controversy is not only to act
without jurisdiction but also to run the risk that, in adjudicating abstract or hypothetical questions, its
decision will be based on speculation rather than experience. Deprived of the opportunity to observe the
impact of the law, the Court is likely to equate questions of constitutionality with questions of wisdom and
is thus likely to intrude into the domain of legislation. Constitutional adjudication, it cannot be too often
repeated, cannot take place in a vacuum.
Some of the brethren contend that not deciding the constitutional issues raised by petitioners will be a
"galling cop out"
4
or an "advocacy of timidity, let alone isolationism."
5
To decline the exercise of
jurisdiction in this case is no more a "cop out" or a sign of "timidity" than it was for Chief Justice Marshall
in Marbury v. Madison
6
to hold that petitioner had the right to the issuance of his commission as justice of
the peace of the District of Columbia only to declare in the end that after all mandamus did not lie,
because 13 of the Judiciary Act of 1789, which conferred original jurisdiction on the United States
Supreme Court to issue the writ of mandamus, was unconstitutional as the courts jurisdiction is mainly
appellate.
Today Marbury v. Madison is remembered for the institution of the power of judicial review, and so that
there can be no doubt of this power of our Court, we in this country have enshrined its principle in Art.
VIII, 1. Now, the exercise of judicial review can result either in the invalidation of an act of Congress or in
upholding it. Hence, the checking and legitimating functions of judicial review so well mentioned in the
decisions
7
of this Court.
To decline, therefore, the exercise of jurisdiction where there is no genuine controversy is not to show
timidity but respect for the judgment of a coequal department of government whose acts, unless shown to
be clearly repugnant to the fundamental law, are presumed to be valid. The polestar of constitutional
adjudication was set forth by Justice Laurel in the Angara case when he said that "this power of judicial
review is limited to actual cases and controversies to be exercised after full opportunity of argument by
the parties, and limited further to the constitutional question raised or the very lis mota, presented."
8
For
the exercise of this power is legitimate only in the last resort, and as a necessity in the determination of
real, earnest, and vital controversy between individuals.
9
Until, therefore, an actual case is brought to test
the constitutionality of the IPRA, the presumption of constitutionality, which inheres in every statute, must
be accorded to it.
Justice Kapunan, on the other hand, cites the statement in Severino v. Governor General,
10
reiterated
in Tanada v. Tuvera,
11
that "when the question is one of public right and the object of mandamus to
procure the enforcement of a public duty, the people are regarded as the real party in interest, and the
relator at whose instigation the proceedings are instituted need not show that he has any legal or special
interest in the result, it being sufficient that he is a citizen and as such is interested in the execution of the
laws." On the basis of this statement, he argues that petitioners have standing to bring these
proceedings.
12

In Severino v. Governor General,
13
the question was whether mandamus lay to compel the Governor
General to call a special election on the ground that it was his duty to do so. The ruling was that he did
not have such a duty. On the other hand, although mandamus was issued in Tanada v. Tuvera, it was
clear that petitioners had standing to bring the suit, because the public has a right to know and the failure
of respondents to publish all decrees and other presidential issuances in the Official Gazette placed
petitioners in danger of violating those decrees and issuances. But, in this case, what public right is there
for petitioners to enforce when the IPRA does not apply to them except in general and in common with
other citizens.
For the foregoing reasons I vote to dismiss the petition in this case.


SEPARATE OPINION
(Concurring and Dissenting)
PANGANIBAN, J .:
I concur with the draft ponencia of Mr. Justice Santiago M. Kapunan in its well-crafted handling of the
procedural or preliminary issues. In particular, I agree that petitioners have shown an actual case or
controversy involving at least two constitutional questions of transcendental importance,
1
which deserve
judicious disposition on the merits directly by the highest court of the land.
2
Further, I am satisfied that the
various aspects of this controversy have been fully presented and impressively argued by the parties.
Moreover, prohibition and mandamus are proper legal remedies
3
to address the problems raised by
petitioners. In any event, this Court has given due course to the Petition, heard oral arguments and
required the submission of memoranda. Indeed, it would then be a galling copout for us to dismiss it on
mere technical or procedural grounds.
Protection of Indigenous Peoples Rights Must Be Within the Constitutional Framework
With due respect, however, I dissent from the ponencias resolution of the two main substantive issues,
which constitute the core of this case. Specifically, I submit that Republic Act (RA) No. 8371, otherwise
known as the Indigenous Peoples Rights Act (IPRA) of 1997, violates and contravenes the Constitution
of the Philippines insofar as -
1. It recognizes or, worse, grants rights of ownership over "lands of the public domain, waters, x x
x and other natural resources" which, under Section 2, Article XII of the Constitution, "are owned
by the State" and "shall not be alienated." I respectfully reject the contention that "ancestral lands
and ancestral domains are not public lands and have never been owned by the State." Such
sweeping statement places substantial portions of Philippine territory outside the scope of the
Philippine Constitution and beyond the collective reach of the Filipino people. As will be
discussed later, these real properties constitute a third of the entire Philippine territory; and the
resources, 80 percent of the nation's natural wealth.
2. It defeats, dilutes or lessens the authority of the State to oversee the "exploration,
development, and utilization of natural resources," which the Constitution expressly requires to
"be under the full control and supervision of the State."
True, our fundamental law mandates the protection of the indigenous cultural communities right to their
ancestral lands, but such mandate is "subject to the provisions of this Constitution."
4
I concede that
indigenous cultural communities and indigenous peoples (ICCs/IPs) may be accorded preferential rights
to the beneficial use of public domains, as well as priority in the exploration, development and utilization
of natural resources. Such privileges, however, must be subject to the fundamental law.
Consistent with the social justice principle of giving more in law to those who have less in life, Congress in
its wisdom may grant preferences and prerogatives to our marginalized brothers and sisters, subject to
the irreducible caveat that the Constitution must be respected. I personally believe in according every
benefit to the poor, the oppressed and the disadvantaged, in order to empower them to equally enjoy the
blessings of nationhood. I cannot, however, agree to legitimize perpetual inequality of access to the
nation's wealth or to stamp the Court's imprimatur on a law that offends and degrades the repository of
the very authority of this Court - the Constitution of the Philippines.
The Constitution Is a Compact
My basic premise is that the Constitution is the fundamental law of the land, to which all other laws must
conform.
5
It is the people's quintessential act of sovereignty, embodying the principles upon which the
State and the government are founded.
6
Having the status of a supreme and all-encompassing law, it
speaks for all the people all the time, not just for the majority or for the minority at intermittent times. Every
constitution is a compact made by and among the citizens of a State to govern themselves in a certain
manner.
7
Truly, the Philippine Constitution is a solemn covenant made by all the Filipinos to govern
themselves. No group, however blessed, and no sector, however distressed, is exempt from its compass.
RA 8371, which defines the rights of indigenous cultural communities and indigenous peoples, admittedly
professes a laudable intent. It was primarily enacted pursuant to the state policy enshrined in our
Constitution to "recognize and promote the rights of indigenous cultural communities within the framework
of national unity and development."
8
Though laudable and well-meaning, this statute, however, has
provisions that run directly afoul of our fundamental law from which it claims origin and authority. More
specifically, Sections 3(a) and (b), 5, 6, 7(a) and (b), 8 and other related provisions contravene the
Regalian Doctrine - the basic foundation of the State's property regime.
Public Domains and Natural Resources Are Owned by the State and Cannot Be Alienated or Ceded
Jura regalia was introduced into our political system upon the "discovery" and the "conquest" of our
country in the sixteenth century. Under this concept, the entire earthly territory known as the Philippine
Islands was acquired and held by the Crown of Spain. The King, as then head of State, had the supreme
power or exclusive dominion over all our lands, waters, minerals and other natural resources. By royal
decrees, though, private ownership of real property was recognized upon the showing of (1) a title deed;
or (2) ancient possession in the concept of owner, according to which a title could be obtained by
prescription.
9
Refusal to abide by the system and its implementing laws meant the abandonment or
waiver of ownership claims.
By virtue of the 1898 Treaty of Paris, the Philippine archipelago was ceded to the United States. The
latter assumed administration of the Philippines and succeeded to the property rights of the Spanish
Crown. But under the Philippine Bill of 1902, the US Government allowed and granted patents to Filipino
and US citizens for the "free and open x x x exploration, occupation and purchase [of mines] and the land
in which they are found."
10
To a certain extent, private individuals were entitled to own, exploit and
dispose of mineral resources and other rights arising from mining patents.
This US policy was, however, rejected by the Philippine Commonwealth in 1935 when it crafted and
ratified our first Constitution. Instead, the said Constitution embodied the Regalian Doctrine, which more
definitively declared as belonging to the State all lands of the public domain, waters, minerals and other
natural resources.
11
Although respecting mining patentees under the Philippine Bill of 1902, it restricted
the further exploration, development and utilization of natural resources, both as to who might be entitled
to undertake such activities and for how long. The pertinent provision reads:
"SECTION 1 [Art. XIII]. All agricultural, timber, and mineral lands of the public domain, waters, minerals,
coal, petroleum, and other mineral oils, all forces of potential energy, and other natural resources of the
Philippines belong to the State, and their disposition, exploitation, development, or utilization shall be
limited to citizens of the Philippines, or to corporations or associations at least sixty per centum of the
capital of which is owned by such citizens, subject to any existing right, grant, lease, or concession at the
time of the inauguration of the Government established under this Constitution. Natural resources, with
the exception of public agricultural land, shall not be alienated, and license, concession, or lease for the
exploitation, development, or utilization of any of the natural resources shall be granted for a period
exceeding twenty-five years, renewable for another twenty-five years, except as to water rights for
irrigation, water supply, fisheries, or industrial uses other than the development of water power, in which
cases beneficial use may be the measure and the limit of the grant."
The concept was carried over in the 1973 and the 1987 Constitutions. Hence, Sections 8 and 9, Article
XIV of the 1973 Constitution, state:
"SEC. 8. All lands of the public domain, waters, minerals, coal, petroleum and other mineral oils, all forces
of potential energy, fisheries, wildlife, and other natural resources of the Philippines belong to the State.
With the exception of agricultural, industrial or commercial, residential, and resettlement lands of the
public domain, natural resources shall not be alienated and no license, concession, or lease for the
exploration, development, exploitation, utilization of any of the natural resources shall be granted for a
period exceeding twenty-five years, renewable for not more than twenty-five years, except as to water
rights for irrigation, water supply, fisheries, or industrial uses other than the development of water power,
in which cases beneficial use may be the measure and the limit of the grant.
SEC. 9. The disposition, exploration, development, exploitation, or utilization of any of the natural
resources of the Philippines shall be limited to citizens of the Philippines, or to corporations or
associations at least sixty per centum of the capital of which is owned by such citizens. The National
Assembly, in the national interest, may allow such citizens, corporations, or associations to enter into
service contracts for financial, technical, management, or other forms of assistance with any foreign
person or entity for the exploration, development, exploitation, or utilization of any of the natural
resources. Existing valid and binding service contracts for financial, technical, management, or other
forms of assistance are hereby recognized as such."
Similarly, Section 2, Article XII of the 1987 Constitution, provides:
"SEC. 2. All lands of the public domain, waters, minerals, coal, petroleum, and other mineral oils, all
forces of potential energy, fisheries, forests or timber, wildlife, flora and fauna, and other natural
resources are owned by the State. With the exception of agricultural lands, all other natural resources
shall not be alienated. The exploration, development, and utilization of natural resources shall be under
the full control and supervision of the State. The State may directly undertake such activities, or it may
enter into co-production, joint venture, or production-sharing agreements with Filipino citizen, or
corporations or associations at least sixty per centum of whose capital is owned by such citizens. Such
agreements may be for a period not exceeding twenty-five years, renewable for not more than twenty-five
years, and under such terms and conditions as may be provided by law. In cases of water rights for
irrigation, water supply, fisheries, or industrial uses other than the development of water power, beneficial
use may be the measure and limit of the grant.
"The State shall protect the nation's marine wealth in its archipelagic waters, territorial sea, and exclusive
economic zone, and reserve its use and enjoyment exclusively to Filipino citizens.
"The Congress may, by law, allow small-scale utilization of natural resources by Filipino citizens, as well
as cooperative fish farming, with priority to subsistence fishermen and fish workers in rivers, lakes, bays
and lagoons.
"The President may enter into agreements with foreign-owned corporations involving either technical or
financial assistance for large-scale exploration, development, and utilization of minerals, petroleum, and
other mineral oils according to the general terms and conditions provided by law, based on real
contributions to the economic growth and general welfare of the country. In such agreements, the State
shall promote the development and use of local scientific and technical resources.
"The President shall notify the Congress of every contract entered into in accordance with this provision,
within thirty days from its execution."
The adoption of the Regalian Doctrine by the Philippine Commonwealth was initially impelled by the
desire to preserve the nation's wealth in the hands of the Filipinos themselves. Nationalism was fervent at
the time, and our constitutional framers decided to embody the doctrine in our fundamental law. Charging
the State with the conservation of the national patrimony was deemed necessary for Filipino posterity.
The arguments in support of the provision are encapsulated by Aruego as follows: "[T]he natural
resources, particularly the mineral resources which constituted a great source of wealth, belonged not
only to the generation then but also to the succeeding generation and consequently should be conserved
for them."
12

Thus, after expressly declaring that all lands of the public domain, waters, minerals, all forces of energy
and other natural resources belonged to the Philippine State, the Commonwealth absolutely prohibited
the alienation of these natural resources. Their disposition, exploitation, development and utilization were
further restricted only to Filipino citizens and entities that were 60 percent Filipino-owned. The present
Constitution even goes further by declaring that such activities "shall be under the full control and
supervision of the State." Additionally, it enumerates land classifications and expressly states that only
agricultural lands of the public domain shall be alienable. We quote below the relevant provision:
13

"SEC. 3. Lands of the public domain are classified into agricultural, forest or timber, mineral lands, and
national parks. Agricultural lands of the public domain may be further classified by law according to the
uses to which they may be devoted. Alienable lands of the public domain shall be limited to agricultural
lands. Private corporations or associations may not hold such alienable lands of the public domain except
by lease, for a period not exceeding twenty-five years, renewable for not more than twenty-five years, and
not to exceed one thousand hectares in area. x x x."
Mr. Justice Kapunan upholds private respondents and intervenors in their claim that all ancestral domains
and lands are outside the coverage of public domain; and that these properties - including forests, bodies
of water, minerals and parks found therein - are private and have never been part of the public domain,
because they have belonged to the indigenous peoples ancestors since time immemorial.
I submit, however, that all Filipinos, whether indigenous or not, are subject to the Constitution. Indeed, no
one is exempt from its all-encompassing provisions. Unlike the 1935 Charter, which was subject to "any
existing right, grant, lease or concession," the 1973 and the 1987 Constitutions spoke in absolute terms.
Because of the States implementation of policies considered to be for the common good, all those
concerned have to give up, under certain conditions, even vested rights of ownership.
In Republic v. Court of Appeals,
14
this Court said that once minerals are found even in private land, the
State may intervene to enable it to extract the minerals in the exercise of its sovereign prerogative. The
land is converted into mineral land and may not be used by any private person, including the registered
owner, for any other purpose that would impede the mining operations. Such owner would be entitled to
just compensation for the loss sustained.
In Atok Big-Wedge Mining Company v. IAC,
15
the Court clarified that while mining claim holders and
patentees have the exclusive right to the possession and enjoyment of the located claim, their rights are
not absolute or strictly one of ownership. Thus, failure to comply with the requirements of pertinent mining
laws was deemed an abandonment or a waiver of the claim.
Verily, as petitioners undauntedly point out, four hundred years of Philippine political history cannot be set
aside or ignored by IPRA, however well-intentioned it may be. The perceived lack of understanding of the
cultural minorities cannot be remedied by conceding the nations resources to their exclusive advantage.
They cannot be more privileged simply because they have chosen to ignore state laws. For having
chosen not to be enfolded by statutes on perfecting land titles, ICCs/IPs cannot now maintain their
ownership of lands and domains by insisting on their concept of "native title" thereto. It would be plain
injustice to the majority of Filipinos who have abided by the law and, consequently, deserve equal
opportunity to enjoy the countrys resources.
Respondent NCIP claims that IPRA does not violate the Constitution, because it does not grant
ownership of public domains and natural resources to ICCs/IPs. "Rather, it recognizes and mandates
respect for the rights of indigenous peoples over their ancestral lands and domains that had never been
lands of the public domain."
16
I say, however, that such claim finds no legal support. Nowhere in the
Constitution is there a provision that exempts such lands and domains from its coverage. Quite the
contrary, it declares that all lands of the public domain and natural resources "are owned by the State";
and "with the exception of agricultural lands, all other natural resources shall not be alienated."
As early as Oh Cho v. Director of Lands,
17
the Court declared as belonging to the public domain all lands
not acquired from the government, either by purchase or by grant under laws, orders or decrees
promulgated by the Spanish government; or by possessory information under Act 496 (Mortgage Law).
On the other hand, Intervenors Flavier et al.
18
differentiate the concept of ownership of ICCs/IPs from that
which is defined in Articles 427 and 428 of the Civil Code. They maintain that "[t]here are variations
among ethnolinguistic groups in the Cordillera, but a fair synthesis of these refers to x x x the tribal right
to use the land or to territorial control x x x, a collective right to freely use the particular territory x x x [in]
the concept of trusteeship.'"
In other words, the "owner" is not an individual. Rather, it is a tribal community that preserves the property
for the common but nonetheless exclusive and perpetual benefit of its members, without the attributes of
alienation or disposition. This concept, however, still perpetually withdraws such property from the control
of the State and from its enjoyment by other citizens of the Republic. The perpetual and exclusive
character of private respondents claims simply makes them repugnant to basic fairness and equality.
Private respondents and intervenors trace their "ownership" of ancestral domains and lands to the pre-
Spanish conquest. I should say that, at the time, their claims to such lands and domains was limited to
the surfaces thereof since their ancestors were agriculture-based. This must be the continuing scope of
the indigenous groups ownership claims: limited to land, excluding the natural resources found within.
In any event, if all that the ICCs/IPs demand is preferential use - not ownership - of ancestral domains,
then I have no disagreement. Indeed, consistent with the Constitution is IPRAs Section 57
19
- without the
too-broad definitions under Section 3 (a) and (b) - insofar as it grants them priority rights in harvesting,
extracting, developing or exploiting natural resources within ancestral domains.
The concerted effort to malign the Regalian Doctrine as a vestige of the colonial past must fail. Our
Constitution vests the ownership of natural resources, not in colonial masters, but in all the Filipino
people. As the protector of the Constitution, this Court has the sworn duty to uphold the tenets of that
Constitution - not to dilute, circumvent or create exceptions to them.
Cario v. Insular Government Was Modified by the Constitution
In this connection, I submit that Cario v. Insular Government
20
has been modified or superseded by our
1935, 1973 and 1987 Constitutions. Its ratio should be understood as referring only to a means by which
public agricultural land may be acquired by citizens. I must also stress that the claim of Petitioner Cario
refers to land ownership only, not to the natural resources underneath or to the aerial and cosmic space
above.
Significantly, in Director of Land Management v. Court of Appeals,
21
a Decision handed down after our
three Constitutions had taken effect, the Court rejected a cultural minority member's registration of land
under CA 141, Section 48 (c).
22
The reason was that the property fell within the Central Cordillera Forest
Reserve. This Court quoted with favor the solicitor generals following statements:
"3. The construction given by respondent Court of Appeals to the particular provision of law involved, as
to include even forest reserves as susceptible to private appropriation, is to unconstitutionally apply such
provision. For, both the 1973 and present Constitutions do not include timber or forest lands as alienable.
Thus, Section 8, Article XIV of 1973 Constitution states that with the exception of agricultural, industrial or
commercial, residential and resettlement lands of the public domain, natural resources shall not be
alienated. The new Constitution, in its Article XII, Section 2, also expressly states that with the exception
of agricultural lands, all other natural resources shall not be alienated."
Just recently, in Gordula v. Court of Appeals,
23
the Court also stated that "forest land is incapable of
registration, and its inclusion in a title nullifies that title. To be sure, the defense of indefeasiblity of a
certificate of title issued pursuant to a free patent does not lie against the state in an action for reversion
of the land covered thereby when such land is a part of a public forest or of a forest reservation, the
patent covering forest land being void ab initio."
RA 8371 Violates the Inalienability of Natural Resources and of Public Domains
The ponencia theorizes that RA 8371 does not grant to ICCs/IPs ownership of the natural resources
found within ancestral domains. However, a simple reading of the very wordings of the law belies this
statement.
Section 3 (a)
24
defines and delineates ancestral domains as "all areas generally belonging to ICCs/IPs
comprising lands, inland waters, coastal areas, and natural resources therein, held under a claim of
ownership, occupied or possessed by ICCs/IPs, by themselves or through their ancestors, communally or
individually since time immemorial, continuously to the present except when interrupted by war, force
majeure or displacement x x x. It shall include ancestral lands, forests, pasture, residential, agricultural,
and other lands individually owned whether alienable and disposable or otherwise, hunting grounds x x x
bodies of water, mineral and other natural resourcesx x x." (Emphasis ours.)
Clearly, under the above-quoted provision of IPRA, ancestral domains of ICCs/IPs encompass the natural
resources found therein. And Section 7 guarantees recognition and protection of their rights of ownership
and possession over such domains.
The indigenous concept of ownership, as defined under Section 5 of the law, "holds that ancestral
domains are the ICCs/IPs private but community property which belongs to all generations and therefore
cannot be sold, disposed or destroyed." Simply put, the law declares that ancestral domains, including the
natural resources found therein, are owned by ICCs/IPs and cannot be sold, disposed or destroyed. Not
only does it vest ownership, as understood under the Civil Code; it adds perpetual exclusivity. This means
that while ICCs/IPs could own vast ancestral domains, the majority of Filipinos who are not indigenous
can never own any part thereof.
On the other hand, Section 3 (b)
25
of IPRA defines ancestral lands as referring to "lands occupied,
possessed and utilized by individuals, families and clans of the ICCs/IPs since time immemorial x x x,
under claims of individual or traditional group ownership, x x x including, but not limited to, residential lots,
rice terraces or paddies, private forests, swidden farms and tree lots." Section 8 recognizes and protects
"the right of ownership and possession of ICCs/IPs to their ancestral lands." Such ownership need not be
by virtue of a certificate of title, but simply by possession since time immemorial.
I believe these statutory provisions directly contravene Section 2, Article XII of the Constitution, more
specifically the declaration that the State owns all lands of the public domain, minerals and natural
resources none of which, except agricultural lands, can be alienated. In several cases, this Court has
consistently held that non-agricultural land must first be reclassified and converted into alienable or
disposable land for agricultural purposes by a positive act of the government.
26
Mere possession or
utilization thereof, however long, does not automatically convert them into private properties.
27
The
presumption is that "all lands not appearing to be clearly within private ownership are presumed to belong
to the State. Hence, x x x all applicants in land registration proceedings have the burden of overcoming
the presumption that the land thus sought to be registered forms part of the public domain. Unless the
applicant succeeds in showing by clear and convincing evidence that the property involved was acquired
by him or his ancestors either by composition title from the Spanish Government or by possessory
information title, or any other means for the proper acquisition of public lands, the property must be held
to be part of the public domain. The applicant must present competent and persuasive proof to
substantiate his claim; he may not rely on general statements, or mere conclusions of law other than
factual evidence of possession and title."
28

Respondents insist, and the ponencia agrees, that paragraphs (a) and (b) of Sections 3 are merely
definitions and should not be construed independently of the other provisions of the law. But, precisely, a
definition is "a statement of the meaning of a word or word group."
29
It determines or settles the nature of
the thing or person defined.
30
Thus, after defining a term as encompassing several items, one cannot
thereafter say that the same term should be interpreted as excluding one or more of the enumerated
items in its definition. For that would be misleading the people who would be bound by the law. In other
words, since RA 8371 defines ancestral domains as including the natural resources found therein and
further states that ICCs/IPs own these ancestral domains, then it means that ICCs/IPs can own natural
resources.
In fact, Intervenors Flavier et al. submit that everything above and below these ancestral domains, with no
specific limits, likewise belongs to ICCs/IPs. I say that this theory directly contravenes the Constitution.
Such outlandish contention further disregards international law which, by constitutional fiat, has been
adopted as part of the law of the land.
31

No Land Area Limits Are Specified by RA 8371
Under Section 3, Article XII of the Constitution, Filipino citizens may acquire no more than 12 hectares of
alienable public land, whether by purchase, homestead or grant. More than that, but not exceeding 500
hectares, they may hold by lease only.
RA 8371, however, speaks of no area or term limits to ancestral lands and domains. In fact, by their mere
definitions, they could cover vast tracts of the nation's territory. The properties under the assailed law
cover everything held, occupied or possessed "by themselves or through their ancestors, communally or
individually since time immemorial." It also includes all "lands which may no longer be exclusively
occupied by [them] but from which they traditionally had access to for their subsistence and traditional
activities, particularly the home ranges of ICCs/IPs who are still nomadic and/or shifting cultivators."
Nomadic groups have no fixed area within which they hunt or forage for food. As soon as they have used
up the resources of a certain area, they move to another place or go back to one they used to occupy.
From year to year, a growing tribe could occupy and use enormous areas, to which they could claim to
have had "traditional access." If nomadic ICCs/IPs succeed in acquiring title to their enlarging ancestral
domain or land, several thousands of hectares of land may yet be additionally delineated as their private
property.
Similarly, the Bangsa Moro people's claim to their ancestral land is not based on compounded or
consolidated title, but "on a collective stake to the right to claim what their forefathers secured for them
when they first set foot on our country."
32
They trace their right to occupy what they deem to be their
ancestral land way back to their ancient sultans and datus, who had settled in many islands that have
become part of Mindanao. This long history of occupation is the basis of their claim to their ancestral
lands.
33

Already, as of June 1998, over 2.5 million hectares have been claimed by various ICCs/IPs as ancestral
domains; and over 10 thousand hectares, as ancestral lands.
34
Based on ethnographic surveys, the
solicitor general estimates that ancestral domains cover 80 percent of our mineral resources and between
8 and 10 million of the 30 million hectares of land in the country.
35
This means that four fifths of its natural
resources and one third of the country's land will be concentrated among 12 million Filipinos constituting
110 ICCs,
36
while over 60 million other Filipinos constituting the overwhelming majority will have to share
the remaining. These figures indicate a violation of the constitutional principle of a "more equitable
distribution of opportunities, income, and wealth" among Filipinos.
RA 8371 Abdicates the State Duty to Take Full Control and Supervision of Natural Resources
Section 2, Article XII of the Constitution, further provides that "[t]he exploration, development, and
utilization of natural resources shall be under the full control and supervision of the State." The State may
(1) directly undertake such activities; or (2) enter into co-production, joint venture or production-sharing
agreements with Filipino citizens or entities, 60 percent of whose capital is owned by Filipinos.
37
Such
agreements, however, shall not exceed 25 years, renewable for the same period and under terms and
conditions as may be provided by law.
But again, RA 8371 relinquishes this constitutional power of full control in favor of ICCs/IPs, insofar as
natural resources found within their territories are concerned. Pursuant to their rights of ownership and
possession, they may develop and manage the natural resources, benefit from and share in the profits
from the allocation and the utilization thereof.
38
And they may exercise such right without any time limit,
unlike non-ICCs/IPs who may do so only for a period not exceeding 25 years, renewable for a like
period.
39
Consistent with the Constitution, the rights of ICCs/IPs to exploit, develop and utilize natural
resources must also be limited to such period.
In addition, ICCs/IPs are given the right to negotiate directly the terms and conditions for the exploration
of natural resources,
40
a right vested by the Constitution only in the State. Congress, through IPRA, has in
effect abdicated in favor of a minority group the State's power of ownership and full control over a
substantial part of the national patrimony, in contravention of our most fundamental law.
I make clear, however, that to the extent that ICCs/IPs may undertake small-scale utilization of natural
resources and cooperative fish farming, I absolutely have no objection. These undertakings are certainly
allowed under the third paragraph of Section 2, Article XII of the Constitution.
Having already disposed of the two major constitutional dilemmas wrought by RA 8371 (1) ownership of
ancestral lands and domains and the natural resources therein; and (2) the ICCs/IPs' control of the
exploration, development and utilization of such resources I believe I should no longer tackle the
following collateral issues petitioners have brought up:
1. Whether the inclusion of private lands within the coverage of ancestral domains amounts to
undue deprivation of private property
2. Whether ICCs/IPs may regulate the entry/exit of migrants
3. Whether ancestral domains are exempt from real property taxes, special levies and other forms
of exaction
4. Whether customary laws and traditions of ICCs/IPs should first be applied in the settlements of
disputes over their rights and claims
5. Whether the composition and the jurisdiction of the National Commission of Indigenous
Peoples (NCIP) violate the due process and equal protection clauses
6. Whether members of the ICCs/IPs may be recruited into the armed forces against their will
I believe that the first three of the above collateral issues have been rendered academic or, at least, no
longer of "transcendental importance," in view of my contention that the two major IPRA propositions are
based on unconstitutional premises. On the other hand, I think that in the case of the last three, it is best
to await specific cases filed by those whose rights may have been injured by specific provisions of RA
8371.
Epilogue
Section 5, Article XII of the Constitution, provides:
"SEC. 5. The State, subject to the provisions of this Constitution and national development policies and
programs, shall protect the rights of indigenous cultural communities to their ancestral lands to ensure
their economic, social, and cultural well being.
"The Congress may provide for the applicability of customary laws governing property rights and relations
in determining the ownership and extent of ancestral domain."
Clearly, there are two parameters that must be observed in the protection of the rights of ICCs/IPs: (1) the
provisions of the 1987 Constitution and (2) national development policies and programs.
Indigenous peoples may have long been marginalized in Philippine politics and society. This does not,
however, give Congress any license to accord them rights that the Constitution withholds from the rest of
the Filipino people. I would concede giving them priority in the use, the enjoyment and the preservation of
their ancestral lands and domains.
41
But to grant perpetual ownership and control of the nation's
substantial wealth to them, to the exclusion of other Filipino citizens who have chosen to live and abide by
our previous and present Constitutions, would be not only unjust but also subversive of the rule of law.
In giving ICCs/IPs rights in derogation of our fundamental law, Congress is effectively mandating "reverse
discrimination." In seeking to improve their lot, it would be doing so at the expense of the majority of the
Filipino people. Such short-sighted and misplaced generosity will spread the roots of discontent and, in
the long term, fan the fires of turmoil to a conflagration of national proportions.
Peace cannot be attained by brazenly and permanently depriving the many in order to coddle the few,
however disadvantaged they may have been. Neither can a just society be approximated by maiming the
healthy to place them at par with the injured. Nor can the nation survive by enclaving its wealth for the
exclusive benefit of favored minorities.
Rather, the law must help the powerless by enabling them to take advantage of opportunities and
privileges that are open to all and by preventing the powerful from exploiting and oppressing them. This is
the essence of social justice empowering and enabling the poor to be able to compete with the rich and,
thus, equally enjoy the blessings of prosperity, freedom and dignity.
WHEREFORE, I vote to partially GRANT the Petition and
to DECLARE as UNCONSTITUTIONAL Sections 3(a) and (b), 5, 6, 7(a) and (b), 8 and related provisions
of RA 8371.


3. G.R. No. 134209 January 24, 2006
REPUBLIC OF THE PHILIPPINES, Petitioner,
vs.
CELESTINA NAGUIAT, Respondent.
D E C I S I O N
GARCIA, J .:
Before the Court is this petition for review under Rule 45 of the Rules of Court seeking the reversal of the
Decision
1
dated May 29, 1998 of the Court of Appeals (CA) in CA-G.R. CV No. 37001 which affirmed an
earlier decision
2
of the Regional Trial Court at Iba, Zambales, Branch 69 in Land Registration Case No.
N-25-1.
The decision under review recites the factual backdrop, as follows:
This is an application for registration of title to four (4) parcels of land located in Panan, Botolan,
Zambales, more particularly described in the amended application filed by Celestina Naguiat on 29
December 1989 with the Regional Trial Court of Zambales, Branch 69. Applicant [herein respondent]
alleges, inter alia, that she is the owner of the said parcels of land having acquired them by purchase
from the LID Corporation which likewise acquired the same from Demetria Calderon, Josefina Moraga
and Fausto Monje and their predecessors-in-interest who have been in possession thereof for more than
thirty (30) years; and that to the best of her knowledge, said lots suffer no MORTGAGE or
encumbrance of whatever kind nor is there any person having any interest, legal or equitable, or in
possession thereof.
On 29 June 1990, the Republic of the Philippines [herein petitioner]. . . filed an opposition to the
application on the ground that neither the applicant nor her predecessors-in interest have been in open,
continuous, exclusive and notorious possession and occupation of the lands in question since 12 June
1945 or prior thereto; that the muniments of title and tax payment receipts of applicant do not constitute
competent and sufficient evidence of a bona-fide acquisition of the lands applied for or of his open,
continuous, exclusive and notorious possession and occupation thereof in the concept of (an) owner; that
the applicants claim of ownership in fee simple on the basis of Spanish title or grant can no longer be
availed of . . .; and that the parcels of land applied for are part of the public domain belonging to the
Republic of the Philippines not subject to private appropriation.
On 15 October 1990, the lower court issued an order of general default as against the whole world, with
the exception of the Office of the SOLICITOR General, and proceeded with the hearing of this
registration case.
After she had presented and formally offered her evidence . . . applicant rested her case.
The SOLICITOR General, thru the Provincial Prosecutor, interposed no objection to the admission of the
exhibits. Later . . . the Provincial Prosecutor manifest (sic) that the Government had no evidence to
adduce.
3

In a decision
4
dated September 30, 1991, the trial court rendered judgment for herein respondent
Celestina Naguiat, adjudicating unto her the parcels of land in question and decreeing the registration
thereof in her name, thus:
WHEREFORE, premises considered, this Court hereby adjudicates the parcels of land situated in Panan,
Botolan, Zambales, appearing on Plan AP-03-003447 containing an area of 3,131 square meters,
appearing on Plan AP-03-003446 containing an area of 15,322 containing an area of 15,387 square
meters to herein applicant Celestina T. Naguiat, of legal age, Filipino citizen, married to Rommel Naguiat
and a resident of Angeles City, Pampanga together with all the improvements existing thereon and orders
and decrees registration in her name in accordance with Act No. 496, Commonwealth Act No. 14, [should
be 141] as amended, and Presidential Decree No. 1529. This adjudication, however, is subject to the
various easements/reservations provided for under pertinent laws, presidential decrees and/or
presidential letters of instructions which should be annotated/ projected on the title to be issued. And once
this decision becomes final, let the corresponding decree of registration be immediately issued. (Words in
bracket added)
With its motion for reconsideration having been denied by the trial court, petitioner Republic went on
appeal to the CA in CA-G.R. CV No. 37001.
As stated at the outset hereof, the CA, in the herein assailed decision of May 29, 1998, affirmed that of
the trial court, to wit:
WHEREFORE, premises considered, the decision appealed from is hereby AFFIRMED.
SO ORDERED.
Hence, the Republics present recourse on its basic submission that the CAs decision "is not in
accordance with law, jurisprudence and the evidence, since respondent has not established with the
required evidence her title in fee simple or imperfect title in respect of the subject lots which would
warrant their registration under (P.D. 1529 or Public Land Act (C.A.) 141." In particular, petitioner
Republic faults the appellate court on its finding respecting the length of respondents occupation of the
property subject of her application for registration and for not considering the fact that she has not
established that the lands in question have been declassified from forest or timber zone to alienable and
disposable property.
Public forest lands or forest reserves, unless declassified and released by positive act of the Government
so that they may form part of the disposable agricultural lands of the public domain, are not capable of
private appropriation.
5
As to these assets, the rules on confirmation of imperfect title do not apply.
6
Given
this postulate, the principal issue to be addressed turns on the question of whether or not the areas in
question have ceased to have the status of forest or other inalienable lands of the public domain.
Forests, in the context of both the Public Land Act
7
and the Constitution
8
classifying lands of the public
domain into "agricultural, forest or timber, mineral lands and national parks," do not necessarily refer to a
large tract of wooded land or an expanse covered by dense growth of trees and underbrush. As we stated
in Heirs of Amunategui
9
-
A forested area classified as forest land of the public domain does not lose such classification simply
because loggers or settlers have stripped it of its forest cover. Parcels of land classified as forest land
may actually be covered with grass or planted to crops by kaingin cultivators or other farmers. "Forest
lands" do not have to be on mountains or in out of the way places. xxx. The classification is merely
descriptive of its legal nature or status and does not have to be descriptive of what the land actually looks
like. xxx
Under Section 2, Article XII of the Constitution,
10
which embodies the Regalian doctrine, all lands of the
public domain belong to the State the source of any asserted right to ownership of land.
11
All lands not
appearing to be clearly of private dominion presumptively belong to the State.
12
Accordingly, public lands
not shown to have been reclassified or released as alienable agricultural land or alienated to a private
person by the State remain part of the inalienable public domain.
13
Under Section 6 of the Public Land
Act, the prerogative of classifying or reclassifying lands of the public domain, i.e., from forest or mineral to
agricultural and vice versa, belongs to the Executive Branch of the government and not the
court.
14
Needless to stress, the onus to overturn, by incontrovertible evidence, the presumption that the
land subject of an application for registration is alienable or disposable rests with the applicant.
15

In the present case, the CA assumed that the lands in question are already alienable and disposable.
Wrote the appellate court:
The theory of [petitioner] that the properties in question are lands of the public domain cannot be
sustained as it is directly against the above doctrine. Said doctrine is a reaffirmation of the principle
established in the earlier cases . . . that open, exclusive and undisputed possession of alienable public
land for period prescribed by law creates the legal fiction whereby the land, upon completion of the
requisite period, ipso jure and without the need of judicial or other sanction, ceases to be public land and
becomes private property . (Word in bracket and underscoring added.)
The principal reason for the appellate courts disposition, finding a registerable title for respondent, is her
and her predecessor-in-interests open, continuous and exclusive occupation of the subject property for
more than 30 years. Prescinding from its above assumption and finding, the appellate court went on to
conclude, citing Director of Lands vs. Intermediate Appellate Court (IAC)
16
and Herico vs. DAR,
17
among
other cases, that, upon the completion of the requisite period of possession, the lands in question cease
to be public land and become private property.
Director of Lands, Herico and the other cases cited by the CA are not, however, winning cards for the
respondent, for the simple reason that, in said cases, the disposable and alienable nature of the land
sought to be registered was established, or, at least, not put in issue. And there lies the difference.
Here, respondent never presented the required certification from the proper government agency or official
proclamation reclassifying the land applied for as alienable and disposable. Matters of land classification
or reclassification cannot be assumed. It calls for proof.
18
Aside from tax receipts, respondent submitted
in evidence the survey map and technical descriptions of the lands, which, needless to state, provided no
information respecting the classification of the property. As the Court has held, however, these
documents are not sufficient to overcome the presumption that the land sought to be registered forms
part of the public domain.
19

It cannot be overemphasized that unwarranted appropriation of public lands has been a notorious
practice resorted to in land registration cases.
20
For this reason, the Court has made it a point to stress,
when appropriate, that declassification of forest and mineral lands, as the case may be, and their
conversion into alienable and disposable lands need an express and positive act from the government.
21

The foregoing considered, the issue of whether or not respondent and her predecessor-in-interest have
been in open, exclusive and continuous possession of the parcels of land in question is now of little
moment. For, unclassified land, as here, cannot be acquired by adverse occupation or possession;
occupation thereof in the concept of owner, however long, cannot ripen into private ownership and be
registered as title.
22

WHEREFORE, the instant petition is GRANTED and the assailed decision dated May 29, 1998 of the
Court of Appeals in CA-G.R. CV No. 37001 is REVERSED and SET ASIDE. Accordingly, respondents
application for original registration of title in Land Registration Case No. N-25-1 of the Regional Trial
Court at Iba, Zambales, Branch 69, is DENIED.
No costs.
SO ORDERED.


4.Carino v. Insular Government, 212 U.S. 449 (1909)
Carino v. Insular Government of the Philippine Islands
No. 72
Argued January 13, 1909
Decided February 23, 1909
212 U.S. 449
ERROR TO THE SUPREME COURT
OF THE PHILIPPINE ISLANDS
Syllabus
Writ of error is the general, and appeal the exceptional, method of bringing Cases to this Court. The latter
method is in the main confined to equity cases, and the former is proper to bring up a judgment of the
Supreme Court of the Philippine Islands affirming a judgment of the Court of Land Registration dismissing
an application for registration of land.
Although a province may be excepted from the operation of Act No. 926 of 1903 of the Philippine
Commission which provides for the registration and perfecting of new titles, one who actually owns
property in such province is entitled to registration under Act No. 496 of 1902, which applies to the whole
archipelago.
While, in legal theory and as against foreign nations, sovereignty is absolute, practically it is a question of
strength and of varying degree, and it is for a new sovereign to decide how far it will insist upon
theoretical relations of the subject to the former sovereign and how far it will recognize actual facts.
Page 212 U. S. 450
The acquisition of the Philippines was not for the purpose of acquiring the lands occupied by the
inhabitants, and under the Organic Act of July 1, 1902, c. 1369, 32 Stat. 691, providing that property
rights are to be administered for the benefit of the inhabitants, one who actually owned land for many
years cannot be deprived of it for failure to comply with certain ceremonies prescribed either by the acts
of the Philippine Commission or by Spanish law.
The Organic Act of the Philippines made a bill of rights embodying safeguards of the Constitution, and,
like the Constitution, extends those safeguards to all.
Every presumption of ownership is in favor of one actually occupying land for many years, and against
the government which seeks to deprive him of it, for failure to comply with provisions of a subsequently
enacted registration act.
Title by prescription against the crown existed under Spanish law in force in the Philippine Islands prior to
their acquisition by the United States, and one occupying land in the Province of Benguet for more than
fifty years before the Treaty of Paris is entitled to the continued possession thereof.
7 Phil. 132 reversed.
The facts are stated in the opinion.
Page 212 U. S. 455
MR. JUSTICE HOLMES delivered the opinion of the Court.
This was an application to the Philippine Court of Land Registration for the registration of certain land.
The application was granted by the court on March 4, 1904. An appeal was taken to the Court of First
Instance of the Province of Benguet on behalf of the government of the Philippines, and also on behalf of
the United States, those governments having taken possession of the property for public and military
purposes. The Court of First Instance found the facts and dismissed the application upon grounds of law.
This judgment was affirmed by the supreme court, 7 Phil. 132, and the case then was brought here by
writ of error.
The material facts found are very few. The applicant and plaintiff in error is an Igorot of the Province of
Benguet, where the land lies. For more than fifty years before the Treaty of
Page 212 U. S. 456
Paris, April 11, 1899, as far back as the findings go, the plaintiff and his ancestors had held the land as
owners. His grandfather had lived upon it, and had maintained fences sufficient for the holding of cattle,
according to the custom of the country, some of the fences, it seems, having been of much earlier date.
His father had cultivated parts and had used parts for pasturing cattle, and he had used it for pasture in
his turn. They all had been recognized as owners by the Igorots, and he had inherited or received the
land from his father in accordance with Igorot custom. No document of title, however, had issued from the
Spanish Crown, and although, in 1893-1894 and again in 1896-1897, he made application for one under
the royal decrees then in force, nothing seems to have come of it, unless, perhaps, information that lands
in Benguet could not be conceded until those to be occupied for a sanatorium, etc., had been designated
-- a purpose that has been carried out by the Philippine government and the United States. In 1901, the
plaintiff filed a petition, alleging ownership, under the MORTGAGE law, and the lands were registered
to him, that process, however, establishing only a possessory title, it is said.
Before we deal with the merits, we must dispose of a technical point. The government has spent some
energy in maintaining that this case should have been brought up by appeal, and not by writ of error. We
are of opinion, however, that the mode adopted was right. The proceeding for registration is likened to
bills in equity to quiet title, but it is different in principle. It is a proceeding in rem under a statute of the
type of the Torrens Act, such as was discussed in Tyler v. Court of Registration, 175 Mass. 71. It is nearer
to law than to equity, and is an assertion of legal title; but we think it unnecessary to put it into either
pigeon hole. A writ of error is the general method of bringing cases to this Court, an appeal the exception,
confined to equity in the main. There is no reason for not applying the general rule to this case. Ormsby v.
Webb,134 U. S. 47, 134 U. S. 65; Campbell v. Porter,162 U. S. 478; Metropolitan R. Co. v. District of
Columbia,195 U. S. 322.
Page 212 U. S. 457
Another preliminary matter may as well be disposed of here. It is suggested that, even if the applicant
have title, he cannot have it registered, because the Philippine Commission's Act No. 926, of 1903,
excepts the Province of Benguet among others from its operation. But that act deals with the acquisition
of new titles by homestead entries, purchase, etc., and the perfecting of titles begun under the Spanish
law. The applicant's claim is that he now owns the land, and is entitled to registration under the Philippine
Commission's Act No. 496, of 1902, which established a court for that purpose with jurisdiction
"throughout the Philippine Archipelago," 2, and authorized in general terms applications to be made by
persons claiming to own the legal estate in fee simple, as the applicant does. He is entitled to registration
if his claim of ownership can be maintained.
We come, then, to the question on which the case was decided below -- namely, whether the plaintiff
owns the land. The position of the government, shortly stated, is that Spain assumed, asserted, and had
title to all the land in the Philippines except so far as it saw fit to permit private titles to be acquired; that
there was no prescription against the Crown, and that, if there was, a decree of June 25, 1880, required
registration within a limited time to make the title good; that the plaintiff's land was not registered, and
therefore became, if it was not always, public land; that the United States succeeded to the title of Spain,
and so that the plaintiff has no rights that the Philippine government is bound to respect.
If we suppose for the moment that the government's contention is so far correct that the Crown of Spain
in form asserted a title to this land at the date of the Treaty of Paris, to which the United States
succeeded, it is not to be assumed without argument that the plaintiff's case is at an end. It is true that
Spain, in its earlier decrees, embodied the universal feudal theory that all lands were held from the
Crown, and perhaps the general attitude of conquering nations toward people not recognized as entitled
to the treatment accorded to those
Page 212 U. S. 458
in the same zone of civilization with themselves. It is true also that, in legal theory, sovereignty is
absolute, and that, as against foreign nations, the United States may assert, as Spain asserted, absolute
power. But it does not follow that, as against the inhabitants of the Philippines, the United States asserts
that Spain had such power. When theory is left on one side, sovereignty is a question of strength, and
may vary in degree. How far a new sovereign shall insist upon the theoretical relation of the subjects to
the head in the past, and how far it shall recognize actual facts, are matters for it to decide.
The Province of Benguet was inhabited by a tribe that the SOLICITOR General, in his argument,
characterized as a savage tribe that never was brought under the civil or military government of the
Spanish Crown. It seems probable, if not certain, that the Spanish officials would not have granted to
anyone in that province the registration to which formerly the plaintiff was entitled by the Spanish laws,
and which would have made his title beyond question good. Whatever may have been the technical
position of Spain, it does not follow that, in the view of the United States, he had lost all rights and was a
mere trespasser when the present government seized his land. The argument to that effect seems to
amount to a denial of native titles throughout an important part of the island of Luzon, at least, for the
want of ceremonies which the Spaniards would not have permitted and had not the power to enforce.
The acquisition of the Philippines was not like the settlement of the white race in the United States.
Whatever consideration may have been shown to the North American Indians, the dominant purpose of
the whites in America was to occupy the land. It is obvious that, however stated, the reason for our taking
over the Philippines was different. No one, we suppose, would deny that, so far as consistent with
paramount necessities, our first object in the internal administration of the islands is to do justice to the
natives, not to exploit their country for private gain. By the Organic Act of July 1, 1902, c. 1369, 12, 32
Stat. 691, all the property and rights acquired there by the
Page 212 U. S. 459
United States are to be administered "for the benefit of the inhabitants thereof." It is reasonable to
suppose that the attitude thus assumed by the United States with regard to what was unquestionably its
own is also its attitude in deciding what it will claim for its own. The same statute made a bill of rights,
embodying the safeguards of the Constitution, and, like the Constitution, extends those safeguards to all.
It provides that
"no law shall be enacted in said islands which shall deprive any person of life, liberty, or property without
due process of law, or deny to any person therein the equal protection of the laws."
5. In the light of the declaration that we have quoted from 12, it is hard to believe that the United
States was ready to declare in the next breath that "any person" did not embrace the inhabitants of
Benguet, or that it meant by "property" only that which had become such by ceremonies of which
presumably a large part of the inhabitants never had heard, and that it proposed to treat as public land
what they, by native custom and by long association -- one of the profoundest factors in human thought --
regarded as their own.
It is true that, by 14, the government of the Philippines is empowered to enact rules and prescribe terms
for perfecting titles to public lands where some, but not all, Spanish conditions had been fulfilled, and to
issue patents to natives for not more than sixteen hectares of public lands actually occupied by the native
or his ancestors before August 13, 1898. But this section perhaps might be satisfied if confined to cases
where the occupation was of land admitted to be public land, and had not continued for such a length of
time and under such circumstances as to give rise to the understanding that the occupants were owners
at that date. We hesitate to suppose that it was intended to declare every native who had not a paper title
a trespasser, and to set the claims of all the wilder tribes afloat. It is true again that there is excepted from
the provision that we have quoted as to the administration of the property and rights acquired by the
United States such land and property as shall be designated by the President for military or other
reservations,
Page 212 U. S. 460
as this land since has been. But there still remains the question what property and rights the United
States asserted itself to have acquired.
Whatever the law upon these points may be, and we mean to go no further than the necessities of
decision demand, every presumption is and ought to be against the government in a case like the
present. It might, perhaps, be proper and sufficient to say that when, as far back as testimony or memory
goes, the land has been held by individuals under a claim of private ownership, it will be presumed to
have been held in the same way from before the Spanish conquest, and never to have been public land.
Certainly, in a case like this, if there is doubt or ambiguity in the Spanish law, we ought to give the
applicant the benefit of the doubt. Whether justice to the natives and the import of the organic act ought
not to carry us beyond a subtle examination of ancient texts, or perhaps even beyond the attitude of
Spanish law, humane though it was, it is unnecessary to decide. If, in a tacit way, it was assumed that the
wild tribes of the Philippines were to be dealt with as the power and inclination of the conqueror might
dictate, Congress has not yet sanctioned the same course as the proper one "for the benefit of the
inhabitants thereof."
If the applicant's case is to be tried by the law of Spain, we do not discover such clear proof that it was
bad by that law as to satisfy us that he does not own the land. To begin with, the older decrees and laws
cited by the counsel for the plaintiff in error seem to indicate pretty clearly that the natives were
recognized as owning some lands, irrespective of any royal grant. In other words, Spain did not assume
to convert all the native inhabitants of the Philippines into trespassers, or even into tenants at will. For
instance, Book 4, Title 12, Law 14 of the Recopilacion de Leyes de las Indias, cited for a contrary
conclusion in Valenton v. Murciano, 3 Phil. 537, while it commands viceroys and others, when it seems
proper, to call for the exhibition of grants, directs them to confirm those who hold by good grants or justa
prescripcion. It is true that it
Page 212 U. S. 461
begins by the characteristic assertion of feudal overlordship and the origin of all titles in the King or his
predecessors. That was theory and discourse. The fact was that titles were admitted to exist that owed
nothing to the powers of Spain beyond this recognition in their books.
Prescription is mentioned again in the royal cedula of October 15, 1754, cited in 3 Phil. 546:
"Where such possessors shall not be able to produce title deeds, it shall be sufficient if they shall show
that ancient possession, as a valid title by prescription."
It may be that this means possession from before 1700; but, at all events, the principle is admitted. As
prescription, even against Crown lands, was recognized by the laws of Spain, we see no sufficient reason
for hesitating to admit that it was recognized in the Philippines in regard to lands over which Spain had
only a paper sovereignty.
The question comes, however, on the decree of June 25, 1880, for the adjustment of royal lands
wrongfully occupied by private individuals in the Philippine Islands. This begins with the usual theoretic
assertion that, for private ownership, there must have been a grant by competent authority; but instantly
descends to fact by providing that, for all legal effects, those who have been in possession for certain
times shall be deemed owners. For cultivated land, twenty years, uninterrupted, is enough. For
uncultivated, thirty. Art. 5. So that, when this decree went into effect, the applicant's father was owner of
the land by the very terms of the decree. But, it is said, the object of this law was to require the
adjustment or registration proceedings that it described, and in that way to require everyone to get a
document of title or lose his land. That purpose may have been entertained, but it does not appear clearly
to have been applicable to all. The regulations purport to have been made "for the adjustment of royal
lands wrongfully occupied by private individuals." (We follow the translation in the government's brief.) It
does not appear that this land ever was royal land or wrongfully occupied. In Article 6, it is provided that
"interested parties not included within the two preceding
Page 212 U. S. 462
articles [the articles recognizing prescription of twenty and thirty years] may legalize their possession, and
thereby acquire the full ownership of the said lands, by means of adjustment proceedings, to be
conducted in the following manner."
This seems, by its very terms, not to apply to those declared already to be owners by lapse of time. Article
8 provides for the case of parties not asking an adjustment of the lands of which they are unlawfully
enjoying the possession, within one year, and threatens that the treasury "will reassert the ownership of
the state over the lands," and will sell at auction such part as it does not reserve. The applicant's
possession was not unlawful, and no attempt at any such proceedings against him or his father ever was
made. Finally, it should be noted that the natural construction of the decree is confirmed by the report of
the council of state. That report puts forward as a reason for the regulations that, in view of the condition
of almost all property in the Philippines, it is important to fix its status by general rules on the principle that
the lapse of a fixed period legalizes completely all possession, recommends in two articles twenty and
thirty years, as adopted in the decree, and then suggests that interested parties not included in those
articles may legalize their possession and acquire ownership by adjustment at a certain price.
It is true that the language of Articles 4 and 5 attributes title to those "who may prove" possession for the
necessary time, and we do not overlook the argument that this means may prove in registration
proceedings. It may be that an English CONVEYANCER would have recommended an application
under the foregoing decree, but certainly it was not calculated to convey to the mind of an Igorot chief the
notion that ancient family possessions were in danger, if he had read every word of it. The words "may
prove" (acrediten), as well, or better, in view of the other provisions, might be taken to mean when called
upon to do so in any litigation. There are indications that registration was expected from all, but none
sufficient to show that, for want of it, ownership actually gained would be lost.
Page 212 U. S. 463
The effect of the proof, wherever made, was not to confer title, but simply to establish it, as already
conferred by the decree, if not by earlier law. The royal decree of February 13, 1894, declaring forfeited
titles that were capable of adjustment under the decree of 1880, for which adjustment had not been
sought, should not be construed as a confiscation, but as the withdrawal of a privilege. As a matter of
fact, the applicant never was disturbed. This same decree is quoted by the Court of Land Registration for
another recognition of the common law prescription of thirty years as still running against alienable Crown
land.
It will be perceived that the rights of the applicant under the Spanish law present a problem not without
difficulties for courts of a different legal tradition. We have deemed it proper on that account to notice the
possible effect of the change of sovereignty and the act of Congress establishing the fundamental
principles now to be observed. Upon a consideration of the whole case, we are of opinion that law and
justice require that the applicant should be granted what he seeks, and should not be deprived of what,
by the practice and belief of those among whom he lived, was his property, through a refined
interpretation of an almost forgotten law of Spain.
Judgment reversed.
Official Supreme Court case law is only found in the print version of the United States Reports. Justia
case law is provided for general informational purposes only, and may not reflect current legal
developments, verdicts or settlements. We make no warranties or guarantees about the accuracy,
completeness, or adequacy of the information contained on this site or information linked to from this site.
Please check official sources.

5. G.R. No. L-59731 January 11, 1990
ALFREDO CHING, petitioner,
vs.
THE HONORABLE COURT OF APPEALS & PEDRO ASEDILLO, respondents.
Joaquin E. Chipeco & Lorenzo D. Fuggan for petitioners.
Edgardo Salandanan for private respondent.

PARAS, J .:
This is a petition for review on certiorari which seeks to nullify the decision of respondent Court of
Appeals (penned by Hon. Rodolfo A. Nocon with the concurrence of Hon. Crisolito Pascual and Juan A.
Sison) in CA-G.R. No. 12358-SP entitled Alfredo Ching v. Hon. M. V. Romillo, et al. which in effect
affirmed the decision of the Court of First Instance of Rizal, now Regional Trial Court (penned by Judge
Manuel V. Romillo, Jr. then District Judge, Branch XXVII Pasay City) granting ex-parte the cancellation of
title registered in the name of Ching Leng in favor of Pedro Asedillo in Civil Case No. 6888-P entitled
Pedro Asedillo v. Ching Leng and/or Estate of Ching Leng.
The facts as culled from the records disclose that:
In May 1960, Decree No. N-78716 was issued to spouses Maximo Nofuente and Dominga Lumandan in
Land Registration Case No. N-2579 of the Court of First Instance of Rizal and Original Certificate of Title
No. 2433 correspondingly given by the Register of Deeds for the Province of Rizal covering a parcel of
land situated at Sitio of Kay-Biga Barrio of San Dionisio, Municipality of Paranaque, Province of Rizal,
with an area of 51,852 square meters (Exhibit "7", p. 80, CA, Rollo).
In August 1960, 5/6 portion of the property was reconveyed by said spouses to Francisco, Regina,
Perfects, Constancio and Matilde all surnamed Nofuente and Transfer Certificate of Title No. 78633 was
issued on August 10, 1960 accordingly (Exhibit "8", pp. 81 and 82, Ibid.).
By virtue of a sale to Ching Leng with postal address at No. 44 Libertad Street, Pasay City, Transfer
Certificate of Title No. 91137 was issued on September 18, 1961 and T.C.T. No. 78633 was deemed
cancelled. (Exhibit "5-2", pp. 76-77 and 83, Ibid.).
On October 19, 1965, Ching Leng died in Boston, Massachusetts, United States of America. His
legitimate son Alfredo Ching filed with the Court of First Instance of Rizal (now RTC) Branch III, Pasay
City a petition for administration of the estate of deceased Ching Leng docketed as Sp. Proc. No. 1956-P.
Notice of hearing on the petition was duly published in the "Daily Mirror", a newspaper of general
circulation on November 23 and 30 and December 7, 1965. No oppositors appeared at the hearing on
December 16, 1965, consequently after presentation of evidence petitioner Alfredo Ching was appointed
administrator of Ching Leng's estate on December 28, 1965 and letters of administration issued on
January 3, 1966 (pp. 51-53, Rollo). The land covered by T.C.T. No. 91137 was among those included in
the inventory submitted to the court (p. 75, Ibid.).
Thirteen (13) years after Ching Leng's death, a suit against him was commenced on December 27, 1978
by private respondent Pedro Asedillo with the Court of First Instance of Rizal (now RTC), Branch XXVII,
Pasay City docketed as Civil Case No. 6888-P for reconveyance of the abovesaid property and
cancellation of T.C.T. No. 91137 in his favor based on possession (p. 33, Ibid.). Ching Leng's last known
address is No. 44 Libertad Street, Pasay City which appears on the face of T.C.T. No. 91137
(not No. 441 Libertad Street, Pasay City, as alleged in private respondent's complaint). (Order dated May
29, 1980, p. 55, Ibid.). An amended complaint was filed by private respondent against Ching Leng and/or
Estate of Ching Leng on January 30, 1979 alleging "That on account of the fact that the defendant has
been residing abroad up to the present, and it is not known whether the defendant is still alive or dead, he
or his estate may be served by summons and other processes only by publication;" (p. 38, Ibid.).
Summons by publication to Ching Leng and/or his estate was directed by the trial court in its order dated
February 7, 1979. The summons and the complaint were published in the "Economic Monitor", a
newspaper of general circulation in the province of Rizal including Pasay City on March 5, 12 and 19,
1979. Despite the lapse of the sixty (60) day period within which to answer defendant failed to file a
responsive pleading and on motion of counsel for the private respondent, the court a quo in its order
dated May 25, 1979, allowed the presentation of evidence ex-parte. A judgment by default was rendered
on June 15, 1979, the decretal portion of which reads:
WHEREFORE, finding plaintiffs causes of action in the complaint to be duly substantiated
by the evidence, judgment is hereby rendered in favor of the plaintiff and against the
defendant declaring the former (Pedro Asedillo) to be the true and absolute owner of the
property covered by T.C.T. No. 91137; ordering the defendant to reconvey the said
property in favor of the plaintiff; sentencing the defendant Ching Leng and/or the
administrator of his estate to surrender to the Register of Deeds of the Province of Rizal
the owner's copy of T.C.T. No. 91137 so that the same may be cancelled failing in which
the said T.C.T. No. 91137 is hereby cancelled and the Register of Deeds of the Province
of Rizal is hereby ordered to issue, in lieu thereof, a new transfer certificate of title over
the said property in the name of the plaintiff Pedro Asedillo of legal age, and a resident of
Estrella Street, Makati, Metro Manila, upon payment of the fees that may be required
therefor, including the realty taxes due the Government.
IT IS SO ORDERED. (pp. 42-44, Ibid.)
Said decision was likewise served by publication on July 2, 9 and 16, 1979 pursuant to Section 7 of Rule
13 of the Revised Rules of Court (CA Decision, pp. 83-84, Ibid.). The title over the property in the name of
Ching Leng was cancelled and a new Transfer Certificate of Title was issued in favor of Pedro Asedillo (p.
77, CA Rollo) who subsequently sold the property to Villa Esperanza Development, Inc. on September 3,
1979 (pp. 125-126, Ibid.).
On October 29, 1979 petitioner Alfredo Ching learned of the abovestated decision. He filed a verified
petition on November 10, 1979 to set it aside as null and void for lack of jurisdiction which was granted by
the court on May 29, 1980 (penned by Hon. Florentino de la Pena, Vacation Judge, pp. 54-59, Rollo).
On motion of counsel for private respondent the said order of May 29, 1980 was reconsidered and set
aside, the decision dated June 15, 1979 aforequoted reinstated in the order dated September 2, 1980.
(pp. 60-63, Ibid.)
On October 30, 1980, petitioner filed a motion for reconsideration of the said latter order but the same
was denied by the trial court on April 12, 1981 (pp. 77-79, Ibid.)
Petitioner filed an original petition for certiorari with the Court of Appeals but the same was dismissed on
September 30, 1981. His motion for reconsideration was likewise denied on February 10, 1982 (pp. 81-
90, Ibid.)
Private respondent Pedro Asedillo died on June 7, 1981 at Makati, Metro Manila during the pendency of
the case with the Court of Appeals (p. 106, CA Rollo).
Hence, the instant petition.
Private respondent's comment was filed on June 1, 1982 (p. 117, Ibid.) in compliance with the resolution
dated April 26, 1982 (p. 109, Ibid.) Petitioner filed a reply to comment on June 18, 1982 (p. 159, Ibid ),
and the Court gave due course to the petition in the resolution of June 28, 1982 (p. 191, Ibid.)
Petitioner raised the following:
ASSIGNMENTS OF ERROR
I
WHETHER OR NOT A DEAD MAN CHING LENG AND/OR HIS ESTATE MAY BE
VALIDLY SERVED WITH SUMMONS AND DECISION BY PUBLICATION.
II
WHETHER OR NOT AN ACTION FOR RECONVEYANCE OF PROPERTY AND
CANCELLATION OF TITLE IS IN PERSONAM, AND IF SO, WOULD A DEAD MAN
AND/OR HIS ESTATE BE BOUND BY SERVICE OF SUMMONS AND DECISION BY
PUBLICATION.
III
WHETHER OR NOT THE PROCEEDINGS FOR RECONVEYANCE AND
CANCELLATION OF TITLE CAN BE HELD EX-PARTE.
IV
WHETHER OR NOT THE TRIAL COURT ACQUIRED JURISDICTION OVER THE
SUBJECT MATTER AND THE PARTIES.
V
WHETHER OR NOT PRIVATE RESPONDENT IS GUILTY OF LACHES IN
INSTITUTING THE ACTION FOR RECONVEYANCE AFTER THE LAPSE OF 19
YEARS FROM THE TIME THE DECREE OF REGISTRATION WAS ISSUED.
Petitioner's appeal hinges on whether or not the Court of Appeals has decided a question of substance in
a way probably not in accord with law or with the applicable decisions of the Supreme Court.
Petitioner avers that an action for reconveyance and cancellation of title is in personam and the court a
quo never acquired jurisdiction over the deceased Ching Leng and/or his estate by means of service of
summons by publication in accordance with the ruling laid down in Ang Lam v. Rosillosa et al., 86 Phil.
448 [1950].
On the other hand, private respondent argues that an action for cancellation of title is quasi in rem, for
while the judgment that may be rendered therein is not strictly a judgment in in rem, it fixes and settles the
title to the property in controversy and to that extent partakes of the nature of the judgment in rem, hence,
service of summons by publication may be allowed unto Ching Leng who on the face of the complaint
was a non-resident of the Philippines in line with the doctrine enunciated in Perkins v. Dizon, 69 Phil. 186
[1939].
The petition is impressed with merit.
An action to redeem, or to recover title to or possession of, real property is not an action in rem or an
action against the whole world, like a land registration proceeding or the PROBATE of a will; it is an
action in personam, so much so that a judgment therein is binding only upon the parties properly
impleaded and duly heard or given an opportunity to be heard. Actions in personam and actions in
rem differ in that the former are directed against specific persons and seek personal judgments, while the
latter are directed against the thing or property or status of a person and seek judgments with respect
thereto as against the whole world. An action to recover a parcel of land is a real action but it is an
action in personam, for it binds a particular individual only although it concerns the right to a tangible thing
(Ang Lam v. Rosillosa, supra).
Private respondent's action for reconveyance and cancellation of title being in personam, the judgment in
question is null and void for lack of jurisdiction over the person of the deceased defendant Ching Leng.
Verily, the action was commenced thirteen (13) years after the latter's death. As ruled by this Court
in Dumlao v. Quality Plastic Products, Inc. (70 SCRA 475 [1976]) the decision of the lower court insofar
as the deceased is concerned, is void for lack of jurisdiction over his person. He was not, and he could
not have been validly served with summons. He had no more civil personality. His juridical personality,
that is fitness to be subject of legal relations, was lost through death (Arts. 37 and 42 Civil Code).
The same conclusion would still inevitably be reached notwithstanding joinder of Ching Leng's estate as
co-defendant. it is a well-settled rule that an estate can sue or be sued through an executor or
administrator in his representative capacity (21 Am. Jr. 872). Contrary to private respondent's claims,
deceased Ching Leng is a resident of 44 Libertad Street, Pasay City as shown in his DEATH
CERTIFICATE and T. C. T. No. 91137 and there is an on-going intestate proceedings in the same
court, Branch III commenced in 1965, and notice of hearing thereof duly published in the same year.
Such misleading and misstatement of facts demonstrate lack of candor on the part of private respondent
and his counsel, which is censurable.
The complaint for cancellation of Ching Leng's Torrens Title must be filed in the original land registration
case, RTC, Pasig, Rizal, sitting as a land registration court in accordance with Section 112 of the Land
Registration Act (Act No. 496, as amended) not in CFI Pasay City in connection with, or as a mere
incident in Civil Case No. 6888-P (Estanislao v. Honrado, 114 SCRA 748 [1982]).
Section 112 of the same law requires "notice to all parties in interest." Since Ching Leng was already in
the other world when the summons was published he could not have been notified at all and the trial court
never acquired jurisdiction over his person. The ex-parte proceedings for cancellation of title could not
have been held (Estanislao v. Honrado, supra).
The cited case of Perkins v. Dizon, supra is inapplicable to the case at bar since petitioner Perkins was a
non-resident defendant sued in Philippine courts and sought to be excluded from whatever interest she
has in 52,874 shares of stocks with Benguet Consolidated MINING COMPANY . The action being
a quasi in rem summons by publication satisfied the constitutional requirement of due process.
The petition to set aside the judgment for lack of jurisdiction should have been granted and the amended
complaint of private respondent based on possession and filed only in 1978 dismissed outrightly. Ching
Leng is an innocent purchaser for value as shown by the evidence adduced in his behalf by petitioner
herein, tracing back the roots of his title since 1960, from the time the decree of registration was issued.
The sole remedy of the landowner whose property has been wrongfully or erroneously registered in
another's nameafter one year from the date of the decreeis not to set aside the decree, but
respecting the decree as incontrovertible and no longer open to review, to bring an ordinary action in the
ordinary court of justice for damages if the property has passed unto the hands of an innocent purchaser
for value (Sy, Sr. v. Intermediate Appellate Court, G.R. No. 66742; Teoville Development Corporation v.
IAC, et al., G.R. No. 75011, June 16, 1988).
Failure to take steps to assert any rights over a disputed land for 19 years from the date of registration of
title is fatal to the private respondent's cause of action on the ground of laches. Laches is the failure or
neglect, for an unreasonable length of time to do that which by exercising due diligence could or should
have been done, earlier; it is negligence or omission to assert a right within a reasonable time warranting
a presumption that the party entitled to assert it either has abandoned it or declined to assert it (Bailon-
Casilao v. Court of Appeals, G.R. No. 78178, April 15, 1988; Villamor v. Court of Appeals, G.R. No.
41508, June 27, 1988).
The real purpose of the Torrens system is to quiet title to land and to stop forever any question as to its
legality. Once a title is registered, the owner may rest secure, without the necessity of waiting in the
portals of the court, or sitting on the "mirador su casa," to avoid the possibility of losing his land (National
Grains Authority v. IAC, 157 SCRA 388 [1988]).
A Torrens title is generally a conclusive evidence of the ownership of the land referred to therein (Section
49, Act 496). A strong presumption exists that Torrens titles are regularly issued and that they are valid. A
Torrens title is incontrovertible against any "information possessoria" or title existing prior to the issuance
thereof not annotated on the title (Salamat Vda. de Medina v. Cruz, G.R. No. 39272, May 4, 1988).
PREMISES CONSIDERED, (1) the instant petition is hereby GRANTED; (2) the appealed decision of the
Court of Appeals is hereby REVERSED and SET ASIDE; (3) the trial court's decision dated June 15,
1979 and the Order dated September 2, 1980 reinstating the same are hereby declared NULL and VOID
for lack of jurisdiction and (4) the complaint in Civil Case No. 6888-P is hereby DISMISSED.
SO ORDERED.
Melencio-Herrera, Sarmiento and Regalado, JJ., concur.
Padilla, J., took no part.

6. THIRD DIVISION


DINAH C. CASTILLO,
Petitioner,



- versus-



ANTONIO M. ESCUTIN, AQUILINA A.
MISTAS,MARIETTA L. LINATOC, AND THE
HONORABLE COURT OF APPEALS,
Respondents.
G.R. No. 171056

Present:

YNARES-SANTIAGO, J.
Chairperson,
MARTINEZ,
CHICO-NAZARIO,
NACHURA, and
PERALTA, JJ.



Promulgated:

March 13, 2009
x- - - - - - - - - - - - - - - - - - - - - - - - - - - - - - - - - - - - - - - - - - - - - - - - - - -x


D E C I S I O N


CHICO-NAZARIO, J.:


Before this Court is a Petition for Review on Certiorari
[1]
under Rule 45 of the Rules of Court filed by
petitioner Dinah C. Castillo seeking the reversal and setting aside of the Decision,
[2]
dated 18 October 2005, of the
Court of Appeals in CA-G.R. SP No. 90533, as well as the Resolution,
[3]
dated 11 January 2006 of the same court
denying reconsideration of its afore-mentioned Decision. The Court of Appeals, in its assailed Decision, affirmed
the Joint Resolution
[4]
dated 28 April 2004 and Joint Order
[5]
dated 20 June 2005 of the Office of the Deputy
Ombudsman for Luzon in OMB-L-A-03-0573-F and OMB-L-C-03-0728-F, dismissing petitioner Dinah C. Castillos
complaint for grave misconduct and violation of Section 3(e) of Republic Act No. 3019, the Anti-Graft and Corrupt
Practices Act, as amended, against respondent public officers Antonio M. Escutin (Escutin), Aquilina A. Mistas
(Mistas) and Marietta L. Linatoc (Linatoc), together with private individuals Lauro S. Leviste II (Leviste) and
Benedicto L. Orense (Orense).

Petitioner is a judgment creditor of a certain Raquel K. Moratilla (Raquel), married to Roel
Buenaventura. In the course of her search for properties to satisfy the judgment in her favor, petitioner
discovered that Raquel, her mother Urbana Kalaw (Urbana), and sister Perla K. Moratilla (Perla), co-
owned Lot 13713, a parcel of land consisting of 15,000 square meters, situated at Brgy. Bugtongnapulo, Lipa City,
Batangas, and covered by Tax Declaration No. 00449.

Petitioner set about verifying the ownership of Lot 13713. She was able to secure an Order
[6]
dated 4
March 1999 issued by Secretary Horacio R. Morales, Jr. of the Department of Agrarian Reform (DAR) approving the
application of Summit Point Golf & Country Club, Inc. for conversion of several agricultural landholdings, including
Lot 13713 owned by Perla K. Mortilla, et al. and covered by Tax Declaration No. 00449, to residential,
commercial, and recreational uses. She was also able to get from the Office of the City Assessor, Lipa City, a
Certification
[7]
stating that Lot 13713, covered by Tax Declaration No. 00554-A, was in the name of co-owners
Raquel, Urbana, and Perla; and a certified true copy of Tax Declaration No. 00554-A itself.
[8]
Lastly, the Register of
Deeds of Lipa City issued a Certification
[9]
attesting that Lot 13713 in the name of co-owners Raquel, Urbana, and
Perla, was not covered by a certificate of title, whether judicial or patent, or subject to the issuance of a Certificate
of Land Ownership Award or patent under the Comprehensive Agrarian Reform Program.

Only thereafter did petitioner proceed to levy on execution Lot 13713, and the public auction sale of the
same was scheduled on 14 May 2002. Sometime in May 2002, before the scheduled public auction sale, petitioner
learned that Lot 13713 was inside the Summit Point Golf and Country Club Subdivision owned by Summit Point
Realty and Development Corporation (Summit Realty). She immediately went to the Makati City office of Summit
Realty to meet with its Vice President,Orense. However, she claimed that Orense did not show her any document
to prove ownership of Lot 13713 by Summit Realty, and even threatened her that the owners of Summit Realty,
the Leviste family, was too powerful and influential for petitioner to tangle with.

The public auction sale pushed through on 14 May 2002, and petitioner bought Raquels 1/3 pro-
indiviso share in Lot 13713.

On 4 June 2002, petitioner had the following documents, on her acquisition of Raquels 1/3 pro-
indiviso share in Lot 13713, recorded in the Primary Entry Book and Registration Book of the Register of Deeds of
Lipa City in accordance with Act No. 3344
[10]
: (a) Notice of Levy;
[11]
(b) Certificate of Sale;
[12]
(c) Affidavit of
Publication;
[13]
and (d) Writ of Execution.
[14]


Subsequently, petitioner was issued by the City Assessor of Lipa City Tax Declaration No. 00942-
A,
[15]
indicating that she owned 5,000 square meters of Lot 13713, while Urbana and Perla owned the other 10,000
square meters.

When petitioner attempted to pay real estate taxes for her 5,000-square-meter share in Lot 13713, she
was shocked to find out that, without giving her notice, her Tax Declaration No. 00942-A was cancelled. Lot 13713
was said to be encompassed in and overlapping with the 105,648 square meter parcel of land known as Lot 1-B,
covered by Transfer Certificate of Title (TCT) No. 129642
[16]
and Tax Declaration No. 00949-A,
[17]
both in the name
of Francisco Catigbac (Catigbac). The reverse side of TCT No. 129642 bore three entries, reflecting the supposed
sale of Lot 1-B to Summit Realty, to wit:

ENTRY NO. 184894: SPECIAL POWER OF ATTORNEY : In favor of LEONARDO YAGIN: For
purposes more particularly stipulated in the contract ratified before Atty. Ernesto M. Vergara
of Lipa City as per Doc. No. 639; Page No. 29; Book No. LXXVI; Series of 1976.
Date of instrument 2-6-1976
Date of inscription 6-26-2002 at 11:20 a.m.

ENTRY NO. 185833: SALE IN FAVOR OF SUMMIT POINT REALTY & DEVELOPMENT CORP:
ENTRY NO. 185834: BIR CLEARANCE: Of the parcel of land described in this cert. of title is
hereby sold and cancelled TCT No. 134609(SN-6672938) Vol. 671-A, having been issued by virtue
of the aforesaid instrument ratified before Perfecto L. Dimayuga, Notary Public for Makati City as
per Doc. No. 148; Page 31, Book No. LXVII, Series of 2002.
Date of instrument: July 22, 2002
Date of inscription: July 25, 2002 at 2:30 P.M.
[18]



On 25 July 2002, at 2:30 p.m., TCT No. 129642 in the name of Catigbac was cancelled and TCT No. T-
134609 in the name of Summit Realty was issued in its place.

The foregoing incidents prompted petitioner to file a Complaint Affidavit
[19]
before the Office of the
Deputy Ombudsman for Luzon charging several public officers and private individuals as follows:

32. I respectfully charge that on or about the months of June 2002 and July 2002 and
onwards in Lipa City, Atty. Antonio M. [Escutin], the Register of Deeds of Lipa City[;]Aquilina A.
Mistas, the Local Assessment Operations Officer III of the City Assessors Office of Lipa
City[;] Marietta Linatoc, Records Clerk, Office of the City Assessor of Lipa City, who are public
officers and acting in concert and conspiring with Lauro S. Leviste II and Benedicto L. Orense,
Executive Vice-President and Vice-President, respectively[,] of Summit Point Realty and
Development Corporation x x x while in the discharge of their administrative functions did then
and there unlawfully, through evident bad faith, gross inexcusable negligence and with manifest
partiality towards Summit caused me injury in the sum of P20,000,000.00 by cancelling my TD
#00942-A in the Office of the City Assessor of Lipa City and instead issuing in the name of
Francisco Catigbac TC #00949-A when aforesaid personalities well knew that TCT No. 129642 was
already cancelled and therefore not legally entitled to a new tax declaration thereby manifestly
favoring Summit Point Realty and Development Corporation who now appears to be the
successor-in-interest of Francisco Catigbac, all to my damage and prejudice.
[20]
(Emphasis ours.)

Petitioners Complaint Affidavit gave rise to simultaneous administrative and preliminary (criminal)
investigations, docketed as OMB-L-A-03-0573-F and OMB-L-C-03-0728-F, respectively.

Petitioner pointed out several irregularities in the circumstances surrounding the alleged sale of Lot 1-B to
Summit Realty and in the documents evidencing the same.

The supposed Deed of Absolute Sale in favor of Summit Realty executed on 22 July 2002 by Leonardo
Yagin (Yagin), as Catigbacs attorney-in-fact, appeared to be a one-way street. It did not express the desire of
Summit Realty, as vendee, to purchase Lot 1-B or indicate its consent and conformity to the terms of the Deed. No
representative of Summit Realty signed the left margin of each and every page of said Deed. It also did not appear
from the Deed that a representative of Summit Realty presented himself before the Notary Public who notarized
the said document. The Tax Identification Numbers of Yagin, as vendor, and Summit Realty, as vendee, were not
stated in the Deed.

Petitioner also averred that, being a corporation, Summit Realty could only act through its Board of
Directors. However, when the Deed of Absolute Sale of Lot 1-B was presented for recording before the Register of
Deeds, it was not accompanied by a Secretarys Certificate attesting to the existence of a Board Resolution which
authorized said purchase by Summit Realty. There was no entry regarding such a Secretarys Certificate and/or
Board Resolution, whether on TCT No. 129642 or TCT No. T-134609. A Secretarys Certificate eventually surfaced,
but it was executed only on 30 July 2002, five days after TCT No. T-134609 in the name of Summit Realty was
already issued.

The Deed of Absolute Sale was presented before and recorded by the Register of Deeds of Lipa City on 25
July 2002 at 2:30 p.m., at exactly the same date and time TCT No. T-134609 was issued to Summit
Realty. Petitioner theorizes that for this to happen, TCT No. T-134609 was already prepared and ready even before
the presentation for recording of the Deed of Absolute Sale before the Register of Deeds.

Moreover, Catigbac had long been dead and buried. The agency Catigbac supposedly executed in favor of
Yagin was extinguished by Catigbacs death. Thus, petitioner argued, Yagin no longer had authority to execute
on 22 July 2002 the Deed of Absolute Sale of Lot 1-B in favor of Summit Realty, making the said Deed null and
void ab initio.

Petitioner asserted that Summit Realty was well-aware of Catigbacs death, having acknowledged the
same in LRC Case No. 00-0376, the Petition for Issuance of New Owners Duplicate of TCT No. 181 In Lieu of Lost
One, filed by Summit Realty before the Regional Trial Court (RTC) of Lipa City. During the ex partepresentation of
evidence in the latter part of 2000, Orense testified on behalf of Summit Realty that Catigbacs property used to
form part of a bigger parcel of land,Lot 1 of Plan Psu-12014, measuring 132,975 square meters, covered by TCT
No. 181 in the name of Catigbac; after Catigbacs death, Lot 1 was informally subdivided into several parts among
his heirs and/or successors-in-interest, some of whom again transferred their shares to other persons; Summit
Realty separately bought subdivided parts of Lot 181 from their respective owners, with a consolidated area
of 105,648 square meters, and identified as Lot 1-B after survey; despite the subdivision and transfer of ownership
of Lot 1, TCT No. 181 covering the same was never cancelled; and the owners duplicate of TCT No. 181 was lost
and the fact of such loss was annotated at the back of the original copy of TCT No. 181 with the Registry of
Deeds. Subsequently, in an Order
[21]
dated 3 January 2001, the RTC granted the Petition in LRC Case No. 00-0376
and directed the issuance of a new owners duplicate of TCT No. 181 in the name of Catigbac, under the same
terms and condition as in its original form.

Petitioner further cast doubt on the acts undertaken by Summit Realty in connection with Catigbacs
property, purportedly without legal personality and capacity. The Special Power of Attorney dated 6 February
1976 granted Yagin the right to sue on behalf of Catigbac, yet it was Summit Realty which instituted LRC Case No.
00-0376, and Yagin had no participation at all in said case. Likewise, it was not Yagin, but Orense, who, through a
letter
[22]
dated 27 June 2001, requested the cancellation of TCT No. 181 covering Lot 1 and the issuance of a new
certificate of title for Lot 1-B. Hence, it was Orenses request which resulted in the issuance of TCT No. 129642 in
the name of Catigbac, later cancelled and replaced by TCT No. T-134609 in the name of Summit Realty.

Lastly, petitioner questioned why, despite the cancellation of TCT No. 129642 in the name of Catigbac and
the issuance in its place of TCT No. T-134609 in the name of Summit Realty, it was the former cancelled title which
was used as basis for canceling petitioners Tax Declaration No. 00942-A. Tax Declaration No. 00949-A was thus
still issued in the name of Catigbac, instead of Summit Realty.

Piecing everything together, petitioner recounted in her Complaint Affidavit the alleged scheme
perpetrated against her and the involvement therein of each of the conspirators:

28. Summit Point Realty and Development Corporation went into action right after I
paid Orense a visit sometime May 2002. Summit resurrected from the grave. (sic) Francisco
Catigbac whom they knew to be long dead to face possible litigation. This is the height of malice
and bad faith on the part of Summit through its Lauro Leviste II, the Executive Vice President and
Benedicto Orense, the Vice President. I had only in my favor a tax declaration to show my
interest and ownership over the 5, 000 sq.m. of the subject parcel of land. Evidently, Leviste
and Orense came to the desperate conclusion that they needed a TCT which is a far better title
than any tax declaration.

Both then methodically commenced their evil and illegal scheme by causing on June 26,
2002 at 11:20 a.m. the inscription with the Register of Deeds of Lipa City of a purported Special
Power of Attorney in favor of Leonardo Yagin (Annex I). Next, the Deed of Absolute Sale
(Annex J) was made the following month in order to make it appear that Yaginunilaterally sold
to Summit the subject parcel of land purportedly belonging to Francisco Catigbac. Since the
latter was already dead and realizing that the agency was already extinguished, Annex J was
not signed or executed by Leviste or Orense. This fact however did not deter the two from
securing a BIR clearance on July 25, 2002. Also, on this same day, July 25, 2002, Annex J was
presented to Atty. [Escutin] at 2:30 p.m. simultaneously, at exactly the same time of 2:30
p.m. TCT No. T-134609 in Summits name was issued by Atty. *Escutin+ WITHOUT benefit of the
submission of the necessary documentation such as the Board Resolution, DAR Clearance,
Revenue Tax Receipts for documentary stamps, real property tax clearance, proof of payment of
transfer tax, tax declaration, articles of incorporation, SEC certification, license to sell and/or
certificate of registration by HLURB, etc. Without the total and lightning speed cooperation of
Atty. [Escutin] to close his eyes to the total absence of said vital documents, the desperately
needed TCT to erase my interest and ownership would not have come into existence. Atty.
[Escutin] had indeed acted in concert and in conspiracy with Leviste and Orense in producing
Annex H and Annex K.

29. Thereafter, Leviste and Orense utilized the already cancelled TCT No. 129642 in
the name of Francisco Catigbac to be the basis in seeking the cancellation of TD #00942A in my
name (Annex F). The Tax Mapping Division of the Office of City Assessor of Lipa City opined
that my 5,000 sq.m. was (sic) part and parcel of the 105,648 sq.m. covered by TCT No. 129642. A
photocopy of the Certification from said division is hereto marked and attached as Annex P,
hereof. Aquilina Mistas, the Local Assessment Operations Officer III of the Office of the City
Assessor of Lipa City then conveniently caused the disappearance of my Notice of Levy and other
supporting documents which she had personally received from me onMarch 13, 2002. For her
part of the conspiracy likewise, Marietta Linatoc, Records Clerk, forthwith cancelled by
TD#00942-A and in lieu thereof she issued TD #00949-A in the name of Francisco Catigbac. I dare
say so because Mistas and Linatoc were presented a cancelled TCT as basis for obliterating my
5,000 sq.m. The fact of cancellation is clearly stated on the posterior side of TCT No.
129642. Both can read. But the two nevertheless proceeded with dispatch in canceling my TD,
though they had ample time and opportunity to reject the request of Summit who is not even
the registered owner appearing on TCT No. 129642. Francisco Catigbac could not have been in
front of Mistas and Linatoc because he was already six feet below the ground. Mistas and
Linatoc could have demanded presentation of the document authorizing Summit in requesting
for the cancellation of my TD. Also, they could have demanded from Summit any document
transferring my interest and ownership in favor of a third party. Or, at least, they could have
annotated in Tax Declaration No. 00949-A the fact that I bought my 5,000 sq.m. from a public
auction sale duly conducted by the court sheriff. Alternatively, Linatoc and Mistas should have
advised Summit to the effect that since they already appear to be the owners of the subject
parcel of land, the new tax declaration should bear their name instead. Mistas and Linatoc
indeed conspired with Summit in the illegal and unwarranted cancellation of my TD and in
covering up the behind-the-scenes activities of Summit by making it appear that it was Francisco
Catigbac who caused the cancellation. Even Leonardo Yagin, the alleged attorney-in-fact did not
appear before Mistas and Linatoc. Yagin could not have appeared because he is rumored to be
long dead. The aforementioned acts of the two benefitted (sic)Summit through their manifest
partiality, evident bad faith and/or gross inexcusable negligence. Perhaps, there is some truth to
the rumor that Yagin is dead because he does not even have a TIN in the questioned Deed of
Absolute Sale. If indeed Yagin is already dead or inexistent[,] the allged payment of the purchase
price of P5,282,400.00 on July 25, 2002 is a mere product of the fertile imagination of Orense and
Leviste. To dispute this assertion[,] the live body of Leonardo Yagin must be presented
by Orense and Leviste.
[23]



After filing her Affidavit Complaint, petitioner attempted to have the Sheriffs Deed of Final
Sale/Conveyance of her 5,000 square meter pro-indiviso share in Lot 13713 registered with the Register of
Deeds of Lipa City. She also sought the annotation of her Affidavit of Adverse Claim on the said 5,000 square
meters on TCT No. T-134609 of Summit Realty.

Escutin, the Register of Deeds of Lipa City, relying on the finding of Examiner Juanita H. Sta. Ana (Sta.
Ana), refused to have the Sheriffs Deed of Final Sale/Conveyance registered, since:

The Sheriffs Deed of Final Sale/Conveyance is a Mode of Transfers (sic) ownership in favor
of the Plaintiff, [Dinah] C. Castillo, (sic) However[,] it happen (sic) that the presented Tax
Declaration [No.] 00942-A is already transfer (sic) in the name of the said [Dinah] C. Castillo,
therefore[,] the registration of Sheriff (sic) Final Sale is no longer necessary.
[24]



Escutin likewise denied petitioners request to have her Affidavit of Adverse Claim annotated on TCT
No. T-134609 on the following grounds:

1. The claimants (sic) rights or interest is not adverse to the registered owner. The
registered owner is Summit Point Realty and Development Corporation under Transfer
Certificate of Title No. T-134609 of the Registry of Deeds for Lipa City.

2. The records of the Registry reveals that the source of the rights or interest of the
adverse claimant is by virtue of a Levy on Execution by the Regional Trial Court Fourth Judicial
Region, Branch 30, San Pablo City, in Civil Case No. SP-4489 (1996), [Dinah] C. Castillo vs. Raquel
Buenaventura. The registered owner, Summit Point Realty and Development Corporation nor its
predecessor-in-interest are not the judgment debtor or a party in the said case. Simply stated,
there is no privity of contract between them (Consulta No. 1044 and 1119). If ever, her adverse
claim is against Raquel Buenaventura, the judgment debtor who holds no title over the
property.
[25]



Escutin did mention, however, that petitioner may elevate en consulta to the Land Registration
Authority (LRA) the denial of her request for registration of the Sheriffs Deed of Final Sale/Conveyance and
annotation of her adverse claim on TCT No. T-134609. This petitioner did on 3 July 2003.

While her Consulta was pending before the LRA, petitioner filed a Supplemental Complaint
Affidavit
[26]
and a Second Supplemental Complaint Affidavit
[27]
with the Office of the Deputy Ombudsman
for Luzon, bringing to its attention the aforementioned developments. In her Second Supplemental Complaint
Affidavit, petitioner prayed that Sta. Ana be included as a co-respondent in OMB-L-A-03-0573-F and OMB-L-C-
03-0728-F, averring that the latters actuation deprived petitioner of a factual basis for securing a new title in
her favor over her 5,000 square meter pro-indiviso share in Lot 13713, because the public auction sale of the
said property to her could never become final without the registration of the Sheriffs Deed.

The persons charged in OMB-L-A-03-0573-F and OMB-L-C-03-0728-F filed their respective Counter-
Affidavits.

Respondent Escutin clarified in his Counter Affidavit that TCT No. T-134609 reflected the same date
and time of entry of the Deed of Absolute Sale between Yagin (as Catigbacs attorney-in-fact) and Summit
Realty, i.e., 25 July 2002 at 2:30 p.m., in accordance with Section 56
[28]
of Presidential Decree No. 1529,
otherwise known as the Property Registration Decree. He emphasized that his duty as Register of Deeds to
register the Deed of Absolute Sale presented before him was purely ministerial. If the document was legal and
in due form, and there was nothing mutilated or irregular on its face, the Register of Deeds had no authority
to inquire into its intrinsic validity based upon proofs aliunde. It was not true that he allowed the registration
of the Deed of Absolute Sale notwithstanding the absence of the required documents supporting the
application for registration thereof. On the contrary, all the required documents such as the DAR Clearance,
Bureau of Internal Revenue (BIR) Certificate Authorizing Registration (CAR), Real Property Tax, Transfer Tax,
Secretarys Certificate and Articles of Incorporation of Summit Realty were submitted. While it was true that
the Secretarys Certificate did not accompany the Deed of Absolute Sale upon the presentation of the latter
for registration, Section 117 of the Property Registration Decree gives the party seeking registration five days
to comply with the rest of the requirements; and only if the party should still fail to submit the same would it
result in the denial of the registration. The License to Sell and the Housing and Land Use Regulatory Board
Registration of Summit Realty are only required when a subdivision project is presented for registration. The
use of TINs in certain documents is a BIR requirement. The BIR itself did not require from Yagin as vendor his
TIN in the Deed of Absolute Sale, and issued the CAR even in the absence thereof. The Register of Deeds,
therefore, was only bound by the CAR. As to the Certification earlier issued by the Register of Deeds of Lipa
City attesting that Lot 13713 in the name of co-owners Raquel, Urbana, and Perla, was not covered by any
certificate of title, Escutin explained that the Register of Deeds was not technically equipped to determine
whether a cadastral lot number was within a titled property or not. Lastly, Escutin denied conspiring or
participating in the cancellation of petitioners Tax Declaration No. 00942-A for, as Register of Deeds, he was
not concerned with the issuance (or cancellation) of tax declarations.

Respondent Mistas, the Assistant City Assessor for Administration of the Office of the City
Assessor, Lipa City, disputed petitioners allegations that she personally received from petitioner copies of the
Notice of Levy and other supporting documents, and that she caused the disappearance thereof. Although
she admitted that said documents were shown to her by petitioner, she referred petitioner to the Receiving
Clerk, Lynie Reyes, who accordingly received the same. Mistas maintained that she was not the custodian of
records of the Office and she should not be held responsible for the missing documents. She opined that
petitioners documents could have been among those misplaced or destroyed when the Office of the City
Assessor was flooded with water leaking from the toilet of the Office of the City Mayor. As Assistant City
Assessor for Administration, Mistas identified her main function to be the control and management of all
phases of administrative matters and support. She had no hand in the cancellation of petitioners Tax
Declaration No. 00942-A, and the issuance of Catigbacs Tax Declaration No. 00949-A for such function
pertained to another division over which she did not exercise authority. Thus, it was also not within her
function or authority to demand the presentation of certain documents to support the cancellation of
petitioners Tax Declaration No. 00942-A or to cause the annotation of petitioners interest on Catigbacs Tax
Declaration No. 00949-A.

Respondent Linatoc averred that as Local Assessment Operation Officer II of the Office of the City
Assessor, Lipa City, she was in charge of safekeeping and updating the North District Records. With respect to
the transfer of a tax declaration from one name to another, her duty was limited only to the act of preparing
the new tax declaration and assigning it a number, in lieu of the cancelled tax declaration. It was a purely
ministerial duty. She had no authority to demand the presentation of any document or question the validity
of the transfer. Neither was it within her jurisdiction to determine whether petitioners interest should have
been annotated on Catigbacs Tax Declaration No. 00949-A. Examining the documents presented in support
of the transfer of the tax declaration to anothers name was a function belonging to other divisions of the
Office of the City Assessors. The flow of work, the same as in any other ordinary transaction, mandated her to
cancel petitioners Tax Declaration No. 00942-A, and to prepare and release Catigbacs Tax Declaration No.
00949-A after the transfer had been reviewed and approved by other divisions of the Office. It was also not
true that TCT No. 129642 in the name of Catigbac was already cancelled when it was presented before the
Office of the City Assessors; the photocopy of said certificate of title with the Office bore no mark of
cancellation.

Leviste and Orense, the private individuals charged with the respondent public officers, admitted that
they were corporate officers of Summit Realty. They related that Summit Realty bought a parcel of land
measuring 105,648 square meters, later identified as Lot 1-B, previously included in TCT No. 181, then
specifically covered by TCT No. 129642, both in the name of Catigbac. As a result of such purchase, ownership
of Lot 1-B was transferred from Catigbac to Summit Realty. Summit Realty had every reason to believe in good
faith that said property was indeed owned by Catigbac on the basis of the latters certificate of title over the
same. Catigbacs right as registered owner of Lot 1-B under TCT No. 181/No. 129642, was superior to
petitioners, which was based on a mere tax declaration. Leviste andOrense rebutted petitioners assertion
that the Deed of Absolute Sale between Yagin, as Catigbacs attorney-in-fact, and Summit Realty was a one-
way street. The Deed was actually signed on the left margin by both Yagin and the representative of Summit
Realty. The inadvertent failure of the representative of Summit Realty to sign the last page of the Deed and of
both parties to indicate their TINs therein did not invalidate the sale, especially since the Deed was signed by
witnesses attesting to its due execution. Questions as regards the scope of Catigbacs Special Power of
Attorney in favor of Yagin and the effectivity of the same after Catigbacs death can only be raised in an action
directly attacking the title of Summit Realty over Lot 1-B, and not in an administrative case and/or preliminary
investigation before the Ombudsman, which constituted a collateral attack against said title. Leviste
and Orense further explained that since the owners duplicate of TCT No. 181 was lost and was judicially
ordered replaced only on 3 January 2001, entries/inscriptions were necessarily made thereon after said
date. As to Orenses failure to show petitioner any document proving ownership of Lot 1-B by Summit Realty
when the latter paid him a visit, it was not due to the lack of such documents, but because of petitioners
failure to establish her right to peruse the same. Orense also denied ever threatening petitioner during their
meeting. Finally, according to Leviste andOrense, petitioners allegations were based on mere conjectures and
unsupported by evidence. That particular acts were done or not done by certain public officials was already
beyond the control of Leviste and Orense, and just because they benefited from these acts did not mean that
they had a hand in the commission or omission of said public officials.

After more exchange of pleadings, OMB-L-A-03-0573-F and OMB-L-C-03-0728-F were finally
submitted for resolution.

In a Joint Resolution
[29]
dated 28 April 2004, the Office of the Deputy Ombudsman for Luzon gave
more credence to respondent Escutins defenses, as opposed to petitioners charges against him:

Going to the charges against respondent Escutin, he convincingly explained that he
allowed the registration of the allegedly defective Deed of Sale because he, as Register of Deeds,
has no power to look into the intrinsic validity [of] the contract presented to him for registration,
owing to the ministerial character of his function. Moreover, as sufficiently explained by said
respondent, all the documents required for the registration of the Deed of Sale were submitted
by the applicant.

We likewise find said respondents explanation satisfactory that Section 56 of P.D. 1529
mandates that the TCT bear the date of registration of the instrument on which the said TCTs
issuance was based. It is for this reason that TCT 134609 bears the same date and time as the
registration of the Deed of Absolute Sale, which deed served as basis for its issuance.

As to his denial to register *herein petitioners+ Affidavit of Adverse Claim and Sheriffs
Certificate of Final Sale, through the issuance by the Registry of Deeds Examiner Juanita H. Sta.
Ana, of the 29 June 2003 Order denying registration thereof, such matter had been raised by
herein [petitioner] in a letter-consulta to the Administrator of the Land Registration Authority
(LRA) on 03 July 2003. As the criminal and administrative charges respecting this issue is
premised, in part, on a matter still pending with the LRA, we find it premature to make a finding
on the same.

It is for the same reason that we deny the motion contained in the Second
Supplemental Complaint Affidavit praying for the inclusion, as additional respondent, of Juanita
H. Sta. Ana, who is impleaded solely on the basis of having signed, by authority of Escutin, the 29
July 2003 Order of denial of *petitioners+ application for registration.

Finally, respondent Escutin was able to successfully demonstrate, through Consulta
2103 dated 25 July 1994, wherein the denial of registration by the Examiner of the Registry of
Deeds of Quezon City was upheld by the LRA Administrator, that the (sic) it was practice in the
different Registries that Examiners are given authority by the Register to sign letters of denial.
[30]



The Office of the Deputy Ombudsman for Luzon declared in the same Joint Resolution that there was
no basis to hold respondents Mistas and Linatoc administratively or criminally liable:

In this respect, this Office notes that while [herein petitioner] alleges that Aquilina
Mistas caused the disappearance of the Notice of Levy and other supporting documents received
from [petitioner] on 13 March 2003 when she applied for the issuance of a Tax Declaration in her
favor, she did not present her receiving copy thereof showing that it was Mistas who received
said documents from her. Neither did she show that Mistas is the employee responsible for
record safekeeping.

Next, we find, as convincingly answered, the allegation that respondent Marietta
Linatoc cancelled Tax Declaration No. 00942-A and issued Tax Declaration 00949-Q (sic) on the
basis of a cancelled Transfer Certificate of Title upon the behest of Summit [Realty], which was
not the registered owner of the property.

Respondent Linatoc, meeting squarely *petitioners+ allegation, admits having physically
cancelled Tax Declaration No. 00942-A and having prepared a new declaration covering the same
property in Catigbacs *name+, as mandated by the flow of work in the City Assessors
Office. However, she denies having the authority or discretion to evaluate the correctness and
sufficiency of the documents supporting the application for the issuance of the Tax Declaration,
arguing that her official function is limited to the physical preparation of a new tax declaration,
the assignment of a new tax declaration number and the cancellation of the old tax declaration,
after the application had passed the other divisions of the City Assessors Office.

Verily, [petitioner] failed to establish that respondent Mistas and Linatoc, are the ones
officially designated to receive applications for issuance of Tax Declaration, evaluate the
sufficiency of the documents supporting such applications, and on the basis of the foregoing
recommend or order the cancellation of an existing Tax Declaration and direct the annotation of
any fact affecting the property and direct the issuance of a new tax declaration covering the
same property.

In fact, there is even a discrepancy as to the official designation of said
respondents. While [petitioner] impleads Mistas, in her capacity as Local Assessment Officer,
and Linatoc, in her capacity as Records Clerk, Mistas, in her counter-affidavit, alleges a different
designation, i.e., Assistant City Assessor for Administration, while Linatoc claims to be the Local
Assessment Operation Officer II of the City Assessors Office.

With the scope of work of said respondents not having been neatly defined by
[petitioner], this Office cannot make a definitive determination of their liability for Grave
Misconduct and violation of Section 3(e) of R.A. No. 3019, which charges both relate to the
performance or discharge of Mistas and Linatocs official duties.
[31]



Neither did the Office of the Deputy Ombudsman for Luzon find any probable cause to criminally
charge private individuals Leviste and Orense for the following reasons:

Anent private respondents, with the alleged conspiracy to unlawfully cause the transfer
of the title of [herein petitioners+ property to Summit sufficiently explained by respondent
Register of Deeds, such allegation against private respondents loses a legal leg to stand on.

Inasmuch as [petitioner] was not able to sufficiently outline the official functions of
respondents Mistas and Linatoc to pin down their specific accountabilities, the imputation that
private respondent (sic) conspired with said public respondents respecting the cancellation of
Tax Declaration No. 00942-A is likewise stripped of any factual and legal bases.
[32]


As to whether petitioner was indeed unlawfully deprived of her 5,000 square meter property, which issue
comprised the very premise of OMB-L-A-03-0573-F and OMB-L-C-03-0728-F, the Office of the Deputy Ombudsman
for Luzon ruled that such matter was not within its jurisdiction and should be raised in a civil action before the
courts of justice.

In the end, the Office of the Ombudsman decreed:

WHEREFORE premises considered, it is respectfully recommended that : (1) the
administrative case against public respondents ANTONIO M. ESCUTIN, AQUILINA A. MISTAS and
MARIETA L. LINATOC be DISMISSED, for lack of substantial evidence; and (2) the criminal case
against the same respondents including private respondent LAURO S. LEVISTE II and BENEDICTO
L. ORENSE, be DISMISSED, for lack of probable cause.
[33]



In a Joint Order
[34]
dated 20 June 2005, the Office of the Deputy Ombudsman for Luzon denied
petitioners Motion for Reconsideration.

The Office of the Deputy Ombudsman for Luzon, in its Joint Order, took notice of the Resolution dated 17
December 2002 of the LRA in Consulta No. 3483, which involved circumstances similar to those in petitioners
case. The LRA distinguished between two systems of land registration: one is the Torrens system for registered
lands under the Property Registration Decree, and the other is the system of registration for unregistered land
under Act No. 3344 (now Section 113 of the Property Registration Decree). These systems are separate and
distinct from each other. For documents involving registered lands, the same should be recorded under the
Property Registration Decree. The registration, therefore, of an instrument under the wrong system produces no
legal effect. Since it appeared that in Consulta No. 3483, the registration of the Kasulatan ng Sanglaan, the
Certificate of Sale and the Affidavit of Consolidation was made under Act No. 3344, it did not produce any legal
effect on the disputed property, because the said property was already titled when the aforementioned
documents were executed and presented for registration, and their registration should have been made under the
Property Registration Decree.

Furthermore, the Office of the Deputy Ombudsman for Luzon, in the same Joint Order, took into account
petitioners withdrawal of her appeal en consultabefore the LRA of the denial by the Register of Deeds of her
request for registration of the Sheriffs Deed of Final Sale/Conveyance and Affidavit of Adverse Claim, which
prompted the LRA Administrator to declare the consulta moot and academic. For want of a categorical declaration
on the registerability of petitioners documents from the LRA, the competent authority to rule on the said matter,
there could be no basis for a finding that respondent public officers could be held administratively or criminally
liable for the acts imputed to them.

Petitioner sought recourse from the Court of Appeals by filing a Petition for Review under Rule 43 of
the Rules of Court challenging the 28 April 2004 Joint Resolution and 20 June 2005 Joint Order of the Office of
the Deputy Ombudsman for Luzon.
[35]
The appeal was docketed as CA-G.R. SP No. 90533.

The Court of Appeals promulgated its Decision
[36]
on 18 October 2005, also finding no reason to
administratively or criminally charge respondents. Essentially, the appellate court adjudged that petitioner can
not impute corrupt motives to respondents acts:

Without evidence showing that respondents received any gift, money or other pay-off or that
they were induced by offers of such, the Court cannot impute any taint of direct corruption in the
questioned acts of respondents. Thus, any indication of intent to violate the laws or of flagrant
disregard of established rule may be negated by respondents honest belief that their acts were
sanctioned under the provisions of existing law and regulations. Such is the situation in the case
at bar. Respondent Register of Deeds acted in the honest belief that the agency recognized by
the court in LRC Case No. 00-0376 between the registered owner Francisco Catigbac and
Leonardo Yagin subsisted with respect to the conveyance or sale of Lot 1 to Summit as the
vendee, and that the Special Power of Attorney and Deed of Absolute Sale presented as evidence
during said proceedings are valid and binding. Hence, respondent Escutin was justified in
believing that there is no legal infirmity or defect in registering the documents and proceeding
with the transfer of title of Lot 1 in the name of the new owner Summit. On the other hand,
respondent Linatoc could not be held administratively liable for effecting the cancellation in the
course of ordinary flow of work in the City Assessors Office after the documents have undergone
the necessary evaluation and verification by her superiors.
[37]



The Court of Appeals referred to the consistent policy of the Supreme Court not to interfere with the
exercise by the Ombudsman of his investigatory power. If the Ombudsman, using professional judgment,
finds the case dismissible, the Court shall respect such findings, unless clothed with grave abuse of
discretion. The appellate court pronounced that there was no grave abuse of discretion on the part of the
Office of the Deputy Ombudsman for Luzon in dismissing petitioners Complaint Affidavit against respondents.

Hence, the dispositive portion of the Decision of the Court of Appeals reads:

WHEREFORE, premises considered, the present petition is hereby DISMISSED for lack of
merit. The challenged Joint Resolution dated April 28, 2004 and Joint Order dated June 20,
2005 in OMB-L-A-03-0573-F and OMB-L-C-03-0728-F are hereby AFFIRMED.
[38]



In its Resolution dated 11 January 2006, the Court of Appeals denied petitioners Motion for
Reconsideration for failing to present new matter which the appellate court had not already considered in its
earlier Decision.

Petitioner now comes before this Court via the instant Petition for Review on Certiorari, with the
following assignment of errors:

I.

THE HONORABLE COURT OF APPEALS PATENTLY ERRED IN AFFIRMING THE CANCELLATION OF
THE TAX DECLARATION 00942 OF PETITIONER IN VIOLATION OF SECTION 109 OF PRESIDENTIAL
DECREE 1529, OTHERWISE KNOWN AS THE PROPERTY REGISTRATION ACT (sic);

II.

THE HONORABLE COURT OF APPEALS PATENTLY ERRED IN RULING THAT RESPONDENTS COULD
NOT BE HELD ADMINISTRATIVELY LIABLE FOR UNDULY FAVORING SUMMIT TO THE DAMAGE
AND PREJUDICE OF PETITIONER.
[39]



The Petition at bar is without merit.

As to the first issue, petitioner invokes Section 109 of the Property, Registration Decree which
provides:

SEC. 109. Notice and replacement of lost duplicate certificate. In case of loss or theft
of an owners duplicate certificate of title, due notice under oath shall be sent by the owner or by
someone in his behalf to the Register of Deeds of the province or city where the land lies as soon
as the loss or theft is discovered. If a duplicate certificate is lost or destroyed, or cannot be
produced by a person applying for the entry of a new certificate to him or for the registration of
any new instrument, a sworn statement of the fact of such loss or destruction may be filed by the
registered owner or other person in interest and registered.

Upon the petition of the registered owner or other person in interest, the court may,
after notice and due hearing, direct the issuance of a new duplicate certificate, which shall
contain a memorandum of the fact that it is issued in place of the lost duplicate certificate, but
shall in all respects be entitled to like faith and credit as the original duplicate, and shall
thereafter be regarded as such for all purposes of this decree.


Petitioner argues that the RTC, in LRC Case No. 00-0376, only ordered the issuance of a new owners
duplicate of TCT No. 181 in lieu of the lost one. However, respondents did not only issue a new owners
duplicate of TCT No. 181, but also cancelled petitioners Tax Declaration No. 00942-A and issued in its place
Tax Declaration No. 00949-A in the name of Catigbac. Respondents did not even annotate petitioners existing
right over 5,000 square meters of Lot 1-B or notify petitioner of the cancellation of her Tax Declaration No.
00942-A. Petitioner maintains that a new owners duplicate of title is not a mode of acquiring ownership, nor
is it a mode of losing one. Under Section 109 of the Property Registration Decree, the new duplicate of title
was issued only to replace the old; it cannot cancel existing titles.

Petitioners position on this issue rests on extremely tenuous arguments and befuddled reasoning.

Before anything else, the Court must clarify that a title is different from a certificate of title. Title is
generally defined as the lawful cause or ground of possessing that which is ours. It is that which is the
foundation of ownership of property, real or personal.
[40]
Title, therefore, may be defined briefly as that which
constitutes a just cause of exclusive possession, or which is the foundation of ownership of
property.
[41]
Certificate of title, on the other hand, is a mere evidence of ownership; it is not the title to the
land itself.
[42]
Under the Torrens system, a certificate of title may be an Original Certificate of Title, which
constitutes a true copy of the decree of registration; or a Transfer Certificate of Title, issued subsequent to the
original registration.

Summit Realty acquired its title to Lot 1-B, not from the issuance of the new owners duplicate of TCT
No. 181, but from its purchase of the same from Yagin, the attorney-in-fact of Catigbac, the registered owner
of the said property. Summit Realty merely sought the issuance of a new owners duplicate of TCT No. 181 in
the name of Catigbac so that it could accordingly register thereon the sale in its favor of a substantial portion
of Lot 1 covered by said certificate, later identified as Lot 1-B. Catigbacs title to Lot 1-B passed on by sale to
Summit Realty, giving the latter the right to seek the separation of the said portion from the rest of Lot 1 and
the issuance of a certificate of title specifically covering the same. This resulted in the issuance of TCT No.
129642 in the name of Catigbac, covering Lot 1-B, which was subsequently cancelled and replaced by TCT No.
T-134609 in the name of Summit Realty.

Petitioners reliance on Section 109 of the Property Registration Decree is totally misplaced. It
provides for the requirements for the issuance of a lost duplicate certificate of title. It cannot, in any way, be
related to the cancellation of petitioners tax declaration.

The cancellation of petitioners Tax Declaration No. 00942-A was not because of the issuance of a new
owners duplicate of TCT No. 181, but of the fact that Lot 1-B, which encompassed the 5,000 square meters
petitioner lays claim to, was already covered by TCT No. 181 (and subsequently by TCT No. 129642) in the name of
Catigbac. A certificate of title issued is an absolute and indefeasible evidence of ownership of the property in favor
of the person whose name appears therein. It is binding and conclusive upon the whole world.
[43]
All persons must
take notice, and no one can plead ignorance of the registration.
[44]
Therefore, upon presentation of TCT No.
129642, the Office of the City Assessor must recognize the ownership of Lot 1-B by Catigbac and issue in his name
a tax declaration for the said property. And since Lot 1-B is already covered by a tax declaration in the name of
Catigbac, accordingly, any other tax declaration for the same property or portion thereof in the name of another
person, not supported by any certificate of title, such that of petitioner, must be cancelled; otherwise, the City
Assessor would be twice collecting a realty tax from different persons on one and the same property.

As between Catigbacs title, covered by a certificate of title, and petitioners title, evidenced only by a
tax declaration, the former is evidently far superior and is, in the absence of any other certificate of title to the
same property, conclusive and indefeasible as to Catigbacs ownership of Lot 1-B. Catigbacs certificate of title
is binding upon the whole world, including respondent public officers and even petitioner herself. Time and
again, the Court has ruled that tax declarations and corresponding tax receipts cannot be used to prove title to
or ownership of a real property inasmuch as they are not conclusive evidence of the same.
[45]
Petitioner
acquired her title to the 5,000 square meter property from Raquel, her judgment debtor who, it is important
to note, likewise only had a tax declaration to evidence her title. In addition, the Court of Appeals aptly
observed that, *c+uriously, as to how and when petitioners alleged predecessor-in-interest, Raquel K.
Moratilla and her supposed co-owners acquired portions of Lot 1 described as Lot 13713 stated in TD No.
00449, petitioner had so far remained utterly silent.
[46]


Petitioners allegations of defects or irregularities in the sale of Lot 1-B to Summit Realty by Yagin, as
Catigbacs attorney-in-fact, are beyond the jurisdiction of the Office of the Deputy Ombudsman for Luzon to
consider. It must be remembered that Summit Realty had already acquired a certificate of title, TCT No. T-
134609, in its name over Lot 1-B, which constitutes conclusive and indefeasible evidence of its ownership of
the said property and, thus, cannot be collaterally attacked in the administrative and preliminary
investigations conducted by the Office of the Ombudsman for Luzon. Section 48 of the Property Registration
Decree categorically provides that a certificate of title shall not be subject to collateral attack. It cannot be
altered, modified, or cancelled except in a direct proceeding in accordance with law. For this same reason, the
Court has no jurisdiction to grant petitioners prayer in the instant Petition for the cancellation of TCT No. T-
134609 in the name of Summit Realty.

Which now brings the Court to the second issue raised by petitioner on the administrative liability of
respondents.

Before the Court proceeds to tackle this issue, it establishes that petitioners Complaint Affidavit
before the Office of the Ombudsman for Luzon gave rise to two charges: (1) OMB-L-A-03-0573-F involved the
administrative charge for Gross Misconduct against respondent public officers; and (2) OMB-L-C-03-0728-F
concerned the criminal charge for violation of Section 3(e) of the Anti-Graft and Corrupt Practices
Act
[47]
against respondent public officers and private individuals Leviste and Orense. The Office of the Deputy
Ombudsman for Luzon, affirmed by the Court of Appeals, dismissed both charges. In the Petition at bar,
petitioner only assails the dismissal of the administrative charge for grave misconduct against respondent
public officers. Since petitioner did not raise as an issue herein the dismissal by the Office of the Deputy
Ombudsman for Luzon, affirmed by the Court of Appeals, of the criminal charge against respondent public
officers for violation of Section 3(e) of the Anti-Graft and Corrupt Practices Act, the same became final and
executory.
[48]


In Domingo v. Quimson,
[49]
the Court adopted the well-written report and recommendation of its
Clerk of Court on the administrative matter then pending and involving the charge of gross or serious
misconduct:

"Under Section 36, par. (b) [1] of PD No. 807, otherwise known as the Civil Service
Decree of the Philippines, 'misconduct' is a ground for disciplinary action. And under MC No. 8, S.
1970, issued by the Civil Service Commission on July 28, 1970, which sets the 'Guidelines in the
Application of Penalties in Administrative Cases and other Matters Relative Thereto,' the
administrative offense of 'grave misconduct' carries with it the maximum penalty of dismissal
from the service (Sec. IV-C[3], MC No. 8, S. 1970). But the term 'misconduct' as an administrative
offense has a well defined meaning. It was defined in Amosco vs. Judge Magno, Adm. Mat. No.
439-MJ, Res. September 30, 1976, as referring 'to a transgression of some established and
definite rule of action, more particularly, unlawful behavior or gross negligence by the public
officer.' It is a misconduct 'such as affects the performance of his duties as an officer and not
such only as effects his character as a private individual.' In the recent case of Oao vs. Pabato,
etc., Adm. Mat. No. 782-MJ, Res. July 29, 1977, the Court defined 'serious misconduct' as follows:

Hence, even assuming that the dismissal of the case is erroneous, this
would be merely an error of judgment and not serious misconduct. The term
`serious misconduct is a transgression of some established and definite rule of
action more particularly, unlawful behavior of gross negligence by the
magistrate. It implies a wrongful intention and not a mere error of judgment.
For serious misconduct to exist, there must be reliable evidence showing that
the judicial acts complained of were corrupt or inspired by intention to violate
the law, or were a persistent disregard of well-known legal rules. We have
previously ruled that negligence and ignorance on the part of a judge are
inexcusable if they imply a manifest injustice which cannot be explained by a
reasonable interpretation. This is not so in the case at bar. (Italics supplied.)


To reiterate, for grave misconduct to exist, there must be reliable evidence showing that the acts
complained of were corrupt or inspired by an intention to violate the law, or were a persistent disregard of
well-known legal rules. Both the Office of the Deputy Ombudsman for Luzon and the Court of Appeals found
that there was no sufficient evidence to substantiate petitioners charge of grave misconduct against
respondents. For this Court to reverse the rulings of the Office of the Deputy Ombudsman for Luzon and the
Court of Appeals, it must necessarily review the evidence presented by the parties and decide on a question of
fact. Once it is clear that the issue invites a review of the evidence presented, the question posed is one of
fact.
[50]


Factual issues are not cognizable by this Court in a Petition for Review under Rule 45 of the Rules of
Court. In order to resolve this issue, the Court would necessarily have to look into the PROBATIVE value of
the evidence presented in the proceedings below. It is not the function of the Court to reexamine or
reevaluate the evidence all over again. This Court is not a trier of facts, its jurisdiction in these cases being
limited to reviewing only errors of law that may have been committed by the lower courts or administrative
bodies performing quasi-judicial functions. It should be emphasized that findings made by an administrative
body, which has acquired expertise, are accorded not only respect but even finality by the Court. In
administrative proceedings, the quantum of evidence required is only substantial.
[51]


Absent a clear showing of grave abuse of discretion, the Court shall not disturb findings of fact. The
Court cannot weigh once more the evidence submitted, not only before the Ombudsman, but also before the
Court of Appeals. Under Section 27 of Republic Act No. 6770, findings of fact by the Ombudsman are
conclusive, as long as they are supported by substantial evidence.
[52]
Substantial evidence is the amount of
relevant evidence which a reasonable mind might accept as adequate to justify a conclusion.
[53]


The Court finds no reason to disturb the finding of the Office of the Deputy Ombudsman
for Luzon and the Court of Appeals that respondents did not commit gross misconduct. Evident from the 28
April 2004 Joint Resolution of the former and the 18 October 2005 Decision of the latter is that they arrived at
such findings only after a meticulous consideration of the evidence submitted by the parties.

Respondents were able to clearly describe their official functions and to convincingly explain that
they had only acted in accordance therewith in their dealings with petitioner and/or her
documents. Respondents also enjoy in their favor the presumption of regularity in the performance of their
official duty. The burden of proving otherwise by substantial evidence falls on petitioner, who failed to
discharge the same.

From the very beginning, petitioner was unable to identify correctly the positions held by
respondents Mistas and Linatoc at the Office of the City Assessor. How then could she even assert that a
particular action was within or without their jurisdiction to perform? While it may be true that petitioner
should have at least been notified that her Tax Declaration No. 00942-A was being cancelled, she was not able
to establish that such would be the responsibility of respondents Mistas or Linatoc. Moreover, petitioner did
not present statutory, regulatory, or procedural basis for her insistence that respondents should have done or
not done a particular act. A perfect example was her assertion that respondents Mistas and Linatoc should
have annotated her interest on Tax Declaration No. 00949-A in the name of Catigbac. However, she failed to
cite any law or rule which authorizes or recognizes the annotation of an adverse interest on a tax
declaration. Finally, absent any reliable evidence, petitioners charge that respondents conspired with one
another and with corporate officers of Summit Realty is nothing more than speculation, surmise, or
conjecture. Just because the acts of respondents were consistently favorable to Summit Realty does not mean
that there was a concerted effort to cause petitioner prejudice. Respondents actions were only consistent
with the recognition of the title of Catigbac over Lot 1-B, transferred by sale to Summit Realty, registered
under the Torrens system, and accordingly evidenced by certificates of title.

WHEREFORE, premises considered, the instant Petition for Review is hereby DENIED. The Decision
dated 18 October 2005 and Resolution dated 11 January 2006 of the Court of Appeals in CA-G.R. SP No. 90533
are hereby AFFIRMED in toto. Costs against the petitioner Dinah C. Castillo.

SO ORDERED.







7. [G.R. No. 133240. November 15, 2000]
RUDOLF LIETZ HOLDINGS, INC., petitioner, vs. THE REGISTRY OF DEEDS OF PARAAQUE
CITY, respondent.
D E C I S I O N
YNARES-SANTIAGO, J .:
The instant petition for review is filed on a pure question of law arising from the Decision rendered by
the Regional Trial Court of Paraaque City, Metro Manila, Branch 257, in LRC Case No. 97-0170.
Petitioner corporation was formerly known as Rudolf Lietz, Incorporated. On July 15, 1996, it
amended its Articles of Incorporation to change its name to Rudolf Lietz Holdings, Inc. The Amended
Articles of Incorporation was approved by the Securities and Exchange Commission on February 20,
1997.
[1]

As a consequence of its change of name, petitioner sought the amendment of the transfer
certificates of title over real properties owned by the said corporation, all of which were under the old
name, Rudolf Lietz, Incorporated. For this purpose, petitioner instituted, on November 20, 1997, a
petition for amendment of titles with the Regional Trial Court of Paraaque City, docketed as LRC Case
No. 97-0170.
[2]

The petition impleaded as respondent the Registry of Deeds of Pasay City, apparently because the
titles sought to be amended, namely, Transfer Certificates of Title Nos. 99446, 99447, 99448, 102486,
102487, 102488 and 102489,
[3]
all state that they were issued by the Registry of Deeds of Pasay
City. Petitioner likewise inadvertently alleged in the body of the petition that the lands covered by the
subject titles are located in Pasay City.
Subsequently, petitioner learned that the subject titles are in the custody of the Register of Deeds of
Paraaque City.
[4]
Hence, on February 16, 1998, petitioner filed an Ex-Parte Motion to Admit Amended
Petition.
[5]
In the attached Amended Petition,
[6]
petitioner impleaded instead as respondent the Registry of
Deeds of Paraaque City, and alleged that its lands are located in Paraaque City.
In the meantime, however, on January 30, 1998, the court a quo had dismissed the petition motu
proprio on the ground of improper venue, it appearing therein that the respondent is the Registry of
Deeds of Pasay City and the properties are located in Pasay City.
[7]

Before counsel for petitioner could receive an official copy of the aforesaid order of dismissal, he filed
with the lower court a Motion for Reconsideration.
[8]
On February 20, 1998, in view of the dismissal of the
petition, the lower court denied the Ex-Parte Motion to Admit Amended Petition.
[9]
On March 30, 1998, the
lower court denied the Motion for Reconsideration.
[10]

Petitioner, thus, is before this Court arguing that
The court a quo acted contrary to the rules and jurisprudence on the matter for the following reasons:
1. It has no power to immediately dismiss an initiatory pleading for improper venue;
2. Assuming the Order of 30 January 1998 was proper, it was nevertheless still a matter of right
on petitioners part to amend its petition in order to correct the wrong entries therein; and
3. The unassailable reality is that the subject parcels of land are located in Paraaque City, so
venue was properly laid despite that erroneous allegation in the original petition.
[11]

The Solicitor General filed on November 4, 1998 his Comment.
[12]
He contends that the trial court did
not acquire jurisdiction over the res because it appeared from the original petition that the lands are
situated in Pasay City; hence, outside the jurisdiction of the Paraaque court. Since it had no jurisdiction
over the case, it could not have acted on the motion to admit amended petition.
On February 15, 1999, petitioner filed its Reply.
[13]
It discussed the distinction between jurisdiction
and venue, and maintained that the trial court had jurisdiction over the petition, but that venue appeared
to be improperly laid based on the erroneous allegation therein on the location of the properties.
The issue involved herein is simple. May the trial court motu proprio dismiss a complaint on the
ground of improper venue? This question has already been answered inDacoycoy v. Intermediate
Appellate Court,
[14]
where this Court held that it may not.
While the ground invoked by the trial court in dismissing the petition below was clearly that of
improper venue,
[15]
the Solicitor General confuses venue with jurisdiction. A distinction between the two
must be drawn. Jurisdiction over the subject matter or nature of an action is conferred only by law.
[16]
It
may not be conferred by consent or waiver upon a court which otherwise would have no jurisdiction over
the subject matter of an action. On the other hand, the venue of an action as fixed by statute may be
changed by the consent of the parties, and an objection on improper venue may be waived by the failure
of the defendant to raise it at the proper time. In such an event, the court may still render a valid
judgment. Rules as to jurisdiction can never be left to the consent or agreement of the parties. Venue is
procedural, not jurisdictional, and hence may be waived. It is meant to provide convenience to the
parties, rather than restrict their access to the courts as it relates to the place of trial.
[17]

In Dacoycoy v. IAC, this Court ruled:
The motu proprio dismissal of petitioners complaint by respondent trial court on the ground of improper
venue is plain error, obviously attributable to its inability to distinguish between jurisdiction and venue.
Questions or issues relating to venue of actions are basically governed by Rule 4 of the Revised Rules of
Court. It is said that the laying of venue is procedural rather than substantive. It relates to the jurisdiction
of the court over the person rather than the subject matter. Provisions relating to venue establish a
relation between the plaintiff and the defendant and not between the court and the subject matter. Venue
relates to trial not to jurisdiction, touches more of the convenience of the parties rather than the substance
of the case.
xxx xxx xxx.
Dismissing the complaint on the ground of improper venue is certainly not the appropriate course of
action at this stage of the proceedings, particularly as venue, in inferior courts as well as in the courts of
first instance (now RTC), may be waived expressly or impliedly. Where the defendant fails to challenge
timely the venue in a motion to dismiss as provided by Section 4 of Rule 4 of the Rules of Court, and
allows the trial to be held and a decision to be rendered, he cannot on appeal or in a special action be
permitted to belatedly challenge the wrong venue, which is deemed waived.
Thus, unless and until the defendant objects to the venue in a motion to dismiss, the venue cannot be
truly said to have been improperly laid, as for all practical intents and purposes, the venue, though
technically wrong, may be acceptable to the parties for whose convenience the rules on venue had been
devised. The trial court cannot pre-empt the defendants prerogative to object to the improper laying of
the venue bymotu proprio dismissing the case.
Indeed, it was grossly erroneous for the trial court to have taken a procedural short-cut by
dismissing motu proprio the complaint on the ground of improper venue without first allowing the
procedure outlined in the rules of court to take its proper course. Although we are for the speedy and
expeditious resolution of cases, justice and fairness take primary importance. The ends of justice require
that respondent trial court faithfully adhere to the rules of procedure to afford not only the defendant, but
the plaintiff as well, the right to be heard on his cause.
[18]

Rule 9, Section 1 of the 1997 Rules of Civil Procedure states that defenses and objections not
pleaded either in a motion to dismiss or in the answer are deemed waived. The court may only dismiss
an action motu proprio in case of lack of jurisdiction over the subject matter, litis pendentia, res
judicata and prescription. Therefore, the trial court in this case erred when it dismissed the petition motu
proprio. It should have waited for a motion to dismiss or a responsive pleading from respondent, raising
the objection or affirmative defense of improper venue, before dismissing the petition. However, this was
overtaken by petitioners motion for leave to amend its petition.
Petitioner correctly invoked the jurisdiction of the Regional Trial Court in seeking the amendment of
its certificates of title. The jurisdiction of the Regional Trial Court over matters involving the registration of
lands and lands registered under the Torrens system is conferred by Section 2 of Presidential Decree No.
1529, The Property Registration Decree, viz:
Nature of registration proceedings; jurisdiction of courts. --- Judicial proceedings for the registration of
lands throughout the Philippines shall be in rem and shall be based on the generally accepted principles
underlying the Torrens system.
Courts of First Instance (now Regional Trial Courts) shall have exclusive jurisdiction over all applications
for original registration of title to lands, including improvements and interest therein, and over all petitions
filed after original registration of title, with power to hear and determine all questions arising upon such
applications or petitions. The court through its clerk of court shall furnish the Land Registration
Commission with two certified copies of all pleadings, exhibits, orders, and decisions filed or issued in
applications or petitions for land registration, with the exception of stenographic notes, within five days
from the filing or issuance thereof.
More specifically, jurisdiction over petitions for amendments of certificates of title, such as the one
brought below, is provided for by Section 108 of P.D. 1529, thus:
Amendment and alteration of certificates. --- No erasure, alteration, or amendment shall be made upon
the registration book after the entry of a certificate of title or of a memorandum thereon and the attestation
of the same by the Register of Deeds, except upon order of the proper Court of First Instance (now
Regional Trial Court). A registered owner or other person having an interest in registered property,
or, in proper cases, the Register of Deeds with the approval of the Commissioner of Land
Registration, may apply by petition to the court upon the ground that the registered interests of any
description, whether vested, contingent, expectant inchoate appearing on the certificate, have terminated
and ceased; or that new interest not appearing upon the certificate have arisen or been created; or that
an omission or error was made in entering a certificate or any memorandum thereon, or on any duplicate
certificate; or that the name of any person on the certificate has been changed; or that the registered
owner has married, or, if registered as married, that the marriage has been terminated and no right or
interest of heirs or creditors will thereby be affected, or that a corporation which owned registered land
and has been dissolved has not conveyed the same within three years after its dissolution; or upon any
other reasonable ground and the court may hear and determine the petition after notice to all parties in
interest, and may order the entry or cancellation of a new certificate, or grant any other relief upon such
terms and conditions, requiring security or bond if necessary, as it may consider proper: xxx. (Emphasis
ours.)
In the case at bar, the lands are located in Paraaque City, as stated on the faces of the
titles. Petitioner, thus, also correctly filed the petition in the place where the lands are situated, pursuant
to the following rule:
Venue of real actions. --- Actions affecting title to or possession of real property, or interest therein, shall
be commenced and tried in the proper court which has jurisdiction over the area wherein the real property
involved, or a portion thereof, is situated.
[19]

Petitioner, however, named as respondent the Register of Deeds of Pasay City, under the mistaken
impression that it was still the custodian of the titles to lands in Paraaque. Later, petitioner learned that a
Register of Deeds for Paraaque City had taken over the record and custody of titles therein. Petitioner,
thus, promptly moved for leave of court to amend its petition. This, to our mind, was justified. In
preparing its amended petition, petitioner likewise corrected its allegation on the location of the lands
involved.
Before the amended petition was filed, the trial court had already dismissed the petition based on
improper venue. It relied on the allegation in the petition that the lands are located in Pasay
City. However, the titles of the land, copies of which were attached to the petition, plainly show that the
lands involved are situated in Paraaque City. The trial court should have considered these annexes, as
these form an integral part of the pleading.
At the very least, the trial court should have allowed petitioner to amend its petition, for this was still a
matter of right on its part.
Amendments as a matter of right. --- A party may amend his pleading once as a matter of right at any
time before a responsive pleading is served or, in the case of a reply, at any time within ten (10) days
after it is served.
[20]

Amendments to pleadings are liberally allowed in furtherance of justice, in order that every case may
so far as possible be determined on its real facts, and in order to speed the trial of cases or prevent the
circuitry of action and unnecessary expense.
[21]
The trial court, therefore, should have allowed the
amendment proposed by petitioner for in so doing, it would have allowed the actual merits of the case to
be speedily determined, without regard to technicalities, and in the most expeditious and inexpensive
manner.
[22]

The courts should be liberal in allowing amendments to pleadings to avoid multiplicity of suits and in
order that the real controversies between the parties are presented, their rights determined and the case
decided on the merits without unnecessary delay. This liberality is greatest in the early stages of a
lawsuit, especially in this case where the amendment to the complaint was made before the trial of the
case thereby giving petitioner all the time allowed by law to answer and to prepare for trial.
[23]

WHEREFORE, the petition for review is GRANTED. The Orders dated January 30, 1998, February
20, 1998, and March 30, 1998 are REVERSED and SET ASIDE. LRC Case No. 97-0170 is ordered
REINSTATED.
SO ORDERED.
Davide, Jr., C.J., (Chairman), Puno, Kapunan, and Pardo, JJ., concur.






8. G.R. No. 103727 December 18, 1996
INTESTATE ESTATE OF THE LATE DON MARIANO SAN PEDRO Y ESTEBAN, represented by its
HEIR-JUDICIAL ADMINISTRATOR, ENGRACIO F. SAN PEDRO, petitioner-appellant,
vs.
COURT OF APPEALS (Second Division) AURELIO OCAMPO, DOMINADOR D. BUHAIN, TERESA C.
DELA CRUZ, respondents-appellees.
G.R. No. 106496 December 18, 1996
ENGRACIO SAN PEDRO, CANDIDO GENER, ROSA PANTALEON, VICENTE PANTALEON,
ELEUTERIO PANTALEON, TRINIDAD SAN PEDRO, RODRIGO SAN PEDRO, RICARDO NICOLAS,
FELISA NICOLAS, and LEONA SAN PEDRO, petitioners,
vs.
THE HONORABLE COURT OF APPEALS, (Sixteenth Division) and REPUBLIC OF THE
PHILIPPINES,respondents.

HERMOSISIMA, JR., J .:p
The most fantastic land claim in the history of the Philippines is the subject of controversy in these two
consolidated cases. The heirs of the late Mariano San Pedro y Esteban laid claim and have been laying
claim to the ownership of, against third persons and the Government itself, a total land area of
approximately 173,000 hectares or "214,047 quiniones,"
1
on the basis of a Spanish title, entitled "Titulo
de Propriedad Numero 4136" dated April 25, 1894. The claim, according to the San Pedro heirs, appears
to cover lands in the provinces of Nueva Ecija, Bulacan, Rizal, Laguna and Quezon; and such Metro
Manila cities as Quezon City, Caloocan City, Pasay City, City of Pasig and City of Manila, thus affecting in
general lands extending from Malolos, Bulacan to the City Hall of Quezon City and the land area between
Dingalan Bay in the north and Tayabas Bay in the south.
2

Considering the vastness of the land claim, innumerable disputes cropped up and land swindles and
rackets proliferated resulting in tedious litigation in various trial courts, in the appellate court and in the
Supreme Court,
3
in connection therewith.
We have had the impression that our decisions in Director of Forestry, et al. v. Muoz, 23 SCRA 1183
[1968];Antonio, et al. v. Barroga, et al., 23 SCRA 357 [1968]; Carabot, et al. v. Court of Appeals, et al.,
145 SCRA 368 [1986]; Republic v. Intermediate Appellate Court, et al., 186 SCRA 88 [1990]; Widows and
Orphans Association, Inc. (WIDORA) v. Court of Appeals, et al., 212 SCRA 360 [1992]; NAPOCOR
v. Court of Appeals, et al., 144 SCRA 318 [1986]; Republic v. Court of Appeals, et al., 135 SCRA 156
[1985]; and Director of lands v. Tesalona, 236 SCRA 336 [1994]
4
terminated the controversy as to
ownership of lands covered by Spanish Land Titles, for it is the rule that, once this Court, as the highest
Tribunal of the land, has spoken, there the matter must rest:
It is withal of the essence of the judicial function that at some point, litigation must end,
Hence, after the procedures and processes for lawsuits have been undergone, and the
modes of review set by law have been exhausted, or terminated, no further ventilation of
the same subject matter is allowed. To be sure, there may be, on the part of the losing
parties, continuing disagreement with the verdict, and the conclusions therein embodied.
This is of no moment, indeed, is to be expected; but, it is not their will, but the Court's,
which must prevail; and, to repeat, public policy demands that at some definite time, the
issues must be laid to rest and the court's dispositions thereon accorded absolute
finality.
5
[Cited cases omitted]
It is, therefore, to the best interest of the people and the Government that we render judgment herein
writing finisto these controversies by laying to rest the issue of validity of the basis of the estate's claim of
ownership over this vast expanse of real property.
The following facts are pertinent in the resolution of these long drawn-out cases:
G.R. NO. 103727
G.R No. 103727, an appeal by certiorari, arose out of a complaint
6
for recovery of possession and/or
damages with a prayer for a writ of preliminary injunction. This was dismissed by the Regional Trial Court,
National Capital Judicial Region, Branch 104, Quezon City in its decision
7
dated July 7, 1989, the
dispositive portion
8
of which reads:
WHEREFORE, judgment is hereby rendered, dismissing the complaint against the
defendants Aurelio Ocampo, Dominador Buhain and Teresa dela Cruz and ordering
plaintiff to pay each of the herein defendants, the sum of FIVE THOUSAND PESOS
(P5,000.00) as and for attorney's fees, and to pay the costs of suit.
The said complaint for recovery of possession of real property and/or reconveyance with damages and
with a prayer for preliminary injunction was filed on August 15, 1988 by Engracio San Pedro as heir-
judicial administrator of the "Intestate Estate of Don Mariano San Pedro y Esteban" against Jose G. De
Ocampo, Aurelio Ocampo, MARECO, Inc., Rey Antonio Noguera, Teresa C. dela Cruz, Gaudencio R
Soliven, Diomedes Millan, Carmen Rayasco, Dominador D. Buhain, Mario D. Buhain, Jose D. Buhain,
Arestedes S. Cauntay, Manuel Chung and Victoria Chung Tiu (El Mavic Investment & Development
Corporation), Capitol Hills Realty Corporation and Jose F. Castro. The complaint was docketed as Civil
Case No. Q-88-447 in Branch 104, Regional Trial Court of Quezon City.
In the complaint, it was alleged, among others: (1) that Engracio San Pedro discovered that the
aforenamed defendants were able to secure from the Registry of Deeds of Quezon City titles to portions
of the subject estate, particularly Transfer Certificates of Title Nos. 1386, 8982, 951975-951977, 313624,
279067, 1412, 353054, 372592, 149120, 86404, 17874-17875, all emanating from Original Certificate of
Title No. 614
9
and Transfer Certificates of Title Nos. 255544 and 264124, both derivatives of Original
Certificate of Title No. 333; (2) that the aforesaid defendants were able to acquire exclusive ownership
and possession of certain portions of the subject estate in their names through deceit, fraud, bad faith and
misrepresentation; (3) that Original Certificates of Title Nos. 614 and 333 had been cancelled by and
through a final and executory decision dated March 21, 1988 in relation to letter recommendations by the
Bureau of Lands, Bureau of Forest Development and the Office of the SOLICITOR General and also in
relation to Central Bank Circulars dated April 7, 1971, April 23, 1971, September 12, 1972 and June 10,
1980; and (4) that the issue of the existence, validity and genuineness of Titulo Propriedad No. 4136
dated April 25, 1894 which covers the subject estate had been resolved in favor of the petitioner estate in
a decision dated April 25, 1978 by the defunct Court of First Instance, Branch 1 of Baliwag, Bulacan
pertaining to a case docketed as Special Proceeding No. 312-B.
10

Summons were served on only five of the aforementioned defendants, namely, Aurelio Ocampo,
MARECO, Inc., Teresita G. dela Cruz, Dominador Buhain and Manuel Chung and Victoria Chung Tiu.
11

On February 7, 1989, the lower court ordered the dismissal of the complaint against Mareco, Inc. for
improper service of summons and against Manuel Chung and Victoria Chung Tiu for lack of cause of
action considering that the registered owner of the parcel of land covered by TCT No. 86404 is El Mavic
Investment and Development Co., Inc., not Manuel Chung and Victoria Chung Tiu.
12

Trial on the merits proceeded against the private respondents Ocampo, Buhain and Dela Cruz.
On July 7, 1989, the lower court rendered judgment dismissing the complaint based on the following
grounds: (a) Ocampo, Buhain and Dela Cruz are already the registered owners of the parcels of land
covered by Torrens titles which cannot be defeated by the alleged Spanish title, Titulo Propriedad No.
4136, covering the subject estate; and (b) the decision of the Court of First Instance of Bulacan entitled
"In the Matter of the Intestate Estate of the late Don Mariano San Pedro y Esteban" specifically stated in
its dispositive portion that all lands which have already been legally and validly titled under the Torrens
System by private persons shall be excluded from the coverage of Titulo Propriedad No. 4136.
13

The motion for reconsideration thereof was denied,
14
and so, the petitioner estate interposed an appeal
with the Court of Appeals. On January 20, 1992, the appeal was dismissed
15
for being unmeritorious and
the lower court's decision was affirmed with costs against the petitioner estate. The appellate court
ratiocinated:
(1) neither the Titulo Propriedad No. 4136 nor a genuine copy thereof was presented in
the proceeding below;
(2) the illegible copy of the Titulo presented in court was not registered under the Torrens
system hence, it cannot be used as evidence of land ownership;
(3) the CFI decision invoked by petitioner estate in its favor expressly excluded from the
Titulo titled lands of private individuals;
(4) the Titulo is inferior to that of the registered titles of Ocampo, Buhain and Dela Cruz
as correctly ruled by the lower court;
(5) there is no evidence showing that OCT No. 614 from which titles of Ocampo, Buhain
and Dela Cruz originated was already cancelled, hence, the lower court did not err in not
declaring the same as null and
void.
16

Not having obtained a favorable judgment on appeal, the petitioner estate, on March 16, 1992, filed the
present petition
1
7 docketed as G.R. No. 103727.
G.R. NO. 106496
G.R. No. 106496, a petition for review on certiorari, began as a
petition
18
for letters of administration over the intestate estate of the late Mariano San Pedro y Esteban
which eventually resulted to an Order
19
dated November 17, 1978 declaring inter alia, Titulo de
Propriedad No. 4136 as null and void and of no legal force and effect.
The dispositive portion
20
of the said Order reads:
WHEREFORE, this Court so orders that:
1) The Decision dated April 25, 1978 is reconsidered and set aside.
2) Titulo de Propriedad No. 4136 is declared null and void and of no legal force and effect
and that therefore no rights could be derived therefrom.
3) All orders approving the sales, conveyances, donations or any other transactions
involving the lands covered by Titulo de Propriedad No. 4136 are declared invalidated,
void and of no force and effect.
4) All lands covered by Titulo de Propriedad No. 4136 are excluded from the inventory of
the estate of the late Mariano San Pedro y Esteban.
5) The heirs, agents, privies or anyone acting for and in behalf of the estate of the late
Mariano San Pedro y Esteban are enjoined from representing or exercising any acts of
possession or ownership or from disposing in any manner portions of all the lands
covered by Titulo de Propriedad No. 4136 and to immediately vacate the same.
6) Engracio San Pedro and Justino Benito as co-administrators submit in Court within
twenty days their final accounting and inventory of all real and personal properties of the
estate which had come into their possession or knowledge under oath.
7) This case is hereby re-opened, to allow movants-intervenors to continue with the
presentation of their evidence in order to rest their case.
The consideration and approval of the administrator's final accounting and inventory of
the presentation of movants-intervenors' evidence as well as the consideration of all
other incidents are hereby set on December 22, 1978 at 8:30 a.m.
The aforementioned petition for letters of administration over the intestate estate of the late Mariano San
Pedro y Esteban was filed on December 29, 1971 with the defunct Court of First Instance of Bulacan,
Fifth Judicial District, Branch IV, Baliuag, Bulacan. The petition docketed as Sp. Proc. No. 312-B was
initiated by Engracio San Pedro and Justino Z. Benito who sought to be appointed as administrator and
co-administrator, respectively.
On February 29, 1972, after the jurisdictional facts were established, evidence for the petitioners was
received by the lower court without any opposition.
21

On March 2, 1972, then Presiding Judge Juan F. Echiverri issued an Order appointing Engracio San
Pedro as Administrator of the subject estate.
22

On March 11, 1972, the Court issued letters of administration in favor of Engracio San Pedro upon
posting of a bond in the sum of Ten Thousand Pesos (P10,000.00).
23

On February 7, 1974, Administrator Engracio San Pedro was ordered to furnish copies of the letters of
administration and other pertinent orders approving certain dispositions of the properties of the estate to
the following entities:
(a) The Commanding General
Philippine Constabulary
Camp Crame, Quezon City
(b) The Solicitor General
Manila
(c) The Government Corporate Counsel
A. Mabini St., Manila
(d) The City Mayors of Quezon City & Caloocan
(e) The Governors of Rizal, Quezon and Bulacan
(f) The City Treasurers of Quezon City and Caloocan
(g) The Provincial Treasurers of Quezon, Bulacan and Rizal
(h) The PHHC, Diliman, Quezon City
(i) The PAHRRA Quezon Boulevard, Quezon City
(j) The Municipal Treasurers of the various municipalities in which properties of the estate
are located; and
(k) Office of Civil Relations, Camp Crame, Quezon City and Camp Aguinaldo, Quezon
City.
24

The above Order was issued so as to protect the general public from any confusion brought about by
various persons who had been misrepresenting themselves as having been legally authorized to act for
the subject estate and to sell its properties by virtue thereof.
On August 30, 1976, a Motion for Intervention and an Opposition to the Petition was filed by the Republic
of the Philippines alleging, inter alia:
4. That under Presidential Decree No. 892, dated February 16, 1976, Spanish titles like
the TITULO is absolutely inadmissible and ineffective as proof of ownership in court
proceedings, except where the holder thereof applies for land registration under Act 496,
which is not true in the proceedings at bar;
5. That no less than the Supreme Court had declared TITULO DE PROPIEDAD NO.
4136 as invalid;
6. That, moreover, the late Don Mariano San Pedro y Esteban and/or his supposed heirs
have lost whatever rights of ownership they might have had to the so-called Estate on the
ground of inaction, laches and/or prescription;
7. That, accordingly, there is no estate or property to be administered for purposes of
inventory, settlement or distribution in accordance with law, and all the inventories so far
submitted, insofar as they embraced lands within the TITULO, are deemed ineffective
and cannot be legally considered; and
8. That the Republic of the Philippines has a legal interest in the land subject matter of
the petition considering that, except such portions thereof had been (sic) already the
subject of valid adjudication or disposition in accordance with law, the same belong in
State ownership.
25

On February 15, 1977, the Republic filed a Motion to Suspend
Proceedings.
26

On February 16, 1977, the Republic's Opposition to the Petition for Letters of Administration was
dismissed by means of the following Order issued by Judge Benigno Puno:
WHEREFORE, for lack of jurisdiction to determine the legal issues raised, the Court
hereby DISMISSES the "Opposition" dated August 30, 1976, filed by the Office of the
Solicitor General; likewise, for lack of merit, the Motion to Suspend Proceedings dated
February 15, 1977, filed by the Office of the Solicitor General is DENIED.
The administrator Engracio San Pedro and the Co-administrator Justino Z. Benito are
ordered to furnish the office of the Solicitor General all copies of inventories already filed
in Court within ten (10) days from notice hereof.
2
7
On March 9, 1977, a motion for reconsideration was filed by the
Republic.
28

On April 25, 1978, the lower court then presided over by Judge Agustin C. Bagasao, rendered a 52-page
decision, the dispositive portion of which reads:
WHEREFORE, judgment is hereby rendered:
(a) Declaring the existence, genuineness and authenticity of Titulo de Propriedad No.
4136 of the Registry of Deeds of Bulacan, issued on April 29, 1984, in the name of the
deceased Don Mariano San Pedro y Esteban, covering a total area of approximately
214,047 quiniones or 173,000 hectares, situated in the Provinces of Bulacan, Rizal,
Quezon, Quezon City and Caloocan City;
(b) Declaring Engracio San Pedro, Candido Gener, Santiago Gener, Rosa Pantaleon,
Vicente Pantaleon, Eleuterio Pantaleon, Trinidad San Pedro, Rodrigo San Pedro,
Ricardo Nicolas, and Teresa Nicolas, as the true and lawful heirs of the deceased Don
Mariano San Pedro y Esteban and entitled to inherit the intestate estate left by the said
deceased, consisting of the above-mentioned tract of private land covered and described
by said above-mentioned Titulo de Propriedad No. 4136 of the Registry of Deeds of
Bulacan, excluding therefrom: (a) all lands which have already been legally and validly
titled under the Torrens System, by private persons, or the Republic of the Philippines, or
any of its instrumentalities or agencies; (b) all lands declared by the government as
reservations for public use and purposes; (c) all lands belonging to the public domain;
and, (d) all portions thereof which had been sold, quitclaimed and/or previously excluded
by the Administrator and duly approved by a final order of the Court, except those which
may hereafter be set aside, after due consideration on a case to case basis, of various
motions to set aside the said Court order which approved the said sales, quit-claims,
and/or exclusions;
(c) The designation of Atty. Justino Z. Benito as co- administrator, is hereby revoked to
take effect immediately, to obviate any confusion in the administration of the Estate, and
to fix the responsibilities of administration to the co-heir Administrator, Engracio San
Pedro, whose appointment as such is hereby confirmed. The said co-administrator
Justino Z. Benito is hereby ordered to render his final accounting of his co-administration
of the Estate, within thirty (30) days from receipt of copy hereof;
(d) The Co-Heir-Administrator, Engracio San Pedro is hereby ordered to amass, collate,
consolidate and take possession of all the net estate of the deceased Don Marino San
Pedro y Esteban, as well as all other sets and credits lawfully belonging to the estate
and/or to take appropriate legal action to recover the same in the proper Courts of
Justice, government offices or any appropriate forum; and to pay all taxes or charges due
from the estate to the Government, and all indebtedness of the estate, and thereafter, to
submit a project of partition of the estate among the lawful heirs as herein recognized and
declared.
It is, however, strongly recommended to His Excellency, President Ferdinand E. Marcos
that, to avoid the concentration of too much land to a few persons and in line with the
projected urban land reform program of the government, corollary to the agricultural land
reform program of the New Society, the above intestate estate of the late Don Mariano
San Pedro y Esteban should be expropriated or purchased by negotiated sale by the
government to be used in its human settlements and low cost housing projects.
No Costs.
SO ORDERED.
29

On May 17, 1978, the Republic moved for a reconsideration of the above decision.
30

On June 5, 1978, administrator Engracio San Pedro filed a Manifestation and Petition for the Inhibition of
the then newly appointed Presiding Judge Oscar Fernandez. On July 12, 1978, after the Republic filed its
Reply to the Petition for Inhibition, Judge Fernandez denied the said petition.
31

After hearings were conducted on the Republic's Motion for Reconsideration, Judge Fernandez issued
the aforestated Order
32
dated November 17, 1978 which, in essence, set aside Judge Bagasao's
decision dated April 25, 1978 by declaring Titulo de Propriedad No. 4136 as null and void and of no legal
force and effect, thus, excluding all lands covered by Titulo de Propriedad No. 4136 from the inventory of
the estate of the late Mariano San Pedro y Esteban.
The petitioners-heirs of the late Mariano San Pedro y Esteban appealed to the Court of Appeals and
alleged that the lower court did not act with impartiality when it granted the Republic's motion for
reconsideration which was merely pro forma, thereby overturning a prior declaration by the same court of
the existence, genuineness and authenticity of Titulo de Propriedad No. 4136 in the name of the
deceased Mariano San Pedro.
33

On March 11, 1992, the Court of Appeals dismissed the appeal of the petitioners-heirs.
34
In affirming the
assailed Order dated November 17, 1978, the appellate court focused its discussion solely on the issue of
whether or not the lower court erred in declaring Titulo de Priopriedad No. 4136 null and void. The
appellate court ruled that the petitioners-heirs failed to controvert the Republic's claim that Titulo de
Propriedad No. 4136 is invalid on the following bases; (a) non-production of the original of the subject
title; (b) inadmissibility of the photostat copies of the said title; and (c) non-registration of the subject
Spanish title under Act No. 496 (Land Registration Act) as required by Presidential Decree No. 892
(Discontinuance of the Spanish MORTGAGE System of Registration and of the Use of Spanish Titles
as Evidence in Land Registration Proceedings).
The petitioners-heirs moved for a reconsideration of the Court of Appeals' decision by invoking certain
cases wherein the validity of Titulo de Propriedad No. 4136 had been allegedly recognized. The Court of
Appeals refused to be swayed and denied the motion for reconsideration for lack of merit.
35

Hence, the herein petition,
36
docketed as G. R. No. 106496, was filed on September 18, 1992.
After the parties filed their respective pleadings in G.R. Nos. 103727 and 106496, this Court resolved to
consolidate both cases on September 15,
1994.
3
7
While these cases were pending before us, several parties filed separate motions for intervention which
we denied on different occasions for lack of merit.
In G.R No. 103727, the grounds relied upon for the grant of the petition are as follows:
I. That petitioner-appellant as plaintiff in Civil Case No. Q-88-447, RTC, Branch 104 was
denied due process of law due to gross negligence of lawyer, which respondent court
grossly failed to take cognizance of.
II. That the respondent court committed grave abuse of discretion tantamount to lack of
jurisdiction in not remanding the case for trial and in affirming the lower court's null and
void judgment.
38

In G.R No. 106496, the petitioners-heirs present the following assignment of errors, to wit:
First. Respondent Court of Appeals affirmed the appealed order which resolved a
question of title or ownership over which the lower court as an intestate court has no
jurisdiction and over the vigorous and repeated objections of the petitioners.
39

Second. Respondent Court of Appeals erred in upholding the order of Judge Fernandez
setting aside the order and decision of Judge Puno and Bagasao; Judge Fernandez
thereby acted as an appellate court reviewing, revising, amending or setting aside the
order and decision of Judges of equal rank.
40

Third. Respondent Court of Appeals has no jurisdiction to uphold the order of Judge
Fernandez who without jurisdiction, set aside the order of Judge Puno and the decision of
Judge Bagasao, both of which were already final.
41

Fourth. Respondent Court of Appeals was unmindful of the fact that Judge Fernandez
was appointed by President Marcos to reverse Judge Bagasao, regardless of the
evidence, thereby unmindful that petitioners were denied the cold neutrality of an
impartial tribunal.
42

Fifth. Respondent Court of Appeals erred in not considering the evidence presented
before Judges Echiverri, Puno and Bagasao and merely adopted the order of Judge
Fernandez who never received a single piece of evidence, notwithstanding the 1906
Guide title over Hacienda Angono in Binangonan, Rizal, the boundary owner stated
therein being Don Mariano San Pedro y Esteban, and the November 1991 en
banc decision of the Supreme Court upholding the Guido title.
43

Of paramount importance over and above the central issue of the probative value of the petitioners'
Spanish title in these cases is the propriety of the lower court's resolution of the question of ownership of
the subject San Pedro estate in the special proceedings case. Thus, before we address ourselves to the
issue of whether or not petitioners' Titulo de Propriedad No. 4136 is null and void and of no legal force
and effect, it is best that we first determine whether or not the lower court, acting as a PROBATE court,
in the petition for letters of administration, committed grave abuse of discretion amounting to lack of
jurisdiction in settling the issue of ownership of the San Pedro estate covered by Titulo Propriedad No.
4136.
Petitioners-heirs, in G.R No. 106496, on the one hand, contend that the lower court, then CFI, Bulacan,
Branch IV, had no jurisdiction as an "intestate court"
44
to resolve the question of title or ownership raised
by the public respondent Republic of the Philippines, through the Office of the Solicitor General in the
intestate proceedings of the estate of Mariano San Pedro y Esteban.
45

The public respondent, on the other hand, invoking its sovereign capacity as parens patriae, argues that
petitioners' contention is misplaced considering that when the Republic questioned the existence of the
estate of Mariano San Pedro y Esteban, the lower court became duty-bound to rule on the genuineness
and validity of Titulo de Propriedad 4136 which purportedly covers the said estate, otherwise, the lower
court in the intestate proceedings would be mistakenly dealing with properties that are proven to be part
of the State's patrimony or improperly included as belonging to the estate of the deceased.
46

A probate court's jurisdiction is not limited to the determination of who the heirs are and what shares are
due them as regards the estate of a deceased person. Neither is it confined to the issue of the validity of
wills. We held in the case of Manigat v. Castillo,
4
7 that "the main function of a probate court is to settle
and liquidate the estates of deceased persons either summarily or through the process of administration."
Thus, its function necessarily includes the examination of the properties, rights and credits of the
deceased so as to rule on whether or not the inventory of the estate properly included them for purposes
of distribution of the net assets of the estate of the deceased to the lawful heirs.
In the case of Trinidad v. Court of Appeals,
48
we stated, thus:
. . . questions of title to any property apparently still belonging to estate of the deceased
maybe passed upon in the Probate Court, with the consent of all the parties, without
prejudice to third persons . . .
Parenthetically, questions of title pertaining to the determination prima facie of whether certain properties
ought to be included or excluded from the inventory and accounting of the estate subject of a petition for
letters of administration, as in the intestate proceedings of the estate of the late Mariano San Pedro y
Esteban, maybe resolved by the probate court. In this light, we echo our pronouncement in the case
of Garcia v. Garcia
49
that:
. . . The court which acquired jurisdiction over the properties of a deceased person
through the filing of the corresponding proceedings, has supervision and control over the
said properties, and under the said power, it is its inherent duty to see that the inventory
submitted by the administrator appointed by it contains all the properties, rights and
credits which the law requires the administrator to set out in his inventory. In compliance
with this duty, the court has also inherent power to determine what properties, rights and
credits of the deceased should be included in or excluded from the inventory. Should an
heir or person interested in the properties of a deceased person duly call the court's
attention to the fact that certain properties, rights or credits have been left out in the
inventory, it is likewise the court's duty to hear the observations, with power to determine
if such observations should be attended to or not and if the properties referred to therein
belong prima facie to the intestate, but no such determination is final and ultimate in
nature as to the ownership of the said properties.
50
[Emphasis Supplied]
In view of these disquisitions of this Court, we hold that the lower court did not commit any reversible error
when it issued the Order dated November 17, 1978 which set aside Judge Bagasao's decision dated April
25, 1978 and declared Titulo de Propriedad No. 4136 as null and void, consequently excluding all lands
covered by the said title from the inventory of the estate of the late Mariano San Pedro y Esteban.
A corollary issue sought to be ventilated by the petitioners-heirs as regards the assailed Order of
November 17, 1978 is the impropriety of Judge Fernandez' act of granting the motion for reconsideration
filed by the public respondent Republic since, Judge Fernandez did not personally hear the intestate
case. Petitioners thus dubbed him as a "reviewing judge." By setting aside the Decision dated April 25,
1978 of his predecessors in CFI, Branch IV, Baliuag, Bulacan, namely, Judge Benigno Puno and Judge
Agustin C. Bagasao, respectively, Judge Fernandez, acting as a "reviewing judge," proceeded without
authority and/or jurisdiction.
51

There is no question that, barring any serious doubts as to whether the decision arrived at is fair and just,
a newly appointed judge who did not try the case can decide the same as long as the record and the
evidence are all available to him and that the same were taken into consideration and thoroughly studied.
The "reviewing judge" argument of the petitioners-heirs has no leg to stand on considering that "the fact
that the judge who penned the decision did not hear a certain case in its entirety is not a compelling
reason to jettison his findings and conclusion inasmuch as the full record was available to him for his
perusal."
52
In the case at bar, it is evident that the 41-page Order dated November 17, 1978 of Judge
Fernandez bespeaks of a knowledgeable and analytical discussion of the rationale for reconsidering and
setting aside Judge Bagasao's Decision dated April 25, 1978.
Considering the definiteness of our holding in regard to the correctness of Judge Fernandez' disposition
of the case, i.e., the issuance by the lower court of the assailed Order of November 17, 1978, we now
focus on the core issue of whether or not the lower court in G.R No. 106496 committed reversible error in
excluding from the inventory of the estate of the deceased Mariano San Pedro y Esteban all lands
covered by Titulo de Propriedad No. 4136 primarily on the ground that the said title is null and void and of
no legal force and effect. Juxtaposed with this is the issue of whether or not the appellate court, in both
cases, G.R. Nos. 103727 and 106496, erred in not recognizing Titulo de Propriedad No. 4136 as
evidence to prove ownership by the Late Mariano San Pedro of the lands covered thereby.
It is settled that by virtue of Presidential Decree No. 892 which took effect on February 16, 1976, the
system of registration under the Spanish Mortgage Law was abolished and all holders of Spanish titles or
grants should cause their lands covered thereby to be registered under the Land Registration Act
53
within
six (6) months from the date of effectivity of the said Decree or until August 16, 1976.
54
Otherwise, non-
compliance therewith will result in a re-classification of their lands.
55
Spanish titles can no longer be
countenanced as indubitable evidence of land ownership.
56

Section 1 of the said Decree provides:
Sec. 1. The system of registration under the Spanish Mortgage Law is discontinued, and
all lands recorded under said system which are not yet covered by Torrens title shall be
considered as unregistered lands.
All holders of Spanish titles or grants should apply for registration of their lands under Act
No. 496, otherwise known as the Land Registration Act, within six (6) months from the
effectivity of this decree. Thereafter, Spanish titles cannot be used as evidence of land
ownership in any registration proceedings under the Torrens system.
Hereafter, all instruments affecting lands originally registered under the
Spanish MORTGAGE Law may be recorded under Section 194 of the Revised
Administrative Code, as amended by Act. 3344.
The Whereas clauses of the aforesaid Decree specify the underlying policies for its passage, to wit:
WHEREAS, fraudulent sales, transfers, and other forms of conveyances of large tracts of
public and private lands to unsuspecting and unwary buyers appear to have been
perpetrated by unscrupulous persons claiming ownership under Spanish titles or grants
of dubious origin;
WHEREAS, these fraudulent transactions have often resulted in conflicting claims and
litigations between legitimate title holders, bona fide occupants or applicants of public
lands, on the one hand, and the holders of, or person claiming rights under the said
Spanish titles or grants, on the other, thus creating confusion and instability in property
ownership and threatening the peace and order renditions in the areas affected;
WHEREAS, statistics in the Land Registration Commission show that recording in the
system of registration under the Spanish MORTGAGE Law is practically nil and that
this system has become obsolete;
WHEREAS, Spanish titles to lands which have not yet been brought under the operation
of the Torrens system, being subject to prescription, are now ineffective to prove
ownership unless accompanied by proof of actual possession;
WHEREAS, there is an imperative need to discontinue the System of registration under
the Spanish MORTGAGE Law and the use of Spanish titles as evidence in registration
proceedings under the Torrens system;
In the case of Director of Lands v. Heirs of Isabel Tesalona, et al.,
5
7 we took cognizance of this Decree
and thus held that caution and care must be exercised in the acceptance and admission of Spanish titles
taking into account the numerous fake titles that have been discovered after their supposed reconstitution
subsequent to World War II.
In both cases, petitioners-heirs did not adduce evidence to show that Titulo de Propriedad 4136 was
brought under the operation of P.D. 892 despite their allegation that they did so on August 13,
1976.
58
Time and again we have held that a mere allegation is not evidence and the party who alleges a
fact has the burden of proving it.
59
Proof of compliance with P.D. 892 should be the Certificate of Title
covering the land registered.
In the petition for letters of administration, it was a glaring error on the part of Judge Bagasao who
rendered the reconsidered Decision dated April 25, 1978 to have declared the existence, genuineness
and authenticity of Titulo de Propriedad No. 4136 in the name of the deceased Mariano San Pedro y
Esteban despite the effectivity of P.D. No. 892. Judge Fernandez, in setting aside Judge Bagasao's
decision, emphasized that Titulo de Propriedad No. 4136, under P.D. 892, is inadmissible and ineffective
as evidence of private ownership in the special proceedings case. He made the following observations as
regards the Titulo, to wit:
The SOLICITOR General, articulating on the dire consequences of recognizing the
nebulous tituloas an evidence of ownership underscored the fact that during the
pendency of this case, smart speculators and wise alecks had inveigled innocent parties
into buying portions of the so-called estate with considerations running into millions of
pesos.
Some, under the guise of being benign heroes even feigned donations to charitable and
religious organizations, including veterans' organizations as smoke screen to the
gargantuan fraud they have committed and to hood wink further other gullible and
unsuspecting victims.
60

In the same light, it does not escape this Court's onomatopoeic observation that the then heir-judicial
administrator Engracio San Pedro who filed the complaint for recovery of possession and/or
reconveyance with damages in G.R No. 103727 on August 15, 1988 invoked Judge Bagasao's Decision
of April 25, 1978 in support of the Titulo's validity notwithstanding the fact that, by then, the said Decision
had already been set aside by Judge Fernandez' Order of November 17, 1978. We are in accord with the
appellate courts' holding in G.R No. 103727 insofar as it concludes that since the Titulo was not
registered under Act No. 496, otherwise known as the Land Registration Act, said Titulo is inferior to the
registered titles of the private respondents Ocampo, Buhain and Dela Cruz.
This Court can only surmise that the reason for the non-registration of the Titulo under the Torrens
system is the lack of the necessary documents to be presented in order to comply with the provisions of
P.D. 892. We do not discount the possibility that the Spanish title in question is not genuine, especially
since its genuineness and due execution have not been proven. In both cases, the petitioners heirs were
not able to present the original of Titulo de Propriedad No. 4136 nor a genuine copy thereof. In the
special proceedings case, the petitioners-heirs failed to produce the Titulo despite a subpoena duces
tecum (Exh. "Q-RP") to produce it as requested by the Republic from the then administrators of the
subject intestate estate, Engracio San Pedro and Justino Benito, and the other interested parties. As an
alternative to prove their claim of the subject intestate estate, the petitioners referred to a document
known as "hypoteca" (the Spanish term is 'hipoteca') allegedly appended to the Titulo. However, the said
hypoteca was neither properly identified nor presented as evidence. Likewise, in the action for recovery of
possession and/or reconveyance with damages, the petitioners-heirs did not submit the Titulo as part of
their evidence. Instead, only an alleged illegible copy of the Titulo was presented. (Exhs. "C-9" to "C-19").
The Best Evidence Rule as provided under Rule 130, section 2 of the Rules of Court is stated in
unequivocal terms. Subparagraphs (a) and (b) of the said Rule read:
Sec. 2. Original writing must be produced; exceptions. There can be no evidence of a
writing the contents of which is the subject of inquiry, other than the original writing itself,
except in the following cases:
(a) When the original has been lost, destroyed, or cannot be produced in court;
(b) When the original is in the possession of the party against whom the evidence is
offered, and the latter fails to produce it after reasonable notice;
xxx xxx xxx
Sections 3 and 4 of the same Rule further read:
Sec 4. Secondary evidence when original is lost or destroyed When the original writing
has been lost or destroyed, or cannot be produced in court, upon proof of its execution
and loss or destruction or unavailability, its contents may be proved by a copy, or by a
recital of its contents in some authentic document, or by the recollection of witnesses.
Sec. 5. Secondary evidence when original is in adverse party's custody. If the writing
be in the custody of the adverse party, he must have reasonable notice to produce it. If
after such notice and after satisfactory proof of its existence, he fails to produce the
writing, the contents thereof may be proved as in the case of its loss. But the notice to
produce it is not necessary where the writing is itself a notice, or where it has been
wrongfully obtained or withheld by the adverse party.
Thus, the court shall not receive any evidence that is merely substitutionary in its nature, such as
photocopies, as long as the original evidence can be had. In the absence of a clear showing that
the original writing has been lost or destroyed or cannot be produced in court, the photocopy
submitted, in lieu thereof, must be disregarded, being unworthy of any probative value and being
an inadmissible piece of evidence.
61

Hence, we conclude that petitioners-heirs failed to establish by competent proof the existence and due
execution of the Titulo. Their explanation as to why the original copy of the Titulo could not be produced
was not satisfactory. The alleged contents thereof which should have resolved the issue as to the exact
extent of the subject intestate estate of the late Mariano San Pedro were not distinctly proved. In the case
of Ong Ching Po v. Court of Appeals,
62
we pointed out that:
Secondary evidence is admissible when the original documents were actually lost or
destroyed. But prior to the introduction of such secondary evidence, the proponent must
establish the former existence of the document. The correct order of proof is as follows:
existence; execution; loss; contents. This order may be changed if necessary in the
discretion of the court.
63

In upholding the genuineness and authenticity of Titulo de Propriedad No. 4136, Judge Bagasao, in his
decision, relied on: (1) the testimony of the NBI expert, Mr. Segundo Tabayoyong, pertaining to a report
dated January 28, 1963 denominated as "Questioned Documents Report No. 230-163"; (2) a photostat
copy of the original of the Titulo duly certified by the then Clerk of Court of the defunct Court of First
Instance of Manila; and (3) the hipotecaRegistered in the Register of Deeds of Bulacan on December 4,
1894.
Judge Fernandez, in his November 1978 Order which set aside Judge Bagasao's April 1978 decision
correctly clarified that the NBI report aforementioned was limited to the genuineness of the two signatures
of Alejandro Garcia and Mariano Lopez Delgado appearing on the last page of the Titulo, not the Titulo
itself. When asked by the counsel of the petitioners-heirs to admit the existence and due execution of the
Titulo, the handling Solicitor testified:
xxx xxx xxx
ATTY. BRINGAS:
With the testimony of this witness, I would like to call the distinguished
counsel for the government whether he admits that there is actually a
titulo propiedad 4136.
COURT:
Would you comment on that Solicitor Agcaoili?
ATTY. AGCAOILI:
We are precisely impugning the titulo and I think the question of counsel
is already answered by witness. The parties have not yet established the
due existence of the titulo.
ATTY. BRINGAS:
We are constrained to ask this matter in order to be candid about the
question. The witness is a witness for the government, so with the
testimony of this witness for the government to the effect that there is
actually in existence titulo propiedad 4136; we are asking the question
candidly to the government counsel whether he is prepared to state that
there is really in existence such titulo propiedad 4136.
ATTY. AGCAOILI:
We are now stating before this court that there was such a document
examined by the NBI insofar as the signatures of Alejandro Garcia and
Manuel Lopez Delgado are concerned and they are found to be
authentic.
64

The following significant findings of Judge Fernandez further lend credence to our pronouncement that
the Titulo is of dubious validity:
. . . the NBI in its Questioned Document Report No. 448-977 dated September 2, 1977
(Exhibit "O-RP") concluded that the document contained material alterations as follows:
a) On line 15 of "p, 1, Title" and on line 5 of "p. 2, Title," the word "Pinagcamaligan" was
written after "Pulo;"
b) On line 16, "p. 1, Title," "un" was converted to "mil;"
c) On Line 18, "p. 1, Title," "mil" was written at the end of "tres" in "tres mil;"
d) On line 19 of "p. 1, Title," a semblance of "mil" was written after "setentay tres;"
e) On line 6, "p. 2, Title," "un" was formed to a semblance of "uni;" and
f) On line 8, "p. 2, Title," "un" was formed to "mil."
The plain and evident purpose was definitely to enlarge the area of the titulo. According
to Mr. Tabayoyong of the NBI, there are still "pieces of black ashes around the rings of
the portions which are indications of burnings". The burnings were made on the very
portions where there were previous erasures, alterations and intercalations.
Understandably, the burnings were done to erase traces of the criminal act.
65

In the case of National Power Corporation v. Court of Appeals, et a1.
66
Justice Ameurfina Melencio-
Herrera, in reinstating the trial court's judgment therein, sustained the finding that:
. . . The photostatic copy (in lieu of the lost original) of the Spanish title in the name of
Mariano San Pedro shows obvious alterations and intercalations in an attempt to vastly
increase the area and change the location of the land described in the original title . . .
Anent the inadmissibility as evidence of the photostat copy of the Titulo, we sustain the lower court's
analysis, as affirmed by the appellate court, viz:
To begin with, the original of Titulo de Propiedad No. 4136 was never presented in Court.
Upon request of the Government, a subpoena duces tecum (Exhibit "Q-RP") was issued
to the two administrators, Engracio San Pedro and Justino Benito as well as to other
interested parties to produce the original of Titulo de Propriedad No. 4136. But no one
produced the titulo. What the parties did was to pass the buck to one another.
Without any plausible explanation at all on as to why the original could not be produced,
the Court cannot take cognizance of any secondary evidence.
It was explained that the titulo after changing hands, finally fell into the hands of a certain
Moon Park of Korea but who later disappeared and that his present whereabouts could
not be known.
Strangely enough, despite the significance of the titulo, no serious efforts on the part of
the claimants-heirs were exerted to retrieve this document of vital importance despite the
Court order to produce it in order to determine its authenticity.
It would not be enough to simply say that Moon Park's whereabouts are unknown or that
there are not enough funds to locate him. The only logical conclusion would be that the
original would be adverse if
produced.
6
7
As regards the hipoteca which allegedly defines the metes and bounds of the subject intestate estate, the
petitioners-heirs have not established the conditions required by law for their admissibility as secondary
evidence to prove that there exists a document designated as Titulo de Propriedad No. 4136. Hence, the
same acquires no probative value.
68

At this juncture, our decision dated June 28, 1968 in Director of Forestry, et al. v. Hon. Emmanuel
M. Muoz, as Judge of the Court of First Instance of Bulacan, Branch I, et al.
69
is enlightening. In said
case, private respondent, Pinaycamaligan Indo-Agro Development Corporation, Inc., (PIADECO),
claimed to be the owner of some 72,000 hectares of land located in the municipalities of Angat,
Norzagaray and San Jose del Monte, province of Bulacan, and in Antipolo and Montalban, province of
Rizal. To prove its ownership Piadeco relied on Titulo de Propriedad No. 4136 dated April 28, 1894.
Scholarly opining that the Titulo is of doubtful validity,
70
Justice Conrado V. Sanchez, speaking for the
Court, stated that:
But an important moiety here is the deeply disturbing intertwine of two undisputed
facts. First. The Title embraces land "located in the Provinces of Bulacan, Rizal, Quezon,
and Quezon City." Second. The title was signed only by the provincial officials of
Bulacan, and inscribed only in the Land Registry of Bulacan. Why? The situation, indeed,
cries desperately for a plausible answer.
To be underscored at this point is the well-embedded principle that private ownership of
land must be proved not only through the genuineness of title but also with a clear
identity of the land claimed. (Oligan v. Mejia, 17 Phil. 494, 496; Villa Abrille v. Banuelos,
20 Phil. 1, 8, citing Sison v. Ramos, 13 Phil. 54 and Belen v. Belen, 13 Phil. 202; Licad v.
Bacani, 51 Phil. 51, 54-56; Lasam v. Director, 65 Phil. 367, 371. This Court ruled in a
case involving a Spanish title acquired by purchase that the land must be concretely
measured per hectare or per quinon, not in mass (cuerpos ciertos), (Valdez v. Director,
62 Phil. 362, 373, 375). The fact that the Royal Decree of August 31, 1888 used 30
hectares as a basis for classifying lands strongly suggests that the land applied for must
be measured per hectare.
Here, no definite area seems to have been mentioned in the title. In Piadeco's "Rejoinder
to Opposition" dated April 28, 1964 filed in Civil Case 3035-M, it specified that area
covered by its Titulo de Propiedad as 74,000 hectares (Rollo in L-24796, p. 36). In its
"Opposition" of May 13, 1964 in the same case, it described the land as containing
72,000 hectares (Id., p. 48). Which is which? This but accentuates the nebulous identity
of Piadeco's land, Piadeco's ownership thereof then equally suffers from vagueness, fatal
at least in these proceedings.
Piadeco asserts that Don Mariano San Pedro y Esteban, the original owner appearing on
the title, acquired his rights over the property by prescription under Articles 4 and 5 of the
Royal Decree of June 25, 1880, (Rollo of L-24796, p. 184) the basic decree that
authorized adjustment of lands. By this decree, applications for adjustment showing
the location, boundaries and area of land applied for were to be filed with the Direccion
General de Administration Civil, which then ordered theclassification and survey of the
land with the assistance of the interested party or his legal representative (Ponce, op. cit.,
p. 22).
The Royal Decree of June 5, 1880 also fixed the period for filing applications for
adjustment at one year from the date of publication of the decree in the Gaceta de
Manila on September 10, 1880, extended for another year by the Royal Order of July 15,
1881 (Ibid.). If Don Mariano sought adjustment within the time prescribed, as he should
have, then, seriously to be considered here are the Royal Orders of November 25, 1880
and of October 26, 1881, which limited adjustment to 1,000 hectares of arids lands, 500
hectares of land with trees and 100 hectares of irrigable lands (See: Government v. Avila,
46 Phil. 146, 154; Bayot v. Director of Lands, 98 Phil. 935, 941. Article 15 of the Royal
Decree of January 26, 1889 limited the area that may be acquired by purchase to 2,500
hectares, with allowable error up to 5%. Ponce, op cit., p. 19). And, at the risk of
repetition, it should be stated again that Piadeco's Titulo is held out to embrace 72,000 or
74,000 hectares of land.
But if more were needed, we have the Maura Law (Royal Decree of February 13, 1894),
published in the Gaceta de Manila on April 17, 1894 (Ibid., p. 26; Venture, op. cit., p. 28).
That decree required a second petition for adjustment within six months from publication,
for those who had not yet secured their titles at the time of the publication of the law
(Ibid.). Said law also abolished the provincial boards for the adjustment of lands
established by Royal Decree of December 26, 1884, and confirmed by Royal Decree of
August 31, 1888, which boards were directed to deliver to their successors, the provincial
boards established by Decree on Municipal Organization issued on May 19, 1893, all
records and documents which they may hold in their possession (Ramirez v. Director of
Land, supra, at p. 124).
Doubt on Piadeco's title here supervenes when we come to consider that title was either
dated April 29 or April 25, 1894, twelve or eight days after the publication of the Maura
Law.
Let us now take a look, as near as the record allows, at how Piadeco exactly acquired its
rights under the Titulo. The original owner appearing thereon was Don Mariano San
Pedro y Esteban. From Piadeco's explanation not its evidence (Rollo of L-24796, pp.
179-188) we cull the following: On December 3, 1894, Don Mariano mortgaged the land
under pacto de retro, redeemable within 10 years, for P8,000.00 to one Don Ignacio
Conrado. This transaction was said to have been registered or inscribed on December 4,
1894. Don Mariano Ignacio died, his daughter, Maria Socorro Conrado, his only heir,
adjudicated the land to herself. At about the same time, Piadeco was organized. Its
certificate of registration was issued by the Securities and Exchange Commission on
June 27, 1932. Later, Maria Socorro, heir of Don Ignacio, became a shareholder of
Piadeco when she conveyed the land to Piadeco's treasurer and an incorporator,
Trinidad B. Estrada, in consideration of a certain amount of Piadeco shares. Thereafter,
Trinidad B. Estrada assigned the land to Piadeco. Then came to the scene a certain
Fabian Castillo, appearing as sole heir of Don Mariano, the original owner of the land.
Castillo also executed an affidavit of adjudication to himself over the same land, and then
sold the same to Piadeco. Consideration therefor was paid partially by
Piadeco, pending the registration of the land under Act 496.
The question may well be asked: Why was full payment of the consideration to Fabian
Castillo made to depend on the registration of the land under the Torrens system, if
Piadeco was sure of the validity of Titulo de Propiedad 4136? This, and other factors
herein pointed out, cast great clouds of doubt that hang most conspicuously over
Piadeco's title.
Moreover, in the case of Widows & Orphans Association, Inc. v. Court of Appeals,
71
we categorically
enunciated that the alleged Spanish title, Titulo de Propriedad No. 4136, had become bereft of any
probative value as evidence of land ownership by virtue of P.D. 892 as contained in our Resolution dated
February 6, 1985 in a related case entitled Benito and WIDORA v. Ortigas docketed as G.R No. 69343.
On March 29, 1985, an entry of final judgment was made respecting G.R. No. 69343.
Under the doctrine of conclusiveness of judgment, the prior declarations by this Court relating to the issue
of the validity of Titulo de Propriedad No. 4136 preclude us from adjudicating otherwise. In
the Muoz case, we had cast doubt on the Titulo's validity. In the WIDORA case, the Titulo's nullification
was definitive. In both cases, the Republic and the estate of Mariano San Pedro y Esteban were on
opposite ends before this bench. In the case en banc of Calalang v. Register of Deeds of Quezon
City,
72
the Court explained the concept of conclusiveness of judgment, viz:
. . . conclusiveness of judgment states that a fact or question which was in issue in a
former suit and was there judicially passed upon and determined by a court of competent
jurisdiction, is conclusively settled by the judgment therein as far as the parties to that
action and persons in privity with them are concerned and cannot be again litigated in
any future action between such parties or their privies, in the same court or any other
court of concurrent jurisdiction on either the same or different cause of action, while the
judgment remains unreversed by proper authority. It has been held that in order that a
judgment in one action can be conclusive as to a particular matter in another action
between the same parties or their privies, it is essential that the issue be identical. If a
particular point or question is in issue in the second action, and the judgment will depend
on the determination of that particular point or question, a former judgment between the
same parties or their privies will be final and conclusive in the second if that same point
or question was in issue and adjudicated in the first suit (Nabus v. Court of Appeals, 193
SCRA 732 [1991]). Identity of cause of action is not required by merely identity of issues.
The issue, whether Titulo de Propriedad No. 4136 is valid or not, must now be laid to rest. The
Titulo cannot be relied upon by the petitioners-heirs or their privies as evidence of ownership. In
the petition for letters of administration the inventory submitted before the probate court consisted
solely of lands covered by the Titulo. Hence, there can be no "net estate" to speak of after the
Titulo's exclusion from the intestate proceedings of the estate of the late Mariano San Pedro.
In G.R No. 103727, the Titulo cannot be superior to the Torrens Titles of private respondents Buhain,
Ocampo and Dela Cruz, namely TCT No. 372592 (Exh. "2", Buhain), TCT No. 8982 (Exh. "2" De
Ocampo) and TCT No. 269707 (Exh. "2" Dela Cruz).
73
Under the Torrens system of registration, the
titles of private respondents became indefeasible and incontrovertible one year from its final
decree.
74
More importantly, TCT Nos. 372592, 8982, 269707, having been issued under the Torrens
system, enjoy the conclusive presumption of validity.
75
As a last hurrah to champion their claim to the
vast estate covered by the subject Spanish title, the petitioners-heirs imputed fraud and bad faith which
they failed to prove on the part of the private respondents as regards their Torrens titles and accused
their own counsel of gross negligence for having failed to call the proper witnesses from the Bureau of
Forestry to substantiate the petitioners-heirs' claim that OCT No. 614 from which private respondents
were derived is null and void. It is an elementary legal principle that the negligence of counsel binds the
client.
76
The records show that the petitioners-heirs were not at all prejudiced by the non-presentation of
evidence to prove that OCT No. 614 is a nullity considering that their ownership itself of the lands being
claimed was not duly proved. In the case of Villa Rhecar Bus v. Dela Cruz, et al.,
7
7 we held:
It is unfortunate that the lawyer of the petitioner neglected his responsibilities to his client.
This negligence ultimately resulted in a judgment adverse to the client. Be that as it may,
such mistake binds the client, the herein petitioner. As a general rule, a client is bound by
the mistakes of his counsel. (Que v. Court of Appeals, 101 SCRA 13 [1980] Only when
the application of the general rule would result in serious injustice should an exception
thereto be called for. Under the circumstances obtaining in this case, no undue prejudice
against the petitioner has been satisfactorily demonstrated. At most, there is only an
unsupported claim that the petitioner bad been prejudiced by the negligence of its
counsel, without an explanation to that effect.
Sans preponderance of evidence in support of the contention that the petitioners-heirs were
denied due process on account of the negligence of their counsel, the writ of certiorari is
unavailing.
It bears repeating that the heirs or successors-in-interest of Mariano San Pedro y Esteban are not without
recourse. Presidential Decree No. 892, quoted hereinabove, grants all holders of Spanish Titles the right
to apply for registration of their lands under Act No. 496, otherwise known as the Land Registration Act,
within six (6) months from the effectivity of the Decree. Thereafter, however, any Spanish Title, if utilized
as evidence of possession, cannot be used as evidence of ownership in any land registration proceedings
under the Torrens system.
All instruments affecting lands originally registered under the Spanish Mortgage Law may be recorded
under Section 194 of the Revised Administrative Code, as amended by Act 3344.
In view hereof, this is as good a time as any, to remind the Solicitor General to be more vigilant in
handling land registration cases and intestate proceedings involving portions of the subject estate. It is
not too late in the day for the Office of the Solicitor General to contest the Torrens titles of those who have
acquired ownership of such portions of land that rightfully belong to the State.
In fine, the release of the matured Land Bank Capital Bonds issued in favor of Mariano San Pedro y
Esteban on August 13, 1968 sought by one Catalino San Pedro, alleged heir, legal holder and owner of
Titulo de Propriedad No. 4136 is a matter not ripe for adjudication in these cases. Firstly, Catalino San
Pedro is not a party in any of the two cases before us for review, hence, this Court in a Resolution dated
May 10, 1993,
78
denied Catalino's motion for leave to reopen and/or new trial. And, secondly, the
aforementioned bonds were not included in the inventory of the subject estate submitted by then
administrators, Engracio San Pedro and Justino Benito before the probate court.
WHEREFORE, in view of all the foregoing, the petitions in G.R Nos. 103727 and 106496 are hereby
DISMISSED for lack of merit.
Consequently, in G.R No. 103727, the decision of the Court of Appeals dated January 20, 1992 is hereby
AFFIRMED.
In G.R No. 106496, judgment is hereby rendered as follows:
(1) Titulo de Propriedad No. 4136 is declared null and void and, therefore, no rights could
be derived therefrom;
(2) All lands covered by Titulo de Propriedad No. 4136 are excluded from the inventory of
the estate of the late Mariano San Pedro y Esteban;
(3) The petition for letters of administration, docketed as Special Proceedings No. 312-B,
should be, as it is, hereby closed and terminated.
(4) The heirs, agents, privies and/or anyone acting for and in behalf of the estate of the
late Mariano San Pedro y Esteban are hereby disallowed to exercise any act of
possession or ownership or to otherwise, dispose of in any manner the whole or any
portion of the estate covered by Titulo de Propriedad No. 4136; and they are hereby
ordered to immediately vacate the same, if they or any of them are in possession thereof.
This judgment is IMMEDIATELY EXECUTORY.
SO ORDERED.
Narvasa, C.J., Padilla, Regalado, Davide, Jr., Romero, Bellosillo, Puno, Panganiban and Torres, Jr., JJ.,
concur.
Melo, Kapunan, Mendoza and Francisco, JJ., took no part.
Vitug, J., is on leave.

9. G.R. No. L-8936 October 2, 1915
CONSUELO LEGARDA, with her husband MAURO PRIETO, plaintiffs-appellants,
vs.
N.M. SALEEBY, defendant-appellee.
Singson, Ledesma and Lim for appellants.
D.R. Williams for appellee.

JOHNSON, J .:
From the record the following facts appear:
First. That the plaintiffs and the defendant occupy, as owners, adjoining lots in the district of Ermita in the
city of Manila.
Second. That there exists and has existed a number of years a stone wall between the said lots. Said wall
is located on the lot of the plaintiffs.
Third. That the plaintiffs, on the 2d day of March, 1906, presented a petition in the Court of Land
Registration for the registration of their lot. After a consideration of said petition the court, on the 25th day
of October, 1906, decreed that the title of the plaintiffs should be registered and issued to them the
original certificate provided for under the torrens system. Said registration and certificate included the
wall.
Fourth. Later the predecessor of the defendant presented a petition in the Court of Land Registration for
the registration of the lot now occupied by him. On the 25th day of March, 1912, the court decreed the
registration of said title and issued the original certificate provided for under the torrens system. The
description of the lot given in the petition of the defendant also included said wall.
Fifth. Several months later (the 13th day of December, 1912) the plaintiffs discovered that the wall which
had been included in the certificate granted to them had also been included in the certificate granted to
the defendant .They immediately presented a petition in the Court of Land Registration for an adjustment
and correction of the error committed by including said wall in the registered title of each of said parties.
The lower court however, without notice to the defendant, denied said petition upon the theory that,
during the pendency of the petition for the registration of the defendant's land, they failed to make any
objection to the registration of said lot, including the wall, in the name of the defendant.
Sixth. That the land occupied by t he wall is registered in the name of each of the owners of the adjoining
lots. The wall is not a joint wall.
Under these facts, who is the owner of the wall and the land occupied by it?
The decision of the lower court is based upon the theory that the action for the registration of the lot of the
defendant was a judicial proceeding and that the judgment or decree was binding upon all parties who did
not appear and oppose it. In other words, by reason of the fact that the plaintiffs had not opposed the
registration of that part of the lot on which the wall was situate they had lost it, even though it had been
theretofore registered in their name. Granting that theory to be correct one, and granting even that the
wall and the land occupied by it, in fact, belonged to the defendant and his predecessors, then the same
theory should be applied to the defendant himself. Applying that theory to him, he had already lost
whatever right he had therein, by permitting the plaintiffs to have the same registered in their name, more
than six years before. Having thus lost hid right, may he be permitted to regain it by simply including it in a
petition for registration? The plaintiffs having secured the registration of their lot, including the wall, were
they obliged to constantly be on the alert and to watch all the proceedings in the land court to see that
some one else was not having all, or a portion of the same, registered? If that question is to be answered
in the affirmative, then the whole scheme and purpose of the torrens system of land registration must fail.
The real purpose of that system is to quiet title to land; to put a stop forever to any question of the legality
of the title, except claims which were noted at the time of registration, in the certificate, or which may arise
subsequent thereto. That being the purpose of the law, it would seem that once a title is registered the
owner may rest secure, without the necessity of waiting in the portals of the court, or sitting in the
"mirador de su casa," to avoid the possibility of losing his land. Of course, it can not be denied that the
proceeding for the registration of land under the torrens system is judicial (Escueta vs. .Director of Lands,
16 Phil. Rep., 482). It is clothed with all the forms of an action and the result is final and binding upon all
the world. It is an action in rem. (Escueta vs. Director of Lands (supra); Grey Alba vs. De la Cruz, 17 Phil.
rep., 49 Roxas vs. Enriquez, 29 Phil. Rep., 31; Tyler vs. Judges, 175 Mass., 51 American Land
Co. vs. Zeiss, 219 U.S., 47.)
While the proceeding is judicial, it involves more in its consequences than does an ordinary action. All the
world are parties, including the government. After the registration is complete and final and there exists
no fraud, there are no innocent third parties who may claim an interest. The rights of all the world are
foreclosed by the decree of registration. The government itself assumes the burden of giving notice to all
parties. To permit persons who are parties in the registration proceeding (and they are all the world) to
again litigate the same questions, and to again cast doubt upon the validity of the registered title, would
destroy the very purpose and intent of the law. The registration, under the torrens system, does not give
the owner any better title than he had. If he does not already have a perfect title, he can not have it
registered. Fee simple titles only may be registered. The certificate of registration accumulates in open
document a precise and correct statement of the exact status of the fee held by its owner. The certificate,
in the absence of fraud, is the evidence of title and shows exactly the real interest of its owner.
The title once registered, with very few exceptions, should not thereafter be impugned, altered, changed,
modified, enlarged, or diminished, except in some direct proceeding permitted by law. Otherwise all
security in registered titles would be lost. A registered title can not be altered, modified, enlarged, or
diminished in a collateralproceeding and not even by a direct proceeding, after the lapse of the period
prescribed by law.
For the difficulty involved in the present case the Act (No. 496) providing for the registration of titles under
the torrens system affords us no remedy. There is no provision in said Act giving the parties relief under
conditions like the present. There is nothing in the Act which indicates who should be the owner of land
which has been registered in the name of two different persons.
The rule, we think, is well settled that the decree ordering the registration of a particular parcel of land is a
bar to future litigation over the same between the same parties .In view of the fact that all the world are
parties, it must follow that future litigation over the title is forever barred; there can be no persons who are
not parties to the action. This, we think, is the rule, except as to rights which are noted in the certificate or
which arise subsequently, and with certain other exceptions which need not be dismissed at present. A
title once registered can not be defeated, even by an adverse, open, and notorious possession.
Registered title under the torrens system can not be defeated by prescription (section 46, Act No. 496).
The title, once registered, is notice to the world. All persons must take notice. No one can plead
ignorance of the registration.
The question, who is the owner of land registered in the name of two different persons, has been
presented to the courts in other jurisdictions. In some jurisdictions, where the "torrens" system has been
adopted, the difficulty has been settled by express statutory provision. In others it has been settled by the
courts. Hogg, in his excellent discussion of the "Australian Torrens System," at page 823, says: "The
general rule is that in the case of two certificates of title, purporting to include the same land, the earlier in
date prevails, whether the land comprised in the latter certificate be wholly, or only in part, comprised in
the earlier certificate. (Oelkers vs. Merry, 2 Q.S.C.R., 193; Miller vs. Davy, 7 N.Z.R., 155;
Lloyd vs. Myfield, 7 A.L.T. (V.) 48; Stevens vs. Williams, 12 V.L. R., 152; Register of Titles, vs. Esperance
Land Co., 1 W.A.R., 118.)" Hogg adds however that, "if it can be very clearly ascertained by the ordinary
rules of construction relating to written documents, that the inclusion of the land in the certificate of title of
prior date is a mistake, the mistake may be rectified by holding the latter of the two certificates of title to
be conclusive." (See Hogg on the "Australian torrens System," supra, and cases cited. See also the
excellent work of Niblack in his "Analysis of the Torrens System," page 99.) Niblack, in discussing the
general question, said: "Where two certificates purport to include the same land the earlier in date
prevails. ... In successive registrations, where more than one certificate is issued in respect of a particular
estate or interest in land, the person claiming under the prior certificates is entitled to the estate or
interest; and that person is deemed to hold under the prior certificate who is the holder of, or whose claim
is derived directly or indirectly from the person who was the holder of the earliest certificate issued in
respect thereof. While the acts in this country do not expressly cover the case of the issue of two
certificates for the same land, they provide that a registered owner shall hold the title, and the effect of
this undoubtedly is that where two certificates purport to include the same registered land, the holder of
the earlier one continues to hold the title" (p. 237).
Section 38 of Act No. 496, provides that; "It (the decree of registration) shall be conclusive upon and
against all persons, including the Insular Government and all the branches thereof, whether mentioned by
name in the application, notice, or citation, or included in the general description "To all whom it may
concern." Such decree shall not be opened by reason of the absence, infancy, or other disability of any
person affected thereby, nor by any proceeding in any court for reversing judgments or decrees; subject,
however, to the right of any person deprived of land or of any estate or interest therein by decree of
registration obtained by fraud to file in the Court of Land Registration a petition for review within one
year after entry of the decree (of registration), provided no innocent purchaser for value has acquired an
interest.
It will be noted, from said section, that the "decree of registration" shall not be opened, for any reason, in
any court, except for fraud, and not even for fraud, after the lapse of one year. If then the decree of
registration can not be opened for any reason, except for fraud, in a direct proceeding for that purpose,
may such decree be opened or set aside in a collateral proceeding by including a portion of the land in a
subsequent certificate or decree of registration? We do not believe the law contemplated that a person
could be deprived of his registered title in that way.
We have in this jurisdiction a general statutory provision which governs the right of the ownership of land
when the same is registered in the ordinary registry in the name of two persons. Article 1473 of the Civil
Code provides, among other things, that when one piece of real property had been sold to two different
persons it shall belong to the person acquiring it, who first inscribes it in the registry. This rule, of course,
presupposes that each of the vendees or purchasers has acquired title to the land. The real ownership in
such a case depends upon priority of registration. While we do not now decide that the general provisions
of the Civil Code are applicable to the Land Registration Act, even though we see no objection thereto,
yet we think, in the absence of other express provisions, they should have a persuasive influence in
adopting a rule for governing the effect of a double registration under said Act. Adopting the rule which we
believe to be more in consonance with the purposes and the real intent of the torrens system, we are of
the opinion and so decree that in case land has been registered under the Land Registration Act in the
name of two different persons, the earlier in date shall prevail.
In reaching the above conclusion, we have not overlooked the forceful argument of the appellee. He says,
among other things; "When Prieto et al. were served with notice of the application of Teus (the
predecessor of the defendant) they became defendants in a proceeding wherein he, Teus, was seeking
to foreclose their right, and that of orders, to the parcel of land described in his application. Through their
failure to appear and contest his right thereto, and the subsequent entry of a default judgment against
them, they became irrevocably bound by the decree adjudicating such land to Teus. They had their day in
court and can not set up their own omission as ground for impugning the validity of a judgment duly
entered by a court of competent jurisdiction. To decide otherwise would be to hold that lands with torrens
titles are above the law and beyond the jurisdiction of the courts".
As was said above, the primary and fundamental purpose of the torrens system is to quiet title. If the
holder of a certificate cannot rest secure in this registered title then the purpose of the law is defeated. If
those dealing with registered land cannot rely upon the certificate, then nothing has been gained by the
registration and the expense incurred thereby has been in vain. If the holder may lose a strip of his
registered land by the method adopted in the present case, he may lose it all. Suppose within the six
years which elapsed after the plaintiff had secured their title, they had MORTGAGED or sold their right,
what would be the position or right of the mortgagee or vendee? That mistakes are bound to occur cannot
be denied, and sometimes the damage done thereby is irreparable. It is the duty of the courts to adjust
the rights of the parties under such circumstances so as to minimize such damages, taking into
consideration al of the conditions and the diligence of the respective parties to avoid them. In the present
case, the appellee was the first negligent (granting that he was the real owner, and if he was not the real
owner he can not complain) in not opposing the registration in the name of the appellants. He was a
party-defendant in an action for the registration of the lot in question, in the name of the appellants, in
1906. "Through his failure to appear and to oppose such registration, and the subsequent entry of a
default judgment against him, he became irrevocably bound by the decree adjudicating such land to the
appellants. He had his day in court and should not be permitted to set up his own omissions as the
ground for impugning the validity of a judgment duly entered by a court of competent jurisdiction."
Granting that he was the owner of the land upon which the wall is located, his failure to oppose the
registration of the same in the name of the appellants, in the absence of fraud, forever closes his mouth
against impugning the validity of that judgment. There is no more reason why the doctrine invoked by the
appellee should be applied to the appellants than to him.
We have decided, in case of double registration under the Land Registration Act, that the owner of the
earliest certificate is the owner of the land. That is the rule between original parties. May this rule be
applied to successive vendees of the owners of such certificates? Suppose that one or the other of the
parties, before the error is discovered, transfers his original certificate to an "innocent purchaser." The
general rule is that the vendee of land has no greater right, title, or interest than his vendor; that he
acquires the right which his vendor had, only. Under that rule the vendee of the earlier certificate would
be the owner as against the vendee of the owner of the later certificate.
We find statutory provisions which, upon first reading, seem to cast some doubt upon the rule that the
vendee acquires the interest of the vendor only. Sections 38, 55, and 112 of Act No. 496 indicate that the
vendee may acquire rights and be protected against defenses which the vendor would not. Said sections
speak of available rights in favor of third parties which are cut off by virtue of the sale of the land to an
"innocent purchaser." That is to say, persons who had had a right or interest in land wrongfully included in
an original certificate would be unable to enforce such rights against an "innocent purchaser," by virtue of
the provisions of said sections. In the present case Teus had his land, including the wall, registered in his
name. He subsequently sold the same to the appellee. Is the appellee an "innocent purchaser," as that
phrase is used in said sections? May those who have been deprived of their land by reason of a mistake
in the original certificate in favor of Teus be deprived of their right to the same, by virtue of the sale by him
to the appellee? Suppose the appellants had sold their lot, including the wall, to an "innocent purchaser,"
would such purchaser be included in the phrase "innocent purchaser," as the same is used in said
sections? Under these examples there would be two innocent purchasers of the same land, is said
sections are to be applied .Which of the two innocent purchasers, if they are both to be regarded as
innocent purchasers, should be protected under the provisions of said sections? These questions indicate
the difficulty with which we are met in giving meaning and effect to the phrase "innocent purchaser," in
said sections.
May the purchaser of land which has been included in a "second original certificate" ever be regarded as
an "innocent purchaser," as against the rights or interest of the owner of the first original certificate, his
heirs, assigns, or vendee? The first original certificate is recorded in the public registry. It is never issued
until it is recorded. The record notice to all the world. All persons are charged with the knowledge of what
it contains. All persons dealing with the land so recorded, or any portion of it, must be charged with notice
of whatever it contains. The purchaser is charged with notice of every fact shown by the record and is
presumed to know every fact which the record discloses .This rule is so well established that it is scarcely
necessary to cite authorities in its support (Northwestern National Bank vs. Freeman, 171 U.S., 620, 629;
Delvin on Real Estate, sections 710, 710 [a]).
When a conveyance has been properly recorded such record is constructive notice of its contents and all
interests, legal and equitable, included therein. (Grandin vs. Anderson, 15 Ohio State, 286, 289;
Orvis vs. Newell, 17 Conn., 97; Buchanan vs. Intentional Bank, 78 Ill., 500; Youngs vs. Wilson, 27 N.Y.,
351; McCabe vs. Grey, 20 Cal., 509; Montefiore vs. Browne, 7 House of Lords Cases, 341.)
Under the rule of notice, it is presumed that the purchaser has examined every instrument of record
affecting the title. Such presumption is irrebutable. He is charged with notice of every fact shown by the
record and is presumed to know every fact which an examination of the record would have disclosed.
This presumption cannot be overcome by proof of innocence or good faith. Otherwise the very purpose
and object of the law requiring a record would be destroyed. Such presumption cannot be defeated by
proof of want of knowledge of what the record contains any more than one may be permitted to show that
he was ignorant of the provisions of the law. The rule that all persons must take notice of the facts which
the PUBLIC RECORD contains is a rule of law. The rule must be absolute. Any variation would lead to
endless confusion and useless litigation.
While there is no statutory provision in force here requiring that original deeds of conveyance of real
property be recorded, yet there is a rule requiring mortgages to be recorded. (Arts. 1875 and 606 of the
Civil Code.) The record of a MORTGAGE is indispensable to its validity. (Art .1875.) In the face of that
statute would the courts allow a mortgage to be valid which had not been recorded, upon the plea of
ignorance of the statutory provision, when third parties were interested? May a purchaser of land,
subsequent to the recorded mortgage, plead ignorance of its existence, and by reason of such ignorance
have the land released from such lien? Could a purchaser of land, after the recorded mortgage, be
relieved from the mortgage lien by the plea that he was a bona fide purchaser? May there be a bona
fide purchaser of said land, bona fide in the sense that he had no knowledge of the existence of the
mortgage? We believe the rule that all persons must take notice of what the PUBLIC RECORD
contains in just as obligatory upon all persons as the rule that all men must know the law; that no one
can plead ignorance of the law. The fact that all men know the law is contrary to the presumption. The
conduct of men, at times, shows clearly that they do not know the law. The rule, however, is mandatory
and obligatory, notwithstanding. It would be just as logical to allow the defense of ignorance of the
existence and contents of a public record.
In view, therefore, of the foregoing rules of law, may the purchaser of land from the owner of the second
original certificate be an "innocent purchaser," when a part or all of such land had theretofore been
registered in the name of another, not the vendor? We are of the opinion that said sections 38, 55, and
112 should not be applied to such purchasers. We do not believe that the phrase "innocent purchaser
should be applied to such a purchaser. He cannot be regarded as an "innocent purchaser" because of the
facts contained in the record of the first original certificate. The rule should not be applied to the
purchaser of a parcel of land the vendor of which is not the owner of the original certificate, or his
successors. He, in nonsense, can be an "innocent purchaser" of the portion of the land included in
another earlier original certificate. The rule of notice of what the record contains precludes the idea of
innocence. By reason of the prior registry there cannot be an innocent purchaser of land included in a
prior original certificate and in a name other than that of the vendor, or his successors. In order to
minimize the difficulties we think this is the safe rule to establish. We believe the phrase "innocent
purchaser," used in said sections, should be limited only to cases where unregistered land has been
wrongfully included in a certificate under the torrens system. When land is once brought under the torrens
system, the record of the original certificate and all subsequent transfers thereof is notice to all the world.
That being the rule, could Teus even regarded as the holder in good fifth of that part of the land included
in his certificate of the appellants? We think not. Suppose, for example, that Teus had never had his lot
registered under the torrens system. Suppose he had sold his lot to the appellee and had included in his
deed of transfer the very strip of land now in question. Could his vendee be regarded as an "innocent
purchaser" of said strip? Would his vendee be an "innocent purchaser" of said strip? Certainly not. The
record of the original certificate of the appellants precludes the possibility. Has the appellee gained any
right by reason of the registration of the strip of land in the name of his vendor? Applying the rule of notice
resulting from the record of the title of the appellants, the question must be answered in the negative. We
are of the opinion that these rules are more in harmony with the purpose of Act No. 496 than the rule
contended for by the appellee. We believe that the purchaser from the owner of the later certificate, and
his successors, should be required to resort to his vendor for damages, in case of a mistake like the
present, rather than to molest the holder of the first certificate who has been guilty of no negligence. The
holder of the first original certificate and his successors should be permitted to rest secure in their title,
against one who had acquired rights in conflict therewith and who had full and complete knowledge of
their rights. The purchaser of land included in the second original certificate, by reason of the facts
contained in the public record and the knowledge with which he is charged and by reason of his
negligence, should suffer the loss, if any, resulting from such purchase, rather than he who has obtained
the first certificate and who was innocent of any act of negligence.
The foregoing decision does not solve, nor pretend to solve, all the difficulties resulting from double
registration under the torrens system and the subsequent transfer of the land. Neither do we now attempt
to decide the effect of the former registration in the ordinary registry upon the registration under the
torrens system. We are inclined to the view, without deciding it, that the record under the torrens system,
supersede all other registries. If that view is correct then it will be sufficient, in dealing with land registered
and recorded alone. Once land is registered and recorded under the torrens system, that record alone
can be examined for the purpose of ascertaining the real status of the title to the land.
It would be seen to a just and equitable rule, when two persons have acquired equal rights in the same
thing, to hold that the one who acquired it first and who has complied with all the requirements of the law
should be protected.
In view of our conclusions, above stated, the judgment of the lower court should be and is hereby
revoked. The record is hereby returned to the court now having and exercising the jurisdiction heretofore
exercised by the land court, with direction to make such orders and decrees in the premises as may
correct the error heretofore made in including the land in the second original certificate issued in favor of
the predecessor of the appellee, as well as in all other duplicate certificates issued.
Without any findings as to costs, it is so ordered.
Arellano, C.J., Torrens, and Araullo, JJ., concur.



Separate Opinions

TRENT, J ., dissenting:
I dissent.
In cases of double or overlapping registration, I am inclined to agree with the reasoning and authority on
which it is held in the majority opinion (first) that the original holder of the prior certificate is entitled to the
land as against the original holder of the later certificate, where there has been no transfer of title by
either party to an innocent purchaser; both, as is shown in the majority opinion, being at fault in permitting
the double registration to take place; (second) that an innocent purchaser claiming under the prior
certificate is entitled to the land as against the original holder of the later certificate, and also as against
innocent purchasers from the holder of the later certificate; the innocent purchaser being in no wise at
fault in connection with the issuance of the later certificate.
But I am of opinion that neither the authorities cited, nor the reasoning of the majority opinion sustains the
proposition that the original holder of the prior certificate is entitled to the land as against an innocent
purchaser from the holder of the later certificate.
As to the text-book authorities cited in the majority opinion, it is sufficient to say that the rules laid down by
both Hogg and Niblack are mere general rules, admittedly subject to exception, and of course of no
binding force or authority where the reasoning upon which these rules are based is applicable to the facts
developed in a particular case.
In its last analysis the general rule laid down in the majority opinion rests upon the proposition set forth in
the last page of the opinion wherein it is said that "it would seem to be a just and equitable rule, when two
persons have acquired equal rights in the same thing, to hold that the one who acquired it first and who
has complied with all the requirements of the law should be protected." The rule, as applied to the matter
in hand, may be stated as follows: It would seem to be a just and equitable rule when two persons have
acquired separate and independent registered titles to the same land, under the Land Registration Act, to
hold that the one who first acquired registered title and who has complied with all the requirements of the
law in that regard should be protected, in the absence of any express statutory provision to the contrary.
Thus stated I have no quarrel with the doctrine as a statement of the general rule to be applied in cases
of double or overlapping registration under the Land Registration Act; for it is true as stated in the majority
opinion that in the adjudication and registration of titles by the Courts of Land Registration "mistakes are
bound to occur, and sometimes the damage done thereby is irreparable;" and that in the absence of
statutory provisions covering such cases, "it is the duty of the courts to adjust the rights of the parties,
under such circumstances, so as to minimize such damages, taking into consideration all of the
conditions, and the diligence of the respective parties to avoid them."
But like most such general rules, it has its exceptions and should not be applied in a case wherein the
reasons on which it is based do not exist, or in cases wherein still more forceful reasons demand the
application of a contrary rule.
The general rule relied upon in the majority opinion is a mere application of a well settled equity rule that:
"Where conflicting equities are otherwise equal in merit, that which first occurred will be given the
preference." But it is universally laid down by all the courts which have had occasion to apply this equity
rule that "it should be the last test resorted to," and that "it never prevails when any other equitable
ground for preference exists." (See 19 Cent. Dig., tit. Equity, par. 181; and may cases cited in 16 Cyc.,
139, note 57.) It follows that the general rules, that in cases of double or overlapping registration the
earlier certificate should be protected, ought not to prevail so as to deprive an innocent purchaser under
the later certificate of his title of the earlier certificate contributed to the issuance of the later certificate.
Hence the holder of the earlier certificate of title should not be heard to invoke the"just and equitable
rule" as laid down in the majority opinion, in order to have his own title protected and the title of an
innocent purchaser of a later certificate cancelled or annulled, in any case wherein it appears that the
holder of the later certificate was wholly without fault, while the holder of the issuance of the later
certificate, in that he might have prevented its issuance by merely entering his appearance in court in
response to lawful summons personally served upon him in the course of the proceedings for the
issuance of the second certificate, and pleading his superior rights under the earlier certificate, instead of
keeping silent and by his silence permitting a default judgment to be entered against him adjudicating title
in favor of the second applicant.
The majority opinion clearly recognizes the soundness of the principles I am contending for by reasoning
(with which I am inclined to agree) whereby it undertakes to demonstrate that as between the original
holders of the double or overlapping registration the general rule should prevail, because both such
original parties must held to have been fault and, their equities being equal, preference should be given to
the earlier title.
The majority opinion further recognizes the soundness of my contention by the reasoning whereby it
undertakes to sustain the application of the general rule in favor of the original holder of the earlier
certificate against purchasers from the original holder of the later certificate, by an attempt to demonstrate
that such purchasers can in no event be held to be innocent purchasers; because, as it is said,
negligence may and should always be imputed to such a purchaser, so that in no event can he claim to
be without fault when it appears that the lands purchased by him from the holder of a duly registered
certificate of title are included within the bounds of the lands described in a certificate of title of an earlier
date.
At considerable length the majority opinion (in reliance upon the general rule laid down under the various
systems of land registration, other than those based on the torrens system) insists that a purchaser of
land land duly registered in the Land Registration Court, is charged with notice of the contents of each
and every one of the thousands and tens of thousands of certificates of registry on file in the land registry
office, so that negligencemay be imputed to him if he does not ascertain that all or any part of the land
purchased by him is included within the boundary lines of any one of the thousands or tens of thousands
of tracts of land whose original registry bears an earlier date than the date of the original registry of the
land purchased by him. It is contended that he cannot claim to be without fault should he buy such land
because, as it is said, it was possible for him to discover that the land purchased by him had been made
the subject of double or overlapping registration by a comparison of the description and boundary lines of
the thousands of tracts and parcels of land to be found in the land registry office.
But such ruling goes far to defeat one of the principal objects sought to be attained by the introduction
and adoption of the so-called torrens system for the registration of land. The avowed intent of that system
of land registration is to relieve the purchase of registered lands from the necessity of looking farther than
the certificate of title of the vendor in order that he may rest secure as to the validity of the title to the
lands conveyed to him. And yet it is said in the majority opinion that he is charged with notice of the
contents of every other certificate of title in the office of the registrar so that his failure to acquaint himself
with its contents may be imputed to him as negligence.
If the rule announced in the majority opinion is to prevail, the new system of land registration, instead of
making transfers of real estate simple, expenditious and secure, and instead of avoiding the necessity for
expensive and oftimes uncertain searches of the land record and registries, in order to ascertain the true
condition of the title before purchase, will, in many instances, add to the labor, expense and uncertainty of
any attempt by a purchaser to satisfy himself as to the validity of the title to lands purchased by him.
As I have said before, one of the principal objects, if not the principal object, of the torrens system of land
registration upon which our Land Registration Act is avowedly modelled is to facilitate the transfer of real
estate. To that end the Legislature undertakes to relieve prospective purchasers and all others dealing in
registered lands from the necessity of looking farther than the certificate of title to such lands furnished by
the Court of Land Registration, and I cannot, therefore, give my consent to a ruling which charges a
purchaser or mortgage of registered lands with notice of the contents of every other certificate of title in
the land registry, so that negligence and fault may be imputed to him should he be exposed to loss or
damages as a result of the lack of such knowledge.
Suppose a prospective purchaser of lands registered under the Land Registration Act desires to avoid the
imputation of negligence in the event that, unknown to him, such lands have been made the subject of
double or overlapping registration, what course should he pursue? What measures should he adopt in
order to search out the information with notice of which he is charged? There are no indexes to guide him
nor is there anything in the record or the certificate of title of the land he proposes to buy which
necessarily or even with reasonable probability will furnish him a clue as to the fact of the existence of
such double or overlapping registration. Indeed the only course open to him, if he desires to assure
himself against the possibility of double or overlapping registration, would even seem to be a careful,
laborious and extensive comparison of the registered boundary lines contained in the certificate of title of
the tract of land he proposes to buy with those contained in all the earlier certificates of title to be found in
the land registry. Assuredly it was never the intention of the author of the new Land Registration Act to
impose such a burden on a purchaser of duly registered real estate, under penalty that a lack of the
knowledge which might thus be acquired may be imputed to him by this court as negligence in ruling
upon the respective equities of the holders of lands which have been the subject of double or overlapping
registration.
On the other hand, I think that negligence and fault may fairly be imputed to a holder of a registered
certificate of title who stood supinely by and let a default judgment be entered against him, adjudicating all
or any part of his registered lands to another applicant, if it appears that he was served with notice or had
actual notice of the pendency of the proceedings in the Court of Land Registration wherein such default
judgment was entered.
The owner of land who enjoys the benefits secured to him by its registry in the Court of Land Registration
may reasonably be required to appear and defend his title when he has actual notice that proceedings
are pending in that court wherein another applicant, claiming the land as his own, is seeking to secure its
registry in his name. All that is necessary for him to do is to enter his appearance in those proceedings,
invite the court's attention to the certificate of title registered in his name, and thus, at the cost of the
applicant, avoid all the damage and inconvenience flowing from the double or overlapping registration of
the land in question. There is nothing in the new system of land registration which seems to render it
either expedient or necessary to relieve a holder of a registered title of the duty of appearing and
defending that title, when he has actual notice that it is being attacked in a court of competent jurisdiction,
and if, as a result of his neglect or failure so to do, his lands become subject to double or overlapping
registration, he should not be permitted to subject an innocent purchaser, holding under the later
certificate, to all the loss and damage resulting from the double or overlapping registration, while he goes
scot free and holds the land under a manifest misapplication of the equitable rule that "where conflicting
equities are otherwise equal in merit, that which first accrued will be given the preference." It is only
where both or neither of the parties are at fault that the rule is properly applicable as between opposing
claimants under an earlier and a later certificate of registry to the same land.
Of course all that is said in the briefs of counsel and the majority opinion as to the right of the holder of a
certificate to rest secure in his registered title so that those dealing with registered lands can confidently
rely upon registry certificates thereto is equally forceful by way of argument in favor of the holder of one or
the other certificate in case of double or overlapping registration. The problem is to determine which of
the certificate holders is entitled to the land. The decision of that question in favor of either one must
necessarily have the effect of destroying the value of the registered title of the other and to that extent
shaking the public confidence in the value of the whole system for the registration of lands. But, in the
language of the majority opinion, "that mistakes are bound to occur cannot be denied and sometimes the
damage done thereby is irreparable. It is the duty of the courts to adjust the rights of the parties under
such circumstances so as to minimize the damages, taking into consideration all the conditions and the
diligence of the respective parties to avoid them."lawphil.net
It will be observed that I limit the exception to the general equitable rule, as laid down in the majority
opinion, to case wherein the holder of the earlier certificate of title has actual notice of the pendency of
the proceedings in the course of which the latter certificate of title was issued, or to cases in which he has
received personal notice of the pendency of those proceedings. Unless he has actual notice of the
pendency of such proceedings I readily agree with the reasoning of the majority opinion so far as it holds
that negligence, culpable negligence, should not be imputed to him for failure to appear and defend his
title so as to defeat his right to the benefit of the equitable rule. It is true that the order of publication in
such cases having been duly complied with, all the world is charged with notice thereof, but it does not
necessarily follow that, in the absence of actual notice, culpable negligence in permitting a default
judgment to be entered against him may be imputed to the holder of the earlier certificate so as to defeat
his right to the land under the equitable rule favoring the earlier certificate. Such a holding would have the
effect (to quote the language of the majority opinion) of requiring the holder of a certificate of title to wait
indefinitely "in the portals of the court" and to sit in the "mirador de su casa" in order to avoid the
possibility of losing his lands; and I agree with the writer of the majority opinion that to do so would place
an unreasonable burden on the holders of such certificate, which was not contemplated by the authors of
the Land Registration Act. But no unreasonable burden is placed upon the holder of a registered title by a
rule which imputes culpable negligence to him when he sits supinely by and lets a judgment in default be
entered against him adjudicating title to his lands in favor of another applicant, despite the fact that he has
actual knowledge of the pendency of the proceedings in which such judgment is entered and despite the
fact that he has been personally served with summons to appear and default his title.
"Taking into consideration all of the conditions and the diligence of the respective parties," it seems to me
that there is no "equality in merit" between the conflicting equities set up by an innocent purchaser who
acquires title to the land under a registered certificate, and the holder of an earlier certificate who
permitted a default judgment to be entered against him, despite actual notice of the pendency of the
proceedings in the course of which the later certificate was issued.
I am convinced, furthermore, that aside from the superior equities of the innocent purchaser in cases
such as that now under discussion, there are strong reasons of convenience and public policy which
militate in favor of the recognition of his title rather than that of the holder of the earlier title.
One ruling exposes all persons purchasing or dealing in registered lands to unknown, unspecified and
uncertain dangers, to guard against which all such persons will be put to additional cost, annoyance and
labor on every occasion when any transaction is had with regard to such lands; while the other ruling
tends to eliminate consequences so directly adverse to the purpose and object for which the land
registration law was enacted, and imposes no burden upon any holder of a certificate of registered lands
other than that of defending his title on those rare, definite and specific occasions wherein he has actual
notice that his title is being challenged in a Court of Land Registration, a proceeding in which the cost and
expense is reduced to the minimum by the conclusive character of his certificate of title in support of his
claim of ownership. Furthermore, judgment against the innocent purchaser and in favor of the holder of
the earlier certificate in a case such as that under consideration must inevitably tend to increase the
danger of double or overlapping registrations by encouraging holders of registered titles, negligently or
fraudulently and conclusively, to permit default judgments to be entered against them adjudicating title to
all or a part of their registered lands in favor of other applicants, despite actual notice of the pendency of
judicial proceedings had for that purpose, and this, without adding in any appreciable degree to the
security of thir titles, and merely to save them the very slight trouble or inconvenience incident to an entry
of appearance in the court in which their own titles were secured, and inviting attention to the fact that
their right, title and ownership in the lands in questions has already been conclusively adjudicated.
The cases wherein there is a practical possibility of double or overlapping registration without actual
notice to the holder of the earlier certificate must in the very nature of things to be so rare as to be
practically negligible. Double or overlapping registration almost invariably occurs in relation to lands held
by adjoining occupants or claimants. It is difficult to conceive of a case wherein double registration can
take place, in the absence of fraud, without personal service of notice of the pendency of the proceedings
upon the holder of the earlier certificate, the statute requiring such notice to be served upon the owner or
occupant of all lands adjoining those for which application for registration is made; and the cases wherein
an adjoining land owner can, even by the use of fraud, conduct proceedings for the registration of his land
to a successful conclusion without actual notice to the adjoining property owners must be rare indeed.
In the case at bar the defendant purchased the land in question from the original holder of a certificate of
title issued by the Court of Land Registration, relying upon the records of the Court of Land Registration
with reference thereto and with no knowledge that any part of the land thus purchased was included in an
earlier certificate of title issued to the plaintiff. The plaintiff, the holder of the earlier certificate of title,
negligently permitted a default judgment to be entered against him in the Court of Land Registration,
adjudicating part of the lands included in his own certificate of title in favor of another applicant, from
whom the defendant in this action acquired title, and this despite the fact that he was an adjoining land
owner, had actual notice of the pendency of the proceedings and was personally served with summons to
appear and defends his rights in the premises. It seems to me that there can be no reason for doubt as to
the respective merits of the equities of the parties, and further that the judgment of the majority in favor of
the plaintiff will inevitably tend to increase the number of cases wherein registered land owners in the
future will fail to appear and defend their titles when challenged in other proceedings in the Courts of
Land Registration, thereby enormously increasing the possibility and probability of loss and damage to
innocent third parties and dealers in registered lands generally, arising out of erroneous, double or
overlapping registration of lands by the Courts of Land Registration.
Carson, J., concurs.

10. [G.R. No. 133698. April 4, 2001]
ANTONIO TALUSAN and CELIA TALUSAN, petitioners, vs. HERMINIGILDO
*
TAYAG and JUAN
HERNANDEZ, respondents.
D E C I S I O N
PANGANIBAN, J .:
For purposes of real property taxation, the registered owner of a property is deemed the taxpayer and, hence,
the only one entitled to a notice of tax delinquency and the resultant proceedings relative to an auction
sale. Petitioners, who allegedly acquired the property through an unregistered deed of sale, are not entitled to such
notice, because they are not the registered owners. Moral lessons: real property buyers must register their purchases
as soon as possible and, equally important, they must pay their taxes on time.
The Case

Before us is a Petition for Review on Certiorari under Rule 45 of the Rules of Court, assailing the November
20, 1997 Decision
[1]
of the Court of Appeals (CA) in CA-GR CV No. 41586. The dispositive portion of the
challenged Decision is hereunder reproduced as follows:
WHEREFORE, premises considered, the appealed decision (dated February 4, 1993) of the Regional Trial Court
(Branch 7) in Baguio City in Civil Case No. 1456-R is hereby AFFIRMED, with costs against plaintiffs/appellants.
Also assailed is the April 27, 1998 CA Resolution
[2]
which denied petitioners Motion for Reconsideration.
The questioned CA ruling affirmed the Decision
[3]
of Branch 7 of the Regional Trial Court (RTC) of Baguio
City in Civil Case No. 1456-R. The RTC, in turn, dismissed an action for the annulment of the auction sale of a
condominium unit, covered by Condominium Certificate of Title No. 651 and located in Building IV, Europa
Condominium Villas, Baguio City.
The Facts

The CA summarized the antecedents of this case in this wise:
[4]

On June 28, 1988, [herein petitioners] filed a complaint wherein they alleged, inter alia, that:
--They bought the subject property covered by Condominium Certificate of Title No. 651, from its former owner,
Elias Imperial, as evidenced by a Deed of Absolute Sale:
--On October 15, 1985, [herein Respondent] Juan D. Hernandez, x x x sued x x x in his capacity as City Treasurer
of Baguio City, wrote a letter to the former owner Elias Imperial informing him that the above described property
would be sold at public auction on December 9, 1985, x x x to satisfy the delinquent real estate taxes, penalties and
cost of sale, and demanded payment of the sum of P4,039.80, representing total taxes due and penalties thereon;
--Elias Imperial and his entire family emigrated to Australia in 1974. Elias Imperial never authorized a certain
Dante Origan x x x to receive any letter or mail matter for and on his behalf;
--[Respondent] Hernandez sold the above-described property to [Respondent] Tayag for P4,400.00 without any
notice to the former owner thereof, [or] to [petitioners], and without compliance with the provisions of PD No. 464,
as evidenced by the Certificate of Sale;
--A final bill of sale was later issued in favor of the [Respondent] Hermenegildo Tayag. The assessed value alone
of the said property is P37,310.00 and the fair market value of the same is more than P300,000.00 and both
[respondents] knew these;
--The bid price of P4,400 is so unconscionably low and shocking to the conscience, thus, the sale for the
alleged UNPAID TAXES in the sum of P4,039.79, including penalties is null and void ab initio;
--[Petitioners] have been in actual possession of the Unit in question, since they bought the same from its former
owners, and their possession is open, public, continuous, adverse and in the concept of owners, while [Respondent]
Hermegildo Tayag has never been in possession of the said property;
--[Petitioners] through intermediaries offered to pay to the [respondents] the sum of P4,400 plus all interests and
expenses which [they] might have incurred x x x but said offer was rejected without any just [or] lawful cause.
There is a need to issue a writ of preliminary injunction to preserve the status quo.
They asked for: moral damages of not less than P50,000.00; exemplary damages of not less than P20,000.00;
attorneys fee of P30,000.00, plus appearance fee of P2,000.00 for every appearance; and litigation expenses of not
less than P5,000.00 to prosecute the case. (pages 3-8 of the Record)
On July 14, 1988, [Respondent] Hermenegildo Tayag filed his [A]nswer with [C]ounterclaim (pages 28-32 of the
Record), wherein he substantially denied the allegations in the complaint and, at the same time, raised the following
affirmative defenses, among others:
--(T)he ownership of the Condominium unit registered under Condominium Certificate of Title No. 651, Baguio
City, has been consolidated in his name by virtue of the decision of the Regional Trial Court of Baguio, Branch 6,
on September 16, 1987 x x x . The said decision has [become] final and executory as evidenced by the Certificate
of Finality issued on October 8, 1987;
--[Petitioners have] no cause of action against him, he being a buyer in good faith in a regular and lawful public
bidding in which any person is qualified to participate.
--The lower court has no jurisdiction over [petitioners] claim because the [petitioners] pray for the annulment of
the Certificate of the Sale and the Final Bill of Sale, which was affirmed by virtue of the decision of the Regional
Trial Court of Baguio, Branch 6, on September 16, 1987 x x x. The said decision has [become] final and executory
as evidenced by the Certificate of Finality issued on October 8, 1987;
--The public auction sale complied with the requirements of Presidential Decree No. 464 hence, the same is
lawful and valid:
--[Respondent] Tayag is not bound by the alleged [D]eed of [S]ale in favor of the [petitioners] by Elias [I]mperial,
because it was not registered and recorded with the Registry of Deeds of Baguio City.
[Respondent] Tayag then prayed for the award in his favor, of: moral damages of at least P50,000.00; exemplary
damages; attorneys fees in the sum of P10,000.00; and, expenses of litigation.
[Respondent] Hernandez likewise filed an [A]nswer on July 18. 1988, wherein he denied the material averments in
the complaint and stated that no irregularity or illegality was committed in the conduct of the proceedings with
respect to the delinquent real property of Elias Imperial and the actuations of the defendant herein were all within
the limits of his authority and in accordance with the provisions of the law pertaining to delinquent real property,
particularly, P.D. 464 otherwise known as the Real PROPERTY TAX Code and therefore, no damages may be
imputed against him. He also claimed, by way of affirmative defenses, that:
--The complaint states no cause of action against the [respondent] herein:
--[Petitioners] have not complied with x x x Section 83 of P.D. No. 464 x x x thus, the case cannot prosper;
--Granting that a Deed of Sale was actually issued in favor of the plaintiffs [because of] the fact that it is
unregistered, the same does not bind third persons including defendant herein.
In their Complaint, petitioners alleged that on December 7, 1981, they had acquired the condominium from
Elias Imperial, the original registered owner, for P100,000. The sale was purportedly evidenced by a Deed of Sale
which, however, had not and thenceforth never been registered with the Register of Deeds.
Petitioners also averred that on December 9, 1985, Baguio City Treasurer Juan Hernandez sold the property at
a public auction due to nonpayment of delinquent real estate taxes thereon. The property was sold to Respondent
Herminigildo Tayag for P4,400 which represented the UNPAID TAXES .
Thus, petitioners filed a Complaint seeking the annulment of the auction sale. They cited irregularities in the
proceedings and noncompliance with statutory requirements.
Dismissing the Complaint, Branch 7 of the RTC of Baguio City cited the December 16, 1987 judgment of
Branch 6 of the same court in LRC Adm. Case No.207-R. This earlier Branch 6 Decision had consolidated
ownership of the condominium unit in favor of Respondent Tayag. The Branch 7 Decision also cited the May 31,
1988 Order of Branch 5 of the same court which had granted a Petition for the Cancellation of
Condominium Certificate of Title No. 651 in the name of Elias Imperial and directed the Register of Deeds to issue
a new Certificate of Title in the name of Respondent Tayag. According to the trial court, the Decision in LRC Adm.
Case No. 207-R had already upheld the legality of the questioned auction sale. Hence, to rule again on the same
issue would amount to passing upon a judgment made by a coequal court, contrary to the principle of
conclusiveness of judgment.
Ruling of the CA

The appellate court affirmed the trial courts ruling and ratiocination. The CA explained that LRC Adm. Case
No. 207-R had already ruled on the validity of the auction sale of the subject condominium unit. It further sustained
the validity of that sale, because the city treasurer complied with the requirements of notice, publication and
posting. It added that [i]f [petitioners] never received the notices sent to Elias Imperial, then they have only
themselves to blame for failing to register the deed of sale between them and the former owner x x x.
Rejecting petitioners contention that the purchase price was inadequate, the CA ruled that such inadequacy
could not nullify the auction sale. It likewise held that petitioners had not established bad faith on the part of
respondents in conducting the auction sale. Finally, it agreed with the latters contention that the former were
remiss in causing the registration of the sale in their favor of the subject property and they likewise did not fulfill
their obligation to pay taxes. It [is] thus clear x x x they should only have themselves to blame. Laws exist to be
followed, failing in which the price must be paid.
Hence, this recourse.
[5]

The Issues

Petitioners assigned the following alleged errors for the consideration of this Court:
[6]

I. FIRST ASSIGNMENT OF ERROR
The Honorable Court of Appeals grievously erred in failing to consider that the petitioners were deprived of their
right to due process in this case due to the gross and inexcusable negligence of their former counsel who failed to
inform them of the decision in this case and protect their interest.
II. SECOND ASSIGNMENT OF ERROR
The Honorable Court of Appeals grievously erred in failing to nullify the auction sale of the subject property of
petitioners due to alleged tax delinquency when there was no compliance with the mandatory requirement of Section
46 of P.D. 464 that such notice of delinquency of the payment of the PROPERTY TAX should be published.
III. THIRD ASSIGNMENT OF ERROR
The Honorable Court of Appeals grievously erred in failing to consider the lack of personal notice of the sale for
public auction of the subject property to its owner which nullifies the said proceeding.
IV. FOURTH ASSIGNMENT OF ERROR
The Honorable Court of Appeals grievously erred in holding that the decision of the trial court in the petition for the
consolidation of the title case filed by the private respondent in LRC Admin. Case 207 is a bar to this proceeding.
V. FIFTH ASSIGNMENT OF ERROR
The Honorable Court of Appeals erred in not nullifying the auction sale of subject property on equitable
considerations.
We deem it appropriate to simplify the issues in this wise: (1) whether the RTC Decision in LRC Adm. Case
No. 207-R is a bar to this proceeding; and (2) whether the auction sale of the subject condominium unit should be
annulled on the grounds of (a) non-publication of the notice of delinquency for the payment of PROPERTY TAX ,
(b) lack of personal notice of the sale or public auction of the subject property and (c) equitable considerations. As a
preliminary matter, we shall also consider petitioners submission that they were deprived of due process because of
their counsels failure to inform them immediately of the receipt of the CA Decision.
Preliminary Matter:

Negligence of Petitioners Former Counsel

Petitioners aver that their former counsel informed them of the CA Decision only on February 5, 1998, more
than two months after he had received a copy on December 3, 1997. According to petitioners, their former
counsels negligence effectively deprived them of their right to due process.
We disagree. Notwithstanding its late filing, their Motion for Reconsideration was accepted and considered by
the CA. Hence, this issue has become moot, a fact which petitioners themselves admitted in their Memorandum:
As a matter of fact, in the very resolution of the Court of Appeals of April 27, 1998 (Annex C to Petition)
denying the motion for reconsideration, wherein the matter of inexcusable negligence of counsel in not informing
petitioners immediately of the decision of the court a quo, were among the grounds thereof, it was held that the
issues raised therein had already been considered in the Decision of November 20, 1997. The Court of Appeals
obviously considered that the Motion for Reconsideration was validly filed by petitioners so that the Court of
Appeals favorably considered the plea of petitioners to be afforded due process by acting on the Motion for
Reconsideration. Otherwise, it could have just denied said Motion for late filing or simply noted the same without
action.
[7]

Moreover, petitioners themselves declared in their Reply Memorandum
[8]
that this matter is no longer in issue:
At any rate this issue was raised in the Motion for Reconsideration of the Decision of the appellate court and
obviously it was favorably considered as the said Court denied the merit of said Motion by stating that the issues
raised have already been treated in the Decision, instead of outrightly denying the same for late filing. Hence, this is
no longer in issue in this proceeding.
[9]

First Issue:

Bar by Earlier J udgment

Petitioners contend that the Decision in LRC Adm. Case No. 207-R, rendered by the Regional Trial Court of
Baguio City (Branch 6), did not preclude the filing of a separate action to annul the auction sale. Citing Tiongco v.
Philippine Veterans Bank,
[10]
they aver that this RTC Branch had no jurisdiction to rule on the validity of that
sale. Hence, its Decision in the LRC case cannot bar the present proceedings.
Petitioners reliance on Tiongco is misplaced, considering that its factual incidents are different from those of
the present controversy. In that case, the trial court was acting on a Petition for the Surrender of Certificates of
Title. In LRC Adm. Case No. 207-R, the trial court was faced with a Petition for Consolidation of Ownership. It
had jurisdiction to rule on all matters necessary for the determination of the issue of ownership, including the
validity of the auction sale.
Indeed, this Court in several cases
[11]
has previously declared that a petition for the surrender of the owners
duplicate certificate involves contentious questions which should be threshed out in an ordinary case, because the
land registration court has no jurisdiction to try them.
Presidential Decree (PD) 1529, however, intended to avoid a multiplicity of suits and to promote the
expeditious termination of cases. In more recent cases,
[12]
therefore, the Court declared that this Decree
had eliminated the distinction between general jurisdiction vested in the regional trial court and the latters limited
jurisdiction when acting merely as a land registration court. Land registration courts, as such, can now hear and
decide even controversial and contentious cases, as well as those involving substantial issues.
[13]

Thus, petitioners err in contending that the RTC is, in a land registration case, barred from ruling on the
validity of the auction sale. That court now has the authority to act not only on applications for original registration,
but also on all petitions filed after the original registration of title. Coupled with this authority is the power to hear
and determine all questions arising upon such applications or petitions.
[14]
Especially where the issue of ownership
is ineluctably tied up with the question of registration, the land registration court commits no error in assuming
jurisdiction.
[15]

It is equally important to consider that a land registration courts decision ordering the confirmation and the
registration of title, being the result of a proceeding in rem, binds the whole world.
[16]
Thus, the trial courts ruling
consolidating the ownership and the title of the property in the name of herein respondent is valid and binding not
only on petitioners, but also on everyone else who may have any claim thereon.
Second Issue:

Validity of theAuction sale

Petitioners contend that the auction sale was invalid, because several requisites regarding notice and
publication were not satisfied. We are not convinced.
It has been held that matters of notice and publication in tax sales are factual questions that cannot be
determined by this Court.
[17]
Moreover, a recourse under Rule 45 of the Rules of Court, as in this case, generally
precludes the determination of factual issues. This Court will not, as a rule, inquire into the evidence relied upon by
the lower courts to support their findings.
[18]
In this case, the CA had already ruled on the question of compliance
with the requirements of notice and publication in this wise:
In the case at bench, it cannot be denied that the requirements of notice, publication and posting have been
complied with by the public defendant prior to the auction sale wherein the subject condominium unit was sold. x x
x Ergo, there was nothing irregular in the questioned public auction -- thus, the validity of the same must be upheld
in accordance with the aforementioned cases.
[19]

The CA ruling notwithstanding, we shall proceed to discuss these factual issues in order to assure petitioners of
a complete adjudication of their case, and not a mere disposition of procedural technicalities.
TheNon-Publication of Noticeof Real PROPERTY TAX Delinquency

Petitioners assert that the tax sale should be annulled because of noncompliance with the requirement of
publication prescribed in Section 65 of PD 464.
In this regard, we note that unlike land registration proceedings which are in rem, cases involving an auction
sale of land for the collection of delinquent taxes are in personam. Thus, notice by publication, though sufficient in
proceedings in rem, does not as a rule satisfy the requirement of proceedings in personam.
[20]
As such, mere
publication of the notice of delinquency would not suffice, considering that the procedure in tax sales is in
personam. It was, therefore, still incumbent upon the city treasurer to send the notice of tax delinquency directly to
the taxpayer in order to protect the interests of the latter.
In the present case, the notice of delinquency was sent by registered mail to the permanent address of the
registered owner in Manila. In that notice, the city treasurer of Baguio City directed him to settle the charges
immediately and to protect his interest in the property. Under the circumstances, we hold that the notice sent by
registered mail adequately protected the rights of the taxpayer, who was the registered owner of the condominium
unit.
For purposes of the real PROPERTY TAX , the registered owner of the property is deemed the
taxpayer. Hence, only the registered owner is entitled to a notice of tax delinquency and other proceedings relative
to the tax sale. Not being registered owners of the property, petitioners cannot claim to have been deprived of such
notice. In fact, they were not entitled to it.
Lack of Personal Noticeof theSaleor of thePublic Auction of theSubject Property

Petitioners also contend that the registered owner was not given personal notice of the public auction. They
cite Section 73 of PD 464, the pertinent portion of which is reproduced hereunder:
x x x. Copy of the notices shall forthwith be sent either by registered mail or by messenger, or through messenger,
or through the barrio captain, to the delinquent taxpayer, at the address shown in the tax rolls or PROPERTY TAX
records of the municipality or city where the property is located, or at his residence, if known to said treasurer or
barrio captain. x x x. (Underscoring supplied by petitioners in their Memorandum)
According to petitioners, the notice of public auction should have been sent to the address appearing in the tax
roll or property records of the City of Baguio. That address is Unit No. 5, Baden #4105, Europa Condominium
Villas, Baguio City; not the known address or residence of the registered owner at 145 Ermin Garcia Street, Cubao,
Quezon City. They contend that notice may be sent to the residence of the taxpayer, only when the tax roll does not
show any address of the property.
The above-cited provision, however, shows that the determination of the taxpayers address to which the notice
may be sent is the treasurers discretionary prerogative. In this case, the city treasurer deemed it best to send the
notice of public auction to the residence of the taxpayer. The former validly exercised this option, inasmuch as the
address of the latter was known to him. Moreover, it was more practical and favorable to the registered owner that
the notice of delinquency be sent to his permanent residence in Manila, because he was using the subject
condominium unit merely as a VACATION HOUSE and not as a residence.
This Court in Pecson v. Court of Appeals
[21]
made a clear and categorical ruling on the matter, when it declared
as follows:
Under the said provisions of law, notices of the sale of the public auction may be sent to the delinquent taxpayer,
either (I) at the address as shown in the tax rolls or property tax record cards of the municipality or city where the
property is located or (ii) at his residence, if known to such treasurer or barrio captain. (emphasis supplied)
To reiterate, for purposes of the collection of real property taxes, the registered owner of the property is
considered the taxpayer. Although petitioners have been in possession of the subject premises by virtue of
an unregistered deed of sale, such transaction has no binding effect with respect to third persons who have no
knowledge of it.
The importance of registration and its binding effect is stated in Section 51 of the Property Registration Decree
or PD 1529, which reads:
Sec. 51. Conveyance and other dealings by registered owner. - An owner of registered land may
convey, MORTGAGE , LEASE , charge or otherwise deal with the same in accordance with existing laws. He
may use such forms, deeds, MORTGAGES , leases or other voluntary instrument as are sufficient in law. But no
deed, mortgage, lease or other voluntary instrument, except a will purporting to convey or effect registered
land, shall take effect as a conveyance or bind the land, but shall operate only as a contract between the parties
and as evidence of authority to the Registry of Deeds to make registration.
The act of registration shall be the operative act to convey or affect the land insofar as third persons are
concerned, and in all cases under this Decree, the registration shall be made in the Office of the Register of Deeds
for the province or the city where the land lies.
Thus, insofar as third persons are concerned, it is the registration of the deed of sale that can validly transfer or
convey a persons interest in a property.
[22]
In the absence of registration, the registered owner whose name appears
on the certificate of title is deemed the taxpayer to whom the notice of auction sale should be sent. Petitioners,
therefore, cannot claim to be taxpayers. For this reason, the annulment of the auction sale may not be invoked
successfully.
TheAnnulment of theAuction Saleon EquitableConsiderations

As correctly pointed out by respondents, equitable considerations will not find application, if the statutes or
rules of procedure explicitly provide for the requisites and standards by which the matters at bench can be resolved.
While it may be assumed that both petitioners and Respondent Tayag are innocent purchasers of the subject
property, it is a well-settled principle that between two purchasers, the one who has registered the sale in ones favor
has a preferred right over the other whose title has not been registered, even if the latter is in actual possession of the
subject property.
[23]

Likewise, we cannot help but point out the fact that petitioners brought this misfortune upon themselves. They
neither registered the Deed of Sale after its execution nor moved for the consolidation of ownership of title to the
property in their name. Worse, they failed to pay the real PROPERTY TAXES due. Although they had been in
possession of the property since 1981, they did not take the necessary steps to protect and legitimize their interest.
Indeed, petitioners suit is now barred by laches.
[24]
The law helps the vigilant, but not those who sleep on their
rights, for time is a means of obliterating actions. Verily, time runs against the slothful and the contemners of their
own rights.
[25]

WHEREFORE, the Petition is hereby DENIED and the assailed Decision and Resolution AFFIRMED. Costs
against petitioners.
SO ORDERED.
Melo, (Chairman), Vitug, Gonzaga-Reyes, and Sandoval-Gutierrez, JJ., concur.


11. HEIRS OF MARIO MALABANAN, G.R. No. 179987
Petitioner,

Present:

PUNO, C.J.,
QUISUMBING,
YNARES-SANTIAGO,
CARPIO,
- versus - AUSTRIA-MARTINEZ,
CORONA,
CARPIO MORALES,
TINGA,
CHICO-NAZARIO,
VELASCO, JR.,
NACHURA,
LEONARDO DE CASTRO,
BRION,
REPUBLIC OF THE PHILIPPINES, PERALTA, and
Respondent. BERSAMIN, JJ.

Promulgated:

April 29, 2009

x--------------------------------------------------------------------------- x


D E C I S I O N

TINGA, J.:

One main reason why the informal sector has not become formal is that
from Indonesia to Brazil, 90 percent of the informal lands are not titled and registered. This is a
generalized phenomenon in the so-called Third World. And it has many consequences.


xxx

The question is: How is it that so many governments, from Suharto's in Indonesia to
Fujimori's in Peru, have wanted to title these people and have not been able to do so effectively?
One reason is that none of the state systems in Asia or Latin America can gather proof of informal
titles. In Peru, the informals have means of proving property ownership to each other which are not
the same means developed by the Spanish legal system. The informals have their own papers, their
own forms of agreements, and their own systems of registration, all of which are very clearly stated
in the maps which they use for their own informal business transactions.
If you take a walk through the countryside, from Indonesia to Peru, and you walk by field
after field--in each field a different dog is going to bark at you. Even dogs know what private
property is all about. The only one who does not know it is the government. The issue is that there
exists a "common law" and an "informal law" which the Latin American formal legal system does
not know how to recognize.
- Hernando De Soto
[1]


This decision inevitably affects all untitled lands currently in possession of persons and entities other than
the Philippine government. The petition, while unremarkable as to the facts, was accepted by the Court en banc in
order to provide definitive clarity to the applicability and scope of original registration proceedings under Sections
14(1) and 14(2) of the Property Registration Decree. In doing so, the Court confronts not only the relevant
provisions of the Public Land Act and the Civil Code, but also the reality on the ground. The countrywide
phenomenon of untitled lands, as well as the problem of informal settlement it has spawned, has unfortunately
been treated with benign neglect. Yet our current laws are hemmed in by their own circumscriptions in addressing
the phenomenon. Still, the duty on our part is primarily to decide cases before us in accord with the Constitution
and the legal principles that have developed our public land law, though our social obligations dissuade us from
casting a blind eye on the endemic problems.

I.

On 20 February 1998, Mario Malabanan filed an application for land registration covering a parcel of land
identified as Lot 9864-A, Cad-452-D, Silang Cadastre,
[2]
situated in Barangay Tibig, Silang Cavite, and consisting of
71,324 square meters. Malabanan claimed that he had purchased the property from Eduardo Velazco,
[3]
and that
he and his predecessors-in-interest had been in open, notorious, and continuous adverse and peaceful possession
of the land for more than thirty (30) years.

The application was raffled to the Regional Trial Court of (RTC) Cavite-Tagaytay City, Branch 18. The Office of
the Solicitor General (OSG) duly designated the Assistant Provincial Prosecutor of Cavite, Jose Velazco, Jr., to
appear on behalf of the State.
[4]
Apart from presenting documentary evidence, Malabanan himself and his witness,
Aristedes Velazco, testified at the hearing. Velazco testified that the property was originally belonged to a twenty-
two hectare property owned by his great-grandfather, Lino Velazco. Lino had four sons Benedicto, Gregorio,
Eduardo and Estebanthe fourth being Aristedess grandfather. Upon Linos death, his four sons inherited the
property and divided it among themselves. But by 1966, Estebans wife, Magdalena, had become the administrator
of all the properties inherited by the Velazco sons from their father, Lino. After the death of Esteban
and Magdalena, their son Virgilio succeeded them in administering the properties, including Lot9864-A, which
originally belonged to his uncle, Eduardo Velazco. It was this property that was sold by Eduardo Velazco to
Malabanan.
[5]


Assistant Provincial Prosecutor Jose Velazco, Jr. did not cross-examine Aristedes Velazco. He further
manifested that he also *knew+ the property and I affirm the truth of the testimony given by Mr. Velazco.
[6]
The
Republic of the Philippines likewise did not present any evidence to controvert the application.

Among the evidence presented by Malabanan during trial was a Certification dated 11 June 2001, issued
by the Community Environment & Natural Resources Office, Department of Environment and Natural Resources
(CENRO-DENR), which stated that the subject property was verified to be within the Alienable or Disposable land
per Land Classification Map No. 3013 established under Project No. 20-A and approved as such under FAO 4-
1656 on March 15, 1982.
[7]


On 3 December 2002, the RTC rendered judgment in favor of Malabanan, the dispositive portion of which
reads:

WHEREFORE, this Court hereby approves this application for registration and thus places
under the operation of Act 141, Act 496 and/or P.D. 1529, otherwise known as Property
Registration Law, the lands described in Plan Csd-04-0173123-D, Lot 9864-A and containing an
area of Seventy One Thousand Three Hundred Twenty Four (71,324) Square Meters, as supported
by its technical description now forming part of the record of this case, in addition to other proofs
adduced in the name of MARIO MALABANAN, who is of legal age, Filipino, widower, and with
residence at Munting Ilog, Silang, Cavite.

Once this Decision becomes final and executory, the corresponding decree of registration
shall forthwith issue.

SO ORDERED.


The Republic interposed an appeal to the Court of Appeals, arguing that Malabanan had failed to prove
that the property belonged to the alienable and disposable land of the public domain, and that the RTC had erred
in finding that he had been in possession of the property in the manner and for the length of time required by law
for confirmation of imperfect title.

On 23 February 2007, the Court of Appeals rendered a Decision
[8]
reversing the RTC and dismissing the
application of Malabanan. The appellate court held that under Section 14(1) of the Property Registration Decree
any period of possession prior to the classification of the lots as alienable and disposable was inconsequential and
should be excluded from the computation of the period of possession. Thus, the appellate court noted that since
the CENRO-DENR certification had verified that the property was declared
alienable and disposable only on 15 March 1982, the Velazcos possession prior to that date could not be
factored in the computation of the period of possession. This interpretation of the Court of Appeals of Section
14(1) of the Property Registration Decree was based on the Courts ruling in Republic v. Herbieto.
[9]


Malabanan died while the case was pending with the Court of Appeals;
[10]
hence, it was his heirs who
appealed the decision of the appellate court. Petitioners, before this Court, rely on our ruling in Republic v.
Naguit,
[11]
which was handed down just four months prior to Herbieto. Petitioners suggest that the discussion
inHerbieto cited by the Court of Appeals is actually obiter dictum since the Metropolitan Trial Court therein which
had directed the registration of the property had no jurisdiction in the first place since the requisite notice of
hearing was published only after the hearing had already begun. Naguit, petitioners argue, remains the controlling
doctrine, especially when the property in question is agricultural land. Therefore, with respect to agricultural lands,
any possession prior to the declaration of the alienable property as disposable may be counted in reckoning the
period of possession to perfect title under the Public Land Act and the Property Registration Decree.



The petition was referred to the Court en banc,
[12]
and on 11 November 2008, the case was heard on oral
arguments. The Court formulated the principal issues for the oral arguments, to wit:

1. In order that an alienable and disposable land of the public domain may be
registered under Section 14(1) of Presidential Decree No. 1529, otherwise known as the Property
Registration Decree, should the land be classified as alienable and disposable as of June 12, 1945
or is it sufficient that such classification occur at any time prior to the filing of the applicant for
registration provided that it is established that the applicant has been in open, continuous,
exclusive and notorious possession of the land under a bona fide claim of ownership since June
12, 1945 or earlier?

2. For purposes of Section 14(2) of the Property Registration Decree may a parcel of
land classified as alienable and disposable be deemed private land and therefore susceptible to
acquisition by prescription in accordance with the Civil Code?

3. May a parcel of land established as agricultural in character either because of its
use or because its slope is below that of forest lands be registrable under Section 14(2) of the
Property Registration Decree in relation to the provisions of the Civil Code on acquisitive
prescription?

4. Are petitioners entitled to the registration of the subject land in their names
under Section 14(1) or Section 14(2) of the Property Registration Decree or both?
[13]


Based on these issues, the parties formulated their respective positions.

With respect to Section 14(1), petitioners reiterate that the analysis of the Court in Naguit is the correct
interpretation of the provision. The seemingly contradictory pronouncement in Herbieto, it is submitted, should be
considered obiter dictum, since the land registration proceedings therein was void ab initio due to lack of
publication of the notice of initial hearing. Petitioners further point out that in Republic v. Bibonia,
[14]
promulgated
in June of 2007, the Court applied Naguitand adopted the same observation that the preferred interpretation by
the OSG of Section 14(1) was patently absurd. For its part, the OSG remains insistent that for Section 14(1) to
apply, the land should have been classified as alienable and disposable as of 12 June 1945. Apart from Herbieto,
the OSG also cites the subsequent rulings in Buenaventura v. Republic,
[15]
Fieldman Agricultural Trading v.
Republic
[16]
and Republic v. Imperial Credit Corporation,
[17]
as well as the earlier case of Director of Lands v. Court of
Appeals.
[18]


With respect to Section 14(2), petitioners submit that open, continuous, exclusive and notorious
possession of an alienable land of the public domain for more than 30 years ipso jure converts the land into private
property, thus placing it under the coverage of Section 14(2). According to them, it would not matter whether the
land sought to be registered was previously classified as agricultural land of the public domain so long as, at the
time of the application, the property had already been converted into private property through prescription. To
bolster their argument, petitioners cite extensively from our 2008 ruling in Republic v. T.A.N. Properties.
[19]



The arguments submitted by the OSG with respect to Section 14(2) are more extensive. The OSG notes
that under Article 1113 of the Civil Code, the acquisitive prescription of properties of the State refers to
patrimonial property, while Section 14(2) speaks of private lands. It observes that the Court has yet to decide a
case that presented Section 14(2) as a ground for application for registration, and that the 30-year possession
period refers to the period of possession under Section 48(b) of the Public Land Act, and not the concept of
prescription under the Civil Code. The OSG further submits that, assuming that the 30-year prescriptive period can
run against public lands, said period should be reckoned from the time the public land was declared alienable and
disposable.

Both sides likewise offer special arguments with respect to the particular factual circumstances
surrounding the subject property and the ownership thereof.

II.

First, we discuss Section 14(1) of the Property Registration Decree. For a full understanding of the provision,
reference has to be made to the Public Land Act.

A.

Commonwealth Act No. 141, also known as the Public Land Act, has, since its enactment, governed the
classification and disposition of lands of the public domain. The President is authorized, from time to time, to
classify the lands of the public domain into alienable and disposable, timber, or mineral lands.
[20]
Alienable and
disposable lands of the public domain are further classified according to their uses into (a) agricultural; (b)
residential, commercial, industrial, or for similar productive purposes; (c) educational, charitable, or other similar
purposes; or (d) reservations for town sites and for public and quasi-public uses.
[21]


May a private person validly seek the registration in his/her name of alienable and disposable lands of the
public domain? Section 11 of the Public Land Act acknowledges that public lands suitable for agricultural purposes
may be disposed of by confirmation of imperfect or incomplete titles through judicial legalization.
[22]
Section
48(b) of the Public Land Act, as amended by P.D. No. 1073, supplies the details and unmistakably grants that right,
subject to the requisites stated therein:

Sec. 48. The following described citizens of the Philippines, occupying lands of the
public domain or claiming to own any such land or an interest therein, but whose titles have not
been perfected or completed, may apply to the Court of First Instance of the province where
the land is located for confirmation of their claims and the issuance of a certificate of title
therefor, under the Land Registration Act, to wit:

xxx

(b) Those who by themselves or through their predecessors in interest have been in
open, continuous, exclusive, and notorious possession and occupation of alienable and
disposable lands of the public domain, under a bona fide claim of acquisition of ownership,
since June 12, 1945, or earlier, immediately preceding the filing of the application for
confirmation of title except when prevented by war or force majeure. These shall be
conclusively presumed to have performed all the conditions essential to a Government grant
and shall be entitled to a certificate of title under the provisions of this chapter.


Section 48(b) of Com. Act No. 141 received its present wording in 1977 when the law was amended by P.D.
No. 1073. Two significant amendments were introduced by P.D. No. 1073. First, the term agricultural lands was
changed to alienable and disposable lands of the public domain. The OSG submits that this amendment
restricted the scope of the lands that may be registered.
[23]
This is not actually the case. Under Section 9 of the
Public Land Act, agricultural lands are a mere subset of lands of the public domain alienable or open to
disposition. Evidently, alienable and disposable lands of the public domain are a larger class than only
agricultural lands.

Second, the length of the requisite possession was changed from possession for thirty (30) years
immediately preceding the filing of the application to possession since June 12, 1945 or earlier. The Court
in Naguit explained:

When the Public Land Act was first promulgated in 1936, the period of possession
deemed necessary to vest the right to register their title to agricultural lands of the public domain
commenced from July 26, 1894. However, this period was amended by R.A. No. 1942, which
provided that the bona fide claim of ownership must have been for at least thirty (30) years. Then
in 1977, Section 48(b) of the Public Land Act was again amended, this time by P.D. No. 1073,
which pegged the reckoning date at June 12, 1945. xxx


It bears further observation that Section 48(b) of Com. Act No, 141 is virtually the same as Section 14(1) of the
Property Registration Decree. Said Decree codified the various laws relative to the registration of property,
including lands of the public domain. It is Section 14(1) that operationalizes the registration of such lands of the
public domain. The provision reads:


SECTION 14. Who may apply. The following persons may file in the proper Court of
First Instance an application for registration of title to land, whether personally or through their
duly authorized representatives:

(1) those who by themselves or through their predecessors-in-interest have
been in open, continuous, exclusive and notorious possession and
occupation of alienable and disposable lands of the public domain under
a bona fide claim of ownership since June 12, 1945, or earlier.


Notwithstanding the passage of the Property Registration Decree and the inclusion of Section 14(1)
therein, the Public Land Act has remained in effect. Both laws commonly refer to persons or their predecessors-in-
interest who have been in open, continuous, exclusive and notorious possession and occupation of alienable and
disposable lands of the public domain under a bona fide claim of ownership since June 12, 1945, or earlier. That
circumstance may have led to the impression that one or the other is a redundancy, or that Section 48(b) of the
Public Land Act has somehow been repealed or mooted. That is not the case.

The opening clauses of Section 48 of the Public Land Act and Section 14 of the Property Registration
Decree warrant comparison:

Sec. 48 [of the Public Land Act]. The following described citizens of the Philippines,
occupying lands of the public domain or claiming to own any such land or an interest therein,
but whose titles have not been perfected or completed, may apply to the Court of First Instance
of the province where the land is located for confirmation of their claims and the issuance of a
certificate of title therefor, under the Land Registration Act, to wit:

xxx

Sec. 14 [of the Property Registration Decree]. Who may apply. The following persons
may file in the proper Court of First Instance an application for registration of title to land,
whether personally or through their duly authorized representatives:

xxx

It is clear that Section 48 of the Public Land Act is more descriptive of the nature of the right enjoyed by
the possessor than Section 14 of the Property Registration Decree, which seems to presume the pre-existence of
the right, rather than establishing the right itself for the first time. It is proper to assert that it is the Public Land Act,
as amended by P.D. No. 1073 effective 25 January 1977, that has primarily established the right of a Filipino citizen
who has been in open, continuous, exclusive, and notorious possession and occupation of alienable and
disposable lands of the public domain, under a bona fide claim of acquisition of ownership, since June 12, 1945 to
perfect or complete his title by applying with the proper court for the confirmation of his ownership claim and the
issuance of the corresponding certificate of title.


Section 48 can be viewed in conjunction with the afore-quoted Section 11 of the Public Land Act, which
provides that public lands suitable for agricultural purposes may be disposed of by confirmation of imperfect or
incomplete titles, and given the notion that both provisions declare that it is indeed the Public Land Act that
primarily establishes the substantive ownership of the possessor who has been in possession of the property since
12 June 1945. In turn, Section 14(a) of the Property Registration Decree recognizes the substantive right granted
under Section 48(b) of the Public Land Act, as well provides the corresponding original registration procedure for
the judicial confirmation of an imperfect or incomplete title.

There is another limitation to the right granted under Section 48(b). Section 47 of the Public Land Act limits
the period within which one may exercise the right to seek registration under Section 48. The provision has been
amended several times, most recently by Rep. Act No. 9176 in 2002. It currently reads thus:

Section 47. The persons specified in the next following section are hereby granted time,
not to extend beyond December 31, 2020 within which to avail of the benefits of this
Chapter: Provided, That this period shall apply only where the area applied for does not exceed
twelve (12) hectares: Provided, further, That the several periods of time designated by the
President in accordance with Section Forty-Five of this Act shall apply also to the lands comprised
in the provisions of this Chapter, but this Section shall not be construed as prohibiting any said
persons from acting under this Chapter at any time prior to the period fixed by the President.
[24]


Accordingly under the current state of the law, the substantive right granted under Section 48(b) may be
availed of only until 31 December 2020.

B.

Despite the clear text of Section 48(b) of the Public Land Act, as amended and Section 14(a) of the
Property Registration Decree, the OSG has adopted the position that for one to acquire the right to seek
registration of an alienable and disposable land of the public domain, it is not enough that the applicant and his/her
predecessors-in-interest be in possession under a bona fide claim of ownership since 12 June 1945; the alienable
and disposable character of the property must have been declared also as of 12 June 1945. Following the OSGs
approach, all lands certified as alienable and disposable after 12 June 1945 cannot be registered either under
Section 14(1) of the Property Registration Decree or Section 48(b) of the Public Land Act as amended. The absurdity
of such an implication was discussed inNaguit.

Petitioner suggests an interpretation that the alienable and disposable character of the
land should have already been established since June 12, 1945 or earlier. This is not borne out by
the plain meaning of Section 14(1). Since June 12, 1945, as used in the provision, qualifies its
antecedent phrase under a bonafide claim of ownership. Generally speaking, qualifying words
restrict or modify only the words or phrases to which they are immediately associated, and not
those distantly or remotely located.
[25]
Ad proximum antecedents fiat relation nisi impediatur
sentencia.

Besides, we are mindful of the absurdity that would result if we adopt petitioners
position. Absent a legislative amendment, the rule would be, adopting the OSGs view, that all
lands of the public domain which were not declared alienable or disposable before June 12,
1945 would not be susceptible to original registration, no matter the length of unchallenged
possession by the occupant. Such interpretation renders paragraph (1) of Section 14 virtually
inoperative and even precludes the government from giving it effect even as it decides to
reclassify public agricultural lands as alienable and disposable. The unreasonableness of the
situation would even be aggravated considering that before June 12, 1945, the Philippines was
not yet even considered an independent state.

Accordingly, the Court in Naguit explained:

[T]he more reasonable interpretation of Section 14(1) is that it merely requires the
property sought to be registered as already alienable and disposable at the time the application
for registration of title is filed. If the State, at the time the application is made, has not yet
deemed it proper to release the property for alienation or disposition, the presumption is that the
government is still reserving the right to utilize the property; hence, the need to preserve its
ownership in the State irrespective of the length of adverse possession even if in good faith.
However, if the property has already been classified as alienable and disposable, as it is in this
case, then there is already an intention on the part of the State to abdicate its exclusive
prerogative over the property.


The Court declares that the correct interpretation of Section 14(1) is that which was adopted in Naguit.
The contrary pronouncement in Herbieto, as pointed out in Naguit, absurdly limits the application of the provision
to the point of virtual inutility since it would only cover lands actually declared alienable and disposable prior to 12
June 1945, even if the current possessor is able to establish open, continuous, exclusive and notorious possession
under a bona fide claim of ownership long before that date.

Moreover, the Naguit interpretation allows more possessors under a bona fide claim of ownership to avail
of judicial confirmation of their imperfect titles than what would be feasible under Herbieto. This balancing fact is
significant, especially considering our forthcoming discussion on the scope and reach of Section 14(2) of the
Property Registration Decree.

Petitioners make the salient observation that the contradictory passages from Herbieto are obiter
dicta since the land registration proceedings therein is void ab initio in the first place due to lack of the requisite
publication of the notice of initial hearing. There is no need to explicitly overturn Herbieto, as it suffices that the
Courts acknowledgment that the particular line of argument used therein concerning Section 14(1) is
indeed obiter.
It may be noted that in the subsequent case of Buenaventura,
[26]
the Court, citing Herbieto, again stated that
[a]ny period of possession prior to the date when the [s]ubject [property was] classified as alienable and disposable
is inconsequential and should be excluded from the computation of the period of possession That statement, in
the context of Section 14(1), is certainly erroneous. Nonetheless, the passage as cited in Buenaventura should again
be considered as obiter. The application therein was ultimately granted, citing Section 14(2). The evidence
submitted by petitioners therein did not establish any mode of possession on their part prior to 1948, thereby
precluding the application of Section 14(1). It is not even apparent from the decision whether petitioners therein had
claimed entitlement to original registration following Section 14(1), their position being that they had been in
exclusive possession under a bona fide claim of ownership for over fifty (50) years, but not before 12 June 1945.
Thus, neither Herbieto nor its principal discipular ruling Buenaventura has any precedental value with
respect to Section 14(1). On the other hand, the ratio ofNaguit is embedded in Section 14(1), since it precisely
involved situation wherein the applicant had been in exclusive possession under a bona fide claim of ownership
prior to 12 June 1945. The Courts interpretation of Section 14(1) therein was decisive to the resolution of the case.
Any doubt as to which between Naguit orHerbieto provides the final word of the Court on Section 14(1) is now
settled in favor of Naguit.

We noted in Naguit that it should be distinguished from Bracewell v. Court of Appeals
[27]
since in the latter,
the application for registration had been filedbefore the land was declared alienable or disposable. The dissent
though pronounces Bracewell as the better rule between the two. Yet two years after Bracewell, itsponente, the
esteemed Justice Consuelo Ynares-Santiago, penned the ruling in Republic v. Ceniza,
[28]
which involved a claim of
possession that extended back to 1927 over a public domain land that was declared alienable and disposable only
in 1980. Ceniza cited Bracewell, quoted extensively from it, and following the mindset of the dissent, the attempt
at registration in Ceniza should have failed. Not so.

To prove that the land subject of an application for registration is alienable, an applicant
must establish the existence of a positive act of the government such as a presidential
proclamation or an executive order; an administrative action; investigation reports of Bureau of
Lands investigators; and a legislative act or a statute.

In this case, private respondents presented a certification dated November 25, 1994,
issued by Eduardo M. Inting, the Community Environment and Natural Resources Officer in the
Department of Environment and Natural Resources Office in Cebu City, stating that the lots
involved were "found to be within the alienable and disposable (sic) Block-I, Land Classification
Project No. 32-A, per map 2962 4-I555 dated December 9, 1980." This is sufficient evidence to
show the real character of the land subject of private respondents application. Further, the
certification enjoys a presumption of regularity in the absence of contradictory evidence, which
is true in this case. Worth noting also was the observation of the Court of Appeals stating that:

[n]o opposition was filed by the Bureaus of Lands and Forestry to
contest the application of appellees on the ground that the property still forms
part of the public domain. Nor is there any showing that the lots in question
are forestal land....

Thus, while the Court of Appeals erred in ruling that mere possession of public land for
the period required by law would entitle its occupant to a confirmation of imperfect title, it did
not err in ruling in favor of private respondents as far as the first requirement in Section 48(b) of
the Public Land Act is concerned, for they were able to overcome the burden of proving the
alienability of the land subject of their application.

As correctly found by the Court of Appeals, private respondents were able to prove their
open, continuous, exclusive and notorious possession of the subject land even before the year
1927. As a rule, we are bound by the factual findings of the Court of Appeals. Although there are
exceptions, petitioner did not show that this is one of them.
[29]



Why did the Court in Ceniza, through the same eminent member who authored Bracewell, sanction the
registration under Section 48(b) of public domain lands declared alienable or disposable thirty-five (35) years and
180 days after 12 June 1945? The telling difference is that in Ceniza, the application for registration was filed
nearly six (6) years after the land had been declared alienable or disposable, while in Bracewell, the application
was filed nine (9) years before the land was declared alienable or disposable. That crucial difference was also
stressed in Naguit to contradistinguish it from Bracewell, a difference which the dissent seeks to belittle.

III.

We next ascertain the correct framework of analysis with respect to Section 14(2). The provision reads:



SECTION 14. Who may apply. The following persons may file in the proper Court of
First Instance an application for registration of title to land, whether personally or through their
duly authorized representatives:

xxx

(2) Those who have acquired ownership over private lands by prescription
under the provisions of existing laws.


The Court in Naguit offered the following discussion concerning Section 14(2), which we did even then
recognize, and still do, to be an obiter dictum, but we nonetheless refer to it as material for further discussion,
thus:

Did the enactment of the Property Registration Decree and the amendatory P.D. No.
1073 preclude the application for registration of alienable lands of the public domain, possession
over which commenced only after June 12, 1945? It did not, considering Section 14(2) of the
Property Registration Decree, which governs and authorizes the application of those who have
acquired ownership of private lands by prescription under the provisions of existing laws.

Prescription is one of the modes of acquiring ownership under the Civil Code.[
[30]
] There
is a consistent jurisprudential rule that properties classified as alienable public land may be
converted into private property by reason of open, continuous and exclusive possession of at least
thirty (30) years.[
[31]
] With such conversion, such property may now fall within the contemplation
of private lands under Section 14(2), and thus susceptible to registration by those who have
acquired ownership through prescription. Thus, even if possession of the alienable public land
commenced on a date later than June 12, 1945, and such possession being been open, continuous
and exclusive, then the possessor may have the right to register the land by virtue of Section 14(2)
of the Property Registration Decree.

Naguit did not involve the application of Section 14(2), unlike in this case where petitioners have based their
registration bid primarily on that provision, and where the evidence definitively establishes their claim of
possession only as far back as 1948. It is in this case that we can properly appreciate the nuances of the provision.

A.

The obiter in Naguit cited the Civil Code provisions on prescription as the possible basis for application for
original registration under Section 14(2). Specifically, it is Article 1113 which provides legal foundation for the
application. It reads:

All things which are within the commerce of men are susceptible of prescription, unless
otherwise provided. Property of the State or any of its subdivisions not patrimonial in character
shall not be the object of prescription.


It is clear under the Civil Code that where lands of the public domain are patrimonial in character, they are
susceptible to acquisitive prescription. On the other hand, among the public domain lands that are not susceptible
to acquisitive prescription are timber lands and mineral lands. The Constitution itself proscribes private ownership
of timber or mineral lands.

There are in fact several provisions in the Civil Code concerning the acquisition of real property through
prescription. Ownership of real property may be acquired by ordinary prescription of ten (10) years,
[32]
or through
extraordinary prescription of thirty (30) years.
[33]
Ordinary acquisitive prescription requires possession in good
faith,
[34]
as well as just title.
[35]


When Section 14(2) of the Property Registration Decree explicitly provides that persons who have
acquired ownership over private lands by prescription under the provisions of existing laws, it unmistakably refers
to the Civil Code as a valid basis for the registration of lands. The Civil Code is the only existing law that specifically
allows the acquisition by prescription of private lands, including patrimonial property belonging to the State. Thus,
the critical question that needs affirmation is whether Section 14(2) does encompass original registration
proceedings over patrimonial property of the State, which a private person has acquired through prescription.

The Naguit obiter had adverted to a frequently reiterated jurisprudence holding that properties classified
as alienable public land may be converted into private property by reason of open, continuous and exclusive
possession of at least thirty (30) years.
[36]
Yet if we ascertain the source of the thirty-year period, additional
complexities relating to Section 14(2) and to how exactly it operates would emerge. For there are in fact two
distinct origins of the thirty (30)-year rule.

The first source is Rep. Act No. 1942, enacted in 1957, which amended Section 48(b) of the Public Land Act
by granting the right to seek original registration of alienable public lands through possession in the concept of an
owner for at least thirty years.

The following-described citizens of the Philippines, occupying lands of the public
domain or claiming to own any such lands or an interest therein, but whose titles have not been
perfected or completed, may apply to the Court of First Instance of the province where the land
is located for confirmation of their claims and the issuance of a certificate of title therefor,
under the Land Registration Act, to wit:

x x x x x x x x x

(b) Those who by themselves or through their predecessors in interest have been in
open, continuous, exclusive and notorious possession and occupation of agricultural lands of
the public domain, under a bona fide claim of acquisition of ownership, for at least thirty years
immediately preceding the filing of the application for confirmation of title, except when
prevented by war or force majeure. These shall be conclusively presumed to have performed all
the conditions essential to a Government grant and shall be entitled to a certificate of title
under the provisions of this Chapter. (emphasis supplied)
[37]



This provision was repealed in 1977 with the enactment of P.D. 1073, which made the date 12 June
1945 the reckoning point for the first time. Nonetheless, applications for registration filed prior to 1977 could have
invoked the 30-year rule introduced by Rep. Act No. 1942.

The second source is Section 14(2) of P.D. 1529 itself, at least by implication, as it applies the rules on
prescription under the Civil Code, particularly Article 1113 in relation to Article 1137. Note that there are two kinds
of prescription under the Civil Codeordinary acquisitive prescription and extraordinary acquisitive prescription,
which, under Article 1137, is completed through uninterrupted adverse possession for thirty years, without need
of title or of good faith.

Obviously, the first source of the thirty (30)-year period rule, Rep. Act No. 1942, became unavailable after
1977. At present, the only legal basis for the thirty (30)-year period is the law on prescription under the Civil Code,
as mandated under Section 14(2). However, there is a material difference between how the thirty (30)-year rule
operated under Rep. Act No. 1942 and how it did under the Civil Code.

Section 48(b) of the Public Land Act, as amended by Rep. Act No. 1942, did not refer to or call into
application the Civil Code provisions on prescription. It merely set forth a requisite thirty-year possession period
immediately preceding the application for confirmation of title, without any qualification as to whether the
property should be declared alienable at the beginning of, and continue as such, throughout the entire thirty-(30)
years. There is neither statutory nor jurisprudential basis to assert Rep. Act No. 1942 had mandated such a
requirement,
[38]
similar to our earlier finding with respect to the present language of Section 48(b), which now
sets 12 June 1945 as the point of reference.

Then, with the repeal of Rep. Act No. 1942, the thirty-year possession period as basis for original
registration became Section 14(2) of the Property Registration Decree, which entitled those who have acquired
ownership over private lands by prescription under the provisions of existing laws to apply for original registration.
Again, the thirty-year period is derived from the rule on extraordinary prescription under Article 1137 of the Civil
Code. At the same time, Section 14(2) puts into operation the entire regime of prescription under the Civil Code, a
fact which does not hold true with respect to Section 14(1).

B.

Unlike Section 14(1), Section 14(2) explicitly refers to the principles on prescription under existing laws.
Accordingly, we are impelled to apply the civil law concept of prescription, as set forth in the Civil Code, in our
interpretation of Section 14(2). There is no similar demand on our part in the case of Section 14(1).

The critical qualification under Article 1113 of the Civil Code is thus: *p+roperty of the State or any of its
subdivisions not patrimonial in character shall not be the object of prescription. The identification what consists of
patrimonial property is provided by Articles 420 and 421, which we quote in full:

Art. 420. The following things are property of public dominion:

(1) Those intended for public use, such as roads, canals, rivers, torrents, ports and bridges
constructed by the State, banks, shores, roadsteads, and others of similar character;

(2) Those which belong to the State, without being for public use, and are intended for
some public service or for the development of the national wealth.

Art. 421. All other property of the State, which is not of the character stated in
the preceding article, is patrimonial property

It is clear that property of public dominion, which generally includes property belonging to the State, cannot
be the object of prescription or, indeed, be subject of the commerce of man.
[39]
Lands of the public domain,
whether declared alienable and disposable or not, are property of public dominion and thus insusceptible to
acquisition by prescription.

Let us now explore the effects under the Civil Code of a declaration by the President or any duly authorized
government officer of alienability and disposability of lands of the public domain. Would such lands so declared
alienable and disposable be converted, under the Civil Code, from property of the public dominion into patrimonial
property? After all, by connotative definition, alienable and disposable lands may be the object of the commerce
of man; Article 1113 provides that all things within the commerce of man are susceptible to prescription; and the
same provision further provides that patrimonial property of the State may be acquired by prescription.

Nonetheless, Article 422 of the Civil Code states that *p+roperty of public dominion, when no longer
intended for public use or for public service, shall form part of the patrimonial property of the State. It is this
provision that controls how public dominion property may be converted into patrimonial property susceptible to
acquisition by prescription. After all, Article 420 (2) makes clear that those property which belong to the State,
without being for public use, and are intended for some public service or for the development of the national
wealth are public dominion property. For as long as the property belongs to the State, although already classified
as alienable or disposable, it remains property of the public dominion if when it is intended for some public
service or for the development of the national wealth.

Accordingly, there must be an express declaration by the State that the public dominion property is no
longer intended for public service or the development of the national wealth or that the property has been
converted into patrimonial. Without such express declaration, the property, even if classified as alienable or
disposable, remains property of the public dominion, pursuant to Article 420(2), and thus incapable of
acquisition by prescription. It is only when such alienable and disposable lands are expressly declared by the
State to be no longer intended for public service or for the development of the national wealth that the period
of acquisitive prescription can begin to run. Such declaration shall be in the form of a law duly enacted by
Congress or a Presidential Proclamation in cases where the President is duly authorized by law.

It is comprehensible with ease that this reading of Section 14(2) of the Property Registration Decree limits
its scope and reach and thus affects the registrability even of lands already declared alienable and disposable to
the detriment of the bona fide possessors or occupants claiming title to the lands. Yet this interpretation is in
accord with the Regalian doctrine and its concomitant assumption that all lands owned by the State, although
declared alienable or disposable, remain as such and ought to be used only by the Government.

Recourse does not lie with this Court in the matter. The duty of the Court is to apply the Constitution and
the laws in accordance with their language and intent. The remedy is to change the law, which is the province of
the legislative branch. Congress can very well be entreated to amend Section 14(2) of the Property Registration
Decree and pertinent provisions of the Civil Code to liberalize the requirements for judicial confirmation of
imperfect or incomplete titles.

The operation of the foregoing interpretation can be illustrated by an actual example. Republic Act No.
7227, entitled An Act Accelerating The Conversion Of Military Reservations Into Other Productive Uses, etc., is
more commonly known as the BCDA law. Section 2 of the law authorizes the sale of certain military reservations
and portions of military camps in Metro Manila, including Fort Bonifacio and Villamor Air Base. For purposes of
effecting the sale of the military camps, the law mandates the President to transfer such military lands to the Bases
Conversion Development Authority (BCDA)
[40]
which in turn is authorized to own, hold and/or administer
them.
[41]
The President is authorized to sell portions of the military camps, in whole or in part.
[42]
Accordingly, the
BCDA law itself declares that the military lands subject thereof are alienable and disposable pursuant to the
provisions of existing laws and regulations governing sales of government properties.
[43]


From the moment the BCDA law was enacted the subject military lands have become alienable and
disposable. However, said lands did not become patrimonial, as the BCDA law itself expressly makes the
reservation that these lands are to be sold in order to raise funds for the conversion of the former American bases
at Clarkand Subic.
[44]
Such purpose can be tied to either public service or the development of national wealth
under Article 420(2). Thus, at that time, the lands remained property of the public dominion under Article 420(2),
notwithstanding their status as alienable and disposable. It is upon their sale as authorized under the BCDA law to
a private person or entity that such lands become private property and cease to be property of the public
dominion.


C.

Should public domain lands become patrimonial because they are declared as such in a duly enacted law
or duly promulgated proclamation that they are no longer intended for public service or for the development of
the national wealth, would the period of possession prior to the conversion of such public dominion into
patrimonial be reckoned in counting the prescriptive period in favor of the possessors? We rule in the negative.

The limitation imposed by Article 1113 dissuades us from ruling that the period of possession before the
public domain land becomes patrimonial may be counted for the purpose of completing the prescriptive period.
Possession of public dominion property before it becomes patrimonial cannot be the object of prescription
according to the Civil Code. As the application for registration under Section 14(2) falls wholly within the
framework of prescription under the Civil Code, there is no way that possession during the time that the land was
still classified as public dominion property can be counted to meet the requisites of acquisitive prescription and
justify registration.

Are we being inconsistent in applying divergent rules for Section 14(1) and Section 14(2)? There is no
inconsistency. Section 14(1) mandates registration on the basis of possession, while Section 14(2) entitles
registration on the basis of prescription. Registration under Section 14(1) is extended under the aegis of
the Property Registration Decree and the Public Land Act while registration under Section 14(2) is made
available both by the Property Registration Decree and the Civil Code.

In the same manner, we can distinguish between the thirty-year period under Section 48(b) of the Public
Land Act, as amended by Rep. Act No. 1472, and the thirty-year period available through Section 14(2) of the
Property Registration Decree in relation to Article 1137 of the Civil Code. The period under the former speaks of
a thirty-year period of possession, while the period under the latter concerns a thirty-year period of
extraordinary prescription. Registration under Section 48(b) of the Public Land Act as amended by Rep. Act No.
1472 is based on thirty years of possession alone without regard to the Civil Code, while the registration under
Section 14(2) of the Property Registration Decree is founded on extraordinary prescription under the Civil Code.

It may be asked why the principles of prescription under the Civil Code should not apply as well to Section
14(1). Notwithstanding the vaunted status of the Civil Code, it ultimately is just one of numerous statutes, neither
superior nor inferior to other statutes such as the Property Registration Decree. The legislative branch is not bound
to adhere to the framework set forth by the Civil Code when it enacts subsequent legislation. Section 14(2)
manifests a clear intent to interrelate the registration allowed under that provision with the Civil Code, but no such
intent exists with respect to Section 14(1).

IV.

One of the keys to understanding the framework we set forth today is seeing how our land registration
procedures correlate with our law on prescription, which, under the Civil Code, is one of the modes for acquiring
ownership over property.

The Civil Code makes it clear that patrimonial property of the State may be acquired by private persons
through prescription. This is brought about by Article 1113, which states that *a+ll things which are within the
commerce of man are susceptible to prescription, and that *p+roperty of the State or any of its subdivisions not
patrimonial in character shall not be the object of prescription.

There are two modes of prescription through which immovables may be acquired under the Civil Code. The
first is ordinary acquisitive prescription, which, under Article 1117, requires possession in good faith and with just
title; and, under Article 1134, is completed through possession of ten (10) years. There is nothing in the Civil Code
that bars a person from acquiring patrimonial property of the State through ordinary acquisitive prescription, nor
is there any apparent reason to impose such a rule. At the same time, there are indispensable requisitesgood
faith and just title. The ascertainment of good faith involves the application of Articles 526, 527, and 528, as well as
Article 1127 of the Civil Code,
[45]
provisions that more or less speak for themselves.

On the other hand, the concept of just title requires some clarification. Under Article 1129, there is just
title for the purposes of prescription when the adverse claimant came into possession of the property through
one of the modes recognized by law for the acquisition of ownership or other real rights, but the grantor was not
the owner or could not transmit any right. Dr. Tolentino explains:

Just title is an act which has for its purpose the transmission of ownership, and which
would have actually transferred ownership if the grantor had been the owner. This vice or defect
is the one cured by prescription. Examples: sale with delivery, exchange, donation, succession,
and dacion in payment.
[46]


The OSG submits that the requirement of just title necessarily precludes the applicability of ordinary
acquisitive prescription to patrimonial property. The major premise for the argument is that the State, as the
owner and grantor, could not transmit ownership to the possessor before the completion of the required period of
possession.
[47]
It is evident that the OSG erred when it assumed that the grantor referred to in Article 1129 is the
State. The grantor is the one from whom the person invoking ordinary acquisitive prescription derived the title,
whether by sale, exchange, donation, succession or any other mode of the acquisition of ownership or other real
rights.

Earlier, we made it clear that, whether under ordinary prescription or extraordinary prescription, the
period of possession preceding the classification of public dominion lands as patrimonial cannot be counted for the
purpose of computing prescription. But after the property has been become patrimonial, the period of prescription
begins to run in favor of the possessor. Once the requisite period has been completed, two legal events ensue: (1)
the patrimonial property is ipso jureconverted into private land; and (2) the person in possession for the periods
prescribed under the Civil Code acquires ownership of the property by operation of the Civil Code.

It is evident that once the possessor automatically becomes the owner of the converted patrimonial
property, the ideal next step is the registration of the property under the Torrens system. It should be
remembered that registration of property is not a mode of acquisition of ownership, but merely a mode of
confirmation of ownership.
[48]


Looking back at the registration regime prior to the adoption of the Property Registration Decree in 1977,
it is apparent that the registration system then did not fully accommodate the acquisition of ownership of
patrimonial property under the Civil Code. What the system accommodated was the confirmation of imperfect
title brought about by the completion of a period of possession ordained under the Public Land Act (either 30
years following Rep. Act No. 1942, or since 12 June 1945following P.D. No. 1073).

The Land Registration Act
[49]
was noticeably silent on the requisites for alienable public lands acquired
through ordinary prescription under the Civil Code, though it arguably did not preclude such registration.
[50]
Still,
the gap was lamentable, considering that the Civil Code, by itself, establishes ownership over the patrimonial
property of persons who have completed the prescriptive periods ordained therein. The gap was finally closed with
the adoption of the Property Registration Decree in 1977, with Section 14(2) thereof expressly authorizing original
registration in favor of persons who have acquired ownership over private lands by prescription under the
provisions of existing laws, that is, the Civil Code as of now.

V.

We synthesize the doctrines laid down in this case, as follows:

(1) In connection with Section 14(1) of the Property Registration Decree, Section 48(b) of the Public Land Act
recognizes and confirms that those who by themselves or through their predecessors in interest have been in
open, continuous, exclusive, and notorious possession and occupation of alienable and disposable lands of the
public domain, under a bona fide claim of acquisition of ownership, since June 12, 1945 have acquired ownership
of, and registrable title to, such lands based on the length and quality of their possession.

(a) Since Section 48(b) merely requires possession since 12 June 1945 and does not require
that the lands should have been alienable and disposable during the entire period of possession, the
possessor is entitled to secure judicial confirmation of his title thereto as soon as it is declared alienable
and disposable, subject to the timeframe imposed by Section 47 of the Public Land Act.
[51]


(b) The right to register granted under Section 48(b) of the Public Land Act is further confirmed
by Section 14(1) of the Property Registration Decree.

(2) In complying with Section 14(2) of the Property Registration Decree, consider that under the Civil Code,
prescription is recognized as a mode of acquiring ownership of patrimonial property. However, public domain
lands become only patrimonial property not only with a declaration that these are alienable or disposable. There
must also be an express government manifestation that the property is already patrimonial or no longer retained
for public service or the development of national wealth, under Article 422 of the Civil Code. And only when the
property has become patrimonial can the prescriptive period for the acquisition of property of the public dominion
begin to run.

(a) Patrimonial property is private property of the government. The person acquires
ownership of patrimonial property by prescription under the Civil Code is entitled to secure registration
thereof under Section 14(2) of the Property Registration Decree.

(b) There are two kinds of prescription by which patrimonial property may be acquired, one
ordinary and other extraordinary. Under ordinary acquisitive prescription, a person acquires ownership of
a patrimonial property through possession for at least ten (10) years, in good faith and with just title.
Under extraordinary acquisitive prescription, a persons uninterrupted adverse possession of patrimonial
property for at least thirty (30) years, regardless of good faith or just title, ripens into ownership.

B.

We now apply the above-stated doctrines to the case at bar.

It is clear that the evidence of petitioners is insufficient to establish that Malabanan has acquired ownership
over the subject property under Section 48(b) of the Public Land Act. There is no substantive evidence to establish
that Malabanan or petitioners as his predecessors-in-interest have been in possession of the property since 12
June 1945 or earlier. The earliest that petitioners can date back their possession, according to their own
evidencethe Tax Declarations they presented in particularis to the year 1948. Thus, they cannot avail
themselves of registration under Section 14(1) of the Property Registration Decree.


Neither can petitioners properly invoke Section 14(2) as basis for registration. While the subject property was
declared as alienable or disposable in 1982, there is no competent evidence that is no longer intended for public
use service or for the development of the national evidence, conformably with Article 422 of the Civil Code. The
classification of the subject property as alienable and disposable land of the public domain does not change its
status as property of the public dominion under Article 420(2) of the Civil Code. Thus, it is insusceptible to
acquisition by prescription.

VI.

A final word. The Court is comfortable with the correctness of the legal doctrines established in this
decision. Nonetheless, discomfiture over the implications of todays ruling cannot be discounted. For, every
untitled property that is occupied in the country will be affected by this ruling. The social implications cannot be
dismissed lightly, and the Court would be abdicating its social responsibility to the Filipino people if we simply
levied the law without comment.

The informal settlement of public lands, whether declared alienable or not, is a phenomenon tied to long-
standing habit and cultural acquiescence, and is common among the so-called Third World countries. This
paradigm powerfully evokes the disconnect between a legal system and the reality on the ground. The law so far
has been unable to bridge that gap. Alternative means of acquisition of these
public domain lands, such as through homestead or free patent, have

proven unattractive due to limitations imposed on the grantee in the encumbrance or alienation of said
properties.
[52]
Judicial confirmation of imperfect title has emerged as the most viable, if not the most attractive
means to regularize the informal settlement of alienable or disposable lands of the public domain, yet even that
system, as revealed in this decision, has considerable limits.

There are millions upon millions of Filipinos who have individually or exclusively held residential lands on
which they have lived and raised their families. Many more have tilled and made productive idle lands of the State
with their hands. They have been regarded for generation by their families and their communities as common law
owners. There is much to be said about the virtues of according them legitimate states. Yet such virtues are not
for the Court to translate into positive law, as the law itself considered such lands as property of the public
dominion. It could only be up to Congress to set forth a new phase of land reform to sensibly regularize and
formalize the settlement of such lands which in legal theory are lands of the public domain before the problem
becomes insoluble. This could be accomplished, to cite two examples, by liberalizing the standards for judicial
confirmation of imperfect title, or amending the Civil Code itself to ease the requisites for the conversion of public
dominion property into patrimonial.

Ones sense of security over land rights infuses into every aspect of well-being not only of that individual,
but also to the persons family. Once that sense of security is deprived, life and livelihood are put on stasis. It is for
the political branches to bring welcome closure to the long pestering problem.

WHEREFORE, the Petition is DENIED. The Decision of the Court of Appeals dated 23 February 2007 and
Resolution dated 2 October 2007 areAFFIRMED. No pronouncement as to costs.

SO ORDERED.

12. G.R. No. L-40912 September 30, 1976
REPUBLIC OF THE PHILIPPINES, represented by the MINDANAO MEDICAL CENTER, petioner,
vs.
HON. COURT OF APPEALS and ALEJANDRO Y DE JESUS, respondents.
Office of the Solicitor for petitioner.
Ananias C. Ona for private respondent.

MARTIN, J .:t.hqw
This is an appeal by certiorari from the decision of the Court of Apiwals in its CA-G.R. No. 39577-R,
raising the question of whether or not petitioner Mindanao Medical Center has registerable title over a full
12.8081-hectare land by virtue of an executive proclamation in 1956 reserving the area for medical center
site purposes.
On January 22, 1921, Eugenio de Jesus, the father of respondent Alejandro de Jesus, applied with the
Bureau of Lands for Sales Patent (Sales Application No. 5436) of a 33-hectare situated in barrio Libaron,
Municipality of Davao (now Davao City).
1
The property applied for was a portion of what was then known
as Lot 522 of the Davao Cadastre.
On January 23, 1934, the Bureau of Lands, through its Davao District Land Officer, accepted sealed bids
for the purchase of the subject land. One Irineo Jose bidded for P20.00 per hectare, while a certain Dr.
Josc Ebro submitted a bid of P100.50 per hectare The Director of Lands, however, annulled the auction
sale for the reason that the sales applicant, Eugenio de Jesus, failed to participate in the bidding for non-
service of notice on him of the scheduled bidding.
In lieu of that sale, another bidding was held on October 4, 1934. Sales applicant Eugenio de Jesus was
the lone bidder. He equalled the bid previously submitted by Dr. Jose Ebro and made a deposit of
P221.00 representing 10% of the price of the land at P100.50 per hectare.
On November 23, 1934, the Director of Lands issued to Eugenio de Jesus an Order of Award, the
dispositive portion of which reads:
2
+.wph!1
In view of the foregoing, and it appearing that the proceedings had in connection with the
Sales Application No. 5436 were in accordance with law and existing regulations, the
land covered thereby is herebyawarded to the said applicant, Eugenio de jesus, at
P100.50 per hectare or P2,211.00 for the whole tract.
This application should be entered in the records of this office as Sales Application No.
3231,covering the tract herein awarded, which is more particularly described as follows:
Location: Central, Davao,+.wph!1
Davao
Area: 22 hectares
Boundaries:+.wph!1
NMaria Villa Abrille and Arenio Suazo;
SEProvincial Road and Mary Gohn;
SWPublic Land;
WMunicipal Road;
Because the area conveyed had not been actually surveyed at the time Eugenio de Jesus filed his Sales
Application, the Bureau of Lands conducted a survey under Plan Bsd-1514. On July 29, 1936, the plan
was approved and the land awarded to Eugenio de Jesus was designated as Lot Nos. 1176-A, 1176-B-1-
A and 1176-B-1-B with an aggregate area of 20.6400 hectares, Bsd-10153, City of Davao.
On August 28, 1936, the Director of Lands ordered an amendment of the Sales Application of Eugenio de
Jesus stating that "a portion of the land covered by Sales Application No. 5436 (E-3231) of Eugenio de
Jesus is needed by the Philippine Army for military camp site purposes, the said application is amended
so as to exclude therefrom portion "A" as shown in the sketch on the back thereof, and as thus amended,
it will continue to be given due course." The area excluded was Identified as Lot 1176-B-2, the very land
in question, consisting of 12.8081 hectares.
On September 7, 1936, President Manuel L. Quezon issued Proclaimation No. 85 withdrawing Lot No.
1176-B-2 from sale and settlement and reserving the same for military purposes, under the administration
of the Chief of Staff, Philippine Army.
On November 29, 1939, Eugenio de Jesus paid P660.45 covering the 8th and 10th installment for
20.6400 hectares, the remaining area after his Sales Application was amended. This payment did not
include the military camp site (Lot No. 1176-B-2) as the same had already been excluded from the Sales
Application at the time the payment was made.
3
Thereafter, or on May 15, 1948, then Director of Lands
Jose P. Dans ordered the issuance of patent to Eugenio de Jesus, pursuant to his Sales Application for "a
tract of land having an area of 20.6400 hectares, situated in the barrio of Poblacion, City of Davao.
4
On
the same date, then Secretary of Agriculture and Natural Resources Mariano Garchitorena granted a
Sales Patent to Eugenio de Jesus for "a tract of agricultural public land situated in the City of Davao,
Island of Mindanao, Philippines,containing an area of 20 hectares, 64 ares, and 00 centares.
5

On August 11, 1956, President Ramon Magsaysay revoked Proclamation No. 85 and declared the
disputed Lot 1176-B-2 open to disposition under the provisions of the Public land Act for resettlement of
the squatters in the Piapi Beach, Davao City.
6
In the following October 9, President Magsaysay revoked
this Proclamation No. 328 and reserved the same Lot No. 1176-B-2 for medical center site purposes
under the administration of the Director of Hospital.
7

Whereupon, on December 6, 1969, petitioner Mindanao Medical Center applied for the Torrens
registration of the 12.8081-hectare Lot 1176-B-2 with the Court of First Instance of Davao. The Medical
Center claimed "fee simple" title to the land on the strength of proclamation No. 350 reserving the area for
medical center site purposes.
Respondent Alejandro de Jesus, the son and successor-in-interest of sale applicant Eugenio de Jesus,
opposed the registration oil the ground that his father, Eugenio de Jesus, had aquired a vested right on
the subject lot by virtue of the Order of Award issued to him by the Director of Lands.
A certain Arsenio Suazo likewise filed his opposition to the registration on the claim that the 2-hectare
portion on the northeastern part of Lot 1176-B-2 belongs to him.
After due hearing, the Court of First Instance of Davao rendered judgment on September 2, 1966,
directing "the registration of the title to Lot No. 1176-B-2 of Subdivision Plan Bsd-5134, shown on Plan
Ap-6512, situated in the Barrio of Central, City of Davao, and containing an area of 128,081 square
meters in the name of the Mindanao Medical Center, Bureau of Medical Services, Department of Health.
The two oppositors, Alejandro de Jesus and Arsenio Suazo, excepted from this judgment of the trial court
and appealed the case to the respondent Court of Appeals.
On July 2, 1974, the Appellate Court held: +.wph!1
WHEREFORE, the appealed judgment is hereby modified insofar as it denies the claim
of appellant Arsenio Suazo, the same is hereby affirmed, in regard the appeal of
appellant Alejandro Y. de Jesus, registration Lot 1176-B-2, situated in Barrio Central,
Davao City, and containing an area of 12.8081 square meters, is hereby decreed in the
name of said appellants, but said appellant is hereby ordered to relinquish to the appellee
that portion of Lot 1176-B-2 which is occupied by the medical center and nervous disease
pavilion and their reasonable appartenances, no costs.
On July 5, 1974, petitioner Mindanao Medical Center moved for reconsideration, maintaining ownership
over the entire area of 12.8081 hectares, but the Appellate Court in a Special Division of Five denied the
motion on June 17, 1975.
8

Forthwith, petitioner Mindanao Medical Center elevated the matter to Us thru the present appeal.
We find petitioner's appeal to b meritorious.
1. Petitioner Mindanao Medical Center has registerable title over the whole contested area of 12.8081
hectares, designated Lot No. 1176-B-2, and not only on a portion thereof occupied by the Medical Center,
its nervous disease pavilion and their reasonable appurtenances. Proclamation No. 350, dated October 9,
1956, of President Magsaysay legally effected a land grant to the Mindanao Medical Center, Bureau of
Medical Services, Department of Health, of the whole lot, validity sufficient for initial registration under the
Land Registration Act. Such land grant is constitutive of a "fee simple" tile or absolute title in favor of
petitioner Mindanao Medical Center. Thus, Section 122 of the Act, which governs the registration of
grants or patents involving public lands, provides that "Whenever public lands in the Philippine Islands
belonging to the Government of the Philippines are alienated, granted, or conveyed to persons or to
public or private corporations, the same shall be brought forthwith under the operation of this Act [Land
Registration Act, Act 496] and shall become registered lands."
9
It would be completely absurd to rule
that, on the basis of Proclamation No. 350, the Medical Center has registerable title on the portion
occupied by it, its nervous disease pavilion and the reasonable appurtenances, and not on the full extent
of the reservation, when the proclamation explicitly reserved the entire Lot 1176-B-2 of 12.8081 hectares
to the Center.
Certainly, proclamation no. 350 is free of any legal infirmity. It proceeds from the recognized competence
of the president to reserve by executive proclamation alienable lands of the public domain for a specific
public use or service.
10
section 64 (e) of the Revised Administrative Code empowers the president "(t)o
reserve from sale oe other disposition and for specific public uses for service, any land belonging to the
private domain of the Government of the Philippines, the use of which is not otherwise directed by law.
the land reserved "shall be used for the specific purposes directed by such executive order until otherwise
provided by law." Similarly, Section 83 of the Public Land Act (CA 141) authorizes the President to
"designate by proclamation any tract or tracts of land of the public domain as reservations for the use
ofthe commonwealth of the Philippines or of any of its branches, or of the inhabitants thereof, ... or for
quasi-public uses or purposes when the public interest requires it, including reservations for ... other
improvements for the public benefit.
2. Respondent Appellate Court erroneously ruled that Alejabdro's father, Eugenio de jesus, had acquired
ownership over the whole 12.8081-hectare Lot 1176-B-2 because the Sales Award issued to him on
November 23, 1934 by then Director of Lands Simeon Ramos covered the 33 hectares applied for,
including the 12.8081 hectares. We fail to see any reasonable basis on record for the Appellate Court to
draw such conclusion. On the contrary, the very Sales Award describes the tract awarded as located in
Central, Davao, Davao, with an area of22 hectares, and bounded on the north by Maria Villa Abrille and
Arsenio Suazo; on the southeast by a provincial road and Mary Gohn; on the southwest by a public land;
and on the west by a municipal road.
11
This area of 22 hectares was even reduced to 20.6400 hectares
upon actual survey made by the Bureau of Lands. The same area was reckoned with by then Lands
Director Jose P. Dans when he directed the issuance of a patent to Eugenio de Jesus on May 15, 1948
for his application filed on January 22, 1921 covering "a tract of land having an area of 20.6400 hectares,
situated in the barrio of Poblacion, City of Davao."
12
In like manner, the Sales Patent issued to Eugenio
de Jesus on the same date, May 15, 1948, by then Secretary of Agriculture and Natural Resources
Mariano Garchitorena indicated therein the sale to Eugenio de Jesus of "a tract of agricultural public land
situated in the City of Davao, Island of Mindanao, Philippines, containing an area of 20 hectares 64, ares
00 centares." Seen in the light of Patent, and Sales Order for Issuance of Patent, and Sales Patent,
invariably bearing the area awarded to sales applicant Eugenio de Jesusas 20.6400 hectares, it becomes
imperative to conclude that what was really awarded to Eugenio de jesus was only 20.6400 hectares and
not 33 hectares as applied for by him.
However, We observe that in the public bidding of october 4, 1934, the succesful bidder, submitted a bid
of 100.50 per hectare and made a cash deposit of only P221.00, which amount represents 10% of the
purchase price of the land.
13
At P100.50 per hectare, the purchase would be P2,221.00 for 22 hectares,
10% deposit of which amounts to P221.00. For 33 hectares, the total purchase price would be P3,316.50
at P100.50 per hectare and the 10% deposit would be P331.65, not P221.00, as what was actually
deposited by sales applicant Eugenio de Jesus. Withal, if Eugenio de Jesus was really awarded 33
hectares in that public bidding, he should have made the required 10% deposit of P331.65. That he
merely deposited P221.00 strongly suggests that what was bidden for and awarded to him was only 22
hectares and not 33 hectares as applied for. As a matter of fact, his last payment of P660.45 on
November 29, 1939 for the 8th te 10th installment intended only to cover 20.6400 hectares, the remaining
area after the amendment of the Sales Application on August 28, 1936, excluding "the military camp site
[Lot 1176B-2 of 12.8081 hectares] for the reason that the said site, at the time of last installment was
already excluded from Sale Application SA-5436 of Eugenio de Jesus, as ordered ... by the Director of
Lands."
14

But, respondent Appellate Court reasons out that if the area bidden for and awarded in 1934 ws only 22
hectares and since two years thereafter the Director of Lands ordered an amendment excluding the
military camp site of 12.8081 hectares, then only 10 hectares, then would have been left to applicant
Eugenio de Jesus and not 20.6400 hectares would have been left in the Sales Patent. The Appellate
Court's reasoning is premised on wrong assumption. What was ordered amended was the Sales
Application for 33 hectares and not the Order of 22 hectares or 20.6400 hectares. The Order states:
"Order: Amendment of Application." Necessarily so, because the amendment was already reflected in the
Order of Award, since only an area of 22 hectares was awarded.
3. The phrase "whole tract" in the Sales Award
15
cannot be licitly seized upon as basis for the conclusion
that the area awarded to applicant Eugenio de Jesus was the applied area of 33 hectares. Such general
description of "whole tract" cannot prevail over the specific description delineating the area in quantity and
in boundaries. Thus, the Sales Award specifies the area awarded as 22 hectares, located at Central,
Davao, Davao, and bounded on the north by the property of Maria Villa Abrille and Arsenio Suazo; on the
southwest by a provincial road and the property by Mary Gohn on the southwest by a public land; and on
the west by a municipal road.
16
Specific description is ordinarily preferred to general description, or that
which is more certain to what which is less certain.
17
More so, when it is considered that the series of
executive proclamations (Proclamation Nos. 85, 328, 350) continuously maintained the intent of the
Government to reserve the subject land for a specific purpose or service.
Besides, patents and land grants are construed favorably to the Governement, and most strongly against
the grantee.
18
Any doubt as to the intention or extent of the grant, or the intention of the Government, is
to be resolved in its favor.
19
In general, the quantity of the land granted must be ascertained from the
description in the patent is exclusive evidence of the land conveyed.
20
And courts do not usually go
beyond a description of a tract in a patent and determine the tract and quantity of land apart from the
patent itself.
21

4. We cannot share the view of respondent Appellate Court that eugenio de jesus's alleged occupation,
cultivation and improvement of the 33-hectare land (including the 12-hectare camp site) since 1916
vested in him a right of preference or pre-empive right in the acquisition of the land, which right was
controverted into "a special propriety right" when the Sales Award was issued to him in 1934. Not only for
the earlier reasons that the Sales Award was only for 22 hectares (later found to be 20,6400 fectares
upon actual survey) and not for 33 hectares, the privilege of occupying public lands a view to preemption
confers np contractual or vested right in the lands occupied and the authority of the President to withdraw
suchlands for sale or acquisition by the public, or to reserve them for public use, prior to the divesting by
the government of title threof stands, even though this may defeat the imperfect right of a settler.
22
Lands
covered by reservation are not subject to entry, and no lawful settlement on them can be acquired.
23
The
claims o0f persons who have settled on occupied, and improved a parcel of public land which is later
included in a reservation are considered worthy of protection and are usually respected, but where the
President, as authorized by law, issuesa proclamation reserving certain lands and warning all persons to
depart therefrom, this terminates any rights previously avquired in such lands by a person who was
settled thereon in order to obtain a preferential right of purchase.
24
And patents for lands which have
been previously granted, reserved from sale, or appropriate, are void.
25

It is true that Proclamation No. 350 states that the same is subject to "privilege rights, if any there be," but
Eugenio de Jesus or his son Alejandro de Jesus failed to prove any private rights over the property
reserved. Wee-settled is the rule that unless the applicant has shown by clear and convincing evidence
that a certain portion of the public domain was acquired by him or his ancestors either by composition title
from the Spanish Government or by possessory information title, or any other means for the acquisition of
public lands, such as grants or patents, the property must be held to be part of the public domain.
26
Nor
could respondent Alejandro de Jesus legetimately claim to have obtained title by prescription over the
disputed 12.8081 hectares, inasmuch as by applying for the sale thereof (assuming hypothetically that the
12.8081-hectare lot was included in the original sales application for 33 hectares), his father, Eugenio de
Jesus, necessarily admits that the portions applied for are part of the public domain, against which no
acquisitive prescription may lie
27
except as provided in Section 48(b) of C.A. 141, as amended.
5. Respondent Appellate Court mistakenly sustained Eugenio de Jesus's pretense that the military "camp
site" (Lot 176-B-2) had been donated by him to the Philippine Army, thru Secretary Serafin Marabut of the
Department of National Defense, sometime in 1936 subject to the condition that it would be returned to
him when the Philippine Army would no longer need it. As found by the trial court in 1936, the Department
of National Defense was not yet in existence, so that no Defense Secretary by the name of Serafin
Marabut could have entered into a deed of donation with Eugenio de Jesus over Lot 1176-B-2 consisting
of 12.8081 hectares. The Department of National Defense was only organized in 1939. Nonetheless,
respondent Alejandro de Jesus, would prove by secondary evidence the existence of such donation thru
the testimony of persons who supposedly saw it. In this regard, the Rules provides that before the terms
of a transaction in realty may be established by secondary evidence, it is n that the due execution and
subsequent loss of the original instrument evidencing the transaction be proved. For it is the due
execution of the document and its subsequent loss that would constitute the foundation for the
introduction of secondary evidence to prove the contents of such document. And the due of the execution
of the document would be proved through the testimony of (1) the person or persons who executed it; (2)
the person before whom its execution was acknowledged, or (3) any who was present and saw it
executed and delivered, or who, after its execution and delivery, saw it and recognized the signatures, or
by a person to whom the parties to the instrument had previously confessed the execution
thereof.
28
None of these modes of proof was ever followed by respondent Alejandro de Jesus. His
predecessor- in-interest, Eugenio de Jesus, merely made a broad statement that he executed a deed f
donation in 1936 with Defense Secretary Marabut when at hat time the Defense Department was not yet
in existence. The notary public who presumptively acknowledged the donation or the witnesses to the
instrument were never presented. It has been ruled that the failure of the party to present the notary
Public and thore s who must have seen the signing of the document as witnesses to testify on its
execution interdicts the admission of a secondary evidence of the terms of the deed.
29
This is especially
true in realty donations where Art. 748 of the new Civil Code requires the accomplishment thereof in a
public document in order to be valid. The testimony of Marcelo Belendres that Sesinando de jesus,
brother of Eugenio de Jesus showed him a copy of the "paper" signed by Secretary Marabut and Eugenio
de Jesus; of Jose Tinio, Acting Register of Deeds of Davao, that in May or June 1937, Col. Simeon de
jesus went to his office to register a document" executed by Eugenio de Jesus and Secretary Marabut; of
former Secretary Brigido Valencia that Col. Simeon de Jesus showed him a deed of donation signed by
Eugenio de Jesus and Serafin Marabut. hardly suffer to satisfy the requisites of the Rules, as to which
very strict compliance is imposed because of the importance of the document involved.
30
First none of
these persons was a witness to the instrument, nor any of them saw the document after its execution and
delivery ind recognized the signatures of the parties nor to whom the parties to the instrument had
previously confessed the execution; second, the reference to a "paper" or "document" ambigous as to be
synonymous with a "deed of donation;" and third, the persons who showed the deed, Sesinando de Jesus
and Col. Simeon de Jesus were not parties to the instrument. Respondent Alejandro de Jesus's narration
of the existence and loss of the document equally deserves no credence. As found by the trial court, he
testified that the copy of the deed which his father kept was sent to him in Manila thru his uncle,
Sesinando de Jesus in July 1942, while his father himself, Eugenio de Jesus, declared that his copy of
the deed was burned in Davao during the Japanese occupation. The replies of the Undersecretary of
Agriculture and Natural Resources and the Acting Executive Secretary that the property was "still needed
for military purposes" and may not therefore be released from the reservation cannot substitute the proof
so required. These replies are not confirmatory of the existence of such donation much less official
admissions thereof.
Even on the gratuitous assumption that a donation of the military "camp site" was executed between
Eugenior de jesus and Serafin Marabut, such donation would anyway be void, because Eugenior de
jesus held no dominical rights over the site when it was allegedly donated by him in 1936. In that year,
proclamation No. 85 of President Quezon already withrew the area from sale or settlement and reserved
it for military purposes. Respondent Appellate Court, however, rationalizes that the subject of the
donation was not the land itself but "the possessory and special proprietary rights" of Eugenio de jesus
over it. We disagree. It is true that the gratiuitous disposal in donation may consist of a thing or
right.
31
But the term "right" must be understood in a "propriety" sense, over which the processor has the
jus disponendi.
32
This is because, in true donations, there results a consequent impoverishment of the
donor or diminution of his assets.
33
Eugenio de Jesus cannot be said to be possessed of that "proprietary
" right over the whole 33 hectares in 1936 including the disputed 12.8081 hectares for at that time this
12.8081-hectare lot had already been severed from the mass of disposable public lands by Proclamation
No. 85 and excluded in the Sales Award. Impoverishment of Eugenio's assets as a consequence of such
donation is therefore farfetehed. In fact, even if We were to assume in gratia argumenti that the 12.8081-
hectare lot was included in the Sales Award, still the same may not be the subject of donation. In Sales
Award, what is conferred on the applicant is merely the right "to take possession of the land so that he
could comply with the requirements prescribed by law."
34
In other words, the right granted to the sales
awardee is only "possessory right" as distinguished from "proprietary right," for the fundamental reason
that prior to the issuance of the sales patent and registration thereof, title to the land is retained by the
State.
35
Admittedly, the land applied for may be considered "disposed of by the Government" upon the
issuance of the Sales Award, but this has the singular effect of withdrawing the land from the public
domian that is "disposable" by the Director of Lands under the Public Land Act. Moreover, the dsiposition
is merely provisional because the applicant has still to comply with the requirements of the law before any
patent is issued. It is only after compliance with such requirements to the satisfaction of the Director of
Lands, that the patent is issued and the land applied for considered "permanently disposed of by the
Government." This again is a circumstance that demeans the irrevocable nature donation, because the
mere desistance of the sales applicant to pursue the requirements called for would cause the virtual
revocation of the donation.
ACCORDINGLY, the appealed judgement of the Court of Appeals, promulgated on July 2, 1974, and its
resolution of Jane 17, 1975, denying petitioner's motion for reconsiderations, are hereby reversed and set
aside. The disputed Lot 1176-B-2, Plan Bsd-1514 of Davao Cadastre and containing an area of 12.8081
hectares, is hereby adjudicated in favor of petitioner Mindanao Medical Center. The urgent motion of the
petitioner for leave to construct essential hospitawl buildings, namely: (a) communicable and contagious
diseas pavilion; (b) hospital motorpool; and (c) physician's quarters, is hereby granted. With costs against
private respondent.
SO ORDERED.
Teehankee (Chairman), Makasiar, Muoz Palma and Concepcion, Jr., JJ., concur.1wph1.t


13. G.R. No. 133250 July 9, 2002
FRANCISCO I. CHAVEZ, petitioner,
vs.
PUBLIC ESTATES AUTHORITY and AMARI COASTAL BAY DEVELOPMENT
CORPORATION, respondents.
CARPIO, J .:
This is an original Petition for Mandamus with prayer for a writ of preliminary injunction and a temporary
restraining order. The petition seeks to compel the Public Estates Authority ("PEA" for brevity) to disclose
all facts on PEA's then on-going renegotiations with Amari Coastal Bay and Development Corporation
("AMARI" for brevity) to reclaim portions of Manila Bay. The petition further seeks to enjoin PEA from
signing a new agreement with AMARI involving such reclamation.
The Facts
On November 20, 1973, the government, through the Commissioner of Public Highways, signed a
contract with the Construction and Development Corporation of the Philippines ("CDCP" for brevity) to
reclaim certain foreshore and offshore areas of Manila Bay. The contract also included the construction of
Phases I and II of the Manila-Cavite Coastal Road. CDCP obligated itself to carry out all the works in
consideration of fifty percent of the total reclaimed land.
On February 4, 1977, then President Ferdinand E. Marcos issued Presidential Decree No. 1084 creating
PEA. PD No. 1084 tasked PEA "to reclaim land, including foreshore and submerged areas," and "to
develop, improve, acquire, x x x lease and sell any and all kinds of lands."
1
On the same date, then
President Marcos issued Presidential Decree No. 1085 transferring to PEA the "lands reclaimed in the
foreshore and offshore of the Manila Bay"
2
under the Manila-Cavite Coastal Road and Reclamation
Project (MCCRRP).
On December 29, 1981, then President Marcos issued a memorandum directing PEA to amend its
contract with CDCP, so that "[A]ll future works in MCCRRP x x x shall be funded and owned by PEA."
Accordingly, PEA and CDCP executed a Memorandum of Agreement dated December 29, 1981, which
stated:
"(i) CDCP shall undertake all reclamation, construction, and such other works in the MCCRRP as
may be agreed upon by the parties, to be paid according to progress of works on a unit
price/lump sum basis for items of work to be agreed upon, subject to price escalation, retention
and other terms and conditions provided for in Presidential Decree No. 1594. All the financing
required for such works shall be provided by PEA.
x x x
(iii) x x x CDCP shall give up all its development rights and hereby agrees to cede and transfer in
favor of PEA, all of the rights, title, interest and participation of CDCP in and to all the areas of
land reclaimed by CDCP in the MCCRRP as of December 30, 1981 which have not yet been
sold, transferred or otherwise disposed of by CDCP as of said date, which areas consist of
approximately Ninety-Nine Thousand Four Hundred Seventy Three (99,473) square meters in the
Financial Center Area covered by land pledge No. 5 and approximately Three Million Three
Hundred Eighty Two Thousand Eight Hundred Eighty Eight (3,382,888) square meters of
reclaimed areas at varying elevations above Mean Low Water Level located outside the Financial
Center Area and the First Neighborhood Unit."
3

On January 19, 1988, then President Corazon C. Aquino issued Special Patent No. 3517, granting and
transferring to PEA "the parcels of land so reclaimed under the Manila-Cavite Coastal Road and
Reclamation Project (MCCRRP) containing a total area of one million nine hundred fifteen thousand eight
hundred ninety four (1,915,894) square meters." Subsequently, on April 9, 1988, the Register of Deeds of
the Municipality of Paraaque issued Transfer Certificates of Title Nos. 7309, 7311, and 7312, in the
name of PEA, covering the three reclaimed islands known as the "Freedom Islands" located at the
southern portion of the Manila-Cavite Coastal Road, Paraaque City. The Freedom Islands have a total
land area of One Million Five Hundred Seventy Eight Thousand Four Hundred and Forty One (1,578,441)
square meters or 157.841 hectares.
On April 25, 1995, PEA entered into a Joint Venture Agreement ("JVA" for brevity) with AMARI, a private
corporation, to develop the Freedom Islands. The JVA also required the reclamation of an additional 250
hectares of submerged areas surrounding these islands to complete the configuration in the Master
Development Plan of the Southern Reclamation Project-MCCRRP. PEA and AMARI entered into the JVA
through negotiation without public bidding.
4
On April 28, 1995, the Board of Directors of PEA, in its
Resolution No. 1245, confirmed the JVA.
5
On June 8, 1995, then President Fidel V. Ramos, through then
Executive Secretary Ruben Torres, approved the JVA.
6

On November 29, 1996, then Senate President Ernesto Maceda delivered a privilege speech in the
Senate and denounced the JVA as the "grandmother of all scams." As a result, the Senate Committee on
Government Corporations and Public Enterprises, and the Committee on Accountability of Public Officers
and Investigations, conducted a joint investigation. The Senate Committees reported the results of their
investigation in Senate Committee Report No. 560 dated September 16, 1997.
7
Among the conclusions of
their report are: (1) the reclaimed lands PEA seeks to transfer to AMARI under the JVA are lands of the
public domain which the government has not classified as alienable lands and therefore PEA cannot
alienate these lands; (2) the certificates of title covering the Freedom Islands are thus void, and (3) the
JVA itself is illegal.
On December 5, 1997, then President Fidel V. Ramos issued Presidential Administrative Order No. 365
creating a Legal Task Force to conduct a study on the legality of the JVA in view of Senate Committee
Report No. 560. The members of the Legal Task Force were the Secretary of Justice,
8
the Chief
Presidential Legal Counsel,
9
and the Government Corporate Counsel.
10
The Legal Task Force upheld the
legality of the JVA, contrary to the conclusions reached by the Senate Committees.
11

On April 4 and 5, 1998, the Philippine Daily Inquirer and Today published reports that there were on-
going renegotiations between PEA and AMARI under an order issued by then President Fidel V. Ramos.
According to these reports, PEA Director Nestor Kalaw, PEA Chairman Arsenio Yulo and retired Navy
Officer Sergio Cruz composed the negotiating panel of PEA.
On April 13, 1998, Antonio M. Zulueta filed before the Court a Petition for Prohibition with Application for
the Issuance of a Temporary Restraining Order and Preliminary Injunction docketed as G.R. No. 132994
seeking to nullify the JVA. The Court dismissed the petition "for unwarranted disregard of judicial
hierarchy, without prejudice to the refiling of the case before the proper court."
12

On April 27, 1998, petitioner Frank I. Chavez ("Petitioner" for brevity) as a taxpayer, filed the
instant Petition for Mandamus with Prayer for the Issuance of a Writ of Preliminary Injunction and
Temporary Restraining Order. Petitioner contends the government stands to lose billions of pesos in the
sale by PEA of the reclaimed lands to AMARI. Petitioner prays that PEA publicly disclose the terms of any
renegotiation of the JVA, invoking Section 28, Article II, and Section 7, Article III, of the 1987 Constitution
on the right of the people to information on matters of public concern. Petitioner assails the sale to AMARI
of lands of the public domain as a blatant violation of Section 3, Article XII of the 1987 Constitution
prohibiting the sale of alienable lands of the public domain to private corporations. Finally, petitioner
asserts that he seeks to enjoin the loss of billions of pesos in properties of the State that are of public
dominion.
After several motions for extension of time,
13
PEA and AMARI filed their Comments on October 19, 1998
and June 25, 1998, respectively. Meanwhile, on December 28, 1998, petitioner filed an Omnibus Motion:
(a) to require PEA to submit the terms of the renegotiated PEA-AMARI contract; (b) for issuance of a
temporary restraining order; and (c) to set the case for hearing on oral argument. Petitioner filed a
Reiterative Motion for Issuance of a TRO dated May 26, 1999, which the Court denied in a Resolution
dated June 22, 1999.
In a Resolution dated March 23, 1999, the Court gave due course to the petition and required the parties
to file their respective memoranda.
On March 30, 1999, PEA and AMARI signed the Amended Joint Venture Agreement ("Amended JVA," for
brevity). On May 28, 1999, the Office of the President under the administration of then President Joseph
E. Estrada approved the Amended JVA.
Due to the approval of the Amended JVA by the Office of the President, petitioner now prays that on
"constitutional and statutory grounds the renegotiated contract be declared null and void."
14

The Issues
The issues raised by petitioner, PEA
15
and AMARI
16
are as follows:
I. WHETHER THE PRINCIPAL RELIEFS PRAYED FOR IN THE PETITION ARE MOOT AND
ACADEMIC BECAUSE OF SUBSEQUENT EVENTS;
II. WHETHER THE PETITION MERITS DISMISSAL FOR FAILING TO OBSERVE THE
PRINCIPLE GOVERNING THE HIERARCHY OF COURTS;
III. WHETHER THE PETITION MERITS DISMISSAL FOR NON-EXHAUSTION OF
ADMINISTRATIVE REMEDIES;
IV. WHETHER PETITIONER HAS LOCUS STANDI TO BRING THIS SUIT;
V. WHETHER THE CONSTITUTIONAL RIGHT TO INFORMATION INCLUDES OFFICIAL
INFORMATION ON ON-GOING NEGOTIATIONS BEFORE A FINAL AGREEMENT;
VI. WHETHER THE STIPULATIONS IN THE AMENDED JOINT VENTURE AGREEMENT FOR
THE TRANSFER TO AMARI OF CERTAIN LANDS, RECLAIMED AND STILL TO BE
RECLAIMED, VIOLATE THE 1987 CONSTITUTION; AND
VII. WHETHER THE COURT IS THE PROPER FORUM FOR RAISING THE ISSUE OF
WHETHER THE AMENDED JOINT VENTURE AGREEMENT IS GROSSLY
DISADVANTAGEOUS TO THE GOVERNMENT.
The Court's Ruling
First issue: whether the principal reliefs prayed for in the petition are moot and academic because
of subsequent events.
The petition prays that PEA publicly disclose the "terms and conditions of the on-going negotiations for a
new agreement." The petition also prays that the Court enjoin PEA from "privately entering into, perfecting
and/or executing any new agreement with AMARI."
PEA and AMARI claim the petition is now moot and academic because AMARI furnished petitioner on
June 21, 1999 a copy of the signed Amended JVA containing the terms and conditions agreed upon in
the renegotiations. Thus, PEA has satisfied petitioner's prayer for a public disclosure of the
renegotiations. Likewise, petitioner's prayer to enjoin the signing of the Amended JVA is now moot
because PEA and AMARI have already signed the Amended JVA on March 30, 1999. Moreover, the
Office of the President has approved the Amended JVA on May 28, 1999.
Petitioner counters that PEA and AMARI cannot avoid the constitutional issue by simply fast-tracking the
signing and approval of the Amended JVA before the Court could act on the issue. Presidential approval
does not resolve the constitutional issue or remove it from the ambit of judicial review.
We rule that the signing of the Amended JVA by PEA and AMARI and its approval by the President
cannot operate to moot the petition and divest the Court of its jurisdiction. PEA and AMARI have still to
implement the Amended JVA. The prayer to enjoin the signing of the Amended JVA on constitutional
grounds necessarily includes preventing its implementation if in the meantime PEA and AMARI have
signed one in violation of the Constitution. Petitioner's principal basis in assailing the renegotiation of the
JVA is its violation of Section 3, Article XII of the Constitution, which prohibits the government from
alienating lands of the public domain to private corporations. If the Amended JVA indeed violates the
Constitution, it is the duty of the Court to enjoin its implementation, and if already implemented, to annul
the effects of such unconstitutional contract.
The Amended JVA is not an ordinary commercial contract but one which seeks to transfer title and
ownership to 367.5 hectares of reclaimed lands and submerged areas of Manila Bay to a single
private corporation. It now becomes more compelling for the Court to resolve the issue to insure the
government itself does not violate a provision of the Constitution intended to safeguard the national
patrimony. Supervening events, whether intended or accidental, cannot prevent the Court from rendering
a decision if there is a grave violation of the Constitution. In the instant case, if the Amended JVA runs
counter to the Constitution, the Court can still prevent the transfer of title and ownership of alienable lands
of the public domain in the name of AMARI. Even in cases where supervening events had made the
cases moot, the Court did not hesitate to resolve the legal or constitutional issues raised to formulate
controlling principles to guide the bench, bar, and the public.
17

Also, the instant petition is a case of first impression. All previous decisions of the Court involving Section
3, Article XII of the 1987 Constitution, or its counterpart provision in the 1973
Constitution,
18
covered agricultural landssold to private corporations which acquired the lands from
private parties. The transferors of the private corporations claimed or could claim the right to judicial
confirmation of their imperfect titles
19
under Title II of Commonwealth Act. 141 ("CA No. 141" for
brevity). In the instant case, AMARI seeks to acquire from PEA, a public corporation, reclaimed lands and
submerged areas for non-agricultural purposes by purchase under PD No. 1084 (charter of PEA)
and Title III of CA No. 141. Certain undertakings by AMARI under the Amended JVA constitute the
consideration for the purchase. Neither AMARI nor PEA can claim judicial confirmation of their titles
because the lands covered by the Amended JVA are newly reclaimed or still to be reclaimed. Judicial
confirmation of imperfect title requires open, continuous, exclusive and notorious occupation of
agricultural lands of the public domain for at least thirty years since June 12, 1945 or earlier. Besides, the
deadline for filing applications for judicial confirmation of imperfect title expired on December 31, 1987.
20

Lastly, there is a need to resolve immediately the constitutional issue raised in this petition because of the
possible transfer at any time by PEA to AMARI of title and ownership to portions of the reclaimed lands.
Under the Amended JVA, PEA is obligated to transfer to AMARI the latter's seventy percent proportionate
share in the reclaimed areas as the reclamation progresses. The Amended JVA even allows AMARI to
mortgage at any time the entirereclaimed area to raise financing for the reclamation project.
21

Second issue: whether the petition merits dismissal for failing to observe the principle governing
the hierarchy of courts.
PEA and AMARI claim petitioner ignored the judicial hierarchy by seeking relief directly from the Court.
The principle of hierarchy of courts applies generally to cases involving factual questions. As it is not a
trier of facts, the Court cannot entertain cases involving factual issues. The instant case, however, raises
constitutional issues of transcendental importance to the public.
22
The Court can resolve this case without
determining any factual issue related to the case. Also, the instant case is a petition for mandamus which
falls under the original jurisdiction of the Court under Section 5, Article VIII of the Constitution. We resolve
to exercise primary jurisdiction over the instant case.
Third issue: whether the petition merits dismissal for non-exhaustion of administrative remedies.
PEA faults petitioner for seeking judicial intervention in compelling PEA to disclose publicly certain
information without first asking PEA the needed information. PEA claims petitioner's direct resort to the
Court violates the principle of exhaustion of administrative remedies. It also violates the rule that
mandamus may issue only if there is no other plain, speedy and adequate remedy in the ordinary course
of law.
PEA distinguishes the instant case from Taada v. Tuvera
23
where the Court granted the petition for
mandamus even if the petitioners there did not initially demand from the Office of the President the
publication of the presidential decrees. PEA points out that in Taada, the Executive Department had
an affirmative statutory duty under Article 2 of the Civil Code
24
and Section 1 of Commonwealth Act No.
638
25
to publish the presidential decrees. There was, therefore, no need for the petitioners in Taada to
make an initial demand from the Office of the President. In the instant case, PEA claims it has no
affirmative statutory duty to disclose publicly information about its renegotiation of the JVA. Thus, PEA
asserts that the Court must apply the principle of exhaustion of administrative remedies to the instant
case in view of the failure of petitioner here to demand initially from PEA the needed information.
The original JVA sought to dispose to AMARI public lands held by PEA, a government corporation. Under
Section 79 of the Government Auditing Code,
26
the disposition of government lands to private parties
requires public bidding. PEA was under a positive legal duty to disclose to the public the terms and
conditions for the sale of its lands. The law obligated PEA to make this public disclosure even without
demand from petitioner or from anyone. PEA failed to make this public disclosure because the original
JVA, like the Amended JVA, was the result of a negotiated contract, not of a public bidding. Considering
that PEA had an affirmative statutory duty to make the public disclosure, and was even in breach of this
legal duty, petitioner had the right to seek direct judicial intervention.
Moreover, and this alone is determinative of this issue, the principle of exhaustion of administrative
remedies does not apply when the issue involved is a purely legal or constitutional question.
27
The
principal issue in the instant case is the capacity of AMARI to acquire lands held by PEA in view of the
constitutional ban prohibiting the alienation of lands of the public domain to private corporations. We rule
that the principle of exhaustion of administrative remedies does not apply in the instant case.
Fourth issue: whether petitioner has locus standi to bring this suit
PEA argues that petitioner has no standing to institute mandamus proceedings to enforce his
constitutional right to information without a showing that PEA refused to perform an affirmative duty
imposed on PEA by the Constitution. PEA also claims that petitioner has not shown that he will suffer any
concrete injury because of the signing or implementation of the Amended JVA. Thus, there is no actual
controversy requiring the exercise of the power of judicial review.
The petitioner has standing to bring this taxpayer's suit because the petition seeks to compel PEA to
comply with its constitutional duties. There are two constitutional issues involved here. First is the right of
citizens to information on matters of public concern. Second is the application of a constitutional provision
intended to insure the equitable distribution of alienable lands of the public domain among Filipino
citizens. The thrust of the first issue is to compel PEA to disclose publicly information on the sale of
government lands worth billions of pesos, information which the Constitution and statutory law mandate
PEA to disclose. The thrust of the second issue is to prevent PEA from alienating hundreds of hectares of
alienable lands of the public domain in violation of the Constitution, compelling PEA to comply with a
constitutional duty to the nation.
Moreover, the petition raises matters of transcendental importance to the public. In Chavez v.
PCGG,
28
the Court upheld the right of a citizen to bring a taxpayer's suit on matters of transcendental
importance to the public, thus -
"Besides, petitioner emphasizes, the matter of recovering the ill-gotten wealth of the Marcoses is
an issue of 'transcendental importance to the public.' He asserts that ordinary taxpayers have a
right to initiate and prosecute actions questioning the validity of acts or orders of government
agencies or instrumentalities, if the issues raised are of 'paramount public interest,' and if they
'immediately affect the social, economic and moral well being of the people.'
Moreover, the mere fact that he is a citizen satisfies the requirement of personal interest, when
the proceeding involves the assertion of a public right, such as in this case. He invokes several
decisions of this Court which have set aside the procedural matter of locus standi, when the
subject of the case involved public interest.
x x x
In Taada v. Tuvera, the Court asserted that when the issue concerns a public right and the
object of mandamus is to obtain the enforcement of a public duty, the people are regarded as the
real parties in interest; and because it is sufficient that petitioner is a citizen and as such is
interested in the execution of the laws, he need not show that he has any legal or special interest
in the result of the action. In the aforesaid case, the petitioners sought to enforce their right to be
informed on matters of public concern, a right then recognized in Section 6, Article IV of the 1973
Constitution, in connection with the rule that laws in order to be valid and enforceable must be
published in the Official Gazette or otherwise effectively promulgated. In ruling for the petitioners'
legal standing, the Court declared that the right they sought to be enforced 'is a public right
recognized by no less than the fundamental law of the land.'
Legaspi v. Civil Service Commission, while reiterating Taada, further declared that 'when a
mandamus proceeding involves the assertion of a public right, the requirement of personal
interest is satisfied by the mere fact that petitioner is a citizen and, therefore, part of the general
'public' which possesses the right.'
Further, in Albano v. Reyes, we said that while expenditure of public funds may not have been
involved under the questioned contract for the development, management and operation of the
Manila International Container Terminal, 'public interest [was] definitely involved considering the
important role [of the subject contract] . . . in the economic development of the country and the
magnitude of the financial consideration involved.' We concluded that, as a consequence, the
disclosure provision in the Constitution would constitute sufficient authority for upholding the
petitioner's standing.
Similarly, the instant petition is anchored on the right of the people to information and access to
official records, documents and papers a right guaranteed under Section 7, Article III of the
1987 Constitution. Petitioner, a former SOLICITOR general, is a Filipino citizen. Because of the
satisfaction of the two basic requisites laid down by decisional law to sustain petitioner's legal
standing, i.e. (1) the enforcement of a public right (2) espoused by a Filipino citizen, we rule that
the petition at bar should be allowed."
We rule that since the instant petition, brought by a citizen, involves the enforcement of constitutional
rights - to information and to the equitable diffusion of natural resources - matters of transcendental public
importance, the petitioner has the requisite locus standi.
Fifth issue: whether the constitutional right to information includes official information on on-
going negotiations before a final agreement.
Section 7, Article III of the Constitution explains the people's right to information on matters of public
concern in this manner:
"Sec. 7. The right of the people to information on matters of public concern shall be recognized.
Access to official records, and to documents, and papers pertaining to official acts,
transactions, or decisions, as well as to government research data used as basis for policy
development, shall be afforded the citizen, subject to such limitations as may be provided by law."
(Emphasis supplied)
The State policy of full transparency in all transactions involving public interest reinforces the people's
right to information on matters of public concern. This State policy is expressed in Section 28, Article II of
the Constitution, thus:
"Sec. 28. Subject to reasonable conditions prescribed by law, the State adopts and implements
a policy of full public disclosure of all its transactions involving public interest." (Emphasis
supplied)
These twin provisions of the Constitution seek to promote transparency in policy-making and in the
operations of the government, as well as provide the people sufficient information to exercise effectively
other constitutional rights. These twin provisions are essential to the exercise of freedom of expression. If
the government does not disclose its official acts, transactions and decisions to citizens, whatever citizens
say, even if expressed without any restraint, will be speculative and amount to nothing. These twin
provisions are also essential to hold public officials "at all times x x x accountable to the people,"
29
for
unless citizens have the proper information, they cannot hold public officials accountable for anything.
Armed with the right information, citizens can participate in public discussions leading to the formulation
of government policies and their effective implementation. An informed citizenry is essential to the
existence and proper functioning of any democracy. As explained by the Court inValmonte v. Belmonte,
J r.
30

"An essential element of these freedoms is to keep open a continuing dialogue or process of
communication between the government and the people. It is in the interest of the State that the
channels for free political discussion be maintained to the end that the government may perceive
and be responsive to the people's will. Yet, this open dialogue can be effective only to the extent
that the citizenry is informed and thus able to formulate its will intelligently. Only when the
participants in the discussion are aware of the issues and have access to information relating
thereto can such bear fruit."
PEA asserts, citing Chavez v. PCGG,
31
that in cases of on-going negotiations the right to information is
limited to "definite propositions of the government." PEA maintains the right does not include access to
"intra-agency or inter-agency recommendations or communications during the stage when common
assertions are still in the process of being formulated or are in the 'exploratory stage'."
Also, AMARI contends that petitioner cannot invoke the right at the pre-decisional stage or before the
closing of the transaction. To support its contention, AMARI cites the following discussion in the 1986
Constitutional Commission:
"Mr. Suarez. And when we say 'transactions' which should be distinguished from contracts,
agreements, or treaties or whatever, does the Gentleman refer to the steps leading to the
consummation of the contract, or does he refer to the contract itself?
Mr. Ople: The 'transactions' used here, I suppose is generic and therefore, it can cover
both steps leading to a contract and already a consummated contract, Mr. Presiding
Officer.
Mr. Suarez: This contemplates inclusion of negotiations leading to the consummation of
the transaction.
Mr. Ople: Yes, subject only to reasonable safeguards on the national interest.
Mr. Suarez: Thank you."
32
(Emphasis supplied)
AMARI argues there must first be a consummated contract before petitioner can invoke the right.
Requiring government officials to reveal their deliberations at the pre-decisional stage will degrade the
quality of decision-making in government agencies. Government officials will hesitate to express their real
sentiments during deliberations if there is immediate public dissemination of their discussions, putting
them under all kinds of pressure before they decide.
We must first distinguish between information the law on public bidding requires PEA to disclose publicly,
and information the constitutional right to information requires PEA to release to the public. Before the
consummation of the contract, PEA must, on its own and without demand from anyone, disclose to the
public matters relating to the disposition of its property. These include the size, location, technical
description and nature of the property being disposed of, the terms and conditions of the disposition, the
parties qualified to bid, the minimum price and similar information. PEA must prepare all these data and
disclose them to the public at the start of the disposition process, long before the consummation of the
contract, because the Government Auditing Code requires public bidding. If PEA fails to make this
disclosure, any citizen can demand from PEA this information at any time during the bidding process.
Information, however, on on-going evaluation or review of bids or proposals being undertaken by the
bidding or review committee is not immediately accessible under the right to information. While the
evaluation or review is still on-going, there are no "official acts, transactions, or decisions" on the bids or
proposals. However, once the committee makes its official recommendation, there arises a "definite
proposition" on the part of the government. From this moment, the public's right to information attaches,
and any citizen can access all the non-proprietary information leading to such definite proposition.
In Chavez v. PCGG,
33
the Court ruled as follows:
"Considering the intent of the framers of the Constitution, we believe that it is incumbent upon the
PCGG and its officers, as well as other government representatives, to disclose sufficient public
information on any proposed settlement they have decided to take up with the ostensible owners
and holders of ill-gotten wealth. Such information, though, must pertain to definite propositions
of the government, not necessarily to intra-agency or inter-agency recommendations or
communications during the stage when common assertions are still in the process of being
formulated or are in the "exploratory" stage. There is need, of course, to observe the same
restrictions on disclosure of information in general, as discussed earlier such as on matters
involving national security, diplomatic or foreign relations, intelligence and other classified
information." (Emphasis supplied)
Contrary to AMARI's contention, the commissioners of the 1986 Constitutional Commission understood
that the right to information "contemplates inclusion of negotiations leading to the consummation of
the transaction." Certainly, a consummated contract is not a requirement for the exercise of the right to
information. Otherwise, the people can never exercise the right if no contract is consummated, and if one
is consummated, it may be too late for the public to expose its defects.1wphi1.nt
Requiring a consummated contract will keep the public in the dark until the contract, which may be
grossly disadvantageous to the government or even illegal, becomes a fait accompli. This negates the
State policy of full transparency on matters of public concern, a situation which the framers of the
Constitution could not have intended. Such a requirement will prevent the citizenry from participating in
the public discussion of any proposedcontract, effectively truncating a basic right enshrined in the Bill of
Rights. We can allow neither an emasculation of a constitutional right, nor a retreat by the State of its
avowed "policy of full disclosure of all its transactions involving public interest."
The right covers three categories of information which are "matters of public concern," namely: (1) official
records; (2) documents and papers pertaining to official acts, transactions and decisions; and (3)
government research data used in formulating policies. The first category refers to any document that is
part of the PUBLIC RECORDS in the custody of government agencies or officials. The second category
refers to documents and papers recording, evidencing, establishing, confirming, supporting, justifying or
explaining official acts, transactions or decisions of government agencies or officials. The third category
refers to research data, whether raw, collated or processed, owned by the government and used in
formulating government policies.
The information that petitioner may access on the renegotiation of the JVA includes evaluation reports,
recommendations, legal and expert opinions, minutes of meetings, terms of reference and other
documents attached to such reports or minutes, all relating to the JVA. However, the right to information
does not compel PEA to prepare lists, abstracts, summaries and the like relating to the renegotiation of
the JVA.
34
The right only affords access to records, documents and papers, which means the opportunity
to inspect and copy them. One who exercises the right must copy the records, documents and papers at
his expense. The exercise of the right is also subject to reasonable regulations to protect the integrity of
the PUBLIC RECORDS and to minimize disruption to government operations, like rules specifying
when and how to conduct the inspection and copying.
35

The right to information, however, does not extend to matters recognized as privileged information under
the separation of powers.
36
The right does not also apply to information on military and diplomatic secrets,
information affecting national security, and information on investigations of crimes by law enforcement
agencies before the prosecution of the accused, which courts have long recognized as confidential.
37
The
right may also be subject to other limitations that Congress may impose by law.
There is no claim by PEA that the information demanded by petitioner is privileged information rooted in
the separation of powers. The information does not cover Presidential conversations, correspondences,
or discussions during closed-door Cabinet meetings which, like internal deliberations of the Supreme
Court and other collegiate courts, or executive sessions of either house of Congress,
38
are recognized as
confidential. This kind of information cannot be pried open by a co-equal branch of government. A frank
exchange of exploratory ideas and assessments, free from the glare of publicity and pressure by
interested parties, is essential to protect the independence of decision-making of those tasked to exercise
Presidential, Legislative and Judicial power.
39
This is not the situation in the instant case.
We rule, therefore, that the constitutional right to information includes official information on on-going
negotiations before a final contract. The information, however, must constitute definite propositions by
the government and should not cover recognized exceptions like privileged information, military and
diplomatic secrets and similar matters affecting national security and public order.
40
Congress has also
prescribed other limitations on the right to information in several legislations.
41

Sixth issue: whether stipulations in the Amended J VA for the transfer to AMARI of lands,
reclaimed or to be reclaimed, violate the Constitution.
The Regalian Doctrine
The ownership of lands reclaimed from foreshore and submerged areas is rooted in the Regalian doctrine
which holds that the State owns all lands and waters of the public domain. Upon the Spanish conquest of
the Philippines, ownership of all "lands, territories and possessions" in the Philippines passed to the
Spanish Crown.
42
The King, as the sovereign ruler and representative of the people, acquired and owned
all lands and territories in the Philippines except those he disposed of by grant or sale to private
individuals.
The 1935, 1973 and 1987 Constitutions adopted the Regalian doctrine substituting, however, the State, in
lieu of the King, as the owner of all lands and waters of the public domain. The Regalian doctrine is the
foundation of the time-honored principle of land ownership that "all lands that were not acquired from the
Government, either by purchase or by grant, belong to the public domain."
43
Article 339 of the Civil Code
of 1889, which is now Article 420 of the Civil Code of 1950, incorporated the Regalian doctrine.
Ownership and Disposition of Reclaimed Lands
The Spanish Law of Waters of 1866 was the first statutory law governing the ownership and disposition of
reclaimed lands in the Philippines. On May 18, 1907, the Philippine Commission enacted Act No. 1654
which provided for the LEASE , but not the sale, of reclaimed lands of the government to
corporations and individuals. Later, on November 29, 1919, the Philippine Legislature approved Act
No. 2874, the Public Land Act, which authorized the LEASE , but not the sale, of reclaimed lands of
the government to corporations and individuals. On November 7, 1936, the National Assembly
passed Commonwealth Act No. 141, also known as the Public Land Act, which authorized the lease,
but not the sale, of reclaimed lands of the government to corporations and individuals. CA No. 141
continues to this day as the general law governing the classification and disposition of lands of the public
domain.
The Spanish Law of Waters of 1866 and the Civil Code of 1889
Under the Spanish Law of Waters of 1866, the shores, bays, coves, inlets and all waters within the
maritime zone of the Spanish territory belonged to the public domain for public use.
44
The Spanish Law of
Waters of 1866 allowed the reclamation of the sea under Article 5, which provided as follows:
"Article 5. Lands reclaimed from the sea in consequence of works constructed by the State, or by
the provinces, pueblos or private persons, with proper permission, shall become the property of
the party constructing such works, unless otherwise provided by the terms of the grant of
authority."
Under the Spanish Law of Waters, land reclaimed from the sea belonged to the party undertaking the
reclamation, provided the government issued the necessary permit and did not reserve ownership of the
reclaimed land to the State.
Article 339 of the Civil Code of 1889 defined property of public dominion as follows:
"Art. 339. Property of public dominion is
1. That devoted to public use, such as roads, canals, rivers, torrents, ports and bridges
constructed by the State, riverbanks, shores, roadsteads, and that of a similar character;
2. That belonging exclusively to the State which, without being of general public use, is employed
in some public service, or in the development of the national wealth, such as walls, fortresses,
and other works for the defense of the territory, and mines, until granted to private individuals."
Property devoted to public use referred to property open for use by the public. In contrast, property
devoted to public service referred to property used for some specific public service and open only to
those authorized to use the property.
Property of public dominion referred not only to property devoted to public use, but also to property not so
used but employed to develop the national wealth. This class of property constituted property of public
dominion although employed for some economic or commercial activity to increase the national wealth.
Article 341 of the Civil Code of 1889 governed the re-classification of property of public dominion into
private property, to wit:
"Art. 341. Property of public dominion, when no longer devoted to public use or to the defense of
the territory, shall become a part of the private property of the State."
This provision, however, was not self-executing. The legislature, or the executive department pursuant to
law, must declare the property no longer needed for public use or territorial defense before the
government could LEASE or alienate the property to private parties.
45

Act No. 1654 of the Philippine Commission
On May 8, 1907, the Philippine Commission enacted Act No. 1654 which regulated the lease of reclaimed
and foreshore lands. The salient provisions of this law were as follows:
"Section 1. The control and disposition of the foreshore as defined in existing law, and
the title to all Government or public lands made or reclaimed by the Government by
dredging or filling or otherwise throughout the Philippine Islands, shall be retained by the
Government without prejudice to vested rights and without prejudice to rights conceded to the
City of Manila in the Luneta Extension.
Section 2. (a) The Secretary of the Interior shall cause all Government or public lands made or
reclaimed by the Government by dredging or filling or otherwise to be divided into lots or blocks,
with the necessary streets and alleyways located thereon, and shall cause plats and plans of
such surveys to be prepared and filed with the Bureau of Lands.
(b) Upon completion of such plats and plans the Governor-General shall give notice to the
public that such parts of the lands so made or reclaimed as are not needed for public
purposes will be LEASED for commercial and business purposes, x x x.
x x x
(e) The leases above provided for shall be disposed of to the highest and best
bidder therefore, subject to such regulations and safeguards as the Governor-General may by
executive order prescribe." (Emphasis supplied)
Act No. 1654 mandated that the government should retain title to all lands reclaimed by the
government. The Act also vested in the government control and disposition of foreshore lands. Private
parties could LEASE lands reclaimed by the government only if these lands were no longer needed for
public purpose. Act No. 1654 mandated public bidding in the LEASE of government reclaimed lands.
Act No. 1654 made government reclaimed lands sui generis in that unlike other public lands which the
government could sell to private parties, these reclaimed lands were available only for lease to private
parties.
Act No. 1654, however, did not repeal Section 5 of the Spanish Law of Waters of 1866. Act No. 1654 did
not prohibit private parties from reclaiming parts of the sea under Section 5 of the Spanish Law of Waters.
Lands reclaimed from the sea by private parties with government permission remained private lands.
Act No. 2874 of the Philippine Legislature
On November 29, 1919, the Philippine Legislature enacted Act No. 2874, the Public Land Act.
46
The
salient provisions of Act No. 2874, on reclaimed lands, were as follows:
"Sec. 6. The Governor-General, upon the recommendation of the Secretary of Agriculture
and Natural Resources, shall from time to time classify the lands of the public domain
into
(a) Alienable or disposable,
(b) Timber, and
(c) Mineral lands, x x x.
Sec. 7. For the purposes of the government and disposition of alienable or disposable public
lands, the Governor-General, upon recommendation by the Secretary of Agriculture and
Natural Resources, shall from time to time declare what lands are open to disposition or
concession under this Act."
Sec. 8. Only those lands shall be declared open to disposition or concession which have
been officially delimited or classified x x x.
x x x
Sec. 55. Any tract of land of the public domain which, being neither timber nor mineral land, shall
be classified as suitable for residential purposes or for commercial, industrial, or other
productive purposes other than agricultural purposes, and shall be open to disposition or
concession, shall be disposed of under the provisions of this chapter, and not otherwise.
Sec. 56. The lands disposable under this title shall be classified as follows:
(a) Lands reclaimed by the Government by dredging, filling, or other means;
(b) Foreshore;
(c) Marshy lands or lands covered with water bordering upon the shores or banks of
navigable lakes or rivers;
(d) Lands not included in any of the foregoing classes.
x x x.
Sec. 58. The lands comprised in classes (a), (b), and (c) of section fifty-six shall be
disposed of to private parties by lease only and not otherwise, as soon as the Governor-
General, upon recommendation by the Secretary of Agriculture and Natural Resources,
shall declare that the same are not necessary for the public service and are open to
disposition under this chapter. The lands included in class (d) may be disposed of by sale
or lease under the provisions of this Act." (Emphasis supplied)
Section 6 of Act No. 2874 authorized the Governor-General to "classify lands of the public domain into x x
x alienable or disposable"
47
lands. Section 7 of the Act empowered the Governor-General to "declare
what lands are open to disposition or concession." Section 8 of the Act limited alienable or disposable
lands only to those lands which have been "officially delimited and classified."
Section 56 of Act No. 2874 stated that lands "disposable under this title
48
shall be classified" as
government reclaimed, foreshore and marshy lands, as well as other lands. All these lands, however,
must be suitable for residential, commercial, industrial or other productive non-agricultural purposes.
These provisions vested upon the Governor-General the power to classify inalienable lands of the public
domain into disposable lands of the public domain. These provisions also empowered the Governor-
General to classify further such disposable lands of the public domain into government reclaimed,
foreshore or marshy lands of the public domain, as well as other non-agricultural lands.
Section 58 of Act No. 2874 categorically mandated that disposable lands of the public domain classified
as government reclaimed, foreshore and marshy lands "shall be disposed of to private parties by
lease only and not otherwise." The Governor-General, before allowing the lease of these lands to
private parties, must formally declare that the lands were "not necessary for the public service." Act No.
2874 reiterated the State policy to lease and not to sell government reclaimed, foreshore and marshy
lands of the public domain, a policy first enunciated in 1907 in Act No. 1654. Government reclaimed,
foreshore and marshy lands remained sui generis, as the only alienable or disposable lands of the public
domain that the government could not sell to private parties.
The rationale behind this State policy is obvious. Government reclaimed, foreshore and marshy public
lands for non-agricultural purposes retain their inherent potential as areas for public service. This is the
reason the government prohibited the sale, and only allowed the lease, of these lands to private parties.
The State always reserved these lands for some future public service.
Act No. 2874 did not authorize the reclassification of government reclaimed, foreshore and marshy lands
into other non-agricultural lands under Section 56 (d). Lands falling under Section 56 (d) were the only
lands for non-agricultural purposes the government could sell to private parties. Thus, under Act No.
2874, the government could not sell government reclaimed, foreshore and marshy lands to private
parties, unless the legislature passed a law allowing their sale.
49

Act No. 2874 did not prohibit private parties from reclaiming parts of the sea pursuant to Section 5 of the
Spanish Law of Waters of 1866. Lands reclaimed from the sea by private parties with government
permission remained private lands.
Dispositions under the 1935 Constitution
On May 14, 1935, the 1935 Constitution took effect upon its ratification by the Filipino people. The 1935
Constitution, in adopting the Regalian doctrine, declared in Section 1, Article XIII, that
"Section 1. All agricultural, timber, and mineral lands of the public domain, waters, minerals, coal,
petroleum, and other mineral oils, all forces of potential energy and other natural resources of the
Philippines belong to the State, and their disposition, exploitation, development, or utilization shall
be limited to citizens of the Philippines or to corporations or associations at least sixty per centum
of the capital of which is owned by such citizens, subject to any existing right, grant, lease, or
concession at the time of the inauguration of the Government established under this
Constitution. Natural resources, with the exception of public agricultural land, shall not be
alienated, and no license, concession, or lease for the exploitation, development, or utilization of
any of the natural resources shall be granted for a period exceeding twenty-five years, renewable
for another twenty-five years, except as to water rights for irrigation, water supply, fisheries, or
industrial uses other than the development of water power, in which cases beneficial use may be
the measure and limit of the grant." (Emphasis supplied)
The 1935 Constitution barred the alienation of all natural resources except public agricultural lands, which
were the only natural resources the State could alienate. Thus, foreshore lands, considered part of the
State's natural resources, became inalienable by constitutional fiat, available only for lease for 25 years,
renewable for another 25 years. The government could alienate foreshore lands only after these lands
were reclaimed and classified as alienable agricultural lands of the public domain. Government reclaimed
and marshy lands of the public domain, being neither timber nor mineral lands, fell under the classification
of public agricultural lands.
50
However, government reclaimed and marshy lands, although subject to
classification as disposable public agricultural lands, could only be leased and not sold to private parties
because of Act No. 2874.
The prohibition on private parties from acquiring ownership of government reclaimed and marshy lands of
the public domain was only a statutory prohibition and the legislature could therefore remove such
prohibition. The 1935 Constitution did not prohibit individuals and corporations from acquiring government
reclaimed and marshy lands of the public domain that were classified as agricultural lands under existing
public land laws. Section 2, Article XIII of the 1935 Constitution provided as follows:
"Section 2. No private corporation or association may acquire, lease, or hold public
agricultural lands in excess of one thousand and twenty four hectares, nor may any
individual acquire such lands by purchase in excess of one hundred and forty hectares, or
by lease in excess of one thousand and twenty-four hectares, or by homestead in excess of
twenty-four hectares. Lands adapted to grazing, not exceeding two thousand hectares, may be
leased to an individual, private corporation, or association." (Emphasis supplied)
Still, after the effectivity of the 1935 Constitution, the legislature did not repeal Section 58 of Act No. 2874
to open for sale to private parties government reclaimed and marshy lands of the public domain. On the
contrary, the legislature continued the long established State policy of retaining for the government title
and ownership of government reclaimed and marshy lands of the public domain.
Commonwealth Act No. 141 of the Philippine National Assembly
On November 7, 1936, the National Assembly approved Commonwealth Act No. 141, also known as the
Public Land Act, which compiled the then existing laws on lands of the public domain. CA No. 141, as
amended, remains to this day the existing general law governing the classification and disposition of
lands of the public domain other than timber and mineral lands.
51

Section 6 of CA No. 141 empowers the President to classify lands of the public domain into "alienable or
disposable"
52
lands of the public domain, which prior to such classification are inalienable and outside the
commerce of man. Section 7 of CA No. 141 authorizes the President to "declare what lands are open to
disposition or concession." Section 8 of CA No. 141 states that the government can declare open for
disposition or concession only lands that are "officially delimited and classified." Sections 6, 7 and 8 of CA
No. 141 read as follows:
"Sec. 6. The President, upon the recommendation of the Secretary of Agriculture and
Commerce, shall from time to time classify the lands of the public domain into
(a) Alienable or disposable,
(b) Timber, and
(c) Mineral lands,
and may at any time and in like manner transfer such lands from one class to another,
53
for the
purpose of their administration and disposition.
Sec. 7. For the purposes of the administration and disposition of alienable or disposable public
lands, the President, upon recommendation by the Secretary of Agriculture and
Commerce, shall from time to time declare what lands are open to disposition or
concession under this Act.
Sec. 8. Only those lands shall be declared open to disposition or concession which have
been officially delimited and classified and, when practicable, surveyed, and which have not
been reserved for public or quasi-public uses, nor appropriated by the Government, nor in
any manner become private property, nor those on which a private right authorized and
recognized by this Act or any other valid law may be claimed, or which, having been reserved or
appropriated, have ceased to be so. x x x."
Thus, before the government could alienate or dispose of lands of the public domain, the President must
first officially classify these lands as alienable or disposable, and then declare them open to disposition or
concession. There must be no law reserving these lands for public or quasi-public uses.
The salient provisions of CA No. 141, on government reclaimed, foreshore and marshy lands of the public
domain, are as follows:
"Sec. 58. Any tract of land of the public domain which, being neither timber nor mineral
land, is intended to be used for residential purposes or for commercial, industrial, or other
productive purposes other than agricultural, and is open to disposition or concession,
shall be disposed of under the provisions of this chapter and not otherwise.
Sec. 59. The lands disposable under this title shall be classified as follows:
(a) Lands reclaimed by the Government by dredging, filling, or other means;
(b) Foreshore;
(c) Marshy lands or lands covered with water bordering upon the shores or banks of
navigable lakes or rivers;
(d) Lands not included in any of the foregoing classes.
Sec. 60. Any tract of land comprised under this title may be leased or sold, as the case may be,
to any person, corporation, or association authorized to purchase or lease public lands for
agricultural purposes. x x x.
Sec. 61. The lands comprised in classes (a), (b), and (c) of section fifty-nine shall be
disposed of to private parties by lease only and not otherwise, as soon as the President,
upon recommendation by the Secretary of Agriculture, shall declare that the same are not
necessary for the public service and are open to disposition under this chapter. The lands
included in class (d) may be disposed of by sale or lease under the provisions of this Act."
(Emphasis supplied)
Section 61 of CA No. 141 readopted, after the effectivity of the 1935 Constitution, Section 58 of Act No.
2874 prohibiting the sale of government reclaimed, foreshore and marshy disposable lands of the public
domain. All these lands are intended for residential, commercial, industrial or other non-agricultural
purposes. As before, Section 61 allowed only the lease of such lands to private parties. The government
could sell to private parties only lands falling under Section 59 (d) of CA No. 141, or those lands for non-
agricultural purposes not classified as government reclaimed, foreshore and marshy disposable lands of
the public domain. Foreshore lands, however, became inalienable under the 1935 Constitution which only
allowed the lease of these lands to qualified private parties.
Section 58 of CA No. 141 expressly states that disposable lands of the public domain intended for
residential, commercial, industrial or other productive purposes other than agricultural "shall be disposed
of under the provisions of this chapter and not otherwise." Under Section 10 of CA No. 141, the term
"disposition" includes lease of the land. Any disposition of government reclaimed, foreshore and marshy
disposable lands for non-agricultural purposes must comply with Chapter IX, Title III of CA No.
141,
54
unless a subsequent law amended or repealed these provisions.
In his concurring opinion in the landmark case of Republic Real Estate Corporation v. Court of
Appeals,
55
Justice Reynato S. Puno summarized succinctly the law on this matter, as follows:
"Foreshore lands are lands of public dominion intended for public use. So too are lands reclaimed
by the government by dredging, filling, or other means. Act 1654 mandated that the control and
disposition of the foreshore and lands under water remained in the national government. Said law
allowed only the 'leasing' of reclaimed land. The Public Land Acts of 1919 and 1936 also declared
that the foreshore and lands reclaimed by the government were to be "disposed of to private
parties by lease only and not otherwise." Before leasing, however, the Governor-General, upon
recommendation of the Secretary of Agriculture and Natural Resources, had first to determine
that the land reclaimed was not necessary for the public service. This requisite must have been
met before the land could be disposed of. But even then, the foreshore and lands under water
were not to be alienated and sold to private parties. The disposition of the reclaimed land
was only by lease. The land remained property of the State." (Emphasis supplied)
As observed by Justice Puno in his concurring opinion, "Commonwealth Act No. 141 has remained in
effect at present."
The State policy prohibiting the sale to private parties of government reclaimed, foreshore and marshy
alienable lands of the public domain, first implemented in 1907 was thus reaffirmed in CA No. 141 after
the 1935 Constitution took effect. The prohibition on the sale of foreshore lands, however, became a
constitutional edict under the 1935 Constitution. Foreshore lands became inalienable as natural
resources of the State, unless reclaimed by the government and classified as agricultural lands of the
public domain, in which case they would fall under the classification of government reclaimed lands.
After the effectivity of the 1935 Constitution, government reclaimed and marshy disposable lands of the
public domain continued to be only leased and not sold to private parties.
56
These lands remained sui
generis, as the only alienable or disposable lands of the public domain the government could not sell to
private parties.
Since then and until now, the only way the government can sell to private parties government reclaimed
and marshy disposable lands of the public domain is for the legislature to pass a law authorizing such
sale. CA No. 141 does not authorize the President to reclassify government reclaimed and marshy lands
into other non-agricultural lands under Section 59 (d). Lands classified under Section 59 (d) are the only
alienable or disposable lands for non-agricultural purposes that the government could sell to private
parties.
Moreover, Section 60 of CA No. 141 expressly requires congressional authority before lands under
Section 59 that the government previously transferred to government units or entities could be sold to
private parties. Section 60 of CA No. 141 declares that
"Sec. 60. x x x The area so leased or sold shall be such as shall, in the judgment of the Secretary
of Agriculture and Natural Resources, be reasonably necessary for the purposes for which such
sale or lease is requested, and shall not exceed one hundred and forty-four hectares: Provided,
however, That this limitation shall not apply to grants, donations, or transfers made to a province,
municipality or branch or subdivision of the Government for the purposes deemed by said entities
conducive to the public interest;but the land so granted, donated, or transferred to a
province, municipality or branch or subdivision of the Government shall not be alienated,
encumbered, or otherwise disposed of in a manner affecting its title, except when
authorized by Congress: x x x." (Emphasis supplied)
The congressional authority required in Section 60 of CA No. 141 mirrors the legislative authority required
in Section 56 of Act No. 2874.
One reason for the congressional authority is that Section 60 of CA No. 141 exempted government units
and entities from the maximum area of public lands that could be acquired from the State. These
government units and entities should not just turn around and sell these lands to private parties in
violation of constitutional or statutory limitations. Otherwise, the transfer of lands for non-agricultural
purposes to government units and entities could be used to circumvent constitutional limitations on
ownership of alienable or disposable lands of the public domain. In the same manner, such transfers
could also be used to evade the statutory prohibition in CA No. 141 on the sale of government reclaimed
and marshy lands of the public domain to private parties. Section 60 of CA No. 141 constitutes by
operation of law a lien on these lands.
57

In case of sale or lease of disposable lands of the public domain falling under Section 59 of CA No. 141,
Sections 63 and 67 require a public bidding. Sections 63 and 67 of CA No. 141 provide as follows:
"Sec. 63. Whenever it is decided that lands covered by this chapter are not needed for public
purposes, the Director of Lands shall ask the Secretary of Agriculture and Commerce (now the
Secretary of Natural Resources) for authority to dispose of the same. Upon receipt of such
authority, the Director of Lands shall give notice by public advertisement in the same manner as
in the case of leases or sales of agricultural public land, x x x.
Sec. 67. The lease or sale shall be made by oral bidding; and adjudication shall be made to
the highest bidder. x x x." (Emphasis supplied)
Thus, CA No. 141 mandates the Government to put to public auction all leases or sales of alienable or
disposable lands of the public domain.
58

Like Act No. 1654 and Act No. 2874 before it, CA No. 141 did not repeal Section 5 of the Spanish Law of
Waters of 1866. Private parties could still reclaim portions of the sea with government permission.
However, the reclaimed land could become private land only if classified as alienable agricultural
land of the public domain open to disposition under CA No. 141. The 1935 Constitution prohibited the
alienation of all natural resources except public agricultural lands.
The Civil Code of 1950
The Civil Code of 1950 readopted substantially the definition of property of public dominion found in the
Civil Code of 1889. Articles 420 and 422 of the Civil Code of 1950 state that
"Art. 420. The following things are property of public dominion:
(1) Those intended for public use, such as roads, canals, rivers, torrents, ports and bridges
constructed by the State, banks, shores, roadsteads, and others of similar character;
(2) Those which belong to the State, without being for public use, and are intended for some
public service or for the development of the national wealth.
x x x.
Art. 422. Property of public dominion, when no longer intended for public use or for public service,
shall form part of the patrimonial property of the State."
Again, the government must formally declare that the property of public dominion is no longer needed for
public use or public service, before the same could be classified as patrimonial property of the State.
59
In
the case of government reclaimed and marshy lands of the public domain, the declaration of their being
disposable, as well as the manner of their disposition, is governed by the applicable provisions of CA No.
141.
Like the Civil Code of 1889, the Civil Code of 1950 included as property of public dominion those
properties of the State which, without being for public use, are intended for public service or the
"development of the national wealth." Thus, government reclaimed and marshy lands of the State,
even if not employed for public use or public service, if developed to enhance the national wealth, are
classified as property of public dominion.
Dispositions under the 1973 Constitution
The 1973 Constitution, which took effect on January 17, 1973, likewise adopted the Regalian doctrine.
Section 8, Article XIV of the 1973 Constitution stated that
"Sec. 8. All lands of the public domain, waters, minerals, coal, petroleum and other mineral oils,
all forces of potential energy, fisheries, wildlife, and other natural resources of the Philippines
belong to the State. With the exception of agricultural, industrial or commercial, residential,
and resettlement lands of the public domain, natural resources shall not be alienated, and
no license, concession, or lease for the exploration, development, exploitation, or utilization of
any of the natural resources shall be granted for a period exceeding twenty-five years, renewable
for not more than twenty-five years, except as to water rights for irrigation, water supply, fisheries,
or industrial uses other than the development of water power, in which cases, beneficial use may
be the measure and the limit of the grant." (Emphasis supplied)
The 1973 Constitution prohibited the alienation of all natural resources with the exception of "agricultural,
industrial or commercial, residential, and resettlement lands of the public domain." In contrast, the 1935
Constitution barred the alienation of all natural resources except "public agricultural lands." However, the
term "public agricultural lands" in the 1935 Constitution encompassed industrial, commercial, residential
and resettlement lands of the public domain.
60
If the land of public domain were neither timber nor mineral
land, it would fall under the classification of agricultural land of the public domain. Both the 1935 and
1973 Constitutions, therefore, prohibited the alienation of all natural resources except agricultural
lands of the public domain.
The 1973 Constitution, however, limited the alienation of lands of the public domain to individuals who
were citizens of the Philippines. Private corporations, even if wholly owned by Philippine citizens, were no
longer allowed to acquire alienable lands of the public domain unlike in the 1935 Constitution. Section 11,
Article XIV of the 1973 Constitution declared that
"Sec. 11. The Batasang Pambansa, taking into account conservation, ecological, and
development requirements of the natural resources, shall determine by law the size of land of the
public domain which may be developed, held or acquired by, or leased to, any qualified individual,
corporation, or association, and the conditions therefor. No private corporation or association
may hold alienable lands of the public domain except by lease not to exceed one thousand
hectares in area nor may any citizen hold such lands by lease in excess of five hundred hectares
or acquire by purchase, homestead or grant, in excess of twenty-four hectares. No private
corporation or association may hold by lease, concession, license or permit, timber or forest lands
and other timber or forest resources in excess of one hundred thousand hectares. However, such
area may be increased by the Batasang Pambansa upon recommendation of the National
Economic and Development Authority." (Emphasis supplied)
Thus, under the 1973 Constitution, private corporations could hold alienable lands of the public domain
only through lease. Only individuals could now acquire alienable lands of the public domain, and private
corporations became absolutely barred from acquiring any kind of alienable land of the public
domain. The constitutional ban extended to all kinds of alienable lands of the public domain, while the
statutory ban under CA No. 141 applied only to government reclaimed, foreshore and marshy alienable
lands of the public domain.
PD No. 1084 Creating the Public Estates Authority
On February 4, 1977, then President Ferdinand Marcos issued Presidential Decree No. 1084 creating
PEA, a wholly government owned and controlled corporation with a special charter. Sections 4 and 8 of
PD No. 1084, vests PEA with the following purposes and powers:
"Sec. 4. Purpose. The Authority is hereby created for the following purposes:
(a) To reclaim land, including foreshore and submerged areas, by dredging, filling or other
means, or to acquire reclaimed land;
(b) To develop, improve, acquire, administer, deal in, subdivide, dispose, lease and sell any and
all kinds of lands, buildings, estates and other forms of real property, owned, managed,
controlled and/or operated by the government;
(c) To provide for, operate or administer such service as may be necessary for the efficient,
economical and beneficial utilization of the above properties.
Sec. 5. Powers and functions of the Authority. The Authority shall, in carrying out the purposes for
which it is created, have the following powers and functions:
(a)To prescribe its by-laws.
x x x
(i) To hold lands of the public domain in excess of the area permitted to private corporations by
statute.
(j) To reclaim lands and to construct work across, or otherwise, any stream, watercourse, canal,
ditch, flume x x x.
x x x
(o) To perform such acts and exercise such functions as may be necessary for the attainment of
the purposes and objectives herein specified." (Emphasis supplied)
PD No. 1084 authorizes PEA to reclaim both foreshore and submerged areas of the public domain.
Foreshore areas are those covered and uncovered by the ebb and flow of the tide.
61
Submerged areas
are those permanently under water regardless of the ebb and flow of the tide.
62
Foreshore and
submerged areas indisputably belong to the public domain
63
and are inalienable unless reclaimed,
classified as alienable lands open to disposition, and further declared no longer needed for public service.
The ban in the 1973 Constitution on private corporations from acquiring alienable lands of the public
domain did not apply to PEA since it was then, and until today, a fully owned government corporation.
The constitutional ban applied then, as it still applies now, only to "private corporations and associations."
PD No. 1084 expressly empowers PEA "to hold lands of the public domain" even "in excess of the
area permitted to private corporations by statute." Thus, PEA can hold title to private lands, as well as
title to lands of the public domain.
In order for PEA to sell its reclaimed foreshore and submerged alienable lands of the public domain, there
must be legislative authority empowering PEA to sell these lands. This legislative authority is necessary in
view of Section 60 of CA No.141, which states
"Sec. 60. x x x; but the land so granted, donated or transferred to a province, municipality, or
branch or subdivision of the Government shall not be alienated, encumbered or otherwise
disposed of in a manner affecting its title, except when authorized by Congress; x x x."
(Emphasis supplied)
Without such legislative authority, PEA could not sell but only lease its reclaimed foreshore and
submerged alienable lands of the public domain. Nevertheless, any legislative authority granted to PEA to
sell its reclaimed alienable lands of the public domain would be subject to the constitutional ban on
private corporations from acquiring alienable lands of the public domain. Hence, such legislative authority
could only benefit private individuals.
Dispositions under the 1987 Constitution
The 1987 Constitution, like the 1935 and 1973 Constitutions before it, has adopted the Regalian doctrine.
The 1987 Constitution declares that all natural resources are "owned by the State," and except for
alienable agricultural lands of the public domain, natural resources cannot be alienated. Sections 2 and 3,
Article XII of the 1987 Constitution state that
"Section 2. All lands of the public domain, waters, minerals, coal, petroleum and other mineral
oils, all forces of potential energy, fisheries, forests or timber, wildlife, flora and fauna, and other
natural resources are owned by the State. With the exception of agricultural lands, all other
natural resources shall not be alienated. The exploration, development, and utilization of
natural resources shall be under the full control and supervision of the State. x x x.
Section 3. Lands of the public domain are classified into agricultural, forest or timber, mineral
lands, and national parks. Agricultural lands of the public domain may be further classified by law
according to the uses which they may be devoted. Alienable lands of the public domain shall
be limited to agricultural lands. Private corporations or associations may not hold such
alienable lands of the public domain except by lease, for a period not exceeding twenty-
five years, renewable for not more than twenty-five years, and not to exceed one thousand
hectares in area. Citizens of the Philippines may lease not more than five hundred hectares, or
acquire not more than twelve hectares thereof by purchase, homestead, or grant.
Taking into account the requirements of conservation, ecology, and development, and subject to
the requirements of agrarian reform, the Congress shall determine, by law, the size of lands of
the public domain which may be acquired, developed, held, or leased and the conditions
therefor." (Emphasis supplied)
The 1987 Constitution continues the State policy in the 1973 Constitution banning private corporations
fromacquiring any kind of alienable land of the public domain. Like the 1973 Constitution, the 1987
Constitution allows private corporations to hold alienable lands of the public domain only through lease.
As in the 1935 and 1973 Constitutions, the general law governing the lease to private corporations of
reclaimed, foreshore and marshy alienable lands of the public domain is still CA No. 141.
The Rationale behind the Constitutional Ban
The rationale behind the constitutional ban on corporations from acquiring, except through lease,
alienable lands of the public domain is not well understood. During the deliberations of the 1986
Constitutional Commission, the commissioners probed the rationale behind this ban, thus:
"FR. BERNAS: Mr. Vice-President, my questions have reference to page 3, line 5 which says:
`No private corporation or association may hold alienable lands of the public domain except by
lease, not to exceed one thousand hectares in area.'
If we recall, this provision did not exist under the 1935 Constitution, but this was introduced in the
1973 Constitution. In effect, it prohibits private corporations from acquiring alienable public
lands. But it has not been very clear in jurisprudence what the reason for this is. In some of
the cases decided in 1982 and 1983, it was indicated that the purpose of this is to prevent
large landholdings. Is that the intent of this provision?
MR. VILLEGAS: I think that is the spirit of the provision.
FR. BERNAS: In existing decisions involving the Iglesia ni Cristo, there were instances where the
Iglesia ni Cristo was not allowed to acquire a mere 313-square meter land where a chapel stood
because the Supreme Court said it would be in violation of this." (Emphasis supplied)
In Ayog v. Cusi,
64
the Court explained the rationale behind this constitutional ban in this way:
"Indeed, one purpose of the constitutional prohibition against purchases of public agricultural
lands by private corporations is to equitably diffuse land ownership or to encourage 'owner-
cultivatorship and the economic family-size farm' and to prevent a recurrence of cases like the
instant case. Huge landholdings by corporations or private persons had spawned social unrest."
However, if the constitutional intent is to prevent huge landholdings, the Constitution could have simply
limited the size of alienable lands of the public domain that corporations could acquire. The Constitution
could have followed the limitations on individuals, who could acquire not more than 24 hectares of
alienable lands of the public domain under the 1973 Constitution, and not more than 12 hectares under
the 1987 Constitution.
If the constitutional intent is to encourage economic family-size farms, placing the land in the name of a
corporation would be more effective in preventing the break-up of farmlands. If the farmland is registered
in the name of a corporation, upon the death of the owner, his heirs would inherit shares in the
corporation instead of subdivided parcels of the farmland. This would prevent the continuing break-up of
farmlands into smaller and smaller plots from one generation to the next.
In actual practice, the constitutional ban strengthens the constitutional limitation on individuals from
acquiring more than the allowed area of alienable lands of the public domain. Without the constitutional
ban, individuals who already acquired the maximum area of alienable lands of the public domain could
easily set up corporations to acquire more alienable public lands. An individual could own as many
corporations as his means would allow him. An individual could even hide his ownership of a corporation
by putting his nominees as stockholders of the corporation. The corporation is a convenient vehicle to
circumvent the constitutional limitation on acquisition by individuals of alienable lands of the public
domain.
The constitutional intent, under the 1973 and 1987 Constitutions, is to transfer ownership of only a limited
area of alienable land of the public domain to a qualified individual. This constitutional intent is
safeguarded by the provision prohibiting corporations from acquiring alienable lands of the public domain,
since the vehicle to circumvent the constitutional intent is removed. The available alienable public lands
are gradually decreasing in the face of an ever-growing population. The most effective way to insure
faithful adherence to this constitutional intent is to grant or sell alienable lands of the public domain only to
individuals. This, it would seem, is the practical benefit arising from the constitutional ban.
The Amended J oint Venture Agreement
The subject matter of the Amended JVA, as stated in its second Whereas clause, consists of three
properties, namely:
1. "[T]hree partially reclaimed and substantially eroded islands along Emilio Aguinaldo Boulevard
in Paranaque and Las Pinas, Metro Manila, with a combined titled area of 1,578,441 square
meters;"
2. "[A]nother area of 2,421,559 square meters contiguous to the three islands;" and
3. "[A]t AMARI's option as approved by PEA, an additional 350 hectares more or less to
regularize the configuration of the reclaimed area."
65

PEA confirms that the Amended JVA involves "the development of the Freedom Islands and further
reclamation of about 250 hectares x x x," plus an option "granted to AMARI to subsequently reclaim
another 350 hectares x x x."
66

In short, the Amended JVA covers a reclamation area of 750 hectares. Only 157.84 hectares of the 750-
hectare reclamation project have been reclaimed, and the rest of the 592.15 hectares are still
submerged areas forming part of Manila Bay.
Under the Amended JVA, AMARI will reimburse PEA the sum of P1,894,129,200.00 for PEA's "actual
cost" in partially reclaiming the Freedom Islands. AMARI will also complete, at its own expense, the
reclamation of the Freedom Islands. AMARI will further shoulder all the reclamation costs of all the other
areas, totaling 592.15 hectares, still to be reclaimed. AMARI and PEA will share, in the proportion of 70
percent and 30 percent, respectively, the total net usable area which is defined in the Amended JVA as
the total reclaimed area less 30 percent earmarked for common areas. Title to AMARI's share in the net
usable area, totaling 367.5 hectares, will be issued in the name of AMARI. Section 5.2 (c) of the
Amended JVA provides that
"x x x, PEA shall have the duty to execute without delay the necessary deed of transfer or
conveyance of the title pertaining to AMARI's Land share based on the Land Allocation
Plan. PEA, when requested in writing by AMARI, shall then cause the issuance and
delivery of the proper certificates of title covering AMARI's Land Share in the name of
AMARI, x x x; provided, that if more than seventy percent (70%) of the titled area at any given
time pertains to AMARI, PEA shall deliver to AMARI only seventy percent (70%) of the titles
pertaining to AMARI, until such time when a corresponding proportionate area of additional land
pertaining to PEA has been titled." (Emphasis supplied)
Indisputably, under the Amended J VA AMARI will acquire and own a maximum of 367.5 hectares
of reclaimed land which will be titled in its name.
To implement the Amended JVA, PEA delegated to the unincorporated PEA-AMARI joint venture PEA's
statutory authority, rights and privileges to reclaim foreshore and submerged areas in Manila Bay. Section
3.2.a of the Amended JVA states that
"PEA hereby contributes to the joint venture its rights and privileges to perform Rawland
Reclamation and Horizontal Development as well as own the Reclamation Area, thereby granting
the Joint Venture the full and exclusive right, authority and privilege to undertake the Project in
accordance with the Master Development Plan."
The Amended JVA is the product of a renegotiation of the original JVA dated April 25, 1995 and its
supplemental agreement dated August 9, 1995.
The Threshold Issue
The threshold issue is whether AMARI, a private corporation, can acquire and own under the Amended
JVA 367.5 hectares of reclaimed foreshore and submerged areas in Manila Bay in view of Sections 2 and
3, Article XII of the 1987 Constitution which state that:
"Section 2. All lands of the public domain, waters, minerals, coal, petroleum, and other mineral
oils, all forces of potential energy, fisheries, forests or timber, wildlife, flora and fauna, and other
natural resources are owned by the State. With the exception of agricultural lands, all other
natural resources shall not be alienated. x x x.
x x x
Section 3. x x x Alienable lands of the public domain shall be limited to agricultural lands. Private
corporations or associations may not hold such alienable lands of the public domain
except by lease, x x x."(Emphasis supplied)
Classification of Reclaimed Foreshore and Submerged Areas
PEA readily concedes that lands reclaimed from foreshore or submerged areas of Manila Bay are
alienable or disposable lands of the public domain. In its Memorandum,
67
PEA admits that
"Under the Public Land Act (CA 141, as amended), reclaimed lands are classified as alienable
and disposable lands of the public domain:
'Sec. 59. The lands disposable under this title shall be classified as follows:
(a) Lands reclaimed by the government by dredging, filling, or other means;
x x x.'" (Emphasis supplied)
Likewise, the Legal Task Force
68
constituted under Presidential Administrative Order No. 365 admitted in
its Report and Recommendation to then President Fidel V. Ramos, "[R]eclaimed lands are classified
as alienable and disposable lands of the public domain."
69
The Legal Task Force concluded that
"D. Conclusion
Reclaimed lands are lands of the public domain. However, by statutory authority, the rights of
ownership and disposition over reclaimed lands have been transferred to PEA, by virtue of which
PEA, as owner, may validly convey the same to any qualified person without violating the
Constitution or any statute.
The constitutional provision prohibiting private corporations from holding public land, except by
lease (Sec. 3, Art. XVII,
70
1987 Constitution), does not apply to reclaimed lands whose ownership
has passed on to PEA by statutory grant."
Under Section 2, Article XII of the 1987 Constitution, the foreshore and submerged areas of Manila Bay
are part of the "lands of the public domain, waters x x x and other natural resources" and consequently
"owned by the State." As such, foreshore and submerged areas "shall not be alienated," unless they are
classified as "agricultural lands" of the public domain. The mere reclamation of these areas by PEA does
not convert these inalienable natural resources of the State into alienable or disposable lands of the
public domain. There must be a law or presidential proclamation officially classifying these reclaimed
lands as alienable or disposable and open to disposition or concession. Moreover, these reclaimed lands
cannot be classified as alienable or disposable if the law has reserved them for some public or quasi-
public use.
71

Section 8 of CA No. 141 provides that "only those lands shall be declared open to disposition or
concession which have been officially delimited and classified."
72
The President has the authority to
classify inalienable lands of the public domain into alienable or disposable lands of the public domain,
pursuant to Section 6 of CA No. 141. In Laurel vs. Garcia,
73
the Executive Department attempted to sell
the Roppongi property in Tokyo, Japan, which was acquired by the Philippine Government for use as the
Chancery of the Philippine Embassy. Although the Chancery had transferred to another location thirteen
years earlier, the Court still ruled that, under Article 422
74
of the Civil Code, a property of public dominion
retains such character until formally declared otherwise. The Court ruled that
"The fact that the Roppongi site has not been used for a long time for actual Embassy service
does not automatically convert it to patrimonial property. Any such conversion happens only if the
property is withdrawn from public use (Cebu Oxygen and Acetylene Co. v. Bercilles, 66 SCRA
481 [1975]. A property continues to be part of the public domain, not available for private
appropriation or ownership 'until there is a formal declaration on the part of the
government to withdraw it from being such' (Ignacio v. Director of Lands, 108 Phil. 335
[1960]." (Emphasis supplied)
PD No. 1085, issued on February 4, 1977, authorized the issuance of special land patents for lands
reclaimed by PEA from the foreshore or submerged areas of Manila Bay. On January 19, 1988 then
President Corazon C. Aquino issued Special Patent No. 3517 in the name of PEA for the 157.84 hectares
comprising the partially reclaimed Freedom Islands. Subsequently, on April 9, 1999 the Register of Deeds
of the Municipality of Paranaque issued TCT Nos. 7309, 7311 and 7312 in the name of PEA pursuant to
Section 103 of PD No. 1529 authorizing the issuance of certificates of title corresponding to land patents.
To this day, these certificates of title are still in the name of PEA.
PD No. 1085, coupled with President Aquino's actual issuance of a special patent covering the Freedom
Islands, is equivalent to an official proclamation classifying the Freedom Islands as alienable or
disposable lands of the public domain. PD No. 1085 and President Aquino's issuance of a land patent
also constitute a declaration that the Freedom Islands are no longer needed for public service. The
Freedom Islands are thus alienable or disposable lands of the public domain, open to disposition
or concession to qualified parties.
At the time then President Aquino issued Special Patent No. 3517, PEA had already reclaimed the
Freedom Islands although subsequently there were partial erosions on some areas. The government had
also completed the necessary surveys on these islands. Thus, the Freedom Islands were no longer part
of Manila Bay but part of the land mass. Section 3, Article XII of the 1987 Constitution classifies lands of
the public domain into "agricultural, forest or timber, mineral lands, and national parks." Being neither
timber, mineral, nor national park lands, the reclaimed Freedom Islands necessarily fall under the
classification of agricultural lands of the public domain. Under the 1987 Constitution, agricultural lands of
the public domain are the only natural resources that the State may alienate to qualified private parties.
All other natural resources, such as the seas or bays, are "waters x x x owned by the State" forming part
of the public domain, and are inalienable pursuant to Section 2, Article XII of the 1987 Constitution.
AMARI claims that the Freedom Islands are private lands because CDCP, then a private corporation,
reclaimed the islands under a contract dated November 20, 1973 with the Commissioner of Public
Highways. AMARI, citing Article 5 of the Spanish Law of Waters of 1866, argues that "if the ownership of
reclaimed lands may be given to the party constructing the works, then it cannot be said that reclaimed
lands are lands of the public domain which the State may not alienate."
75
Article 5 of the Spanish Law of
Waters reads as follows:
"Article 5. Lands reclaimed from the sea in consequence of works constructed by the State, or by
the provinces, pueblos or private persons, with proper permission, shall become the property of
the party constructing such works, unless otherwise provided by the terms of the grant of
authority." (Emphasis supplied)
Under Article 5 of the Spanish Law of Waters of 1866, private parties could reclaim from the sea only with
"proper permission" from the State. Private parties could own the reclaimed land only if not "otherwise
provided by the terms of the grant of authority." This clearly meant that no one could reclaim from the sea
without permission from the State because the sea is property of public dominion. It also meant that the
State could grant or withhold ownership of the reclaimed land because any reclaimed land, like the sea
from which it emerged, belonged to the State. Thus, a private person reclaiming from the sea without
permission from the State could not acquire ownership of the reclaimed land which would remain property
of public dominion like the sea it replaced.
76
Article 5 of the Spanish Law of Waters of 1866 adopted the
time-honored principle of land ownership that "all lands that were not acquired from the government,
either by purchase or by grant, belong to the public domain."
77

Article 5 of the Spanish Law of Waters must be read together with laws subsequently enacted on the
disposition of public lands. In particular, CA No. 141 requires that lands of the public domain must first be
classified as alienable or disposable before the government can alienate them. These lands must not be
reserved for public or quasi-public purposes.
78
Moreover, the contract between CDCP and the
government was executed after the effectivity of the 1973 Constitution which barred private corporations
from acquiring any kind of alienable land of the public domain. This contract could not have converted the
Freedom Islands into private lands of a private corporation.
Presidential Decree No. 3-A, issued on January 11, 1973, revoked all laws authorizing the reclamation of
areas under water and revested solely in the National Government the power to reclaim lands. Section 1
of PD No. 3-A declared that
"The provisions of any law to the contrary notwithstanding, the reclamation of areas under
water, whether foreshore or inland, shall be limited to the National Government or any person
authorized by it under a proper contract. (Emphasis supplied)
x x x."
PD No. 3-A repealed Section 5 of the Spanish Law of Waters of 1866 because reclamation of areas
under water could now be undertaken only by the National Government or by a person contracted by the
National Government. Private parties may reclaim from the sea only under a contract with the National
Government, and no longer by grant or permission as provided in Section 5 of the Spanish Law of Waters
of 1866.
Executive Order No. 525, issued on February 14, 1979, designated PEA as the National Government's
implementing arm to undertake "all reclamation projects of the government," which "shall be undertaken
by the PEA or through a proper contract executed by it with any person or entity." Under such
contract, a private party receives compensation for reclamation services rendered to PEA. Payment to the
contractor may be in cash, or in kind consisting of portions of the reclaimed land, subject to the
constitutional ban on private corporations from acquiring alienable lands of the public domain. The
reclaimed land can be used as payment in kind only if the reclaimed land is first classified as alienable or
disposable land open to disposition, and then declared no longer needed for public service.
The Amended JVA covers not only the Freedom Islands, but also an additional 592.15 hectares which
are still submerged and forming part of Manila Bay. There is no legislative or Presidential act
classifying these submerged areas as alienable or disposable lands of the public domain open to
disposition. These submerged areas are not covered by any patent or certificate of title. There can be no
dispute that these submerged areas form part of the public domain, and in their present state
are inalienable and outside the commerce of man. Until reclaimed from the sea, these submerged
areas are, under the Constitution, "waters x x x owned by the State," forming part of the public domain
and consequently inalienable. Only when actually reclaimed from the sea can these submerged areas be
classified as public agricultural lands, which under the Constitution are the only natural resources that the
State may alienate. Once reclaimed and transformed into public agricultural lands, the government may
then officially classify these lands as alienable or disposable lands open to disposition. Thereafter, the
government may declare these lands no longer needed for public service. Only then can these reclaimed
lands be considered alienable or disposable lands of the public domain and within the commerce of man.
The classification of PEA's reclaimed foreshore and submerged lands into alienable or disposable lands
open to disposition is necessary because PEA is tasked under its charter to undertake public services
that require the use of lands of the public domain. Under Section 5 of PD No. 1084, the functions of PEA
include the following: "[T]o own or operate railroads, tramways and other kinds of land transportation, x x
x; [T]o construct, maintain and operate such systems of sanitary sewers as may be necessary; [T]o
construct, maintain and operate such storm drains as may be necessary." PEA is empowered to issue
"rules and regulations as may be necessary for the proper use by private parties of any or all of the
highways, roads, utilities, buildings and/or any of its properties and to impose or collect fees or tolls
for their use." Thus, part of the reclaimed foreshore and submerged lands held by the PEA would actually
be needed for public use or service since many of the functions imposed on PEA by its charter constitute
essential public services.
Moreover, Section 1 of Executive Order No. 525 provides that PEA "shall be primarily responsible for
integrating, directing, and coordinating all reclamation projects for and on behalf of the National
Government." The same section also states that "[A]ll reclamation projects shall be approved by the
President upon recommendation of the PEA, and shall be undertaken by the PEA or through a proper
contract executed by it with any person or entity; x x x." Thus, under EO No. 525, in relation to PD No. 3-
A and PD No.1084, PEA became the primary implementing agency of the National Government to
reclaim foreshore and submerged lands of the public domain. EO No. 525 recognized PEA as the
government entity "to undertake the reclamation of lands and ensure their maximum utilization
in promoting public welfare and interests."
79
Since large portions of these reclaimed lands would
obviously be needed for public service, there must be a formal declaration segregating reclaimed lands
no longer needed for public service from those still needed for public service.1wphi1.nt
Section 3 of EO No. 525, by declaring that all lands reclaimed by PEA "shall belong to or be owned by the
PEA," could not automatically operate to classify inalienable lands into alienable or disposable lands of
the public domain. Otherwise, reclaimed foreshore and submerged lands of the public domain would
automatically become alienable once reclaimed by PEA, whether or not classified as alienable or
disposable.
The Revised Administrative Code of 1987, a later law than either PD No. 1084 or EO No. 525, vests in
the Department of Environment and Natural Resources ("DENR" for brevity) the following powers and
functions:
"Sec. 4. Powers and Functions. The Department shall:
(1) x x x
x x x
(4) Exercise supervision and control over forest lands, alienable and disposable public
lands, mineral resources and, in the process of exercising such control, impose appropriate
taxes, fees, charges, rentals and any such form of levy and collect such revenues for the
exploration, development, utilization or gathering of such resources;
x x x
(14) Promulgate rules, regulations and guidelines on the issuance of licenses, permits,
concessions, LEASE AGREEMENTS and such other privileges concerning the
development, exploration and utilization of the country's marine, freshwater, and brackish
water and over all aquatic resources of the country and shall continue to oversee,
supervise and police our natural resources; cancel or cause to cancel such privileges upon
failure, non-compliance or violations of any regulation, order, and for all other causes which are in
furtherance of the conservation of natural resources and supportive of the national interest;
(15) Exercise exclusive jurisdiction on the management and disposition of all lands of the
public domain and serve as the sole agency responsible for classification, sub-
classification, surveying and titling of lands in consultation with appropriate
agencies."
80
(Emphasis supplied)
As manager, conservator and overseer of the natural resources of the State, DENR exercises
"supervision and control over alienable and disposable public lands." DENR also exercises "exclusive
jurisdiction on the management and disposition of all lands of the public domain." Thus, DENR decides
whether areas under water, like foreshore or submerged areas of Manila Bay, should be reclaimed or not.
This means that PEA needs authorization from DENR before PEA can undertake reclamation projects in
Manila Bay, or in any part of the country.
DENR also exercises exclusive jurisdiction over the disposition of all lands of the public domain. Hence,
DENR decides whether reclaimed lands of PEA should be classified as alienable under Sections 6
81
and
7
82
of CA No. 141. Once DENR decides that the reclaimed lands should be so classified, it then
recommends to the President the issuance of a proclamation classifying the lands as alienable or
disposable lands of the public domain open to disposition. We note that then DENR Secretary Fulgencio
S. Factoran, Jr. countersigned Special Patent No. 3517 in compliance with the Revised Administrative
Code and Sections 6 and 7 of CA No. 141.
In short, DENR is vested with the power to authorize the reclamation of areas under water, while PEA is
vested with the power to undertake the physical reclamation of areas under water, whether directly or
through private contractors. DENR is also empowered to classify lands of the public domain into alienable
or disposable lands subject to the approval of the President. On the other hand, PEA is tasked to
develop, sell or LEASE the reclaimed alienable lands of the public domain.
Clearly, the mere physical act of reclamation by PEA of foreshore or submerged areas does not make the
reclaimed lands alienable or disposable lands of the public domain, much less patrimonial lands of PEA.
Likewise, the mere transfer by the National Government of lands of the public domain to PEA does not
make the lands alienable or disposable lands of the public domain, much less patrimonial lands of PEA.
Absent two official acts a classification that these lands are alienable or disposable and open to
disposition and a declaration that these lands are not needed for public service, lands reclaimed by PEA
remain inalienable lands of the public domain. Only such an official classification and formal declaration
can convert reclaimed lands into alienable or disposable lands of the public domain, open to disposition
under the Constitution, Title I and Title III
83
of CA No. 141 and other applicable laws.
84

PEA's Authority to Sell Reclaimed Lands
PEA, like the Legal Task Force, argues that as alienable or disposable lands of the public domain, the
reclaimed lands shall be disposed of in accordance with CA No. 141, the Public Land Act. PEA, citing
Section 60 of CA No. 141, admits that reclaimed lands transferred to a branch or subdivision of the
government "shall not be alienated, encumbered, or otherwise disposed of in a manner affecting its
title, except when authorized by Congress: x x x."
85
(Emphasis by PEA)
In Laurel vs. Garcia,
86
the Court cited Section 48 of the Revised Administrative Code of 1987, which
states that
"Sec. 48. Official Authorized to Convey Real Property. Whenever real property of the
Government is authorized by law to be conveyed, the deed of conveyance shall be executed in
behalf of the government by the following: x x x."
Thus, the Court concluded that a law is needed to convey any real property belonging to the Government.
The Court declared that -
"It is not for the President to convey real property of the government on his or her own sole
will. Any such conveyance must be authorized and approved by a law enacted by the
Congress. It requires executive and legislative concurrence." (Emphasis supplied)
PEA contends that PD No. 1085 and EO No. 525 constitute the legislative authority allowing PEA to sell
its reclaimed lands. PD No. 1085, issued on February 4, 1977, provides that
"The land reclaimed in the foreshore and OFFSHORE area of Manila Bay pursuant to the
contract for the reclamation and construction of the Manila-Cavite Coastal Road Project between
the Republic of the Philippines and the Construction and Development Corporation of the
Philippines dated November 20, 1973 and/or any other contract or reclamation covering the same
area is hereby transferred, conveyed and assigned to the ownership and administration of
the Public Estates Authority established pursuant to PD No. 1084; Provided, however, That the
rights and interests of the Construction and Development Corporation of the Philippines pursuant
to the aforesaid contract shall be recognized and respected.
Henceforth, the Public Estates Authority shall exercise the rights and assume the obligations of
the Republic of the Philippines (Department of Public Highways) arising from, or incident to, the
aforesaid contract between the Republic of the Philippines and the Construction and
Development Corporation of the Philippines.
In consideration of the foregoing transfer and assignment, the Public Estates Authority shall issue
in favor of the Republic of the Philippines the corresponding shares of stock in said entity with an
issued value of said shares of stock (which) shall be deemed fully paid and non-assessable.
The Secretary of Public Highways and the General Manager of the Public Estates Authority shall
execute such contracts or agreements, including appropriate agreements with the Construction
and Development Corporation of the Philippines, as may be necessary to implement the above.
Special land patent/patents shall be issued by the Secretary of Natural Resources in favor
of the Public Estates Authority without prejudice to the subsequent transfer to the
contractor or his assignees of such portion or portions of the land reclaimed or to be
reclaimed as provided for in the above-mentioned contract. On the basis of such patents,
the Land Registration Commission shall issue the corresponding certificate of title."
(Emphasis supplied)
On the other hand, Section 3 of EO No. 525, issued on February 14, 1979, provides that -
"Sec. 3. All lands reclaimed by PEA shall belong to or be owned by the PEA which shall be
responsible for its administration, development, utilization or disposition in accordance with the
provisions of Presidential Decree No. 1084. Any and all income that the PEA may derive from the
sale, lease or use of reclaimed lands shall be used in accordance with the provisions of
Presidential Decree No. 1084."
There is no express authority under either PD No. 1085 or EO No. 525 for PEA to sell its reclaimed lands.
PD No. 1085 merely transferred "ownership and administration" of lands reclaimed from Manila Bay to
PEA, while EO No. 525 declared that lands reclaimed by PEA "shall belong to or be owned by PEA." EO
No. 525 expressly states that PEA should dispose of its reclaimed lands "in accordance with the
provisions of Presidential Decree No. 1084," the charter of PEA.
PEA's charter, however, expressly tasks PEA "to develop, improve, acquire, administer, deal in,
subdivide, dispose, lease and sell any and all kinds of lands x x x owned, managed, controlled and/or
operated by the government."
87
(Emphasis supplied) There is, therefore, legislative authority granted
to PEA to sell its lands, whether patrimonial or alienable lands of the public domain. PEA may sell
to private parties its patrimonial properties in accordance with the PEA charter free from constitutional
limitations. The constitutional ban on private corporations from acquiring alienable lands of the public
domain does not apply to the sale of PEA's patrimonial lands.
PEA may also sell its alienable or disposable lands of the public domain to private individuals since,
with the legislative authority, there is no longer any statutory prohibition against such sales and the
constitutional ban does not apply to individuals. PEA, however, cannot sell any of its alienable or
disposable lands of the public domain to private corporations since Section 3, Article XII of the 1987
Constitution expressly prohibits such sales. The legislative authority benefits only individuals. Private
corporations remain barred from acquiring any kind of alienable land of the public domain, including
government reclaimed lands.
The provision in PD No. 1085 stating that portions of the reclaimed lands could be transferred by PEA to
the "contractor or his assignees" (Emphasis supplied) would not apply to private corporations but only to
individuals because of the constitutional ban. Otherwise, the provisions of PD No. 1085 would violate both
the 1973 and 1987 Constitutions.
The requirement of public auction in the sale of reclaimed lands
Assuming the reclaimed lands of PEA are classified as alienable or disposable lands open to disposition,
and further declared no longer needed for public service, PEA would have to conduct a public bidding in
selling or leasing these lands. PEA must observe the provisions of Sections 63 and 67 of CA No. 141
requiring public auction, in the absence of a law exempting PEA from holding a public auction.
88
Special
Patent No. 3517 expressly states that the patent is issued by authority of the Constitution and PD No.
1084, "supplemented by Commonwealth Act No. 141, as amended." This is an acknowledgment that the
provisions of CA No. 141 apply to the disposition of reclaimed alienable lands of the public domain unless
otherwise provided by law. Executive Order No. 654,
89
which authorizes PEA "to determine the kind and
manner of payment for the transfer" of its assets and properties, does not exempt PEA from the
requirement of public auction. EO No. 654 merely authorizes PEA to decide the mode of payment,
whether in kind and in installment, but does not authorize PEA to dispense with public auction.
Moreover, under Section 79 of PD No. 1445, otherwise known as the Government Auditing Code, the
government is required to sell valuable government property through public bidding. Section 79 of PD No.
1445 mandates that
"Section 79. When government property has become unserviceable for any cause, or is no
longer needed, it shall, upon application of the officer accountable therefor, be inspected by the
head of the agency or his duly authorized representative in the presence of the auditor concerned
and, if found to be valueless or unsaleable, it may be destroyed in their presence. If found to be
valuable, it may be sold at public auction to the highest bidder under the supervision of the
proper committee on award or similar body in the presence of the auditor concerned or other
authorized representative of the Commission, after advertising by printed notice in the Official
Gazette, or for not less than three consecutive days in any newspaper of general
circulation, or where the value of the property does not warrant the expense of publication, by
notices posted for a like period in at least three public places in the locality where the property is
to be sold. In the event that the public auction fails, the property may be sold at a private
sale at such price as may be fixed by the same committee or body concerned and
approved by the Commission."
It is only when the public auction fails that a negotiated sale is allowed, in which case the Commission on
Audit must approve the selling price.
90
The Commission on Audit implements Section 79 of the
Government Auditing Code through Circular No. 89-296
91
dated January 27, 1989. This circular
emphasizes that government assets must be disposed of only through public auction, and a negotiated
sale can be resorted to only in case of "failure of public auction."
At the public auction sale, only Philippine citizens are qualified to bid for PEA's reclaimed foreshore and
submerged alienable lands of the public domain. Private corporations are barred from bidding at the
auction sale of any kind of alienable land of the public domain.
PEA originally scheduled a public bidding for the Freedom Islands on December 10, 1991. PEA imposed
a condition that the winning bidder should reclaim another 250 hectares of submerged areas to regularize
the shape of the Freedom Islands, under a 60-40 sharing of the additional reclaimed areas in favor of the
winning bidder.
92
No one, however, submitted a bid. On December 23, 1994, the Government Corporate
Counsel advised PEA it could sell the Freedom Islands through negotiation, without need of another
public bidding, because of the failure of the public bidding on December 10, 1991.
93

However, the original JVA dated April 25, 1995 covered not only the Freedom Islands and the additional
250 hectares still to be reclaimed, it also granted an option to AMARI to reclaim another 350 hectares.
The original JVA, a negotiated contract, enlarged the reclamation area to 750 hectares.
94
The failure of
public bidding on December 10, 1991, involving only 407.84 hectares,
95
is not a valid justification for a
negotiated sale of 750 hectares, almost double the area publicly auctioned. Besides, the failure of public
bidding happened on December 10, 1991, more than three years before the signing of the original JVA on
April 25, 1995. The economic situation in the country had greatly improved during the intervening period.
Reclamation under the BOT Law and the Local Government Code
The constitutional prohibition in Section 3, Article XII of the 1987 Constitution is absolute and clear:
"Private corporations or associations may not hold such alienable lands of the public domain except by
lease, x x x." Even Republic Act No. 6957 ("BOT Law," for brevity), cited by PEA and AMARI as
legislative authority to sell reclaimed lands to private parties, recognizes the constitutional ban. Section 6
of RA No. 6957 states
"Sec. 6. Repayment Scheme. - For the financing, construction, operation and maintenance of any
infrastructure projects undertaken through the build-operate-and-transfer arrangement or any of
its variations pursuant to the provisions of this Act, the project proponent x x x may likewise be
repaid in the form of a share in the revenue of the project or other non-monetary payments, such
as, but not limited to, the grant of a portion or percentage of the reclaimed land, subject to the
constitutional requirements with respect to the ownership of the land: x x x." (Emphasis
supplied)
A private corporation, even one that undertakes the physical reclamation of a government BOT project,
cannot acquire reclaimed alienable lands of the public domain in view of the constitutional ban.
Section 302 of the Local Government Code, also mentioned by PEA and AMARI, authorizes local
governments in land reclamation projects to pay the contractor or developer in kind consisting of a
percentage of the reclaimed land, to wit:
"Section 302. Financing, Construction, Maintenance, Operation, and Management of
Infrastructure Projects by the Private Sector. x x x
x x x
In case of land reclamation or construction of industrial estates, the repayment plan may consist
of the grant of a portion or percentage of the reclaimed land or the industrial estate constructed."
Although Section 302 of the Local Government Code does not contain a proviso similar to that of the BOT
Law, the constitutional restrictions on land ownership automatically apply even though not expressly
mentioned in the Local Government Code.
Thus, under either the BOT Law or the Local Government Code, the contractor or developer, if a
corporate entity, can only be paid with leaseholds on portions of the reclaimed land. If the contractor or
developer is an individual, portions of the reclaimed land, not exceeding 12 hectares
96
of non-agricultural
lands, may be conveyed to him in ownership in view of the legislative authority allowing such conveyance.
This is the only way these provisions of the BOT Law and the Local Government Code can avoid a direct
collision with Section 3, Article XII of the 1987 Constitution.
Registration of lands of the public domain
Finally, PEA theorizes that the "act of conveying the ownership of the reclaimed lands to public
respondent PEA transformed such lands of the public domain to private lands." This theory is echoed by
AMARI which maintains that the "issuance of the special patent leading to the eventual issuance of title
takes the subject land away from the land of public domain and converts the property into patrimonial or
private property." In short, PEA and AMARI contend that with the issuance of Special Patent No. 3517
and the corresponding certificates of titles, the 157.84 hectares comprising the Freedom Islands have
become private lands of PEA. In support of their theory, PEA and AMARI cite the following rulings of the
Court:
1. Sumail v. Judge of CFI of Cotabato,
97
where the Court held
"Once the patent was granted and the corresponding certificate of title was issued, the land
ceased to be part of the public domain and became private property over which the Director of
Lands has neither control nor jurisdiction."
2. Lee Hong Hok v. David,
98
where the Court declared -
"After the registration and issuance of the certificate and duplicate certificate of title based on a
public land patent, the land covered thereby automatically comes under the operation of Republic
Act 496 subject to all the safeguards provided therein."3. Heirs of Gregorio Tengco v. Heirs of
Jose Aliwalas,
99
where the Court ruled -
"While the Director of Lands has the power to review homestead patents, he may do so only so
long as the land remains part of the public domain and continues to be under his exclusive
control; but once the patent is registered and a certificate of title is issued, the land ceases to be
part of the public domain and becomes private property over which the Director of Lands has
neither control nor jurisdiction."
4. Manalo v. Intermediate Appellate Court,
100
where the Court held
"When the lots in dispute were certified as disposable on May 19, 1971, and free patents were
issued covering the same in favor of the private respondents, the said lots ceased to be part of
the public domain and, therefore, the Director of Lands lost jurisdiction over the same."
5.Republic v. Court of Appeals,
101
where the Court stated
"Proclamation No. 350, dated October 9, 1956, of President Magsaysay legally effected a land
grant to the Mindanao Medical Center, Bureau of Medical Services, Department of Health, of the
whole lot, validly sufficient for initial registration under the Land Registration Act. Such land grant
is constitutive of a 'fee simple' title or absolute title in favor of petitioner Mindanao Medical Center.
Thus, Section 122 of the Act, which governs the registration of grants or patents involving public
lands, provides that 'Whenever public lands in the Philippine Islands belonging to the
Government of the United States or to the Government of the Philippines are alienated, granted
or conveyed to persons or to public or private corporations, the same shall be brought forthwith
under the operation of this Act (Land Registration Act, Act 496) and shall become registered
lands.'"
The first four cases cited involve petitions to cancel the land patents and the corresponding certificates of
titlesissued to private parties. These four cases uniformly hold that the Director of Lands has no
jurisdiction over private lands or that upon issuance of the certificate of title the land automatically comes
under the Torrens System. The fifth case cited involves the registration under the Torrens System of a
12.8-hectare public land granted by the National Government to Mindanao Medical Center, a government
unit under the Department of Health. The National Government transferred the 12.8-hectare public land to
serve as the site for the hospital buildings and other facilities of Mindanao Medical Center, which
performed a public service. The Court affirmed the registration of the 12.8-hectare public land in the name
of Mindanao Medical Center under Section 122 of Act No. 496. This fifth case is an example of a public
land being registered under Act No. 496 without the land losing its character as a property of public
dominion.
In the instant case, the only patent and certificates of title issued are those in the name of PEA, a wholly
government owned corporation performing public as well as proprietary functions. No patent or certificate
of title has been issued to any private party. No one is asking the Director of Lands to cancel PEA's
patent or certificates of title. In fact, the thrust of the instant petition is that PEA's certificates of title should
remain with PEA, and the land covered by these certificates, being alienable lands of the public domain,
should not be sold to a private corporation.
Registration of land under Act No. 496 or PD No. 1529 does not vest in the registrant private or public
ownership of the land. Registration is not a mode of acquiring ownership but is merely evidence of
ownership previously conferred by any of the recognized modes of acquiring ownership. Registration
does not give the registrant a better right than what the registrant had prior to the registration.
102
The
registration of lands of the public domain under the Torrens system, by itself, cannot convert public lands
into private lands.
103

Jurisprudence holding that upon the grant of the patent or issuance of the certificate of title the alienable
land of the public domain automatically becomes private land cannot apply to government units and
entities like PEA. The transfer of the Freedom Islands to PEA was made subject to the provisions of CA
No. 141 as expressly stated in Special Patent No. 3517 issued by then President Aquino, to wit:
"NOW, THEREFORE, KNOW YE, that by authority of the Constitution of the Philippines and in
conformity with the provisions of Presidential Decree No. 1084, supplemented by
Commonwealth Act No. 141, as amended, there are hereby granted and conveyed unto the
Public Estates Authority the aforesaid tracts of land containing a total area of one million nine
hundred fifteen thousand eight hundred ninety four (1,915,894) square meters; the technical
description of which are hereto attached and made an integral part hereof." (Emphasis supplied)
Thus, the provisions of CA No. 141 apply to the Freedom Islands on matters not covered by PD No.
1084. Section 60 of CA No. 141 prohibits, "except when authorized by Congress," the sale of alienable
lands of the public domain that are transferred to government units or entities. Section 60 of CA No. 141
constitutes, under Section 44 of PD No. 1529, a "statutory lien affecting title" of the registered land even if
not annotated on the certificate of title.
104
Alienable lands of the public domain held by government entities
under Section 60 of CA No. 141 remain public lands because they cannot be alienated or encumbered
unless Congress passes a law authorizing their disposition. Congress, however, cannot authorize the
sale to private corporations of reclaimed alienable lands of the public domain because of the
constitutional ban. Only individuals can benefit from such law.
The grant of legislative authority to sell public lands in accordance with Section 60 of CA No. 141 does
not automatically convert alienable lands of the public domain into private or patrimonial lands. The
alienable lands of the public domain must be transferred to qualified private parties, or to government
entities not tasked to dispose of public lands, before these lands can become private or patrimonial lands.
Otherwise, the constitutional ban will become illusory if Congress can declare lands of the public domain
as private or patrimonial lands in the hands of a government agency tasked to dispose of public lands.
This will allow private corporations to acquire directly from government agencies limitless areas of lands
which, prior to such law, are concededly public lands.
Under EO No. 525, PEA became the central implementing agency of the National Government to
reclaim foreshore and submerged areas of the public domain. Thus, EO No. 525 declares that
"EXECUTIVE ORDER NO. 525
Designating the Public Estates Authority as the Agency Primarily Responsible for all Reclamation
Projects
Whereas, there are several reclamation projects which are ongoing or being proposed to be
undertaken in various parts of the country which need to be evaluated for consistency with
national programs;
Whereas, there is a need to give further institutional support to the Government's declared policy
to provide for a coordinated, economical and efficient reclamation of lands;
Whereas, Presidential Decree No. 3-A requires that all reclamation of areas shall be limited to the
National Government or any person authorized by it under proper contract;
Whereas, a central authority is needed to act on behalf of the National Government which
shall ensure a coordinated and integrated approach in the reclamation of lands;
Whereas, Presidential Decree No. 1084 creates the Public Estates Authority as a
government corporation to undertake reclamation of lands and ensure their maximum
utilization in promoting public welfare and interests; and
Whereas, Presidential Decree No. 1416 provides the President with continuing authority to
reorganize the national government including the transfer, abolition, or merger of functions and
offices.
NOW, THEREFORE, I, FERDINAND E. MARCOS, President of the Philippines, by virtue of the
powers vested in me by the Constitution and pursuant to Presidential Decree No. 1416, do
hereby order and direct the following:
Section 1. The Public Estates Authority (PEA) shall be primarily responsible for integrating,
directing, and coordinating all reclamation projects for and on behalf of the National
Government. All reclamation projects shall be approved by the President upon recommendation
of the PEA, and shall be undertaken by the PEA or through a proper contract executed by it with
any person or entity; Provided, that, reclamation projects of any national government agency or
entity authorized under its charter shall be undertaken in consultation with the PEA upon approval
of the President.
x x x ."
As the central implementing agency tasked to undertake reclamation projects nationwide, with authority to
sell reclaimed lands, PEA took the place of DENR as the government agency charged with leasing or
selling reclaimed lands of the public domain. The reclaimed lands being leased or sold by PEA are not
private lands, in the same manner that DENR, when it disposes of other alienable lands, does not
dispose of private lands but alienable lands of the public domain. Only when qualified private parties
acquire these lands will the lands become private lands. In the hands of the government agency
tasked and authorized to dispose of alienable of disposable lands of the public domain, these
lands are still public, not private lands.
Furthermore, PEA's charter expressly states that PEA "shall hold lands of the public domain" as well
as "any and all kinds of lands." PEA can hold both lands of the public domain and private lands. Thus, the
mere fact that alienable lands of the public domain like the Freedom Islands are transferred to PEA and
issued land patents or certificates of title in PEA's name does not automatically make such lands private.
To allow vast areas of reclaimed lands of the public domain to be transferred to PEA as private lands will
sanction a gross violation of the constitutional ban on private corporations from acquiring any kind of
alienable land of the public domain. PEA will simply turn around, as PEA has now done under the
Amended J VA, and transfer several hundreds of hectares of these reclaimed and still to be reclaimed
lands to a single private corporation in only one transaction. This scheme will effectively nullify the
constitutional ban in Section 3, Article XII of the 1987 Constitution which was intended to diffuse equitably
the ownership of alienable lands of the public domain among Filipinos, now numbering over 80 million
strong.
This scheme, if allowed, can even be applied to alienable agricultural lands of the public domain since
PEA can "acquire x x x any and all kinds of lands." This will open the floodgates to corporations and even
individuals acquiring hundreds of hectares of alienable lands of the public domain under the guise that in
the hands of PEA these lands are private lands. This will result in corporations amassing huge
landholdings never before seen in this country - creating the very evil that the constitutional ban was
designed to prevent. This will completely reverse the clear direction of constitutional development in this
country. The 1935 Constitution allowed private corporations to acquire not more than 1,024 hectares of
public lands.
105
The 1973 Constitution prohibited private corporations from acquiring any kind of public
land, and the 1987 Constitution has unequivocally reiterated this prohibition.
The contention of PEA and AMARI that public lands, once registered under Act No. 496 or PD No. 1529,
automatically become private lands is contrary to existing laws. Several laws authorize lands of the public
domain to be registered under the Torrens System or Act No. 496, now PD No. 1529, without losing their
character as public lands. Section 122 of Act No. 496, and Section 103 of PD No. 1529, respectively,
provide as follows:
Act No. 496
"Sec. 122. Whenever public lands in the Philippine Islands belonging to the x x x Government of
the Philippine Islands are alienated, granted, or conveyed to persons or the public or private
corporations, the same shall be brought forthwith under the operation of this Act and shall
become registered lands."
PD No. 1529
"Sec. 103. Certificate of Title to Patents. Whenever public land is by the Government alienated,
granted or conveyed to any person, the same shall be brought forthwith under the operation of
this Decree." (Emphasis supplied)
Based on its legislative history, the phrase "conveyed to any person" in Section 103 of PD No. 1529
includes conveyances of public lands to public corporations.
Alienable lands of the public domain "granted, donated, or transferred to a province, municipality, or
branch or subdivision of the Government," as provided in Section 60 of CA No. 141, may be registered
under the Torrens System pursuant to Section 103 of PD No. 1529. Such registration, however, is
expressly subject to the condition in Section 60 of CA No. 141 that the land "shall not be alienated,
encumbered or otherwise disposed of in a manner affecting its title, except when authorized by
Congress." This provision refers to government reclaimed, foreshore and marshy lands of the public
domain that have been titled but still cannot be alienated or encumbered unless expressly authorized by
Congress. The need for legislative authority prevents the registered land of the public domain from
becoming private land that can be disposed of to qualified private parties.
The Revised Administrative Code of 1987 also recognizes that lands of the public domain may be
registered under the Torrens System. Section 48, Chapter 12, Book I of the Code states
"Sec. 48. Official Authorized to Convey Real Property. Whenever real property of the Government
is authorized by law to be conveyed, the deed of conveyance shall be executed in behalf of the
government by the following:
(1) x x x
(2) For property belonging to the Republic of the Philippines, but titled in the name of any
political subdivision or of any corporate agency or instrumentality, by the executive head of
the agency or instrumentality." (Emphasis supplied)
Thus, private property purchased by the National Government for expansion of a public wharf may be
titled in the name of a government corporation regulating port operations in the country. Private property
purchased by the National Government for expansion of an airport may also be titled in the name of the
government agency tasked to administer the airport. Private property donated to a municipality for use as
a town plaza or public school site may likewise be titled in the name of the municipality.
106
All these
properties become properties of the public domain, and if already registered under Act No. 496 or PD No.
1529, remain registered land. There is no requirement or provision in any existing law for the de-
registration of land from the Torrens System.
Private lands taken by the Government for public use under its power of eminent domain become
unquestionably part of the public domain. Nevertheless, Section 85 of PD No. 1529 authorizes the
Register of Deeds to issue in the name of the National Government new certificates of title covering such
expropriated lands. Section 85 of PD No. 1529 states
"Sec. 85. Land taken by eminent domain. Whenever any registered land, or interest therein, is
expropriated or taken by eminent domain, the National Government, province, city or municipality,
or any other agency or instrumentality exercising such right shall file for registration in the proper
Registry a certified copy of the judgment which shall state definitely by an adequate description,
the particular property or interest expropriated, the number of the certificate of title, and the nature
of the public use. A memorandum of the right or interest taken shall be made on each certificate
of title by the Register of Deeds, and where the fee simple is taken, a new certificate shall be
issued in favor of the National Government, province, city, municipality, or any other
agency or instrumentality exercising such right for the land so taken. The legal expenses incident
to the memorandum of registration or issuance of a new certificate of title shall be for the account
of the authority taking the land or interest therein." (Emphasis supplied)
Consequently, lands registered under Act No. 496 or PD No. 1529 are not exclusively private or
patrimonial lands. Lands of the public domain may also be registered pursuant to existing laws.
AMARI makes a parting shot that the Amended JVA is not a sale to AMARI of the Freedom Islands or of
the lands to be reclaimed from submerged areas of Manila Bay. In the words of AMARI, the Amended
JVA "is not a sale but a joint venture with a stipulation for reimbursement of the original cost incurred by
PEA for the earlier reclamation and construction works performed by the CDCP under its 1973 contract
with the Republic." Whether the Amended JVA is a sale or a joint venture, the fact remains that the
Amended JVA requires PEA to "cause the issuance and delivery of the certificates of title conveying
AMARI's Land Share in the name of AMARI."
107

This stipulation still contravenes Section 3, Article XII of the 1987 Constitution which provides that private
corporations "shall not hold such alienable lands of the public domain except by lease." The transfer of
title and ownership to AMARI clearly means that AMARI will "hold" the reclaimed lands other than by
lease. The transfer of title and ownership is a "disposition" of the reclaimed lands, a transaction
considered a sale or alienation under CA No. 141,
108
the Government Auditing Code,
109
and Section 3,
Article XII of the 1987 Constitution.
The Regalian doctrine is deeply implanted in our legal system. Foreshore and submerged areas form part
of the public domain and are inalienable. Lands reclaimed from foreshore and submerged areas also form
part of the public domain and are also inalienable, unless converted pursuant to law into alienable or
disposable lands of the public domain. Historically, lands reclaimed by the government are sui generis,
not available for sale to private parties unlike other alienable public lands. Reclaimed lands retain their
inherent potential as areas for public use or public service. Alienable lands of the public domain,
increasingly becoming scarce natural resources, are to be distributed equitably among our ever-growing
population. To insure such equitable distribution, the 1973 and 1987 Constitutions have barred private
corporations from acquiring any kind of alienable land of the public domain. Those who attempt to
dispose of inalienable natural resources of the State, or seek to circumvent the constitutional ban on
alienation of lands of the public domain to private corporations, do so at their own risk.
We can now summarize our conclusions as follows:
1. The 157.84 hectares of reclaimed lands comprising the Freedom Islands, now covered by
certificates of title in the name of PEA, are alienable lands of the public domain. PEA
may LEASE these lands to private corporations but may not sell or transfer ownership of these
lands to private corporations. PEA may only sell these lands to Philippine citizens, subject to the
ownership limitations in the 1987 Constitution and existing laws.
2. The 592.15 hectares of submerged areas of Manila Bay remain inalienable natural resources
of the public domain until classified as alienable or disposable lands open to disposition and
declared no longer needed for public service. The government can make such classification and
declaration only after PEA has reclaimed these submerged areas. Only then can these lands
qualify as agricultural lands of the public domain, which are the only natural resources the
government can alienate. In their present state, the 592.15 hectares of submerged areas are
inalienable and outside the commerce of man.
3. Since the Amended JVA seeks to transfer to AMARI, a private corporation, ownership of 77.34
hectares
110
of the Freedom Islands, such transfer is void for being contrary to Section 3, Article
XII of the 1987 Constitution which prohibits private corporations from acquiring any kind of
alienable land of the public domain.
4. Since the Amended JVA also seeks to transfer to AMARI ownership of 290.156 hectares
111
of
still submerged areas of Manila Bay, such transfer is void for being contrary to Section 2, Article
XII of the 1987 Constitution which prohibits the alienation of natural resources other than
agricultural lands of the public domain. PEA may reclaim these submerged areas. Thereafter, the
government can classify the reclaimed lands as alienable or disposable, and further declare them
no longer needed for public service. Still, the transfer of such reclaimed alienable lands of the
public domain to AMARI will be void in view of Section 3, Article XII of the 1987 Constitution
which prohibits private corporations from acquiring any kind of alienable land of the public
domain.
Clearly, the Amended JVA violates glaringly Sections 2 and 3, Article XII of the 1987 Constitution. Under
Article 1409
112
of the Civil Code, contracts whose "object or purpose is contrary to law," or whose "object
is outside the commerce of men," are "inexistent and void from the beginning." The Court must perform
its duty to defend and uphold the Constitution, and therefore declares the Amended J VA null and void
ab initio.
Seventh issue: whether the Court is the proper forum to raise the issue of whether the Amended
J VA is grossly disadvantageous to the government.
Considering that the Amended JVA is null and void ab initio, there is no necessity to rule on this last
issue. Besides, the Court is not a trier of facts, and this last issue involves a determination of factual
matters.
WHEREFORE, the petition is GRANTED. The Public Estates Authority and Amari Coastal Bay
Development Corporation are PERMANENTLY ENJOINED from implementing the Amended Joint
Venture Agreement which is hereby declared NULL and VOID ab initio.
SO ORDERED.
Davide, Jr., C.J., Bellosillo, Puno, Vitug, Kapunan, Mendoza, Panganiban, Quisumbing, Ynares-Santiago,
Sandoval-Gutierrez, Austria-Martinez, and Corona, JJ., concur.


14. G.R. No. 132963 September 10, 1998
REPUBLIC OF THE PHILIPPINES, (represented by Opol National Secondary Technical
School), petitioner,
vs.
NICANOR DOLDOL, respondent.

ROMERO, J .:
Before us is a petition for review of the decision of the Court of Appeals dated October 27, 1997,
reversing the decision of the Regional Trial Court and dismissing herein petitioner's complaint, as well as
its resolution of March 5, 1998, denying petitioner's motion for reconsideration.
The facts are as follows:
Sometime in 1959, respondent Nicanor Doldol occupied a portion of land in Barrio Pontacan, Municipality
of Opol, Misamis Oriental. On October 23, 1963, he filed an application for saltwork purposes for the said
area with the Bureau of Forest Development. The Director of Forestry, however, rejected the same on
April 1, 1968. Meanwhile, the Provincial Board of Misamis Oriental passed a resolution in 1965 reserving
Lot 4932, Cad-237, Opol Cadastre as a school site. This reserved lot unfortunately included the area
occupied by Doldol.
In accordance with said resolution, the Opol High School transferred to the site in 1970. Seventeen years
later, on November 2, 1987, then President Corazon Aquino issued Proclamation No. 180 reserving the
area, including the portion in dispute, for the Opol High School, now renamed the Opol National
Secondary Technical School (hereafter Opol National School). Needing the area occupied by Doldol for
its intended projects, the school made several demands for him to vacate said portion, but he refused to
move.
In view of Doldol's refusal to vacate, Opol National School filed in 1991 a complaint for accion
possessoria with the Regional Trial Court of Cagayan de Oro. The trial court ruled in the school's favor
and ordered Doldol to vacate the land. On appeal, the Court of Appeals reversed the decision of the
court a quo, ruling that Doldol was entitled to the portion he occupied, he having possessed the same for
thirty-two years, from 1959 up to the time of the filing of the complaint in 1991.
Opol National School's motion for reconsideration of said decision having been denied by the Court of
Appeals in its resolution of March 5, 1998, Opol National School elevated its case to this Court, claiming
that the Court of Appeals erred on a question of law when it held, contrary to the evidence on record, that
respondent had been in open, continuous, notorious and exclusive possession of the land in dispute for
thirty-two years.
The petition is meritorious.
In ruling in Doldol's favor, the Court of Appeals grounded its decision on Section 48 of Commonwealth Act
No. 141 (otherwise known as the Public Land Act). Said provision, as amended by Republic Act No.
1942, provides as follows:
Sec. 48. The following described citizens of the Philippines, occupying lands of the public
domain or claiming interest therein, but whose titles have not been perfected or
completed, may apply to the Court of First Instance (now Regional Trial Court) of the
province where the land is located for confirmation of their claims and the issuance of a
certification of title therefor under the Land Registration Act, to wit:
xxx xxx xxx
(b) Those who by themselves or through their predecessors-in-interest have been in
open, continuous, exclusive and notorious possession and occupation of agricultural
lands of the public domain, under a bona fide claim of acquisition or ownership for at
least thirty years immediately preceding the filing of the application for confirmation of
title, except when prevented by wars or force majeure. Those shall be conclusively
presumed to have performed all the conditions essential to a GOVERNMENT GRANT
and shall be entitled to a certificate of title under the provisions of this chapter.
(Emphasis ours)
In accordance with the above provision, the appellate court averred that a citizen of the Philippines may
acquire alienable land of the public domain if he has possessed the same for thirty years. Finding Doldol
to have occupied the disputed lot for thirty-two years, it ruled that the former had acquired ownership of
the same, thereby negating Opol National School's claim over the questioned area.
To further bolster its argument, the appellate court cited Republic vs.
CA
1
where this Court, citing Director of Lands vs. Iglesia ni Cristo, 200 SCRA 606 (1991) declared that:
The weight of authority is that open, exclusive and undisputed possession of alienable
public land for the period prescribed by law creates the legal fiction whereby the land
upon completion of the requisite period ipso jure and without the need of judicial or other
sanction, ceases to be public land and becomes private property.
xxx xxx xxx
. . . with the latter's proven occupation and cultivation for more than 30 years since 1914,
by himself and by his predecessors-in-interest, title over the land has vested on petitioner
so as to segregate the land from the mass of public land.
xxx xxx xxx
As interpreted in several cases, when the conditions as specified in the foregoing
provision are complied with, the possessor is deemed to have acquired, by operation of
law, a right to a grant, a GOVERNMENT GRANT , without the necessity of a certificate
of title being issued. The land, therefore, ceases to be of the public domain and beyond
the authority of the Director of Lands to dispose of. The application for confirmation is
mere formality, the lack of which does not affect the legal sufficiency of the title as would
he evidenced by the patent and the Torrens title to be issued upon the strength of said
patent.
The appellate court has resolved the question as to who between the parties had a better right to possess
the lot through the erroneous application of an outdated version of Section 48 of the Public Land Act.
Likewise, SOLICITOR Renan E. Ramos of the Office of the SOLICITOR General erred in assuming
that the thirty-year proviso in the aforementioned section was still good law. The original Section 48(b) of
C.A. No. 141 provided for possession and occupation of lands of the public domain since July 26, 1894.
This was superseded by R.A. No. 1942,
2
which provided for a simple thirty year prescriptive period of
occupation by an applicant for judicial confirmation of imperfect title. The same, however, has already
been amended by Presidential Decree No. 1073, approved on January 25, 1977. As amended, Section
48(b) now reads:
(b) Those who by themselves or through their predecessors-in-interest have been in
open, continuous, exclusive and notorious possession and occupation of agricultural
lands of the public domain, under a bona fide claim of acquisition or ownership, since
June 12, 1945, or earlier, immediately preceding the filing of the application for
confirmation of title, except when prevented by wars or force majeure. Those shall be
conclusively presumed to have performed all the conditions essential to
a GOVERNMENT GRANT and shall be entitled to a certificate of title under the
provisions of this chapter. (Emphasis ours)
Thus, in the aforecited Republic vs. CA case, we stated that the Public Land Act requires that the
applicant must prove (a) that the land is alienable public land and (b) that his open, continuous, exclusive
and notorious possession and occupation of the same must either be since time immemorial or for the
period prescribed in the Public Land Act. When the conditions set by law are complied with, the
possessor of the land, by operation of law, acquires a right to a grant, a GOVERNMENT GRANT ,
without the necessity of a certificate of title being issued.
The evidence presented shows that the land in dispute is alienable and disposable, in accordance with
the District Forester's Certification dated September 20, 1978, that the subject area is within Project 8, an
alienable and disposable tract of public land, as appearing in Bureau of Forest Land Classification Map
No. 585. Doldol, thus, meets the first requirement.
The parties, however, stipulated during the pre-trial hearing that Doldol had been occupying the portion
reserved for the school site only since 1959. The law, as presently phrased, requires that possession of
lands of the pubic domain must be from June 12, 1945 or earlier, for the same to be acquired through
judicial confirmation of imperfect title.
Consequently, Doldol could not have acquired an imperfect title to the disputed lot since his occupation of
the same started only in 1959, much later than June 12, 1945. Not having complied with the conditions
set by law, Doldol cannot be said to have acquired a right to the land in question as to segregate the
same from the public domain. Doldol cannot, therefore, assert a right superior to the school, given that
then President Corazon Aquino had reserved the lot for Opol National School. As correctly pointed out by
the Solicitor General:
(T)he privilege of occupying public lands with a view of preemption confers no contractual
or vested right in the lands occupied and the authority of the President to withdraw such
lands for sale or acquisition by the public, or to reserve them for public use, prior to the
divesting by the government of title thereof stands, even though this may defeat the
imperfect right of a settler. Lands covered by reservation are not subject to entry, and no
lawful settlement on them can be acquired.
3

In sum, Opol National School has the better right of possession over the land in dispute.
WHEREFORE, premises considered, the decision of the Court of Appeals dated October 27, 1997, and
Resolution dated March 27, 1998, are hereby ANNULLED and SET ASIDE and the Decision of the
Regional Trial Court dated August 25, 1992, is hereby REINSTATED.
SO ORDERED.
Narvasa, C.J., Kapunan and Purisima, JJ., concur.


15. G.R. No. 155012 April 14, 2004
REPUBLIC OF THE PHILIPPINES, petitioner,
vs.
CARMENCITA M. ALCONABA; LUISITO B. MELENDEZ; CONCEPCION M. LAZARO; MAURICIO B.
MELENDEZ, JR.; and MYRNA M. GALVEZ, represented by CONCEPCION M. LAZARO, respondents.
DECISION
DAVIDE, JR., C.J .:
To serve the ends of social justice, which is the heart of the 1987 Constitution, the State promotes an
equitable distribution of alienable agricultural lands of the public domain to deserving citizens, especially
the underprivileged. A land registration court must, therefore, exercise extreme caution and prudent care
in deciding an application for judicial confirmation of an imperfect title over such lands so that the public
domain may not be raided by unscrupulous land speculators.
1

At bar is a petition for review under Rule 45 of the Rules of Civil Procedure seeking to set aside the
decision
2
of the Court of Appeals of 26 August 2002 in CA-G.R. CV No. 64323, which affirmed the
decision
3
of the Municipal Trial Court (MTC) of Cabuyao, Laguna,
4
of 1 September 1998 in MTC LRC
Case No. 06 ordering the registration in favor of the respondents of parcels of land situated at Barangay
Sala, Cabuyao, Laguna, designated as Lot 2111-A, 2111-B, 2111-C, 2111-D, and 2111-E.
The pertinent facts are as follows:
On 14 November 1996, the respondents filed before the MTC of Cabuyao, Laguna, an
application
5
for registration of title over five parcels of land, each with an area of 5,220 square
meters, situated in Barangay Sala, Cabuyao, Laguna. In their application, they stated, among
other things, that they are the sole heirs of Spouses Melencio E. Melendez, Sr., and Luz
Batallones Melendez, original owners of Lot 2111 of CAD-455, with an area of 2.6 hectares. Their
parents had been in possession of the said property since 1949, more or less. After the death of
their mother and father on 19 February 1967 and 5 May 1976, respectively, they partitioned the
property among themselves and subdivided it into five lots, namely, Lots 2111-A, 2111-B, 2111-
C, 2111-D, and 2111-E. Since then they have been in actual possession of the property in the
concept of owners and in a public and peaceful manner.
Petitioner Republic of the Philippines, through the Office of the SOLICITOR General (OSG), opposed
the application on the following grounds: (a) neither the respondents nor their predecessors-in-interest
possess sufficient title to the property or have been in open, continuous, exclusive, and notorious
possession and occupation of the land in question since 1945 or prior thereto; (b) the muniments of
title, i.e., tax declaration and tax receipts, presented by the respondents do not constitute competent and
sufficient evidence of a bona fideright to registration of the land under Section 48(b), Commonwealth Act
No. 141, otherwise known as The Public Land Act,
6
as amended by Presidential Decree No. 1073; (c) the
claim of ownership in fee simple on the basis of a Spanish title or grant can no longer be availed of by the
respondents; and (d) the land is part of the public domain belonging to the Republic of the Philippines.
7

At the trial on the merits, respondents Mauricio B. Melendez, Jr., and Carmencita M. Alconaba testified to
establish their claim over the subject lots. Mauricio claimed that he and his co-respondents acquired by
inheritance from their deceased parents Lot 2111 of Cad-455, which is an agricultural land. Their parents
had been in possession of the said land since 1949 and had been religiously paying the taxes due
thereon. When their parents died, he and his siblings immediately took possession of said property in the
concept of an owner, paid taxes, and continued to plant rice thereon. On 24 June 1996, he and his co-
heirs executed an Extrajudicial Settlement with Partition over the said lot and subdivided it into five lots.
8

For her part, Carmencita testified that Lot 2111 of Cad-455 had been in the possession of their parents
since 1940 and that after the death of their parents she and her siblings immediately took possession of it
and religiously paid the taxes thereon. The land is being cultivated by Julia Garal, their tenant. She
admitted that no improvements have been introduced by their family on the lot. On cross examination,
she admitted that plans to sell the property were at hand.
9

In its decision of 1 September 1998, the trial court found that the respondents have sufficiently
established their family's actual, continuous, adverse, and notorious possession of the subject property
for more than fifty-seven years, commencing from the possession of their predecessors-in-interest in
1940, and that such possession was in an adverse and public manner. Likewise, it found that the land in
question is alienable and disposable and is not within any reservation or forest zone. Thus, it confirmed
the title of the respondents over the said lots; directed the Register of Deeds of Laguna, Calamba Branch,
to cause the registration of said parcels of land in the name of the respondents upon payment of fees;
and ordered the issuance of a Decree of Registration once the decision becomes final and executory.
Upon appeal
10
by the petitioner, the Court of Appeals affirmed the decision of the trial court. Hence, this
petition.
The OSG argues that both the trial court and the Court of Appeals erred in (a) giving weight to the self-
serving testimonies of Mauricio and Carmencita that the respondents and their predecessors-in-interest
had been in open, continuous, and adverse possession of the lots in question in the concept of an owner
for at least thirty years; and (b) holding that respondents' tax declaration is sufficient proof that they and
their parents have been in possession of the property for at least thirty years, despite the fact that the said
tax declaration was only for the year 1994 and the PROPERTY TAX receipts presented by the
respondents were all of recent dates, i.e., 1990, 1991,1992, 1994, 1996, and 1997. Finally, the OSG
states that even granting for the sake of argument that the respondents have been in possession of the
property since 1940, their adverse possession should be reckoned only from 28 September 1981 when
the property was declared to be within alienable and disposable zone.
The petition is meritorious.
While the rule is well settled that the findings of fact of appellate courts are conclusive upon us,
11
there
are recognized exceptions thereto, among which is where the findings of fact are not supported by the
record or are so glaringly erroneous as to constitute a serious abuse of discretion.
12
This exception is
present in this case.
Section 48(b) of C.A. No. 141, as amended by Republic Act No. 1942,
13
reads as follows:
Section 48. The following described citizens of the Philippines, occupying lands of the public
domain or claiming to own any such lands or an interest therein, but whose titles have not been
perfected or completed, may apply to the Court of First Instance of the province where the land is
located for confirmation of their claims and the issuance of a certificate of title therefor, under the
Land Registration Act, to wit:

(b) Those who by themselves or through their predecessors-in-interest have been in
open, continuous, exclusive, and notorious possession and occupation of agricultural
lands of the public domain, under a bona fide claim of acquisition of ownership, for at
least thirty years immediately preceding the filing of the application for confirmation of title
except when prevented by war or force majeure. These shall be conclusively presumed
to have performed all the conditions essential to a Government grant and shall be entitled
to a certificate of title under the provisions of this chapter.
This provision was further amended by P.D. No. 1073
14
by substituting the phrase "for at least thirty
years" with "since June 12, 1945"; thus:
SEC. 4. The provisions of Section 48(b) and Section 48(c), Chapter VIII, of the Public Land Act
are hereby amended in the sense that these provisions shall apply only to alienable and
disposable lands of the public domain which have been in open, continuous, exclusive and
notorious possession and occupation by the applicant himself or through his predecessor-in-
interest, under a bona fide claim of acquisition of ownership, since June 12, 1945.
The date "12 June 1945" was reiterated in Section 14(1) of P. D. No. 1529,
15
otherwise known as
the Property Registration Decree, provides:
SEC. 14. Who may apply. The following persons may file in the proper Court of First Instance
[now Regional Trial Court] an application for registration of title to land, whether personally or
through their duly authorized representatives:
(1) Those who by themselves or through their predecessors-in-interest have been in
open, continuous, exclusive and notorious possession and occupation of alienable and
disposable lands of the public domain under a bona fide claim of ownership since June
12, 1945, or earlier. (Emphasis supplied).
Applicants for confirmation of imperfect title must, therefore, prove the following: (a) that the land forms
part of the disposable and alienable agricultural lands of the public domain; and (b) that they have been in
open, continuous, exclusive, and notorious possession and occupation of the same under a bona fide
claim of ownership either since time immemorial or since 12 June 1945.
There is no doubt that the subject property is part of the disposable and alienable agricultural lands of the
public domain. But it is not clear as to when it was classified as alienable and disposable by proper
authorities.
We do not find merit in OSG's claim that the subject property was classified as within the alienable and
disposable zone only on 28 September 1981, and hence, possession by respondents' predecessors-in-
interest before that date cannot be considered. In support of this claim, the OSG relies on a statement
appearing in the survey plan marked as Exhibit "Q," which reads:
This survey is inside alienable and disposable area as per Project No. 23-A L.C. Map No. 004
certified on September 28, 1981 and is outside any civil or military reservation.
As postulated by the respondents, the phrase "certified on September 28, 1981" could not have meant
that Lot 2111 became alienable and disposable only on 28 September 1981. That date obviously refers to
the time thatProject No. 23-A L.C. Map No. 004 was certified.
Neither can we give weight to the contention of the respondents that since Project No. 23-A L.C. Map No.
004 of which Lot 2111 forms part was approved on 31 December 1925 by the then Bureau of Forestry,
Lot 2111 must have been disposable and alienable as early as of that date. There is nothing to support
their claim that 31 December 1925 is the date of the approval of such project or the date of the
classification of the subject property as disposable and alienable public land. It is settled that a person
who seeks registration of title to a piece of land must prove his claim by clear and convincing
evidence.
16
The respondents have failed to discharge the burden of showing that Lot 2111 was classified
as part of the disposable and alienable agricultural lands of public domain as of 12 June 1945 or earlier.
Likewise, the respondent have miserably failed to prove that they and their predecessors-in-interest have
been in open, continuous, exclusive, and notorious possession and occupation of the subject property
under a bona fide claim of ownership either since time immemorial or since 12 June 1945.1awphil.net
The trial court and the Court of Appeals based the finding of fifty-seven years of possession by the
respondents and their predecessors-in-interest on the testimonies of Carmencita and Mauricio. The two
were aged 62
17
and 60,
18
respectively, when they testified in 1997. Thus, they must have been born in
1935 and 1937, respectively. If the asserted possession lasted for a period of fifty-seven years at the time
they testified, the same must have commenced sometime in 1940, or at the time that Carmencita was just
5 years old and Mauricio, about 3 years old. It is quite impossible that they could fully grasp, before
coming to the age of reason, the concept of possession of such a big tract of land and testify thereon
nearly six decades later. In short their testimonies could not be relied upon to prove the adverse
possession of the subject parcel of land by their parents.
In any case, respondents' bare assertions of possession and occupation by their predecessors-in-interest
since 1940 (as testified to by Carmencita
19
) or since 1949 (as testified to by Mauricio
20
and declared in
respondents' application for registration) are hardly "the well-nigh incontrovertible" evidence required in
cases of this nature. Proof of specific acts of ownership must be presented to substantiate their claim.
They cannot just offer general statements which are mere conclusions of law than factual evidence of
possession.
21
Even granting that the possession by the respondents' parents commenced in 1940, still
they failed to prove that their predecessors-in-interest had been in open, continuous, exclusive, and
notorious possession and occupation of the subject land under a bona fide claim of acquisition of
ownership.
The law speaks of possession and occupation. Since these words are separated by the conjunction and,
the clear intention of the law is not to make one synonymous with the other. Possession is broader than
occupation because it includes constructive possession. When, therefore, the law adds the
word occupation, it seeks to delimit the all encompassing effect of constructive possession. Taken
together with the words open, continuous, exclusive and notorious, the word occupation serves to
highlight the fact that for an applicant to qualify, his possession must not be a mere fiction.
22
Actual
possession of a land consists in the manifestation of acts of dominion over it of such a nature as a party
would naturally exercise over his own property.
23

No evidence on record shows that Spouses Mauricio and Luz Melendez cultivated, had control over, or
used the whole or even a greater portion of the tract of land for agricultural purposes.
24
Moreover, only
one tenant worked on the land, and there is no evidence as to how big was the portion occupied by the
tenant. Moreover, there is no competent proof that the Melendez Spouses declared the land in their name
for taxation purposes or paid its taxes. While tax receipts and declarations are not incontrovertible
evidence of ownership, they constitute, at the least, proof that the holder has a claim of title over the
property.
25
The voluntary declaration of a piece of property for taxation purposes not only manifests one's
sincere and honest desire to obtain title to the property, but also announces an adverse claim against the
State and all other interested parties with an intention to contribute needed revenues to the government.
Such an act strengthens one's bona fide claim of acquisition of ownership.
26

The respondents claim that they immediately took possession of the subject land upon the death of their
parents, Mauricio and Luz Melendez, who died on 5 May 1976 and 19 February 1967, respectively, and
that they had been religiously paying the taxes thereon. If that were so, why had they not themselves
introduced any improvement on the land?
27
We even find unsubstantiated the claim of Carmencita that
they had a tenant on the land. They did not present any tenant. In any case, we wonder how one tenant
could have cultivated such a vast tract of land with an area of 2.6 hectares.
The records also reveal that the subject property was declared for taxation purposes by the respondents
only for the year 1994. They paid the taxes thereon only for the years 1990, 1991, 1992, 1994, 1996, and
1997. Being of recent dates, we cannot trust the assertion of the respondents that they immediately took
possession of the property in the concept of an owner after the death of their parents. While belated
declaration of a property for taxation purposes does not necessarily negate the fact of possession,
28
tax
declarations or realty tax payments of property are, nevertheless, good indicia of possession in the
concept of an owner, for no one in his right mind would be paying taxes for a property that is not in his
actual or, at least, constructive possession.
29

Likewise, it is noteworthy that none of the respondents reside on the subject property. Carmencita even
admitted that plans of selling the property were at hand. Thus, it would be rational to conclude that this
move for registration is just but a camouflage by smart land speculators who saw in the land applied for
expected profits from its existence.
In a nutshell, the respondents did not have in their favor an imperfect title over the land subject of the
application at the time MTC LRC Case No. 06 was filed with the trial court. They failed to prove that (1)
Lot 2111 was classified as part of the disposable and alienable agricultural lands of public domain as of
12 June 1945 or earlier; (2) they and their predecessors-in-interest have been in continuous, exclusive,
and adverse possession and occupation thereof in the concept of owners from 12 June 1945 or earlier.
WHEREFORE, the petition is GRANTED, and the decisions of the Court of Appeals of 26 August 2002 in
CA-G.R. CV No. 64323 and of the Municipal Trial Court of Cabuyao, Laguna, of 1 September 1998 in
MTC LRC Case No. 06 are hereby REVERSED and SET ASIDE. The land registration case MTC LRC
Case No. 06 is hereby orderedDISMISSED.
Costs de oficio.
SO ORDERED.
Panganiban, Ynares-Santiago, Carpio, and Azcuna, JJ., concur.

You might also like